You are on page 1of 813

Cambridge IGCSE

BIOLOGY
Question papers. Marking schemes

0610/41/M/J/22--- page 1 0610/41/M/J/22--- page 637

0610/42/M/J/22---page 21 0610/42/M/J/22---page 645

0610/43/M/J/22---page 41 0610/43/M/J/22---page 653

0610/42/F/M/22---page 65 0610/42/F/M/22---page 650

0610/41/M/J/21---page 85 0610/41/M/J/21---page 668

0610/42/M/J/21---page 105 0610/42/M/J/21---page 677

0610/43/M/J/21---page 125 0610/43/M/J/21---page 685

0610/41/O/N/21---page 145 0610/41/O/N/21---page 693

0610/42/O/N/21---page 165 0610/42/O/N/21---page 700

0610/43/O/N/21---page 185 0610/43/O/N/21---page 709

0610/42/F/M/21---page 205 0610/42/F/M/21---page 718

0610/41/M/J/20---page 225 0610/41/M/J/20---page 724

0610/42/M/J/20---page 245 0610/42/M/J/20---page 732

0610/43/M/J/20---page 269 0610/43/M/J/20---page 738

0610/41/O/N/20---page 289 0610/41/O/N/20---page 747

0610/42/O/N/20---page 305 0610/42/O/N/20---page 756

0610/43/O/N/20---page 325 0610/43/O/N/20---page 765

0610/42/F/M/20---page 341 0610/42/F/M/20---page 776

0610/41/M/J/19---page 361 0610/41/M/J/19---page 783

0610/42/M/J/19---page 381 0610/42/M/J/19---page 794

0610/43/M/J/19---page 401 0610/43/M/J/19---page 803

0610/41/O/N/19---page 421 0610/41/O/N/19---page 812

0610/42/O/N/19---page 441 0610/43/O/N/19---page 821

0610/43/O/N/19---page 465 0610/42/F/M/19---page 830

0610/42/F/M/19---page 485 0610/41/M/J/18---page 836

0610/41/M/J/18---page 501 0610/42/M/J/18---page 842

0610/42/M/J/18---page 521 0610/43/M/J/18---page 849

0610/43/M/J/18---page 541 0610/41/O/N/18---page 857

0610/41/O/N/18---page 561 0610/42/O/N/18---page 864

0610/42/O/N/18---page 577 0610/42/F/M/18---page 871

0610/43/O/N/18---page 597
0610/42/F/M/18---page 617

Complied by Wisely Natugonza, King David Academy +250782298261


Cambridge IGCSE™
* 6 3 7 6 0 2 7 4 8 9 *

BIOLOGY 0610/41
Paper 4 Theory (Extended) May/June 2022

1 hour 15 minutes

You must answer on the question paper.

No additional materials are needed.

INSTRUCTIONS
● Answer all questions.
● Use a black or dark blue pen. You may use an HB pencil for any diagrams or graphs.
● Write your name, centre number and candidate number in the boxes at the top of the page.
● Write your answer to each question in the space provided.
● Do not use an erasable pen or correction fluid.
● Do not write on any bar codes.
● You may use a calculator.
● You should show all your working and use appropriate units.

INFORMATION
● The total mark for this paper is 80.
● The number of marks for each question or part question is shown in brackets [ ].

This document has 20 pages. Any blank pages are indicated.

DC (RW/CB) 302037/5
© UCLES 2022 [Turn over

Page 1 of 877
2

1 (a) Some students were studying the activity of yeast. They made a fact file, as shown in Fig. 1.1.

Organism: yeast

Scientific name: Saccharomyces cerevisiae

Structure: single cells

Main cell structures: cell wall, cell membrane,


vacuole, nucleus, mitochondria, rough
endoplasmic reticulum

Main energy source: sugars

Fig. 1.1

(i) State the kingdom in which yeast is classified.

..................................................................................................................................... [1]

(ii) State the process that occurs in mitochondria to provide energy for yeast cells.

..................................................................................................................................... [1]

(b) Yeast cells make the enzyme sucrase. Sucrase catalyses the breakdown of sucrose to
glucose and fructose.

Enzymes are made of protein.

Explain how the shape of a sucrase molecule is related to its function.

...................................................................................................................................................

...................................................................................................................................................

...................................................................................................................................................

...................................................................................................................................................

............................................................................................................................................. [2]

(c) The students made an extract of sucrase from yeast cells.

They investigated the activity of the sucrase extract at different pH values. They determined
the rate of reaction at each pH.

They then calculated the rate of each reaction as a percentage of the fastest reaction, to give
the percentage activity of sucrase.

© UCLES 2022 0610/41/M/J/22

Page 2 of 877
3

The results of this investigation are shown in Fig. 1.2.

100

90

80

70

60
percentage
activity of 50
sucrase
40

30

20

10

0
2 3 4 5 6 7 8 9 10 11 12
pH

Fig. 1.2

Describe and explain the effect of pH on the activity of sucrase shown in Fig. 1.2.

...................................................................................................................................................

...................................................................................................................................................

...................................................................................................................................................

...................................................................................................................................................

...................................................................................................................................................

...................................................................................................................................................

...................................................................................................................................................

...................................................................................................................................................

...................................................................................................................................................

...................................................................................................................................................

...................................................................................................................................................

............................................................................................................................................. [6]

[Total: 10]
© UCLES 2022 0610/41/M/J/22 [Turn over
Page 3 of 877
4

2 Mammals have a double circulation.

Fig. 2.1 is a diagram of a section through the heart of a mammal. The arrows show the direction of
blood flow through the heart and blood vessels.

C
A

X septum

Fig. 2.1

(a) (i) State the name of the chamber of the heart with the thickest wall.

..................................................................................................................................... [1]

(ii) D is a vein. State the name of this vein and describe its structure.

name .................................................................................................................................

description of structure ......................................................................................................

...........................................................................................................................................

...........................................................................................................................................

...........................................................................................................................................

...........................................................................................................................................

...........................................................................................................................................
[3]

© UCLES 2022 0610/41/M/J/22

Page 4 of 877
5

(iii) Identify the structure labelled X in Fig. 2.1 and state its role in the heart.

...........................................................................................................................................

...........................................................................................................................................

...........................................................................................................................................

...........................................................................................................................................

..................................................................................................................................... [2]

© UCLES 2022 0610/41/M/J/22 [Turn over


Page 5 of 877
6

(b) Fig. 2.2 is a diagram that shows the double circulation of a mammal. The arrows indicate the
movement of oxygen and carbon dioxide in and out of the blood.

capillaries in
the lungs

capillaries in
respiring tissues

Fig. 2.2

(i) Shade the blood vessel in Fig. 2.2 that transports blood with the highest oxygen
concentration. [1]

© UCLES 2022 0610/41/M/J/22

Page 6 of 877
7

(ii) Describe the evidence shown in Fig. 2.2 that the mammal has a double circulatory
system.

...........................................................................................................................................

...........................................................................................................................................

...........................................................................................................................................

...........................................................................................................................................

..................................................................................................................................... [2]

(iii) Explain the advantages of a double circulation.

...........................................................................................................................................

...........................................................................................................................................

...........................................................................................................................................

...........................................................................................................................................

...........................................................................................................................................

...........................................................................................................................................

...........................................................................................................................................

...........................................................................................................................................

..................................................................................................................................... [4]

[Total: 13]

© UCLES 2022 0610/41/M/J/22 [Turn over


Page 7 of 877
9

3 The pancreas is an organ that has roles in the digestive and hormonal systems of humans.

Fig. 3.1 shows part of the alimentary canal and some of the associated organs.

J
stomach

K
pancreas
L
pancreatic duct

Fig. 3.1

(a) State the names of organs J, K and L.

J ................................................................................................................................................

K ...............................................................................................................................................

L ...............................................................................................................................................
[3]

(b) The pancreas secretes hormones into the blood and enzymes into the pancreatic duct. The
enzymes are released into the alimentary canal.

Complete Table 3.1 by stating the hormones and enzymes that are secreted by the pancreas.

Table 3.1

hormones secreted by the pancreas enzymes secreted by the pancreas

...................................................................... ......................................................................

...................................................................... ......................................................................

......................................................................

[5]

© UCLES 2022 0610/41/M/J/22 [Turn over


Page 9 of 877
10

(c) Chloride ions also move along the pancreatic duct.

CFTR proteins in the cells lining the pancreatic duct move chloride ions out of the cells into
the duct.

Fig. 3.2 is a diagram of a cell from the lining of the pancreatic duct showing the location and
activity of CFTR proteins.

inside the pancreatic duct

CFTR
protein

pancreatic Key:
duct cell chloride ion

not to scale

Fig. 3.2

Explain how CFTR proteins move chloride ions across the membrane of the cell shown in
Fig. 3.2.

...................................................................................................................................................

...................................................................................................................................................

...................................................................................................................................................

...................................................................................................................................................

...................................................................................................................................................

...................................................................................................................................................

............................................................................................................................................. [3]

© UCLES 2022 0610/41/M/J/22

Page 10 of 877
11

(d) The movement of chloride ions into the pancreatic duct causes water to move from the cells
into the duct to help the flow of liquid in the duct.

Explain how water moves from the cell shown in Fig. 3.2 into the pancreatic duct.

...................................................................................................................................................

...................................................................................................................................................

...................................................................................................................................................

...................................................................................................................................................

...................................................................................................................................................

...................................................................................................................................................

............................................................................................................................................. [3]

© UCLES 2022 0610/41/M/J/22 [Turn over


Page 11 of 877
12

(e) If CFTR proteins do not move chloride ions, the liquid in the pancreatic duct becomes very
sticky and the duct can become blocked.

Blocked pancreatic ducts are one effect of cystic fibrosis, which is an inherited disease. Cystic
fibrosis is caused by a mutation of the gene that codes for the CFTR protein.

Fig. 3.3 shows the pedigree diagram of a family that has two people who have cystic fibrosis.

1 2 3

4 5 6 7
Key:

female without cystic fibrosis

male without cystic fibrosis

female with cystic fibrosis

male with cystic fibrosis

Fig. 3.3

(i) The allele that causes cystic fibrosis is a recessive allele.

Describe and explain the evidence shown in Fig. 3.3 that cystic fibrosis is caused by a
recessive allele.

...........................................................................................................................................

...........................................................................................................................................

...........................................................................................................................................

...........................................................................................................................................

..................................................................................................................................... [2]

© UCLES 2022 0610/41/M/J/22

Page 12 of 877
13

(ii) Person 7 is expecting a child with a man who is heterozygous for cystic fibrosis.

Complete the genetic diagram to predict the probability of person 7 and the heterozygous
man having a child with cystic fibrosis.

Use the symbol A for the dominant allele and a for the recessive allele.

parental genotypes ................ ................

gametes +

genotypes of offspring .......................................................................................................

phenotypes of offspring .....................................................................................................

probability of having a child with cystic fibrosis .................................................................


[5]

[Total: 21]

© UCLES 2022 0610/41/M/J/22 [Turn over


Page 13 of 877
14

4 Researchers investigated the effect of adding cattle manure (cattle faeces) to fields where snap
bean plants, Phaseolus vulgaris, were grown. Cattle manure contains some protein.

(a) Explain how protein in the cattle manure is converted to the type of ions that plants can
absorb.

...................................................................................................................................................

...................................................................................................................................................

...................................................................................................................................................

...................................................................................................................................................

...................................................................................................................................................

...................................................................................................................................................

...................................................................................................................................................

...................................................................................................................................................

...................................................................................................................................................

...................................................................................................................................................

............................................................................................................................................. [5]

(b) Snap bean plants are legumes which have root nodules that contain nitrogen‑fixing bacteria.

Fig. 4.1 shows some root nodules.

root nodules

Fig. 4.1

(i) Suggest the advantage to farmers of having snap bean plants that have a large number
of root nodules.

...........................................................................................................................................

...........................................................................................................................................

..................................................................................................................................... [1]

© UCLES 2022 0610/41/M/J/22

Page 14 of 877
15

(ii) The researchers investigated the effect of adding cattle manure to fields of snap bean
plants.

• Field 1 was treated with a small quantity of cattle manure.


• Field 2 was treated with a medium quantity of cattle manure.
• Field 3 was treated with a large quantity of cattle manure.
• Field 4 was not treated with any cattle manure.

The researchers counted the number of root nodules on samples of plants from each
field when the snap beans were harvested.

The results of the investigation are shown in Fig. 4.2.

160

140

120

100
average number
of root nodules 80
per plant
60

40

20

0
1 2 3 4
field

Fig. 4.2

Calculate the percentage increase in the average number of root nodules per plant when
snap bean plants were grown with a large quantity of cattle manure (field 3) compared
with no cattle manure (field 4).

Give your answer to two significant figures.

Space for working.

.............................................................%
[3]
© UCLES 2022 0610/41/M/J/22 [Turn over
Page 15 of 877
16

(iii) When large quantities of manure are put on fields it can lead to eutrophication of streams
and rivers. This can lead to the death of fish.

Describe how eutrophication of streams and rivers can lead to the death of fish.

...........................................................................................................................................

...........................................................................................................................................

...........................................................................................................................................

...........................................................................................................................................

...........................................................................................................................................

...........................................................................................................................................

...........................................................................................................................................

...........................................................................................................................................

..................................................................................................................................... [4]

[Total: 13]

5 The Mulanje cedar, Widdringtonia whytei, is the national tree of Malawi. This species of tree
grows naturally only on Mount Mulanje in Malawi. Many of the trees have been overharvested or
destroyed by wildfires, resulting in deforestation, as shown in Fig. 5.1.

Fig. 5.1

(a) State the genus name of the Mulanje cedar tree.

............................................................................................................................................. [1]

© UCLES 2022 0610/41/M/J/22

Page 16 of 877
17

(b) Explain the undesirable effects of deforestation on habitats that are on mountains, such as
Mount Mulanje.

...................................................................................................................................................

...................................................................................................................................................

...................................................................................................................................................

...................................................................................................................................................

...................................................................................................................................................

...................................................................................................................................................

............................................................................................................................................. [3]

(c) Scientists in Malawi are working to prevent the extinction of the Mulanje cedar tree in its
natural habitat.

Explain the benefits to other organisms on Mount Mulanje of conserving the Mulanje cedar
tree in its natural habitat.

...................................................................................................................................................

...................................................................................................................................................

...................................................................................................................................................

...................................................................................................................................................

............................................................................................................................................. [2]

(d) The seeds of many endangered tree species are kept in seed banks.

Suggest why it is important to collect seeds from many individual trees of each species rather
than just one tree.

...................................................................................................................................................

...................................................................................................................................................

...................................................................................................................................................

...................................................................................................................................................

............................................................................................................................................. [2]

[Total: 8]

© UCLES 2022 0610/41/M/J/22 [Turn over


Page 17 of 877
18

6 (a) Fig. 6.1 is a flow diagram showing the events that occur to form a human fetus.

organ P ovary

diploid diploid
cell cell

process Q

haploid haploid
sperm cell egg cell

process R

diploid
cell S

process T

embryo

process U

fetus

Fig. 6.1

© UCLES 2022 0610/41/M/J/22

Page 18 of 877
19

Complete Table 6.1 by using the information in the flow diagram to identify the cell, the organ
and the processes shown in Fig. 6.1.

Table 6.1

cell, organ or process name of the cell, organ or process

organ P

cell S

process Q produces
haploid sperm and eggs
process R produces
diploid cell S
process T occurs so that
cell S can grow into an
embryo
process U occurs so that
the embryo can gain
oxygen and nutrients
from the mother’s blood
[6]

(b) (i) State why it is important that sperm and egg cells are haploid and not diploid.

...........................................................................................................................................

...........................................................................................................................................

..................................................................................................................................... [1]

(ii) State the function of the jelly coat that surrounds egg cells.

...........................................................................................................................................

...........................................................................................................................................

..................................................................................................................................... [1]

© UCLES 2022 0610/41/M/J/22 [Turn over


Page 19 of 877
20

(c) Complete the sentences with the appropriate words.

The placenta provides a large surface area for the ............................................... of oxygen

and carbon dioxide between maternal and fetal blood. Dissolved nutrients also pass across

the placenta. Examples of dissolved nutrients are: ............................................... acids,

............................................... and ............................................... .

Antibodies pass from the maternal blood giving natural ...............................................

immunity to the baby for some infections that the mother has had or has been vaccinated

against. Each different type of vaccine contains one or more ...............................................

taken from the ............................................... that causes the disease.


[7]

[Total: 15]

Permission to reproduce items where third‑party owned material protected by copyright is included has been sought and cleared where possible. Every
reasonable effort has been made by the publisher (UCLES) to trace copyright holders, but if any items requiring clearance have unwittingly been included, the
publisher will be pleased to make amends at the earliest possible opportunity.

To avoid the issue of disclosure of answer‑related information to candidates, all copyright acknowledgements are reproduced online in the Cambridge
Assessment International Education Copyright Acknowledgements Booklet. This is produced for each series of examinations and is freely available to download
at www.cambridgeinternational.org after the live examination series.

Cambridge Assessment International Education is part of Cambridge Assessment. Cambridge Assessment is the brand name of the University of Cambridge
Local Examinations Syndicate (UCLES), which is a department of the University of Cambridge.

© UCLES 2022 0610/41/M/J/22

Page 20 of 877
Cambridge IGCSE™
* 1 7 9 7 2 1 3 5 0 4 *

BIOLOGY 0610/42
Paper 4 Theory (Extended) May/June 2022

1 hour 15 minutes

You must answer on the question paper.

No additional materials are needed.

INSTRUCTIONS
● Answer all questions.
● Use a black or dark blue pen. You may use an HB pencil for any diagrams or graphs.
● Write your name, centre number and candidate number in the boxes at the top of the page.
● Write your answer to each question in the space provided.
● Do not use an erasable pen or correction fluid.
● Do not write on any bar codes.
● You may use a calculator.
● You should show all your working and use appropriate units.

INFORMATION
● The total mark for this paper is 80.
● The number of marks for each question or part question is shown in brackets [ ].

This document has 20 pages. Any blank pages are indicated.

DC (RW/SW) 302038/4
© UCLES 2022 [Turn over

Page 21 of 877
2

1 Fig. 1.1 shows a spongy mesophyll cell from the leaf of a plant. The arrows show the net direction
of movement of carbon dioxide molecules during daylight.

A
G
B
F

0.01 mm

Fig. 1.1

(a) The scale bar in Fig. 1.1 represents 0.01 mm.

Convert 0.01 mm to micrometres.

..................................................... μm [1]

(b) Table 1.1 shows:

• the functions of some of the structures in plant cells


• some of the names of the structures where these functions occur
• some of the letters that label these structures in Fig. 1.1.

Complete Table 1.1.

Table 1.1

function structure letter in Fig. 1.1

nucleus

chloroplast

aerobic respiration

contains cell sap and stores water

[5]

© UCLES 2022 0610/42/M/J/22

Page 22 of 877
3

(c) Carbon dioxide is a raw material for photosynthesis.

(i) State the process by which carbon dioxide travels into the leaf from the air.

..................................................................................................................................... [1]

(ii) Describe the pathway taken by a molecule of carbon dioxide, from the air outside a leaf
to a spongy mesophyll cell.

...........................................................................................................................................

...........................................................................................................................................

...........................................................................................................................................

...........................................................................................................................................

..................................................................................................................................... [2]

(d) Low concentrations of carbon dioxide in the air may restrict the rate of photosynthesis in
plants.

(i) State the term given to something present in the environment in such short supply that it
restricts life processes.

..................................................................................................................................... [1]

(ii) State one other feature of the environment that may also restrict the rate of
photosynthesis.

..................................................................................................................................... [1]

(e) Researchers have devised a process of artificial photosynthesis. They use gold nanoparticles
as a catalyst to utilise green light to convert carbon dioxide to fuels, such as propane.

Suggest the advantages to the environment of using artificial photosynthesis on a large scale.

...................................................................................................................................................

...................................................................................................................................................

...................................................................................................................................................

...................................................................................................................................................

............................................................................................................................................. [2]

[Total: 13]

© UCLES 2022 0610/42/M/J/22 [Turn over


Page 23 of 877
4

2 The kidneys filter blood, separate useful molecules from excretory wastes and control the water
content of the blood.

Fig. 2.1 is a diagram of a kidney tubule and associated blood vessels. The arrows show the
direction of blood flow.

Fig. 2.2 is a drawing of a vertical section through a cell from the lining of region 2 of the tubule.

Q
P

1
2

R
S

Fig. 2.1 Fig. 2.2

(a) (i) State the name of structure P.

..................................................................................................................................... [1]

(ii) Blood vessel Q has the highest blood pressure.

Suggest why.

...........................................................................................................................................

...........................................................................................................................................

..................................................................................................................................... [1]

(iii) The structures labelled S on Fig. 2.2 are microvilli.

Explain the importance of the microvilli on the surface of these cells.

...........................................................................................................................................

...........................................................................................................................................

...........................................................................................................................................

...........................................................................................................................................

..................................................................................................................................... [2]
© UCLES 2022 0610/42/M/J/22

Page 24 of 877
5

(b) Table 2.1 shows the concentrations of some substances in blood plasma and in the regions
labelled 1 and 3 on the tubule shown in Fig. 2.1.

Table 2.1

concentration / mg per cm3


substance
blood plasma region 1 region 3
protein 8000 0 0
glucose 100 100 0
salts 320 320 300
urea 30 30 2000

Outline how the kidney tubules function to produce urine from the substances in blood
plasma.

Use the information in Fig. 2.1, Fig. 2.2 and Table 2.1 to support your answer.

...................................................................................................................................................

...................................................................................................................................................

...................................................................................................................................................

...................................................................................................................................................

...................................................................................................................................................

...................................................................................................................................................

...................................................................................................................................................

...................................................................................................................................................

...................................................................................................................................................

...................................................................................................................................................

...................................................................................................................................................

...................................................................................................................................................

............................................................................................................................................. [6]

© UCLES 2022 0610/42/M/J/22 [Turn over


Page 25 of 877
6

(c) The kidneys are examples of organs that help the body to maintain a constant internal
environment.

(i) State the term for maintaining a constant internal environment by negative feedback.

..................................................................................................................................... [1]

(ii) Explain how negative feedback controls the blood glucose concentration of a person
who has not eaten for a day.

...........................................................................................................................................

...........................................................................................................................................

...........................................................................................................................................

...........................................................................................................................................

...........................................................................................................................................

...........................................................................................................................................

..................................................................................................................................... [3]

[Total: 14]

© UCLES 2022 0610/42/M/J/22

Page 26 of 877
7

3 The eye is adapted for focusing on near and distant objects.

Fig. 3.1 shows the parts of the eye involved in focusing. The eye is focused on a distant object.

cornea

Fig. 3.1

(a) (i) State the term used to describe what happens to light as it passes from the air into the
cornea.

..................................................................................................................................... [1]

(ii) Describe and explain the changes that occur in the eye when adjusting focus from a
distant object to a near object.

...........................................................................................................................................

...........................................................................................................................................

...........................................................................................................................................

...........................................................................................................................................

...........................................................................................................................................

...........................................................................................................................................

..................................................................................................................................... [3]

© UCLES 2022 0610/42/M/J/22 [Turn over


Page 27 of 877
8

(b) Rods and cones are the receptors in the retina of the eye.

(i) Describe the functions of rods and cones in the eye.

...........................................................................................................................................

...........................................................................................................................................

...........................................................................................................................................

...........................................................................................................................................

...........................................................................................................................................

...........................................................................................................................................

...........................................................................................................................................

...........................................................................................................................................

..................................................................................................................................... [4]

© UCLES 2022 0610/42/M/J/22

Page 28 of 877
9

(ii) Fig. 3.2 is a photograph showing regions of a human retina, as viewed through the pupil
at the front of the eye.

peripheral retina

blind spot fovea

blood
vessel

Fig. 3.2

Complete Table 3.1 to show the distribution of rods and cones across the retina.

Use these words to complete the table, each word may be used once, more than once or
not at all:

many few none

Table 3.1

distribution across the retina


receptor
peripheral retina blind spot fovea

rods

cones

[3]

© UCLES 2022 0610/42/M/J/22 [Turn over


Page 29 of 877
10

(c) Colour blindness can be caused by a mutation in a gene. The gene is located on the
X chromosome.

Fig. 3.3 is a pedigree diagram of a family which has several people who are colour‑blind.

1 2

3 4

5 6

Key:
female with normal
colour vision
male with normal
colour vision
male with colour
blindness

Fig. 3.3

(i) Colour blindness is sex‑linked.

State the evidence from Fig. 3.3 that supports the idea that colour blindness is sex‑linked.

...........................................................................................................................................

...........................................................................................................................................

..................................................................................................................................... [1]

(ii) State the genotype of person 5.

Use the symbols X and Y for the sex chromosomes and A for the dominant allele and a
for the recessive allele of the gene for colour blindness.

..................................................................................................................................... [2]

© UCLES 2022 0610/42/M/J/22

Page 30 of 877
11

(iii) Use the information in Fig. 3.3 to complete the genetic diagram to show the probability of
person 3 and person 4 having another child with colour blindness.

person 3 person 4

parental female with normal male with normal


phenotypes colour vision colour vision

parental
genotypes ................................ ................................

gametes +

offspring
genotypes ..........................................................................................................................

offspring
phenotypes ........................................................................................................................

probability of a child having colour blindness ....................................................................


[5]

[Total: 19]

© UCLES 2022 0610/42/M/J/22 [Turn over


Page 31 of 877
12

4 Fig. 4.1 is a diagram of a virus.

Fig. 4.1

(a) Identify the parts of the virus labelled N and O.

N ...............................................................................................................................................

O ...............................................................................................................................................
[2]

(b) The human immunodeficiency virus (HIV) infects and destroys lymphocytes.

The number of lymphocytes in the blood of a person infected with HIV was measured over a
period of 84 months.

The results are shown in Fig. 4.2.

800

700

600

500
number of
lymphocytes 400
per mm3 of blood
300

200

100

0
0 10 20 30 40 50 60 70 80 90
time after HIV infection / months

Fig. 4.2

© UCLES 2022 0610/42/M/J/22

Page 32 of 877
13

(i) Use the information shown in Fig. 4.2 to calculate the percentage change in the number
of lymphocytes from month 10 to month 60.

Give your answer to two significant figures.

Space for working.

.............................................................%
[3]

(ii) Describe the changes in the number of lymphocytes, over the 84 months following
infection with HIV, shown in Fig. 4.2.

...........................................................................................................................................

...........................................................................................................................................

...........................................................................................................................................

...........................................................................................................................................

..................................................................................................................................... [2]

(iii) Outline the consequences of the changes in the number of lymphocytes for the health of
the person infected with HIV.

...........................................................................................................................................

...........................................................................................................................................

...........................................................................................................................................

...........................................................................................................................................

...........................................................................................................................................

...........................................................................................................................................

...........................................................................................................................................

...........................................................................................................................................

..................................................................................................................................... [4]

© UCLES 2022 0610/42/M/J/22 [Turn over


Page 33 of 877
14

(iv) Explain why antibiotics are not used to treat viral infections.

...........................................................................................................................................

...........................................................................................................................................

...........................................................................................................................................

...........................................................................................................................................

..................................................................................................................................... [2]

[Total: 13]

5 Fig. 5.1 is a diagram of the human gas exchange system.

Fig. 5.1

(a) (i) Identify the parts of the gas exchange system labelled X, Y and Z in Fig. 5.1.

X ........................................................................................................................................

Y ........................................................................................................................................

Z ........................................................................................................................................
[3]

(ii) State the name of the tissue that prevents the collapse of Y and Z during breathing.

..................................................................................................................................... [1]

© UCLES 2022 0610/42/M/J/22

Page 34 of 877
15

(b) Breathing involves the movement of the ribs and the diaphragm.

Describe the process of inspiration.

...................................................................................................................................................

...................................................................................................................................................

...................................................................................................................................................

...................................................................................................................................................

...................................................................................................................................................

...................................................................................................................................................

...................................................................................................................................................

...................................................................................................................................................

............................................................................................................................................. [4]

(c) State the name of the gas exchange surface in the lungs.

............................................................................................................................................. [1]

[Total: 9]

© UCLES 2022 0610/42/M/J/22 [Turn over


Page 35 of 877
16

6 Coral reefs are important ecosystems.

(a) Complete the sentence about ecosystems.

An ecosystem can be defined as a unit containing the ................................................. of

organisms and their ................................................., interacting together.


[2]

Fig. 6.1 is a photograph of a coral reef.

Fig. 6.1

(b) Fig. 6.2 is part of a food web for a coral reef ecosystem which is similar to the one shown in
Fig. 6.1.

reef shark
pufferfish

fan worm blue chromis butterflyfish angelfish


fish

coral polyps sea sponge

zooplankton

phytoplankton

Fig. 6.2

© UCLES 2022 0610/42/M/J/22

Page 36 of 877
17

(i) State the number of secondary consumers shown in Fig. 6.2.

..................................................................................................................................... [1]

(ii) State the name of a species that feeds at more than one trophic level in Fig. 6.2.

..................................................................................................................................... [1]

(iii) State the number of different organisms in the shortest food chain in Fig. 6.2.

..................................................................................................................................... [1]

(iv) State the evidence from the food web in Fig. 6.2, that phytoplankton are producers.

..................................................................................................................................... [1]

(v) It is rare for there to be more than five trophic levels in an ecosystem.

Explain what limits the number of trophic levels in food webs.

...........................................................................................................................................

...........................................................................................................................................

...........................................................................................................................................

...........................................................................................................................................

...........................................................................................................................................

...........................................................................................................................................

..................................................................................................................................... [3]

(c) One threat to aquatic ecosystems, such as coral reefs, is global climate change.

Discuss the threats to aquatic ecosystems, other than climate change.

...................................................................................................................................................

...................................................................................................................................................

...................................................................................................................................................

...................................................................................................................................................

...................................................................................................................................................

...................................................................................................................................................

............................................................................................................................................. [3]

[Total: 12]

© UCLES 2022 0610/42/M/J/22

Page 37 of 877
Cambridge IGCSE™
* 8 9 4 8 6 9 2 8 8 4 *

BIOLOGY 0610/43
Paper 4 Theory (Extended) May/June 2022

1 hour 15 minutes

You must answer on the question paper.

No additional materials are needed.

INSTRUCTIONS
● Answer all questions.
● Use a black or dark blue pen. You may use an HB pencil for any diagrams or graphs.
● Write your name, centre number and candidate number in the boxes at the top of the page.
● Write your answer to each question in the space provided.
● Do not use an erasable pen or correction fluid.
● Do not write on any bar codes.
● You may use a calculator.
● You should show all your working and use appropriate units.

INFORMATION
● The total mark for this paper is 80.
● The number of marks for each question or part question is shown in brackets [ ].

This document has 24 pages. Any blank pages are indicated.

DC (CE/CT) 303967/4
© UCLES 2022 [Turn over

Page 41 of 877
2

1 Penicillin is produced by biotechnology industries.

(a) (i) State the name of the type of pathogen penicillin is used to treat.

..................................................................................................................................... [1]

(ii) State the name of the group of medicinal drugs that includes penicillin.

..................................................................................................................................... [1]

(b) Fig. 1.1 is a flow diagram of some of the steps in the production of penicillin.

organism A in step 4
a test-tube

X Y Z
step 5
gas outflow
pipe

step 1 filter

step 3
nutrient
mixing tank step 8
step 6
sterilising tank penicillin

step 2 step 7
fermenter not to scale

Fig. 1.1

(i) Organism A belongs to the fungus kingdom.

State two main features of fungal cells that are used to distinguish them from the cells of
prokaryotes.

1 ........................................................................................................................................

2 ........................................................................................................................................
[2]

(ii) State the genus name of organism A in Fig. 1.1.

..................................................................................................................................... [1]

© UCLES 2022 0610/43/M/J/22

Page 42 of 877
3

(iii) Penicillin is produced in the fermenter shown in Fig. 1.1. A variety of nutrients, X, Y and Z,
are mixed together and added to the fermenter in step 1.

List two nutrients that need to be added to a fermenter to produce penicillin.

1 ........................................................................................................................................

2 ........................................................................................................................................
[2]

(iv) Explain why the nutrients are sterilised (step 2) before they are added to the fermenter
(step 3).

...........................................................................................................................................

...........................................................................................................................................

..................................................................................................................................... [1]

(v) Explain why the fermenter has a gas outflow pipe.

...........................................................................................................................................

...........................................................................................................................................

...........................................................................................................................................

...........................................................................................................................................

..................................................................................................................................... [2]

(vi) Using the information in Fig. 1.1, outline the events occurring from step 4 to step 8 during
the production of penicillin.

...........................................................................................................................................

...........................................................................................................................................

...........................................................................................................................................

...........................................................................................................................................

...........................................................................................................................................

...........................................................................................................................................

...........................................................................................................................................

...........................................................................................................................................

..................................................................................................................................... [4]

[Total: 14]

© UCLES 2022 0610/43/M/J/22 [Turn over


Page 43 of 877
4

2 Red blood cells contain the protein haemoglobin.

(a) (i) State the names of the four chemical elements that are found in all proteins.

...........................................................................................................................................

..................................................................................................................................... [2]

(ii) State the role of haemoglobin.

...........................................................................................................................................

...........................................................................................................................................

..................................................................................................................................... [1]

(b) Fig. 2.1 shows a photomicrograph of some red blood cells from a person with sickle cell
anaemia.

Abnormal red blood cells occur because of a mutation in the gene for haemoglobin.

abnormal red
blood cell

Fig. 2.1

Suggest how the shape of the abnormal red blood cell shown in Fig. 2.1 will affect blood flow.

...................................................................................................................................................

...................................................................................................................................................

...................................................................................................................................................

...................................................................................................................................................

............................................................................................................................................. [2]

© UCLES 2022 0610/43/M/J/22

Page 44 of 877
5

(c) The allele for the normal form of haemoglobin is HbA.

The allele for the abnormal form of haemoglobin is HbS.

Draw a genetic diagram to determine the probability of two heterozygous parents having a
child who does not have the HbS allele.

parental phenotypes ................................ x ................................

parental genotypes ................................ x ................................

gametes , x ,
............. ............. ............. .............

offspring genotypes ..................................................................................................................

probability of offspring not having the HbS allele .....................................................................


[5]

© UCLES 2022 0610/43/M/J/22 [Turn over


Page 45 of 877
6

(d) Fig. 2.2 and Fig. 2.3 are maps showing some of the different regions in a country. Scientists
studied the distribution of the HbS allele in the country.

Fig. 2.2 shows the estimated frequency of the allele within the population.

Fig. 2.3 shows the estimated number of babies born with sickle cell anaemia in each region.

low

estimated frequency A
of the HbS allele C
in the population
B

high

Fig. 2.2

estimated number A
low
of babies born C
medium
with sickle cell
anaemia high B

Fig. 2.3

© UCLES 2022 0610/43/M/J/22

Page 46 of 877
7

The scientists made a statement:

‘There is a relationship between the frequency of the HbS allele and the number of babies
born with sickle cell anaemia in regions A, B and C.’

(i) Using the information in Fig. 2.2 and Fig. 2.3, discuss the evidence for and against this
statement for regions A, B and C only.

...........................................................................................................................................

...........................................................................................................................................

...........................................................................................................................................

...........................................................................................................................................

...........................................................................................................................................

...........................................................................................................................................

...........................................................................................................................................

...........................................................................................................................................

...........................................................................................................................................

...........................................................................................................................................

..................................................................................................................................... [5]

(ii) Suggest how the scientists would identify the presence of the HbS allele in tissue
samples.

...........................................................................................................................................

...........................................................................................................................................

..................................................................................................................................... [1]

© UCLES 2022 0610/43/M/J/22 [Turn over


Page 47 of 877
8

(e) Mutations are always inherited in single-celled organisms that reproduce asexually but are
not always inherited in organisms that reproduce sexually.

Explain why.

...................................................................................................................................................

...................................................................................................................................................

...................................................................................................................................................

...................................................................................................................................................

...................................................................................................................................................

...................................................................................................................................................

...................................................................................................................................................

...................................................................................................................................................

............................................................................................................................................. [4]

[Total: 20]

© UCLES 2022 0610/43/M/J/22

Page 48 of 877
9

3 Acid rain has destroyed many forests including the forest shown in Fig. 3.1.

Fig. 3.1

(a) Describe how acid rain destroys forests.

...................................................................................................................................................

...................................................................................................................................................

...................................................................................................................................................

...................................................................................................................................................

...................................................................................................................................................

...................................................................................................................................................

............................................................................................................................................. [3]

© UCLES 2022 0610/43/M/J/22 [Turn over


Page 49 of 877
10

(b) Explain the negative consequences to the environment of destroying forests.

...................................................................................................................................................

...................................................................................................................................................

...................................................................................................................................................

...................................................................................................................................................

...................................................................................................................................................

...................................................................................................................................................

...................................................................................................................................................

...................................................................................................................................................

............................................................................................................................................. [4]

(c) Acid rain can also damage aquatic organisms such as the amphibian shown in Fig. 3.2.

Fig. 3.2

Suggest why amphibians are vulnerable to pollutants such as acid rain.

...................................................................................................................................................

...................................................................................................................................................

...................................................................................................................................................

...................................................................................................................................................

............................................................................................................................................. [2]

© UCLES 2022 0610/43/M/J/22

Page 50 of 877
11

(d) Many countries have strict laws to prevent acid rain.

Describe how countries have reduced acid rain.

...................................................................................................................................................

...................................................................................................................................................

...................................................................................................................................................

...................................................................................................................................................

...................................................................................................................................................

...................................................................................................................................................

............................................................................................................................................. [3]

[Total: 12]

© UCLES 2022 0610/43/M/J/22 [Turn over


Page 51 of 877
12

4 (a) Fig. 4.1 is a diagram of the alimentary canal.

Fig. 4.1

© UCLES 2022 0610/43/M/J/22

Page 52 of 877
13

Complete Table 4.1 by stating:

• the names of the organs from Fig. 4.1


• the letters of all the processes shown in the key that occur in each organ.

Key:
A – absorption
C – chemical digestion
E – egestion
I – ingestion
M – mechanical digestion

Table 4.1

number from letter or letters of all the


name of the organ
Fig. 4.1 processes that occur in the organ

[6]

© UCLES 2022 0610/43/M/J/22 [Turn over


Page 53 of 877
14

(b) Fig. 4.2 is a diagram of a villus. The arrow indicates the direction of blood flow.

U S
T

not to scale

Fig. 4.2

© UCLES 2022 0610/43/M/J/22

Page 54 of 877
15

Describe the structure of a villus and its role in the alimentary canal.

Use the letters in Fig. 4.2 to support your answer.

...................................................................................................................................................

...................................................................................................................................................

...................................................................................................................................................

...................................................................................................................................................

...................................................................................................................................................

...................................................................................................................................................

...................................................................................................................................................

...................................................................................................................................................

...................................................................................................................................................

...................................................................................................................................................

...................................................................................................................................................

...................................................................................................................................................

............................................................................................................................................. [6]

[Total: 12]

© UCLES 2022 0610/43/M/J/22 [Turn over


Page 55 of 877
16

5 Glasshouses are designed to maximise crop plant yield.

(a) (i) Explain why carbon dioxide enrichment is used in many glasshouses to increase crop
plant yield.

...........................................................................................................................................

...........................................................................................................................................

...........................................................................................................................................

...........................................................................................................................................

...........................................................................................................................................

...........................................................................................................................................

..................................................................................................................................... [3]

(ii) Suggest how the carbon dioxide concentration in a glasshouse can be enriched.

...........................................................................................................................................

...........................................................................................................................................

..................................................................................................................................... [1]

(iii) Outline how carbon dioxide in a glasshouse moves into leaves.

...........................................................................................................................................

...........................................................................................................................................

...........................................................................................................................................

...........................................................................................................................................

..................................................................................................................................... [2]

© UCLES 2022 0610/43/M/J/22

Page 56 of 877
17

(b) Additional lighting is often installed in glasshouses in countries with temperate climates.

Table 5.1 summarises some of the factors that are considered by plant growers when
choosing the type of lamps to install in a glasshouse.

Table 5.1

type of electrical energy used light intensity output


notes
lamp by the lamp / J per s / arbitrary units
• releases lots of heat
sodium 1041 1767 • best when used in
addition to sunlight
• releases very little heat
LED 423 378 • can be used as an
alternative to sunlight
• releases some heat
metal
651 817 • can be used as an
halide
alternative to sunlight
• releases some heat
fluorescent 394 374 • best when used in
addition to sunlight

(i) Calculate the percentage increase in the energy used by the metal halide lamp compared
to the energy used by the fluorescent lamp.

Give your answer to two significant figures.

Space for working.

............................................................ %
[2]

(ii) State which type of lamp has the highest light intensity output per unit of electrical energy
used.

..................................................................................................................................... [1]

© UCLES 2022 0610/43/M/J/22 [Turn over


Page 57 of 877
18

(iii) Some types of lamp release a lot of heat.

Explain the possible effects of excessive heat on the plants in a glasshouse.

...........................................................................................................................................

...........................................................................................................................................

...........................................................................................................................................

...........................................................................................................................................

...........................................................................................................................................

...........................................................................................................................................

..................................................................................................................................... [3]

[Total: 12]

© UCLES 2022 0610/43/M/J/22

Page 58 of 877
19

6 (a) Fig. 6.1 shows a photograph of fruit attached to the branch of an orange tree, Citrus sinensis.

Fig. 6.1

State one reason why orange trees are classified as dicotyledonous plants.

...................................................................................................................................................

...................................................................................................................................................

............................................................................................................................................. [1]

(b) Biotechnologists use enzymes to extract juice from fruit such as oranges.

Define the term enzyme.

...................................................................................................................................................

...................................................................................................................................................

...................................................................................................................................................

...................................................................................................................................................

............................................................................................................................................. [2]

© UCLES 2022 0610/43/M/J/22 [Turn over


Page 59 of 877
20

(c) Fig. 6.2 shows the results of leaving pieces of orange fruit in an enzyme solution for different
lengths of time.

total volume
7
of juice
extracted
/ cm3 6

4
0 20 40 60 80 100
time / minutes

Fig. 6.2

(i) State the name of the enzyme used to extract juice from fruit.

..................................................................................................................................... [1]

(ii) Using the information in Fig. 6.2, state the optimum length of time for efficient extraction
of juice from oranges.

..................................................................................................................................... [1]

(iii) State the name of the vitamin found in high concentrations in citrus fruit such as oranges.

..................................................................................................................................... [1]

(d) The genes in some plants have been changed to increase the concentration of vitamins that
these plants produce.

(i) State the name of the process of changing the genes of a plant.

..................................................................................................................................... [1]

© UCLES 2022 0610/43/M/J/22

Page 60 of 877
21

(ii) Discuss the possible disadvantages of people changing the genes in a plant.

...........................................................................................................................................

...........................................................................................................................................

...........................................................................................................................................

...........................................................................................................................................

...........................................................................................................................................

...........................................................................................................................................

..................................................................................................................................... [3]

[Total: 10]

© UCLES 2022 0610/43/M/J/22

Page 61 of 877
Cambridge IGCSE™
* 8 4 6 4 9 7 8 6 0 0 *

BIOLOGY 0610/42
Paper 4 Theory (Extended) February/March 2022

1 hour 15 minutes

You must answer on the question paper.

No additional materials are needed.

INSTRUCTIONS
● Answer all questions.
● Use a black or dark blue pen. You may use an HB pencil for any diagrams or graphs.
● Write your name, centre number and candidate number in the boxes at the top of the page.
● Write your answer to each question in the space provided.
● Do not use an erasable pen or correction fluid.
● Do not write on any bar codes.
● You may use a calculator.
● You should show all your working and use appropriate units.

INFORMATION
● The total mark for this paper is 80.
● The number of marks for each question or part question is shown in brackets [ ].

This document has 20 pages. Any blank pages are indicated.

DC (LK/SG) 303960/5
© UCLES 2022 [Turn over

Page 65 of 877
2

1 (a) (i) State the name of the gas exchange surface in humans.

..................................................................................................................................... [1]

(ii) State two features of the gas exchange surface in humans.

1 ........................................................................................................................................

2 ........................................................................................................................................
[2]

(b) Fig. 1.1 is a diagram of the gas exchange system in humans.

trachea

diaphragm

Fig. 1.1

(i) Draw a label line and the letter X on Fig. 1.1 to identify an external intercostal muscle. [1]

(ii) State the name of the tissue that forms C-shaped structures in the wall of the trachea
and state its function.

name .................................................................................................................................

function ..............................................................................................................................

...........................................................................................................................................
[2]

© UCLES 2022 0610/42/F/M/22

Page 66 of 877
3

(iii) Describe the effects on the thorax of contraction of the diaphragm.

...........................................................................................................................................

...........................................................................................................................................

...........................................................................................................................................

...........................................................................................................................................

..................................................................................................................................... [2]

(c) Table 1.1 compares the composition of inspired and expired air.

Table 1.1

percentage in percentage in
gas name of the gas
inspired air expired air

A nitrogen 78 78

B 21 16

C 0.04 4

D variable saturated

(i) Complete Table 1.1 by writing the names of gases B, C and D. [3]

(ii) For gas B and gas C, explain the differences in the percentages shown in Table 1.1
between inspired and expired air.

...........................................................................................................................................

...........................................................................................................................................

...........................................................................................................................................

...........................................................................................................................................

...........................................................................................................................................

...........................................................................................................................................

..................................................................................................................................... [3]

[Total: 14]
© UCLES 2022 0610/42/F/M/22 [Turn over
Page 67 of 877
4

2 (a) Fig. 2.1 is a photomicrograph showing the fertilisation of one human egg cell.

sperm

Fig. 2.1

Describe and explain the adaptations of the cells shown in Fig. 2.1 that enable fertilisation
and early development of the embryo to occur.

...................................................................................................................................................

...................................................................................................................................................

...................................................................................................................................................

...................................................................................................................................................

...................................................................................................................................................

...................................................................................................................................................

...................................................................................................................................................

...................................................................................................................................................

...................................................................................................................................................

...................................................................................................................................................

...................................................................................................................................................

...................................................................................................................................................

............................................................................................................................................. [6]
© UCLES 2022 0610/42/F/M/22

Page 68 of 877
5

(b) People can use artificial insemination (AI) or in vitro fertilisation (IVF) to increase their chance
of becoming pregnant.

(i) Outline the process of artificial insemination.

...........................................................................................................................................

...........................................................................................................................................

...........................................................................................................................................

...........................................................................................................................................

...........................................................................................................................................

...........................................................................................................................................

..................................................................................................................................... [3]

(ii) Outline how the process of in vitro fertilisation (IVF) differs from artificial insemination (AI).

...........................................................................................................................................

...........................................................................................................................................

...........................................................................................................................................

...........................................................................................................................................

..................................................................................................................................... [2]

(iii) Describe the social implications of fertility treatments.

...........................................................................................................................................

...........................................................................................................................................

...........................................................................................................................................

...........................................................................................................................................

...........................................................................................................................................

...........................................................................................................................................

...........................................................................................................................................

...........................................................................................................................................

..................................................................................................................................... [4]

[Total: 15]

© UCLES 2022 0610/42/F/M/22 [Turn over


Page 69 of 877
6

3 (a) A scientist investigated the effect of temperature on the mass of leaves picked from a
tea plant, Camellia sinensis.

• Three samples of leaves were picked and the mass of each sample of leaves was
recorded.
• Each sample of leaves was kept at a different temperature for four hours.
• After four hours, the mass of each sample of leaves was measured and recorded
again.
• The scientist then calculated the final mass as a percentage of the initial mass for
each sample.

Fig. 3.1 shows the results.

100

final mass as a 90
percentage of the
initial mass 80

70
0 10 20 30 40
temperature at which the leaves were kept / °C

Fig. 3.1

(i) Explain the results shown in Fig. 3.1.

...........................................................................................................................................

...........................................................................................................................................

...........................................................................................................................................

...........................................................................................................................................

...........................................................................................................................................

...........................................................................................................................................

...........................................................................................................................................

...........................................................................................................................................

...........................................................................................................................................

...........................................................................................................................................

..................................................................................................................................... [5]

(ii) State one factor, other than temperature, that would affect the loss of mass from the
leaves of a plant.

..................................................................................................................................... [1]

© UCLES 2022 0610/42/F/M/22

Page 70 of 877
7

(b) Fig. 3.2 is a photomicrograph of the tissue that transports water and mineral ions in a plant.

Fig. 3.2

(i) State the name of the tissue shown in Fig. 3.2.

..................................................................................................................................... [1]

(ii) Describe how the tissue shown in Fig. 3.2 is adapted for its functions in the plant.

...........................................................................................................................................

...........................................................................................................................................

...........................................................................................................................................

...........................................................................................................................................

...........................................................................................................................................

...........................................................................................................................................

..................................................................................................................................... [3]

© UCLES 2022 0610/42/F/M/22 [Turn over


Page 71 of 877
8

(c) Explain how mineral ions enter a plant.

...................................................................................................................................................

...................................................................................................................................................

...................................................................................................................................................

...................................................................................................................................................

...................................................................................................................................................

...................................................................................................................................................

............................................................................................................................................. [3]

[Total: 13]

© UCLES 2022 0610/42/F/M/22

Page 72 of 877
9

4 (a) Fig. 4.1 is a diagram of Vibrio cholerae, the bacterium that causes cholera.

plasmid
DNA

ribosomes

capsule

cell wall

cytoplasm

flagellum

Fig. 4.1

(i) Describe two similarities and two differences between a palisade mesophyll cell and the
bacterial cell shown in Fig. 4.1.

similarity 1 .........................................................................................................................

...........................................................................................................................................

similarity 2 .........................................................................................................................

...........................................................................................................................................

difference 1 ........................................................................................................................

...........................................................................................................................................

difference 2 ........................................................................................................................

...........................................................................................................................................
[4]

© UCLES 2022 0610/42/F/M/22 [Turn over


Page 73 of 877
10

(ii) Explain how the cholera bacterium causes diarrhoea.

...........................................................................................................................................

...........................................................................................................................................

...........................................................................................................................................

...........................................................................................................................................

...........................................................................................................................................

...........................................................................................................................................

..................................................................................................................................... [3]

(b) A scientist tested the resistance of one strain of bacteria to different antibiotics.

The scientist tested solutions of five different antibiotics, A to E.

She soaked a paper disc in each antibiotic solution.

The paper discs with antibiotics were placed in a Petri dish containing bacteria on agar jelly.

Fig. 4.2 is a diagram of the appearance of the Petri dish after 48 hours. The shaded areas
show where bacteria grew. The clear areas show where bacteria did not grow.

area where
bacteria grew
A

B
clear area paper disc
where bacteria with antibiotic
did not grow
D

Fig. 4.2

© UCLES 2022 0610/42/F/M/22

Page 74 of 877
11

(i) The strain of bacteria used in this investigation causes a disease.

Using the information in Fig. 4.2, explain why antibiotic E would be the most effective at
treating this disease.

...........................................................................................................................................

...........................................................................................................................................

..................................................................................................................................... [1]

(ii) The results in Fig. 4.2 show that this strain of bacteria is resistant to antibiotic A.

Five years ago, a similar investigation found that the clear area for antibiotic A was the
same size as antibiotic B is in Fig. 4.2.

Explain how bacteria become resistant to antibiotics.

...........................................................................................................................................

...........................................................................................................................................

...........................................................................................................................................

...........................................................................................................................................

...........................................................................................................................................

...........................................................................................................................................

...........................................................................................................................................

...........................................................................................................................................

..................................................................................................................................... [4]

(iii) Describe how to minimise the risk of antibiotic B developing the same results as
antibiotic A.

...........................................................................................................................................

...........................................................................................................................................

..................................................................................................................................... [1]

[Total: 13]

© UCLES 2022 0610/42/F/M/22 [Turn over


Page 75 of 877
13

5 The Arabian oryx and the northern white rhinoceros are both mammals.

Fig. 5.1 is a photograph of an Arabian oryx. Fig. 5.2 is a photograph of a northern white rhinoceros.

Fig. 5.1

Fig. 5.2

(a) Describe two pieces of evidence visible in Fig. 5.1 and Fig. 5.2 that show these animals are
mammals.

1 ................................................................................................................................................

2 ................................................................................................................................................
[2]

© UCLES 2022 0610/42/F/M/22 [Turn over


Page 77 of 877
14

(b) Different conservation methods are used to try to prevent species from becoming extinct.

A population of the Arabian oryx and a population of northern white rhinoceros were monitored.

Fig. 5.3 shows how the population size of each species has changed over time.

350 35

300 30

250 25

number of 200 20 number of


northern
Arabian
white
oryx
150 15 rhinoceros

100 10

50 5

0 0
1975 1980 1985 1990 1995 2000 2005 2010 2015 2020
year
key:
Arabian oryx
northern white rhinoceros

Fig. 5.3

(i) Calculate the percentage increase in the number of Arabian oryx between 1990 and
2000.

Give your answer to three significant figures.

Space for working.

............................................................ %
[3]

© UCLES 2022 0610/42/F/M/22

Page 78 of 877
15

(ii) Describe the data for the northern white rhinoceros shown in Fig. 5.3.

...........................................................................................................................................

...........................................................................................................................................

...........................................................................................................................................

...........................................................................................................................................

...........................................................................................................................................

...........................................................................................................................................

..................................................................................................................................... [3]

(c) Suggest the conservation methods that were used to increase the number of Arabian oryx
between 1978 and 2000.

...................................................................................................................................................

...................................................................................................................................................

...................................................................................................................................................

...................................................................................................................................................

...................................................................................................................................................

...................................................................................................................................................

............................................................................................................................................. [3]

(d) Explain the risks to the northern white rhinoceros species as a result of its population size.

...................................................................................................................................................

...................................................................................................................................................

...................................................................................................................................................

...................................................................................................................................................

...................................................................................................................................................

...................................................................................................................................................

............................................................................................................................................. [3]

[Total: 14]

© UCLES 2022 0610/42/F/M/22 [Turn over


Page 79 of 877
17

6 (a) Complete Table 6.1 to show the names, functions and sites of action of the three different
digestive enzymes.

Table 6.1

name of enzyme function site of action

pepsin

trypsin

breaks down maltose to glucose

[3]

© UCLES 2022 0610/42/F/M/22 [Turn over


Page 81 of 877
18

(b) Enzymes are proteins.

Fig. 6.1 shows the stages involved in protein synthesis.

A – codes for a protein


DNA

stage 1

nuclear membrane

DNA

stage 2

nuclear membrane

stage 3 B

not to scale
Fig. 6.1
© UCLES 2022 0610/42/F/M/22

Page 82 of 877
19

(i) State the name of the parts represented by the letters A and C in Fig. 6.1.

A ........................................................................................................................................

C ........................................................................................................................................
[2]

(ii) Describe the events that occur during stage 2 in Fig. 6.1.

...........................................................................................................................................

...........................................................................................................................................

...........................................................................................................................................

...........................................................................................................................................

..................................................................................................................................... [2]

(iii) State what determines the order in which the parts labelled B are assembled.

...........................................................................................................................................

...........................................................................................................................................

..................................................................................................................................... [1]

(c) The shape of a protein is very important for its function.

Explain the importance of shape for the function of an enzyme.

...................................................................................................................................................

...................................................................................................................................................

...................................................................................................................................................

...................................................................................................................................................

...................................................................................................................................................

...................................................................................................................................................

............................................................................................................................................. [3]

[Total: 11]

© UCLES 2022 0610/42/F/M/22

Page 83 of 877
Cambridge IGCSE™
* 0 7 5 4 4 5 0 6 9 8 *

BIOLOGY 0610/41
Paper 4 Theory (Extended) May/June 2021

1 hour 15 minutes

You must answer on the question paper.

No additional materials are needed.

INSTRUCTIONS
● Answer all questions.
● Use a black or dark blue pen. You may use an HB pencil for any diagrams or graphs.
● Write your name, centre number and candidate number in the boxes at the top of the page.
● Write your answer to each question in the space provided.
● Do not use an erasable pen or correction fluid.
● Do not write on any bar codes.
● You may use a calculator.
● You should show all your working and use appropriate units.

INFORMATION
● The total mark for this paper is 80.
● The number of marks for each question or part question is shown in brackets [ ].

This document has 20 pages. Any blank pages are indicated.

DC (LK/CB) 202934/5
© UCLES 2021 [Turn over

Page 85 of 877
2

1 (a) Baker’s yeast, Saccharomyces cerevisiae, is a single-celled organism that is classified in the
kingdom Fungi.

Fig. 1.1 is a drawing of a section through a yeast cell.

G B

F
C

1 μm

Fig. 1.1

(i) State one other kingdom that contains organisms that all have structure A.

..................................................................................................................................... [1]

(ii) Table 1.1 shows some cell functions.

Complete Table 1.1 by naming the cell structure responsible for each cell function and
give the letter that identifies each cell structure in Fig. 1.1.

Table 1.1

cell function cell structure letter from Fig. 1.1

storage of genes

aerobic respiration

amino acids are assembled to


make protein
[3]

© UCLES 2021 0610/41/M/J/21

Page 86 of 877
3

(b) A student made a drawing of one Escherichia coli bacterium. Fig. 1.2 shows the student’s
drawing.

Fig. 1.2

The actual length of the bacterial cell is 2 μm.

(i) Convert the actual length of the cell to millimetres.

................................................... mm [1]

(ii) State the other information that the student needs in order to calculate the magnification
of the drawing in Fig. 1.2.

..................................................................................................................................... [1]

(c) Describe the similarities and differences between the structure of the yeast cell and the
structure of the bacterial cell.

Use the information in Fig. 1.1 and Fig. 1.2 in your answer.

...................................................................................................................................................

...................................................................................................................................................

...................................................................................................................................................

...................................................................................................................................................

...................................................................................................................................................

...................................................................................................................................................

...................................................................................................................................................

...................................................................................................................................................

...................................................................................................................................................

...................................................................................................................................................

...................................................................................................................................................

...................................................................................................................................................

............................................................................................................................................. [6]

© UCLES 2021 0610/41/M/J/21 [Turn over


Page 87 of 877
4

(d) Some bacteria are involved in the nitrogen cycle.

Fig. 1.3 shows part of the nitrogen cycle.

nitrogen (N2)
1
3 3

ammonium ions nitrite ions nitrate ions


(NH4+) (NO2–) (NO3–)
amino acids
2

proteins

Fig. 1.3

State the processes that are represented by 1, 2 and 3 on Fig. 1.3.

1 ................................................................................................................................................

2 ................................................................................................................................................

3 ................................................................................................................................................
[3]

[Total: 15]

© UCLES 2021 0610/41/M/J/21

Page 88 of 877
5

2 Large quantities of plastic waste are polluting the oceans.

(a) A survey published in March 2018 showed the increase in plastic waste in the Pacific Ocean.
One area of the Pacific Ocean is known as the Great Pacific Garbage Patch (GPGP).

Data were collected from areas inside and outside the GPGP between 1965 and 2015 to
estimate the quantity of plastic waste.

The results are shown in Fig. 2.1.

1.4

1.2

1.0
estimated
quantity of
0.8
plastic waste
/ kg per km2
of ocean 0.6
surface
0.4

0.2

0.0
1965 – 1974 1975 – 1984 1985 – 1994 1995 – 2004 2005 – 2014 2015
years
key:
inside GPGP
outside GPGP

Fig. 2.1

Describe the results of the survey shown in Fig. 2.1.

...................................................................................................................................................

...................................................................................................................................................

...................................................................................................................................................

...................................................................................................................................................

...................................................................................................................................................

...................................................................................................................................................

...................................................................................................................................................

...................................................................................................................................................

............................................................................................................................................. [4]

© UCLES 2021 0610/41/M/J/21 [Turn over


Page 89 of 877
6

(b) The green turtle, Chelonia mydas, is a species of marine animal that is harmed by plastic
waste.

Fig. 2.2 shows a green turtle swimming past a plastic bag in the Pacific Ocean.

Fig. 2.2

(i) Turtles are classified as reptiles.

State one feature shown by all reptiles that is not found in amphibians.

..................................................................................................................................... [1]

(ii) Outline the dangers of non-biodegradable plastic waste to marine animals, such as
green turtles.

...........................................................................................................................................

...........................................................................................................................................

...........................................................................................................................................

...........................................................................................................................................

...........................................................................................................................................

...........................................................................................................................................

...........................................................................................................................................

...........................................................................................................................................

..................................................................................................................................... [4]

© UCLES 2021 0610/41/M/J/21

Page 90 of 877
7

(iii) Suggest ways to reduce the quantity of plastic waste.

...........................................................................................................................................

...........................................................................................................................................

...........................................................................................................................................

...........................................................................................................................................

..................................................................................................................................... [2]

[Total: 11]

© UCLES 2021 0610/41/M/J/21 [Turn over


Page 91 of 877
8

3 Bacteria are used in many biotechnological processes.

(a) Explain why bacteria are useful in biotechnology.

...................................................................................................................................................

...................................................................................................................................................

...................................................................................................................................................

...................................................................................................................................................

...................................................................................................................................................

...................................................................................................................................................

............................................................................................................................................. [3]

(b) Insulin is one of many human proteins that are made by genetically engineered bacteria.

Some people cannot produce insulin because their immune system has destroyed the cells
that make insulin.

(i) State the organ that contains the cells that have been destroyed.

..................................................................................................................................... [1]

(ii) State the name of the disease caused by the destruction of these cells.

..................................................................................................................................... [1]

(iii) State the function of insulin in the body.

..................................................................................................................................... [1]

(c) Genetically engineered bacteria that are used to make insulin were grown in a fermenter for
five days.

Samples were taken from the fermenter every six hours and the number of bacteria in 1.0 mm3
of the nutrient solution were counted.

Changes in the numbers of living bacteria in the samples taken from the fermenter are shown
in Fig. 3.1.

© UCLES 2021 0610/41/M/J/21

Page 92 of 877
9

1 S
Q R
.................................

.................................

.................................
P
.................................

2 ..........................................................................

Fig. 3.1

(i) Complete Fig. 3.1 by adding labels for the axes at 1 and 2 . [1]

(ii) State the names of the stages of population growth of the bacteria labelled P to S.

P ........................................................................................................................................

Q .......................................................................................................................................

R ........................................................................................................................................

S ........................................................................................................................................
[2]

(iii) Explain, with reference to Fig. 3.1, why the bacteria did not grow in the fermenter for
longer than five days.

...........................................................................................................................................

...........................................................................................................................................

...........................................................................................................................................

...........................................................................................................................................

...........................................................................................................................................

...........................................................................................................................................

..................................................................................................................................... [3]

© UCLES 2021 0610/41/M/J/21 [Turn over


Page 93 of 877
10

(d) Mineral salts are important in the human diet. One of the most important is iron.

Explain:
• the importance of iron in the human diet
• the effects of an iron deficiency.

...................................................................................................................................................

...................................................................................................................................................

...................................................................................................................................................

...................................................................................................................................................

...................................................................................................................................................

...................................................................................................................................................

............................................................................................................................................. [3]

(e) Fig. 3.2 shows a field of cassava, Manihot esculenta, which is a crop plant grown in parts of
Africa and Asia.

Fig. 3.2

The plants store starch in their roots, which form a large part of the diet for many people.
Cassava does not provide many vitamins or mineral ions.

Genetic engineers have modified cassava to increase its iron content. They have done this
by incorporating a gene for a membrane protein from the plant Arabidopsis thaliana.

(i) State the name of the enzyme that is used to cut out the gene from the DNA of
A. thaliana.

..................................................................................................................................... [1]

© UCLES 2021 0610/41/M/J/21

Page 94 of 877
11

(ii) Describe how the gene from A. thaliana and the DNA from cassava form recombinant
DNA.

...........................................................................................................................................

...........................................................................................................................................

...........................................................................................................................................

...........................................................................................................................................

..................................................................................................................................... [2]

(iii) Scientists who develop genetically engineered varieties of crop plants often breed them
for several generations before releasing them for farmers to use.

Suggest why the scientists do this.

...........................................................................................................................................

...........................................................................................................................................

...........................................................................................................................................

...........................................................................................................................................

..................................................................................................................................... [2]

[Total: 20]

© UCLES 2021 0610/41/M/J/21 [Turn over


Page 95 of 877
12

4 Two identical potted plants were used to investigate plant responses.

Plant A was placed on a clinostat that continually rotated. Plant B was not rotated.

Both plants were then placed on their sides and kept in the dark.

Fig. 4.1 shows the two plants at the start of the experiment and after seven days.

at the start

box to exclude light

after seven days

plant A plant B

Fig. 4.1

(a) State the name of the response shown by the shoot of plant B.

............................................................................................................................................. [1]

(b) Explain the reason for constantly rotating plant A.

...................................................................................................................................................

...................................................................................................................................................

...................................................................................................................................................

...................................................................................................................................................

............................................................................................................................................. [2]

(c) (i) State the name of the plant hormone that causes the response of the shoot of plant B.

..................................................................................................................................... [1]

© UCLES 2021 0610/41/M/J/21

Page 96 of 877
13

(ii) Explain how the plant hormone causes the response of plant B.

...........................................................................................................................................

...........................................................................................................................................

...........................................................................................................................................

...........................................................................................................................................

...........................................................................................................................................

...........................................................................................................................................

..................................................................................................................................... [3]

(d) Seeds germinate in the soil. The seedlings that grow from seeds show the same response as
shown by plant B in Fig. 4.1.

Explain the advantages of this response to the survival of seedlings and mature plants.

...................................................................................................................................................

...................................................................................................................................................

...................................................................................................................................................

...................................................................................................................................................

...................................................................................................................................................

...................................................................................................................................................

............................................................................................................................................. [3]

[Total: 10]

© UCLES 2021 0610/41/M/J/21 [Turn over


Page 97 of 877
14

5 (a) Fig. 5.1 shows the female reproductive system.

Fig. 5.1

Label Fig. 5.1 using the letters listed to show the position of the organs that are identified by
their functions.

The first one (P) has been completed for you.

P site of secretion of mucus

Q site of fertilisation

R site of implantation

S site of oestrogen secretion

T site where sperm are deposited during sexual intercourse


[4]

(b) Fig. 5.2 shows a section through an egg cell at the time of ovulation.

jelly coat cytoplasm with stores


of protein and fat

nucleus

Fig. 5.2

© UCLES 2021 0610/41/M/J/21

Page 98 of 877
15

(i) Explain why the egg cell contains stores of protein and fat.

...........................................................................................................................................

...........................................................................................................................................

...........................................................................................................................................

...........................................................................................................................................

...........................................................................................................................................

...........................................................................................................................................

..................................................................................................................................... [3]

(ii) Describe the function of the jelly coat.

...........................................................................................................................................

...........................................................................................................................................

...........................................................................................................................................

...........................................................................................................................................

..................................................................................................................................... [2]

(c) Fertilisation results in the formation of a zygote.

Describe how an embryo is formed from a zygote.

...................................................................................................................................................

...................................................................................................................................................

...................................................................................................................................................

...................................................................................................................................................

...................................................................................................................................................

...................................................................................................................................................

...................................................................................................................................................

...................................................................................................................................................

............................................................................................................................................. [4]

[Total: 13]

© UCLES 2021 0610/41/M/J/21 [Turn over


Page 99 of 877
16

6 (a) Antibodies are proteins that are produced by lymphocytes. Antitoxins are antibodies which
neutralise the toxins released by some bacteria.

The transmissible disease diphtheria is caused by a bacterium that releases a toxin that can
cause serious damage to the body.

A person is suspected of having caught diphtheria.

At a clinic, the person is given an injection of antitoxin antibodies that provide protection
against the diphtheria toxin. She is also given an injection of the vaccine for diphtheria.

A few weeks later she is given a second injection of the diphtheria vaccine.

Fig. 6.1 shows the changes in concentration of the antitoxin antibodies and the antibodies
produced in response to the vaccine.

injection of antitoxin antibodies

key:
concentration
antitoxin antibodies
of antibodies
in the blood second antibodies produced in
injection of response to vaccination
vaccine
first injection
of vaccine

time

Fig. 6.1

(i) Explain the advantage of giving the person an injection of antitoxin antibodies.

...........................................................................................................................................

...........................................................................................................................................

...........................................................................................................................................

...........................................................................................................................................

..................................................................................................................................... [2]

© UCLES 2021 0610/41/M/J/21

Page 100 of 877


17

(ii) Explain how the two injections of the vaccine result in better protection against diphtheria
than the injection of antitoxin antibodies.

...........................................................................................................................................

...........................................................................................................................................

...........................................................................................................................................

...........................................................................................................................................

...........................................................................................................................................

...........................................................................................................................................

..................................................................................................................................... [3]

(b) Explain how antibodies protect the body against pathogens.

...................................................................................................................................................

...................................................................................................................................................

...................................................................................................................................................

...................................................................................................................................................

...................................................................................................................................................

...................................................................................................................................................

...................................................................................................................................................

...................................................................................................................................................

............................................................................................................................................. [4]

(c) Antibodies can travel through the body in the lymphatic system.

State two functions of the lymphatic system other than defence against disease.

1 ................................................................................................................................................

2 ................................................................................................................................................
[2]

[Total: 11]

© UCLES 2021 0610/41/M/J/21

Page 101 of 877


Cambridge IGCSE™
* 1 2 6 0 5 8 1 3 5 8 *

BIOLOGY 0610/42
Paper 4 Theory (Extended) May/June 2021

1 hour 15 minutes

You must answer on the question paper.

No additional materials are needed.

INSTRUCTIONS
● Answer all questions.
● Use a black or dark blue pen. You may use an HB pencil for any diagrams or graphs.
● Write your name, centre number and candidate number in the boxes at the top of the page.
● Write your answer to each question in the space provided.
● Do not use an erasable pen or correction fluid.
● Do not write on any bar codes.
● You may use a calculator.
● You should show all your working and use appropriate units.

INFORMATION
● The total mark for this paper is 80.
● The number of marks for each question or part question is shown in brackets [ ].

This document has 20 pages. Any blank pages are indicated.

DC (KN/FC) 202969/5
© UCLES 2021 [Turn over

Page 105 of 877


2

1 (a) Complete the sentence about the nervous system.

The brain and spinal cord form the ............................................... nervous system and the

nerves coming into and out of the spinal cord are part of the ...............................................

nervous system. [1]

(b) Fig. 1.1 shows part of a human eye and three neurones that conduct electrical impulses
between the eye and the brain. These neurones are involved in the pupil reflex.

J
A B
H
C

not to scale

Fig. 1.1

(i) State the type of neurone identified as A in Fig. 1.1.

..................................................................................................................................... [1]

© UCLES 2021 0610/42/M/J/21

Page 106 of 877


3

(ii) Table 1.1 shows the names of some parts of the eye, their functions and the letters in
Fig. 1.1 that identify the parts of the eye.

Complete Table 1.1.

Table 1.1

part of the eye function letter in Fig. 1.1

suspensory ligament G

contracts in response to a bright light

cornea

contains a high density of cones for colour


vision

[4]

(c) (i) The eye can adjust how light is refracted through it in order to focus on a near object.

State one process that uses energy when focusing on a near object.

..................................................................................................................................... [1]

(ii) Mitochondria require oxygen to release energy. Oxygen is transported to cells in the eye
by red blood cells.

State the name of the molecule in red blood cells that carries oxygen.

..................................................................................................................................... [1]

(iii) Explain how oxygen in the capillaries reaches the cells in the eye.

...........................................................................................................................................

...........................................................................................................................................

...........................................................................................................................................

...........................................................................................................................................

..................................................................................................................................... [2]

© UCLES 2021 0610/42/M/J/21 [Turn over


Page 107 of 877
4

(d) Eyelashes and eyelids are mechanical barriers that help to prevent particles and pathogens
entering the eye.

(i) Give two other mechanical barriers that defend the body against pathogens.

1 ........................................................................................................................................

2 ........................................................................................................................................
[2]

(ii) State the name of the white blood cells that digest pathogens.

..................................................................................................................................... [1]

(iii) Conjunctivitis can be caused by pathogens and affects the tissues lining the eyelids and
covering the sclera. People with conjunctivitis that is caused by a pathogen can develop
active immunity.

Explain why the shape of specific parts of a pathogen is important in the development of
active immunity.

...........................................................................................................................................

...........................................................................................................................................

...........................................................................................................................................

...........................................................................................................................................

...........................................................................................................................................

...........................................................................................................................................

..................................................................................................................................... [3]

(e) Most insects and some crustaceans have compound eyes.

State the name of the group that contains insects and crustaceans.

............................................................................................................................................. [1]

[Total: 17]

© UCLES 2021 0610/42/M/J/21

Page 108 of 877


5

2 The classification of giant pandas, Ailuropoda melanoleuca, is debated by many scientists.

Fig. 2.1 shows a giant panda eating bamboo plants.

Fig. 2.1

Fig. 2.2 shows a red panda, Ailurus fulgens, and a polar bear, Ursus maritimus.

red panda eating bamboo plants polar bear eating fish


Fig. 2.2

(a) State one dietary component that is more likely to be found in bamboo plants than in fish.

............................................................................................................................................. [1]

(b) (i) State two features, visible in Fig. 2.1 and Fig. 2.2, that identify the three animals as all
belonging to the same vertebrate group.

1 ........................................................................................................................................

2 ........................................................................................................................................
[2]

© UCLES 2021 0610/42/M/J/21 [Turn over


Page 109 of 877
6

(ii) DNA can also be used to classify species.

Molecular biologists compared the DNA base sequences of eight species from the same
vertebrate group. They used the differences to draw a classification diagram.

Fig. 2.3 shows the classification diagram for these eight species. The shorter the
horizontal distance from two species to the branching point that they share, the more
similar their DNA sequences are and the more closely the two species are related.

The scale on Fig. 2.3 shows the time at which the molecular biologists estimate that
each branching point occurred.

red panda

ferret

giant panda

polar bear

wolf

tiger

human

mouse

100 80 60 40 20 0
million years ago

Fig. 2.3

© UCLES 2021 0610/42/M/J/21

Page 110 of 877


7

Morphology can also be used to classify species. Some scientists think that morphology
suggests that the giant panda is more closely related to the red panda than it is to the
polar bear.

Discuss the evidence for and against the giant panda being more closely related to
the red panda than it is to the polar bear. Use the information in Fig. 2.1, Fig. 2.2 and
Fig. 2.3 in your answer.

...........................................................................................................................................

...........................................................................................................................................

...........................................................................................................................................

...........................................................................................................................................

...........................................................................................................................................

...........................................................................................................................................

...........................................................................................................................................

...........................................................................................................................................

...........................................................................................................................................

...........................................................................................................................................

..................................................................................................................................... [5]

(iii) State one other type of evidence that is used to classify species.

..................................................................................................................................... [1]

[Total: 9]

© UCLES 2021 0610/42/M/J/21 [Turn over


Page 111 of 877
8

3 (a) Fig. 3.1 shows some of the events that occur in the menstrual cycle.

A follicle is fully developed

B gamete is released into the oviduct

C lining of the uterus is removed from the body

D lining of the uterus reaches a maximum thickness

E lining of the uterus gets thicker

Fig. 3.1

(i) Put the events shown in Fig. 3.1 into the correct sequence.
One has been done for you.

B
[1]

(ii) State the name of the hormone that stimulates event A to occur.

..................................................................................................................................... [1]

(iii) Event C means that menstruating females lose blood regularly.

Two females of the same age have different dietary needs because one has started
menstruating and the other has not started menstruating.

Suggest why the dietary needs of the two females are different.

...........................................................................................................................................

...........................................................................................................................................

...........................................................................................................................................

...........................................................................................................................................

..................................................................................................................................... [2]

© UCLES 2021 0610/42/M/J/21

Page 112 of 877


9

(b) During pregnancy menstruation does not occur.

Fig. 3.2 shows some of the organs of a pregnant woman, viewed from the side.

fetus

V
Q
U

T R

Fig. 3.2

Describe what happens between ovulation and the formation of a fetus.

Use the letters in Fig. 3.2 to support your answer.

...................................................................................................................................................

...................................................................................................................................................

...................................................................................................................................................

...................................................................................................................................................

...................................................................................................................................................

...................................................................................................................................................

...................................................................................................................................................

...................................................................................................................................................

...................................................................................................................................................

...................................................................................................................................................

...................................................................................................................................................

...................................................................................................................................................

............................................................................................................................................. [6]
[Total: 10]
© UCLES 2021 0610/42/M/J/21 [Turn over
Page 113 of 877
10

4 (a) Climate change is one reason why a plant species could become endangered.

(i) State other reasons why a plant species could become endangered.

...........................................................................................................................................

...........................................................................................................................................

...........................................................................................................................................

...........................................................................................................................................

...........................................................................................................................................

...........................................................................................................................................

..................................................................................................................................... [3]

(ii) Describe how human actions are causing climate change.

...........................................................................................................................................

...........................................................................................................................................

...........................................................................................................................................

...........................................................................................................................................

...........................................................................................................................................

...........................................................................................................................................

...........................................................................................................................................

...........................................................................................................................................

..................................................................................................................................... [4]

© UCLES 2021 0610/42/M/J/21

Page 114 of 877


11

(b) Seed banks are used to conserve endangered plant species.

Fig. 4.1 shows some of the steps involved in managing a seed bank.

seeds are collected from


many populations of
each species

seeds are checked using


X-rays

seeds are stored at low


temperatures in the
seed bank

a small sample of seeds


is germinated

Fig. 4.1

(i) Explain why seeds from one species are collected from many populations to store in a
seed bank.

...........................................................................................................................................

...........................................................................................................................................

...........................................................................................................................................

...........................................................................................................................................

...........................................................................................................................................

...........................................................................................................................................

..................................................................................................................................... [3]

© UCLES 2021 0610/42/M/J/21 [Turn over


Page 115 of 877
12

(ii) Seeds are X-rayed before they are stored to check that they contain an embryo.

State one possible consequence of using ionising radiation on seeds.

...........................................................................................................................................

..................................................................................................................................... [1]

(iii) Seeds stored at low temperatures have very low respiration rates.

Explain why.

...........................................................................................................................................

...........................................................................................................................................

...........................................................................................................................................

...........................................................................................................................................

..................................................................................................................................... [2]

(c) One purpose of seed banks is to reintroduce plant species into their natural environment.

A survey was done to find out why some reintroduction programmes are not successful.

Fig. 4.2 shows the results of the survey.

40

35

30

percentage of 25
species that
were not 20
successfully
reintroduced 15

10

0
reason wrong changing seeds did seedlings too few wrong
unknown habitat habitat not too young individuals season
germinate

reasons why reintroductions failed

Fig. 4.2

© UCLES 2021 0610/42/M/J/21

Page 116 of 877


13

(i) Some of the seeds in the reintroduction programmes did not germinate.

State the conditions that are necessary for seeds to germinate.

...........................................................................................................................................

...........................................................................................................................................

...........................................................................................................................................

..................................................................................................................................... [3]

(ii) Some reintroduction programmes failed because the seedlings were too young. Young
seedlings only have a few small roots.

Explain why it would be important to reintroduce plants with many large roots.

...........................................................................................................................................

...........................................................................................................................................

...........................................................................................................................................

...........................................................................................................................................

...........................................................................................................................................

...........................................................................................................................................

...........................................................................................................................................

...........................................................................................................................................

..................................................................................................................................... [4]

(iii) The low number of individuals also caused some reintroduction programmes to fail.

State the name of the phase in a population growth curve where the number of individuals
is very low.

..................................................................................................................................... [1]

[Total: 21]

© UCLES 2021 0610/42/M/J/21 [Turn over


Page 117 of 877
14

5 Fig. 5.1 shows some of the stages in the reproduction of the bacterium Escherichia coli.

loop of DNA

original bacterial cell

bacterial cell expands to a critical length


and a copy of DNA is synthesised

daughter cells are formed

daughter cells grow

Fig. 5.1

(a) Complete the sentences about the cells in Fig. 5.1.

The DNA is in the form of a double ............................................... . The DNA is copied so

that the number of loops of DNA after cell division is ............................................... in each

daughter cell. The daughter cells are genetically ............................................... to the original

cell.
[3]

(b) Students used a microscope and time-lapse photography to observe E. coli cells reproducing.

They used the series of photographs to identify which cells were dividing.

They measured the lengths of the dividing cells and put their data into two groups:
• cell lengths immediately before cell division
• cell lengths immediately after cell division.

Fig. 5.2 shows their results.

© UCLES 2021 0610/42/M/J/21

Page 118 of 877


15

key: 100
immediately after division
90
immediately before division
80

70

60
number
of 50
cells
40

30

20

10

0
0 1 2 3 4 5 6 7 8 9 10
cell length / μm

Fig. 5.2

(i) Use the information in Fig. 5.2 to state the most frequent cell length of the E. coli cells
immediately after cell division.

Give your answer in millimetres.

......................................................... mm
[2]

(ii) Some students concluded that the cells must be at least 6 µm in length before cell
division can occur.

Describe the evidence against the students’ conclusion. Use the information in Fig. 5.2
to support your answer.

...........................................................................................................................................

...........................................................................................................................................

...........................................................................................................................................

...........................................................................................................................................

..................................................................................................................................... [2]

© UCLES 2021 0610/42/M/J/21 [Turn over


Page 119 of 877
16

(c) Bacteria are useful in genetic engineering because they contain plasmids.

(i) Describe how a plasmid is cut so that a new gene can be inserted into the plasmid.

...........................................................................................................................................

...........................................................................................................................................

..................................................................................................................................... [2]

(ii) List two reasons, other than the presence of plasmids, that make bacteria and
single-celled fungi useful to biotechnology industries.

1 ........................................................................................................................................

2 ........................................................................................................................................
[2]

[Total: 11]

© UCLES 2021 0610/42/M/J/21

Page 120 of 877


17

6 (a) Some students set up the apparatus shown in Fig. 6.1 to compare transpiration in two sets of
leaves.

Set A was kept in a transparent bag and set B was left in the open air.

All other conditions were kept constant.

The mass of the leaves in each set was measured at the start of the investigation and after
five hours.

string line paper clip

clamp stand

transparent bag

set A

set B

Fig. 6.1

(i) Predict the results for this investigation.

Explain the reason for your prediction.

prediction ...........................................................................................................................

...........................................................................................................................................

explanation ........................................................................................................................

...........................................................................................................................................

...........................................................................................................................................
[3]
© UCLES 2021 0610/42/M/J/21 [Turn over
Page 121 of 877
18

(ii) Explain how transpiration occurred in the leaves shown in Fig. 6.1.

...........................................................................................................................................

...........................................................................................................................................

...........................................................................................................................................

...........................................................................................................................................

...........................................................................................................................................

...........................................................................................................................................

..................................................................................................................................... [3]

(iii) The students needed two additional pieces of apparatus to take measurements so that
they could calculate the rate of transpiration from their results.

State the two additional pieces of apparatus the students needed to take the
measurements.

1 ........................................................................................................................................

2 ........................................................................................................................................
[2]

(b) Fig. 6.2 shows the positions of the different tissues in part of a dicotyledonous leaf.

Fig. 6.2

© UCLES 2021 0610/42/M/J/21

Page 122 of 877


19

Identify the tissues described in Table 6.1 by:


• drawing label lines with the corresponding letter on Fig. 6.2 and
• stating the name of each tissue in Table 6.1.

The label, line and name of the tissue for letter P has been completed for you on Fig. 6.2 and
in Table 6.1.

Table 6.1

letter description name of the tissue

a protective transparent layer that allows light


P upper epidermis
to reach the inner tissues

Q conducts water from the stem

R contains many interconnected air spaces

S transports sucrose and amino acids

traps the most light energy to synthesise


T
carbohydrates

[4]

[Total: 12]

© UCLES 2021 0610/42/M/J/21

Page 123 of 877


20

BLANK PAGE

Permission to reproduce items where third-party owned material protected by copyright is included has been sought and cleared where possible. Every
reasonable effort has been made by the publisher (UCLES) to trace copyright holders, but if any items requiring clearance have unwittingly been included, the
publisher will be pleased to make amends at the earliest possible opportunity.

To avoid the issue of disclosure of answer-related information to candidates, all copyright acknowledgements are reproduced online in the Cambridge
Assessment International Education Copyright Acknowledgements Booklet. This is produced for each series of examinations and is freely available to download
at www.cambridgeinternational.org after the live examination series.

Cambridge Assessment International Education is part of the Cambridge Assessment Group. Cambridge Assessment is the brand name of the University of
Cambridge Local Examinations Syndicate (UCLES), which itself is a department of the University of Cambridge.

© UCLES 2021 0610/42/M/J/21

Page 124 of 877


Cambridge IGCSE™
* 3 6 0 5 4 1 7 5 1 9 *

BIOLOGY 0610/43
Paper 4 Theory (Extended) May/June 2021

1 hour 15 minutes

You must answer on the question paper.

No additional materials are needed.

INSTRUCTIONS
● Answer all questions.
● Use a black or dark blue pen. You may use an HB pencil for any diagrams or graphs.
● Write your name, centre number and candidate number in the boxes at the top of the page.
● Write your answer to each question in the space provided.
● Do not use an erasable pen or correction fluid.
● Do not write on any bar codes.
● You may use a calculator.
● You should show all your working and use appropriate units.

INFORMATION
● The total mark for this paper is 80.
● The number of marks for each question or part question is shown in brackets [ ].

This document has 20 pages. Any blank pages are indicated.

DC (CE/SG) 203218/4
© UCLES 2021 [Turn over

Page 125 of 877


2

1 (a) (i) The main component of milk is a liquid that is an important solvent in the body.

State the name of this solvent.

..................................................................................................................................... [1]

(ii) Table 1.1 shows some of the nutrients found in human milk, the elements that make up
the nutrients, the enzymes that digest them and the products of digestion.

Complete Table 1.1.

Table 1.1

nutrient elements enzyme products of digestion

protein

fat

lactose (milk galactose and glucose


C, H, O
sugar) (simple sugars)

[4]

(b) Milk also contains vitamins and mineral salts.

(i) Explain why vitamin D is important for the nutrition of children.

...........................................................................................................................................

...........................................................................................................................................

...........................................................................................................................................

...........................................................................................................................................

...........................................................................................................................................

...........................................................................................................................................

..................................................................................................................................... [3]

© UCLES 2021 0610/43/M/J/21

Page 126 of 877


3

(ii) Explain why iron is important in the diet.

...........................................................................................................................................

...........................................................................................................................................

...........................................................................................................................................

...........................................................................................................................................

...........................................................................................................................................

..................................................................................................................................... [2]

(c) Rubella and kwashiorkor are two diseases that affect children.

Rubella is a transmissible disease and kwashiorkor is a non-transmissible disease.

(i) Define the term transmissible disease.

...........................................................................................................................................

...........................................................................................................................................

...........................................................................................................................................

..................................................................................................................................... [2]

(ii) Explain why women should be vaccinated against rubella before they reach reproductive
age.

...........................................................................................................................................

...........................................................................................................................................

...........................................................................................................................................

...........................................................................................................................................

...........................................................................................................................................

...........................................................................................................................................

...........................................................................................................................................

..................................................................................................................................... [4]

© UCLES 2021 0610/43/M/J/21 [Turn over


Page 127 of 877
4

(iii) Outline the causes of kwashiorkor.

...........................................................................................................................................

...........................................................................................................................................

...........................................................................................................................................

...........................................................................................................................................

..................................................................................................................................... [2]

[Total: 18]

2 Insulin is a protein made by cells in the pancreas.

Fig. 2.1 shows how insulin is made in a pancreatic cell.

DNA
amino acids

nucleus

amino acids assembled


to form insulin
A

B not to scale

Fig. 2.1

(a) (i) State the name of molecule A in Fig. 2.1.

..................................................................................................................................... [1]

(ii) State the name of structure B in Fig. 2.1.

..................................................................................................................................... [1]

© UCLES 2021 0610/43/M/J/21

Page 128 of 877


5

(iii) Explain the role of molecule A in protein synthesis, as shown in Fig. 2.1.

...........................................................................................................................................

...........................................................................................................................................

...........................................................................................................................................

...........................................................................................................................................

...........................................................................................................................................

...........................................................................................................................................

..................................................................................................................................... [3]

(b) Insulin is secreted from cells in the pancreas when the concentration of glucose in the blood
increases.

Outline the role of insulin.

...................................................................................................................................................

...................................................................................................................................................

...................................................................................................................................................

...................................................................................................................................................

...................................................................................................................................................

...................................................................................................................................................

...................................................................................................................................................

............................................................................................................................................. [4]

(c) State the names of two substances, other than insulin, that are secreted by the pancreas.

1 ................................................................................................................................................

2 ................................................................................................................................................
[2]

[Total: 11]

© UCLES 2021 0610/43/M/J/21 [Turn over


Page 129 of 877
6

3 Scientists measured the rate of photosynthesis in the leaves of a sunflower plant, Helianthus
annuus.

The scientists used the apparatus shown in Fig. 3.1 to measure the rate of photosynthesis.

direction of air flow

air flow regulator chamber 1 chamber 2 chamber 3

temperature 20 °C
CO2 360 ppm

CO2 400 ppm timer


00:00

Fig. 3.1

(a) The apparatus in Fig. 3.1 maintains a constant temperature and a constant humidity.

(i) Explain why temperature has an effect on the rate of photosynthesis.

...........................................................................................................................................

...........................................................................................................................................

...........................................................................................................................................

...........................................................................................................................................

...........................................................................................................................................

...........................................................................................................................................

...........................................................................................................................................

...........................................................................................................................................

..................................................................................................................................... [4]

© UCLES 2021 0610/43/M/J/21

Page 130 of 877


7

(ii) Explain why the rate of photosynthesis will decrease if the humidity in the apparatus
becomes very low.

...........................................................................................................................................

...........................................................................................................................................

...........................................................................................................................................

...........................................................................................................................................

...........................................................................................................................................

...........................................................................................................................................

..................................................................................................................................... [3]

(iii) The apparatus was left for 15 minutes.

Explain how the scientists would use the readings for the concentration of carbon dioxide
in chambers 1 and 3 to calculate the rate of photosynthesis.

...........................................................................................................................................

...........................................................................................................................................

...........................................................................................................................................

...........................................................................................................................................

..................................................................................................................................... [2]

© UCLES 2021 0610/43/M/J/21 [Turn over


Page 131 of 877
8

(b) The scientists carried out another investigation using the same apparatus at different
temperatures.

They measured the rate of uptake of carbon dioxide in the light and then they measured the
rate of release of carbon dioxide in the dark.

The results are shown in Fig. 3.2.

160

140
CO2 uptake
120 in the light

rate of carbon 100


dioxide uptake
and release 80
/ μg per hour
per cm2 of leaf
60

CO2 release
40
in the dark

20

0
15 20 25 30 35 40
temperature / °C

Fig. 3.2

(i) Use the information in Fig. 3.2 to complete Table 3.1.

Table 3.1

temperature rate of uptake of carbon dioxide in rate of release of carbon dioxide in


/ °C the light / μg per hour per cm2 of leaf the dark / μg per hour per cm2 of leaf

20

35 90 45

[2]

© UCLES 2021 0610/43/M/J/21

Page 132 of 877


9

(ii) The scientists determined that photosynthesis in the leaves at 35 °C used carbon dioxide
at a rate of 135 μg per hour per cm2 of leaf. Fig. 3.2 shows that the rate of carbon dioxide
uptake at 35 °C was 90 μg per hour per cm2 of leaf.

Explain why the rate at which carbon dioxide is used in photosynthesis is greater than
the rate of carbon dioxide uptake.

...........................................................................................................................................

...........................................................................................................................................

...........................................................................................................................................

...........................................................................................................................................

..................................................................................................................................... [2]

(iii) Explain why the results in Table 3.1 are expressed as ‘per cm2 of leaf’ rather than ‘per
leaf’.

...........................................................................................................................................

...........................................................................................................................................

..................................................................................................................................... [1]

© UCLES 2021 0610/43/M/J/21 [Turn over


Page 133 of 877
11

(c) Some crops are grown in controlled environments. Glasshouses are an example of a
controlled environment. Fig. 3.3 shows a glasshouse where lettuces are growing.

Fig. 3.3

Discuss the advantages of growing crops in glasshouses.

...................................................................................................................................................

...................................................................................................................................................

...................................................................................................................................................

...................................................................................................................................................

...................................................................................................................................................

...................................................................................................................................................

...................................................................................................................................................

...................................................................................................................................................

...................................................................................................................................................

...................................................................................................................................................

...................................................................................................................................................

...................................................................................................................................................

...................................................................................................................................................

...................................................................................................................................................

...................................................................................................................................................

............................................................................................................................................. [6]

[Total: 20]

© UCLES 2021 0610/43/M/J/21 [Turn over


Page 135 of 877
12

4 There are three species of rhinoceros in Asia. Fig. 4.1 shows an Indian rhinoceros, Rhinoceros
unicornis, in a national park being observed by tourists who are riding on an elephant.

Fig. 4.1

The International Union for Conservation of Nature (IUCN) collects data on animal species and
places them into categories based on estimated numbers.

Table 4.1 summarises the conservation status of the three species of rhinoceros, as available in
2020.

Table 4.1

approximate approximate
species location number in number in IUCN category
the wild captivity
Indian rhinoceros national parks
Rhinoceros in south Nepal 3588 160 vulnerable
unicornis and north India
Javan rhinoceros one national
Rhinoceros park in Java, 72 0 critically endangered
sondaicus Indonesia
Sumatran national parks
rhinoceros in Sumatra
<80 7 critically endangered
Dicerorhinus and Borneo in
sumatrensis Indonesia

© UCLES 2021 0610/43/M/J/21

Page 136 of 877


13

(a) Outline the reasons why large mammals, such as some species of rhinoceros, are
endangered.

...................................................................................................................................................

...................................................................................................................................................

...................................................................................................................................................

...................................................................................................................................................

...................................................................................................................................................

...................................................................................................................................................

...................................................................................................................................................

...................................................................................................................................................

...................................................................................................................................................

...................................................................................................................................................

............................................................................................................................................. [5]

(b) In 2013, Cincinnati Zoo in the USA announced that they were hoping to breed together
rhinoceros that were brother and sister as part of their captive breeding programme.

(i) Discuss the problems that using such closely related animals might have for the
conservation of rhinoceros.

...........................................................................................................................................

...........................................................................................................................................

...........................................................................................................................................

...........................................................................................................................................

...........................................................................................................................................

..................................................................................................................................... [2]

© UCLES 2021 0610/43/M/J/21 [Turn over


Page 137 of 877
14

(ii) Captive breeding programmes often use artificial insemination.

Outline the process of artificial insemination.

...........................................................................................................................................

...........................................................................................................................................

...........................................................................................................................................

...........................................................................................................................................

...........................................................................................................................................

...........................................................................................................................................

...........................................................................................................................................

..................................................................................................................................... [3]

[Total: 10]

© UCLES 2021 0610/43/M/J/21

Page 138 of 877


15

5 (a) Table 5.1 shows four functions of the human gas exchange system.

Complete Table 5.1 by identifying the part of the human gas exchange system that carries out
each function.

Table 5.1

part of the human gas exchange


function
system

contracts to decrease air pressure in the


lungs

allows air to flow between the larynx and


the lungs

contracts to lower the ribs in forced


expiration

site of gas exchange

[4]

© UCLES 2021 0610/43/M/J/21 [Turn over


Page 139 of 877
16

(b) Scientists investigated the effect of increasing carbon dioxide concentration in the air on
the rate and depth of breathing. The people who took part in the investigation were healthy
volunteers.

The people breathed atmospheric air containing 0.04% carbon dioxide for five minutes.
They then breathed air containing 2% carbon dioxide for five minutes and then returned to
breathing atmospheric air for a further five minutes.

The results are shown in Fig. 5.1.

0.04% CO2 2% CO2 0.04% CO2

70

60

50

40

percentage 30
change in rate
and depth of
breathing 20

10

0
0 2 4 6 8 10 12 14 16

–10

–20
time / minutes
key:
rate of breathing
depth of breathing

Fig. 5.1

© UCLES 2021 0610/43/M/J/21

Page 140 of 877


17

Describe the effects of changing the carbon dioxide concentration of the air on the rate and
depth of breathing, as shown in Fig. 5.1.

...................................................................................................................................................

...................................................................................................................................................

...................................................................................................................................................

...................................................................................................................................................

...................................................................................................................................................

...................................................................................................................................................

...................................................................................................................................................

...................................................................................................................................................

............................................................................................................................................. [4]

(c) Complete the sentences by writing the appropriate word or phrase in the spaces provided.

During physical activity the carbon dioxide concentration in the blood increases. The

reason for this is an increase in the rate of ............................................... that occurs in the

............................................... in muscle cells.

The increase in the concentration of carbon dioxide in the blood is detected by the

............................................... , which increases the rate and depth of breathing so that carbon

dioxide can be ............................................... from the body. This is important because carbon

dioxide is a ............................................... substance and can cause harm if it accumulates.


[5]

[Total: 13]

© UCLES 2021 0610/43/M/J/21 [Turn over


Page 141 of 877
18

6 There are four blood groups in the ABO system in humans: A, B, AB and O.

The gene that determines blood group has three alleles: IA, IB and Io.

(a) Parents with the genotypes IAIo and IBIo are planning to have more children.

Complete the genetic diagram to determine the probability that the next child will have blood
group O.

parental blood groups A B

parental genotypes IAIo IBIo

Punnett square

phenotypes of
the children ...............................................................................................................................

probability that the child will have blood group O .....................................................................


[4]

(b) Explain why the ABO blood group system is an example of co-dominance.

...................................................................................................................................................

...................................................................................................................................................

...................................................................................................................................................

...................................................................................................................................................

............................................................................................................................................. [2]

© UCLES 2021 0610/43/M/J/21

Page 142 of 877


19

(c) Fig. 6.1 shows the percentages of the global population with the four different blood groups in
the ABO system.

45

40

35

30

percentage 25
of the global
population
20

15

10

0
A B AB O
blood group

Fig. 6.1

With reference to Fig. 6.1, explain why the ABO blood group system is an example of
discontinuous variation.

...................................................................................................................................................

...................................................................................................................................................

...................................................................................................................................................

...................................................................................................................................................

...................................................................................................................................................

............................................................................................................................................. [2]

[Total: 8]

© UCLES 2021 0610/43/M/J/21

Page 143 of 877


Cambridge IGCSE™
* 8 1 4 7 8 1 8 2 3 7 *

BIOLOGY 0610/41
Paper 4 Theory (Extended) October/November 2021

1 hour 15 minutes

You must answer on the question paper.

No additional materials are needed.

INSTRUCTIONS
● Answer all questions.
● Use a black or dark blue pen. You may use an HB pencil for any diagrams or graphs.
● Write your name, centre number and candidate number in the boxes at the top of the page.
● Write your answer to each question in the space provided.
● Do not use an erasable pen or correction fluid.
● Do not write on any bar codes.
● You may use a calculator.
● You should show all your working and use appropriate units.

INFORMATION
● The total mark for this paper is 80.
● The number of marks for each question or part question is shown in brackets [ ].

This document has 20 pages. Any blank pages are indicated.

DC (ST/JG) 202999/5
© UCLES 2021 [Turn over

Page 145 of 877


2

1 (a) Fish, mammals and birds are all groups of vertebrates.

(i) State the names of the two other main groups of vertebrates.

................................................................... and .......................................................... [1]

(ii) Complete Table 1.1 to compare the features of fish, mammals and birds.

Table 1.1

feature fish mammals birds

organ involved in gas


gills lungs
exchange

circulatory system double

body covering fur

presence of external ears


no
(pinnae)

[3]

© UCLES 2021 0610/41/O/N/21

Page 146 of 877


3

(b) Fig. 1.1 is a micrograph of part of some fish gills.

magnification ×110

Fig. 1.1

Fish gills are adapted for gas exchange by diffusion.

(i) Define the term diffusion.

...........................................................................................................................................

...........................................................................................................................................

...........................................................................................................................................

...........................................................................................................................................

..................................................................................................................................... [2]

(ii) Suggest one adaptation, visible in Fig. 1.1, that shows that fish gills are efficient structures
for gas exchange by diffusion.

...........................................................................................................................................

...........................................................................................................................................

..................................................................................................................................... [1]

© UCLES 2021 0610/41/O/N/21 [Turn over


Page 147 of 877
4

(c) Some pollutants decrease the concentration of dissolved oxygen in rivers. This can result in
the death of fish.

(i) State one type of pollutant that can result in a decrease in the concentration of dissolved
oxygen in rivers.

..................................................................................................................................... [1]

(ii) Researchers investigated the effect of the concentration of dissolved oxygen in water on
gas diffusion distance in tissues. The thickness of fish gills was used to determine the
gas diffusion distance.

The researchers changed the concentration of dissolved oxygen by bubbling different


concentrations of oxygen into water. The temperature of the water was kept constant at
15 °C.

Their results are shown in Fig. 1.2.

3.5

3.0

2.5

gas diffusion 2.0


distance / μm
1.5

1.0

0.5

0.0
6 7 8 9 10 11 12
dissolved oxygen concentration / mg per dm3

Fig. 1.2

© UCLES 2021 0610/41/O/N/21

Page 148 of 877


5

Fig. 1.3 shows the relationship between the concentration of dissolved oxygen and water
temperature.

12

11

10
dissolved
oxygen
9
concentration
/ mg per dm3
8

6
0 5 10 15 20 25 30 35
temperature / °C

Fig. 1.3

Complete steps 1 to 3 to determine the effect on gas diffusion distance of increasing the
temperature of the water from 15 °C to 25 °C.

Step 1
Find the concentration of dissolved oxygen from Fig. 1.3 at:

15 °C: ................................. mg per dm3 25 °C: ................................. mg per dm3

Step 2
Use the values from step 1 to find the gas diffusion distances from Fig. 1.2:

................................. µm ................................. µm

Step 3
Calculate the difference in the values from step 2.

................................. µm
[3]

[Total: 11]

© UCLES 2021 0610/41/O/N/21 [Turn over


Page 149 of 877
6

2 (a) Two tomato plants that produce red fruit were bred together.

This cross produced 71 offspring plants with red fruit and 26 offspring plants with yellow fruit.

Complete the genetic diagram to show this cross.

Select a suitable letter to represent the alleles and decide which allele will need a capital
letter and which allele will need a lower case letter.

letter representing the allele for red fruit ...................................................................................

letter representing the allele for yellow fruit ..............................................................................

parental phenotypes red fruit × red fruit

parental genotypes .................................. × ..................................

gametes , × ,
............. ............. ............. .............

offspring genotypes ........................................................................................................

expected phenotype ratio ...................... red fruit : ...................... yellow fruit

actual phenotype ratio 71 red fruit : 26 yellow fruit


[6]

(b) Researchers carried out some experiments on tomato plants that were homozygous for fruit
colour.

State how the researchers could be sure that the fruit came from homozygous plants.

...................................................................................................................................................

...................................................................................................................................................

............................................................................................................................................. [1]

© UCLES 2021 0610/41/O/N/21

Page 150 of 877


7

(c) The researchers analysed two pigments, chlorophyll and lycopene, in homozygous red
tomato fruit and homozygous yellow tomato fruit. Chlorophyll is found in unripe tomato fruit.

(i) Describe the function of chlorophyll.

...........................................................................................................................................

...........................................................................................................................................

...........................................................................................................................................

...........................................................................................................................................

..................................................................................................................................... [2]

(ii) State the name of one mineral required for the synthesis of chlorophyll.

..................................................................................................................................... [1]

(iii) The researchers analysed the concentration of the pigments in tomato fruits:
• before they were ready to eat (unripe)
• when they were ready to eat (ripe).

The results of the analysis are shown in Table 2.1.

Table 2.1

chlorophyll concentration lycopene concentration


/ mg per g of tomato fruit / mg per g of tomato fruit
unripe red fruit 10.0 0.0
ripe red fruit 1.2 105.7
unripe yellow fruit 6.2 0.0
ripe yellow fruit 0.4 0.7

Use the information in Table 2.1 to compare the changes in pigments in red fruit and
yellow fruit as they ripen.

...........................................................................................................................................

...........................................................................................................................................

...........................................................................................................................................

...........................................................................................................................................

...........................................................................................................................................

...........................................................................................................................................

..................................................................................................................................... [3]

© UCLES 2021 0610/41/O/N/21 [Turn over


Page 151 of 877
8

(d) A gene is responsible for the production of lycopene in fruits. Geneticists have recently
produced genetically modified pink pineapples using the gene associated with the production
of lycopene.

(i) Genes are found at specific locations on an important biological molecule.

State the name of this biological molecule.

..................................................................................................................................... [1]

(ii) Describe the disadvantages of genetically modifying crops.

...........................................................................................................................................

...........................................................................................................................................

...........................................................................................................................................

...........................................................................................................................................

..................................................................................................................................... [2]

[Total: 16]

© UCLES 2021 0610/41/O/N/21

Page 152 of 877


9

3 Some washing powders contain enzymes.

(a) Fig. 3.1 shows a box of biological washing powder containing enzymes.

Wonder washing
powder
Biological washing powder
removes stains made from oil, egg, juice,
grass and many more.
Directions:
• add 1–2 scoops to main wash
• works best at 30 °C but do not use above 50 °C
Ingredients:
• soap
• enzymes

Fig. 3.1

(i) Eggs contain protein.

Describe how the biological washing powder removes egg stains.

...........................................................................................................................................

...........................................................................................................................................

...........................................................................................................................................

...........................................................................................................................................

...........................................................................................................................................

...........................................................................................................................................

..................................................................................................................................... [3]

© UCLES 2021 0610/41/O/N/21 [Turn over


Page 153 of 877
10

(ii) Explain why the manufacturer states that the washing powder works best at 30 °C and
should not be used above 50 °C.

...........................................................................................................................................

...........................................................................................................................................

...........................................................................................................................................

...........................................................................................................................................

...........................................................................................................................................

...........................................................................................................................................

...........................................................................................................................................

...........................................................................................................................................

..................................................................................................................................... [4]

(b) Scientists used enzymes and bile in the early development of biological washing powders.

Outline the roles of bile in the body.

...................................................................................................................................................

...................................................................................................................................................

...................................................................................................................................................

...................................................................................................................................................

...................................................................................................................................................

...................................................................................................................................................

...................................................................................................................................................

...................................................................................................................................................

............................................................................................................................................. [4]

[Total: 11]

© UCLES 2021 0610/41/O/N/21

Page 154 of 877


11

4 Cholera is a transmissible disease.

(a) State the name of the type of pathogen that causes cholera.

............................................................................................................................................. [1]

(b) A study was designed to test the effectiveness of a cholera vaccine in an area where
outbreaks of cholera occur frequently.

The doctors gave some people in this area the new vaccine.

(i) Suggest what the cholera vaccine must contain to be effective.

...........................................................................................................................................

...........................................................................................................................................

..................................................................................................................................... [1]

(ii) Explain why the people were not protected from cholera immediately after receiving the
vaccine.

...........................................................................................................................................

...........................................................................................................................................

...........................................................................................................................................

...........................................................................................................................................

...........................................................................................................................................

...........................................................................................................................................

..................................................................................................................................... [3]

(iii) The doctors ensured that the people who received the new vaccine had not had cholera
before.

Suggest why.

...........................................................................................................................................

...........................................................................................................................................

..................................................................................................................................... [1]

© UCLES 2021 0610/41/O/N/21 [Turn over


Page 155 of 877
12

(iv) There was an outbreak of cholera in the area two months after the new vaccine was
given.

The people who had symptoms of cholera were monitored in hospital for two days.

Some of the infected people had received the new vaccine and others had not received
the vaccine.

The results are shown in Fig. 4.1.

100

90

80 Key:
vaccinated
70
not vaccinated
60
percentage
of people in
hospital with 50
the symptom
40

30

20

10

0
pathogen present diarrhoea high body
in faeces temperature
factors monitored in hospital

Fig. 4.1

Using the information in Fig. 4.1, discuss how effective the vaccine was at preventing the
symptoms of cholera.

...........................................................................................................................................

...........................................................................................................................................

...........................................................................................................................................

...........................................................................................................................................

..................................................................................................................................... [2]

© UCLES 2021 0610/41/O/N/21

Page 156 of 877


13

(c) (i) Explain how cholera causes diarrhoea.

...........................................................................................................................................

...........................................................................................................................................

...........................................................................................................................................

...........................................................................................................................................

...........................................................................................................................................

...........................................................................................................................................

...........................................................................................................................................

...........................................................................................................................................

..................................................................................................................................... [4]

(ii) Describe how diarrhoea is treated.

...........................................................................................................................................

...........................................................................................................................................

...........................................................................................................................................

...........................................................................................................................................

..................................................................................................................................... [2]

[Total: 14]

© UCLES 2021 0610/41/O/N/21 [Turn over


Page 157 of 877
14

5 The circulatory system is comprised of the heart, blood vessels and the blood.

(a) Explain how the structures of the heart ensure that blood flows in one direction.

Include the names of these structures in your answer.

...................................................................................................................................................

...................................................................................................................................................

...................................................................................................................................................

...................................................................................................................................................

...................................................................................................................................................

...................................................................................................................................................

...................................................................................................................................................

...................................................................................................................................................

............................................................................................................................................. [4]

(b) Fig. 5.1 shows a diagram of some blood vessels associated with the skin.

J
Key:
indicates the
direction of
blood flow

P K

N M

not to scale
Fig. 5.1

Table 5.1 shows the functions of some blood vessels.

Complete Table 5.1 by:


• naming the type of blood vessel
• stating the letter of the type of blood vessel from Fig. 5.1.
© UCLES 2021 0610/41/O/N/21

Page 158 of 877


15

Table 5.1

function type of blood vessel letter on Fig. 5.1

regulates blood flow by constricting


and dilating

collects blood from a network of the


narrowest blood vessels

withstands the highest blood


pressure

allows the transfer of substances to


and from tissue fluid

transports blood towards the heart

redirects blood flow deeper under the


surface of the skin

[6]

(c) State the name of the blood vessels that deliver blood to the:

kidneys .....................................................................................................................................

heart muscle .............................................................................................................................


[2]

(d) Describe how blockages in the vessels that deliver blood to the heart muscle can be treated.

...................................................................................................................................................

...................................................................................................................................................

...................................................................................................................................................

...................................................................................................................................................

...................................................................................................................................................

...................................................................................................................................................

...................................................................................................................................................

...................................................................................................................................................

............................................................................................................................................. [4]

[Total: 16]
© UCLES 2021 0610/41/O/N/21 [Turn over
Page 159 of 877
16

6 Many crop plants are grown as monocultures.

Fig. 6.1 shows the destruction of a monoculture of maize as a result of drought.

Fig. 6.1

(a) Describe the negative impacts to an ecosystem of large-scale monocultures, such as the one
shown in Fig. 6.1.

...................................................................................................................................................

...................................................................................................................................................

...................................................................................................................................................

...................................................................................................................................................

...................................................................................................................................................

...................................................................................................................................................

............................................................................................................................................. [3]

© UCLES 2021 0610/41/O/N/21

Page 160 of 877


17

(b) The fruits of maize are produced on structures called cobs. Each cob has many fruits.

Fig. 6.2 shows how the maize cobs have changed over thousands of years as a result of
selective breeding.

fruit

length of
maize cob maize cob

time

Fig. 6.2

State two desirable features, visible in Fig. 6.2, that have been selected from the more
ancient varieties of maize.

1 ................................................................................................................................................

2 ................................................................................................................................................
[2]

© UCLES 2021 0610/41/O/N/21 [Turn over


Page 161 of 877
18

(c) The process of selective breeding involves a series of steps, as shown in Fig. 6.3.

Step 1 select two plants with desirable features

Step 2 transfer pollen from the flower of one plant to a


flower on the second plant

Step 3 use bags to cover the flowers that are pollinated

Step 4 collect the seeds, germinate them and wait for


the plants to grow

Step 5 select the plants from step 4 that have the


desirable features

Step 6 continue step 2 to step 5 for many generations

Fig. 6.3

(i) Explain why the flowers are covered with bags in step 3.

...........................................................................................................................................

...........................................................................................................................................

..................................................................................................................................... [1]

(ii) Explain why the seeds in step 4 need oxygen to germinate.

...........................................................................................................................................

...........................................................................................................................................

...........................................................................................................................................

...........................................................................................................................................

..................................................................................................................................... [2]
© UCLES 2021 0610/41/O/N/21

Page 162 of 877


19

(iii) Explain why some of the plants in step 5 may show features not visible in the parent
plants.

...........................................................................................................................................

...........................................................................................................................................

..................................................................................................................................... [1]

(iv) Suggest why selective breeding should continue for many generations (step 6).

...........................................................................................................................................

...........................................................................................................................................

...........................................................................................................................................

...........................................................................................................................................

..................................................................................................................................... [2]

(v) State how new features, which did not exist in ancient varieties of a crop plant, could
appear.

...........................................................................................................................................

...........................................................................................................................................

..................................................................................................................................... [1]

[Total: 12]

© UCLES 2021 0610/41/O/N/21

Page 163 of 877


Cambridge IGCSE™
* 5 6 6 0 8 4 5 5 8 2 *

BIOLOGY 0610/42
Paper 4 Theory (Extended) October/November 2021

1 hour 15 minutes

You must answer on the question paper.

No additional materials are needed.

INSTRUCTIONS
● Answer all questions.
● Use a black or dark blue pen. You may use an HB pencil for any diagrams or graphs.
● Write your name, centre number and candidate number in the boxes at the top of the page.
● Write your answer to each question in the space provided.
● Do not use an erasable pen or correction fluid.
● Do not write on any bar codes.
● You may use a calculator.
● You should show all your working and use appropriate units.

INFORMATION
● The total mark for this paper is 80.
● The number of marks for each question or part question is shown in brackets [ ].

This document has 20 pages. Any blank pages are indicated.

DC (KN/CGW) 203280/4
© UCLES 2021 [Turn over

Page 165 of 877


2

1 Fig. 1.1 shows several villi from the ileum, which is part of the small intestine.

(a) State the name of one other part of the small intestine.

............................................................................................................................................. [1]

area enlarged
in Fig. 1.2

lacteal

goblet cells

Fig. 1.1

Fig. 1.2 shows the tip of a villus in more detail.

goblet cell
microvilli

epithelial cells

Fig. 1.2

© UCLES 2021 0610/42/O/N/21

Page 166 of 877


3

(b) The epithelial cells of the villi absorb nutrients by diffusion and active transport.

(i) Describe how active transport differs from diffusion.

...........................................................................................................................................

...........................................................................................................................................

...........................................................................................................................................

...........................................................................................................................................

..................................................................................................................................... [3]

(ii) Explain the importance of the microvilli shown in Fig. 1.2.

...........................................................................................................................................

...........................................................................................................................................

...........................................................................................................................................

...........................................................................................................................................

..................................................................................................................................... [2]

(c) Goblet cells provide protection for the epithelial cells that line the intestine.

(i) State the name of the protective substance produced by goblet cells.

..................................................................................................................................... [1]

(ii) Suggest why a protective substance is necessary in the intestines.

...........................................................................................................................................

...........................................................................................................................................

...........................................................................................................................................

...........................................................................................................................................

..................................................................................................................................... [2]

(d) Fig. 1.1 shows a lacteal in the centre of each villus.

Describe the roles of lacteals.

...................................................................................................................................................

...................................................................................................................................................

...................................................................................................................................................

...................................................................................................................................................

............................................................................................................................................. [2]
© UCLES 2021 0610/42/O/N/21 [Turn over
Page 167 of 877
4

(e) Complete Table 1.1 by identifying the level of organisation of each structure.

Choose your answers from the list. Each word or phrase may be used once, more than once
or not at all.

cell cell structure organ organ system organism tissue

Table 1.1

structure level of organisation

gall bladder

endoplasmic reticulum

intestinal epithelium

ileum

[4]

(f) Many fungi are decomposers that feed on dead plants. The fungi secrete enzymes to digest
large molecules.

Students made an extract from a species of fungus. The extract contained digestive enzymes.

The students carried out an investigation to find out if amylase and pectinase were present in
the fungal extract.

They made agar plates by filling Petri dishes with agar jelly containing either starch or pectin.
They cut four holes of the same size in the agar jelly in each Petri dish.

The holes in each Petri dish contained the same volume of:
A – 1% amylase solution
B – 1% pectinase solution
C – distilled water
D – fungal extract.

The Petri dishes were kept at 27 °C for four days. After this time a dye was poured into each
dish to stain the areas where starch and pectin remained.

Fig. 1.3 shows drawings of the stained agar in the Petri dishes. The clear zones indicate the
areas where no starch or pectin remained.

© UCLES 2021 0610/42/O/N/21

Page 168 of 877


5

Petri dish containing starch Petri dish containing pectin


in agar jelly in agar jelly

A B A B
Key:

stained area

D clear zone
D C C
hole in agar jelly

Fig. 1.3

(i) State what conclusions can be made about the enzymes in the fungal extract and give
evidence from Fig. 1.3 to support your conclusions.

...........................................................................................................................................

...........................................................................................................................................

...........................................................................................................................................

...........................................................................................................................................

...........................................................................................................................................

...........................................................................................................................................

...........................................................................................................................................

...........................................................................................................................................

...........................................................................................................................................

...........................................................................................................................................

..................................................................................................................................... [5]

(ii) The investigation was repeated but at 5 °C instead of at 27 °C.

Predict the effect of the lower temperature on the results.

...........................................................................................................................................

...........................................................................................................................................

..................................................................................................................................... [1]

[Total: 21]

© UCLES 2021 0610/42/O/N/21 [Turn over


Page 169 of 877
6

2 (a) A scientist investigated sexual reproduction in flowering plants.

Fig. 2.1 shows the procedure for crossing two plants of the same species.

paint brush used


anthers are removed to pick up pollen
from flower 1 from flower 2

pollen transferred
stigma ready to to the stigma of
receive pollen flower 1

bag placed
around flower 1
and tied tightly

seeds develop in flower 1


after fertilisation

Fig. 2.1

The scientist collected the seeds and germinated them. The leaves and flowers of the offspring
plants showed phenotypic variation as they were not all identical to the parent plants.

The scientist then investigated the chromosomes of all the offspring plants and found that
they had exactly the same number of chromosomes as the parent plants.

(i) Define the term chromosome.

...........................................................................................................................................

...........................................................................................................................................

..................................................................................................................................... [2]

© UCLES 2021 0610/42/O/N/21

Page 170 of 877


7

(ii) Suggest why the scientist placed a bag around flower 1.

...........................................................................................................................................

...........................................................................................................................................

..................................................................................................................................... [1]

(iii) Explain how sexual reproduction results in the variation that the scientist discovered in
the offspring plants.

...........................................................................................................................................

...........................................................................................................................................

...........................................................................................................................................

...........................................................................................................................................

..................................................................................................................................... [2]

(iv) The chromosome number of the offspring plants is the same as the chromosome number
of the parent plants in this investigation.

Explain how the chromosome number is maintained from one generation to the next.

...........................................................................................................................................

...........................................................................................................................................

...........................................................................................................................................

...........................................................................................................................................

..................................................................................................................................... [2]

© UCLES 2021 0610/42/O/N/21 [Turn over


Page 171 of 877
8

(b) The plant Camellia japonica has flowers that can be white, red or a mixture of these two
colours. When red-flowered plants are crossed with white-flowered plants, all the offspring
plants have flowers with petals that are a mixture of red and white, as shown in Fig. 2.2.

Fig. 2.2

• The gene for petal colour in C. japonica is given the symbol P.


• The allele for white petals is given the symbol PW.
• The allele for red petals is given the symbol PR.

(i) Table 2.1 shows the phenotypes of three different pairs of parent plants.

Complete Table 2.1 by giving all the possible genotypes of the offspring plants that could
be produced by these parent plants.

Space for working.

© UCLES 2021 0610/42/O/N/21

Page 172 of 877


9

Table 2.1

phenotype of female all the possible genotypes of offspring


phenotype of male parent
parent plants produced by this cross

red petals red petals

white petals red petals

petals that are both red petals that are both red
and white and white

[3]

(ii) State the type of inheritance that is shown by petal colour in C. japonica.

..................................................................................................................................... [1]

[Total: 11]

© UCLES 2021 0610/42/O/N/21 [Turn over


Page 173 of 877
10

3 Fig. 3.1 shows the changes in the concentrations of the hormones FSH and LH during a menstrual
cycle.

45 Key:
= LH
40
= FSH
35

30

concentration 25
of hormone
/ arbitrary units 20

15

10

0
0 5 10 15 20 25 30
days in the menstrual cycle

Fig. 3.1

(a) (i) Suggest the target organ for FSH.

..................................................................................................................................... [1]

(ii) State how FSH reaches its target organ.

..................................................................................................................................... [1]

(iii) Describe the relationship shown by the two hormones in Fig. 3.1.

...........................................................................................................................................

...........................................................................................................................................

...........................................................................................................................................

...........................................................................................................................................

..................................................................................................................................... [2]

© UCLES 2021 0610/42/O/N/21

Page 174 of 877


11

(b) Describe the roles of FSH and LH in the menstrual cycle.

...................................................................................................................................................

...................................................................................................................................................

...................................................................................................................................................

...................................................................................................................................................

...................................................................................................................................................

...................................................................................................................................................

...................................................................................................................................................

...................................................................................................................................................

............................................................................................................................................. [4]

(c) Describe the changes that occur in the lining of the uterus during one menstrual cycle.

...................................................................................................................................................

...................................................................................................................................................

...................................................................................................................................................

...................................................................................................................................................

...................................................................................................................................................

...................................................................................................................................................

............................................................................................................................................. [3]

(d) Oral contraceptives are a method of birth control taken by women.

Outline how the hormones in contraceptives act as a method of birth control.

...................................................................................................................................................

...................................................................................................................................................

...................................................................................................................................................

...................................................................................................................................................

...................................................................................................................................................

...................................................................................................................................................

............................................................................................................................................. [3]

[Total: 14]

© UCLES 2021 0610/42/O/N/21 [Turn over


Page 175 of 877
12

4 Involuntary actions occur because nerve impulses travel along the components of reflex arcs.

An example of an involuntary action is the rapid movement of a hand after unexpectedly touching
a very hot object.

Fig. 4.1 shows the structures that are involved in the movement of the hand.

U X
V
hot object Y

Fig. 4.1

© UCLES 2021 0610/42/O/N/21

Page 176 of 877


13

(a) Table 4.1 shows the functions of some of the structures shown in Fig. 4.1, the names of the
structures and the letter from Fig. 4.1 that identifies each structure.

Complete Table 4.1.


Table 4.1

function name letter on Fig. 4.1

conducts impulses to central nervous


system (CNS)

conducts impulses to an effector

conducts impulses only within the


CNS

receptor

[5]

© UCLES 2021 0610/42/O/N/21 [Turn over


Page 177 of 877
14

(b) Fig. 4.2 shows the structure of the synapse at W on Fig. 4.1.

Fig. 4.2

Describe how an impulse travels across the synapse shown in Fig. 4.2.

...................................................................................................................................................

...................................................................................................................................................

...................................................................................................................................................

...................................................................................................................................................

...................................................................................................................................................

...................................................................................................................................................

...................................................................................................................................................

...................................................................................................................................................

............................................................................................................................................. [4]

(c) State one example of a reflex action that occurs in the eye.

...................................................................................................................................................

...................................................................................................................................................

............................................................................................................................................. [1]

[Total: 10]

© UCLES 2021 0610/42/O/N/21

Page 178 of 877


16

5 (a) State two factors that affect the volume of urine produced in the human body.

1 ................................................................................................................................................

2 ................................................................................................................................................
[2]

(b) Nitrogen is an important element for organisms.

In a livestock farm, waste from animals contains protein. This waste is often spread on
farmland as a fertiliser.

Describe how the nitrogen in protein is recycled in the soil into a form that plants can absorb
and use.

...................................................................................................................................................

...................................................................................................................................................

...................................................................................................................................................

...................................................................................................................................................

...................................................................................................................................................

...................................................................................................................................................

...................................................................................................................................................

...................................................................................................................................................

...................................................................................................................................................

...................................................................................................................................................

............................................................................................................................................. [5]

© UCLES 2021 0610/42/O/N/21

Page 180 of 877


17

(c) Waste from livestock farms often pollutes water courses, such as streams and rivers, leading
to a reduction in biodiversity.

Explain how the pollution of water courses by animal waste leads to a reduction in biodiversity.

...................................................................................................................................................

...................................................................................................................................................

...................................................................................................................................................

...................................................................................................................................................

...................................................................................................................................................

...................................................................................................................................................

...................................................................................................................................................

...................................................................................................................................................

...................................................................................................................................................

...................................................................................................................................................

...................................................................................................................................................

...................................................................................................................................................

............................................................................................................................................. [6]

[Total: 13]

© UCLES 2021 0610/42/O/N/21 [Turn over


Page 181 of 877
18

6 Enzymes are catalysts.

(a) Define the term catalyst.

...................................................................................................................................................

...................................................................................................................................................

............................................................................................................................................. [2]

(b) Fig. 6.1 shows diagrams of three enzymes and eight different substrates.

W
1
3
V
P

U Q

2 R
T S

Fig. 6.1

(i) State the letter of the substrate that will be broken down by enzyme 1.

..................................................................................................................................... [1]

(ii) Explain, in terms of enzyme structure, the reason for your choice in 6(b)(i).

...........................................................................................................................................

...........................................................................................................................................

...........................................................................................................................................

...........................................................................................................................................

..................................................................................................................................... [2]

© UCLES 2021 0610/42/O/N/21

Page 182 of 877


19

(c) Table 6.1 lists some enzymes and the reactions that they catalyse.

Complete Table 6.1.

Table 6.1

enzyme reaction

maltase breakdown of maltose to ...............................

............................... breakdown of proteins to amino acids

lipase breakdown of fats to ............................... and ...............................

............................... breakdown of lactose to simpler sugars

............................... insertion of a short length of DNA into a plasmid

restriction enzyme .......................................................................................................

[6]

[Total: 11]

© UCLES 2021 0610/42/O/N/21

Page 183 of 877


Cambridge IGCSE™
* 3 9 5 6 9 4 5 6 1 7 *

BIOLOGY 0610/43
Paper 4 Theory (Extended) October/November 2021

1 hour 15 minutes

You must answer on the question paper.

No additional materials are needed.

INSTRUCTIONS
● Answer all questions.
● Use a black or dark blue pen. You may use an HB pencil for any diagrams or graphs.
● Write your name, centre number and candidate number in the boxes at the top of the page.
● Write your answer to each question in the space provided.
● Do not use an erasable pen or correction fluid.
● Do not write on any bar codes.
● You may use a calculator.
● You should show all your working and use appropriate units.

INFORMATION
● The total mark for this paper is 80.
● The number of marks for each question or part question is shown in brackets [ ].

This document has 20 pages. Any blank pages are indicated.

DC (LK) 308227/5 R
© UCLES 2021 [Turn over

Page 185 of 877


2

1 Enzymes are used in genetic engineering.

(a) Define the term enzyme.

...................................................................................................................................................

...................................................................................................................................................

............................................................................................................................................. [2]

(b) The process of genetic engineering often starts with the steps shown in Fig. 1.1.

upper DNA strand

G A A T T C
step 1 lower DNA strand

step 2 enzyme 1

A A T T C
G
step 3

Fig. 1.1

(i) State the sequence of bases on the lower strand of the DNA molecule in step 1.

upper DNA strand G A A T T C

lower DNA strand

[1]

(ii) State the name of enzyme 1 in step 2 of Fig. 1.1.

..................................................................................................................................... [1]

© UCLES 2021 0610/43/O/N/21

Page 186 of 877


3

(iii) Describe the effect of enzyme 1 on the DNA molecule in step 3.

...........................................................................................................................................

...........................................................................................................................................

...........................................................................................................................................

...........................................................................................................................................

..................................................................................................................................... [2]

(iv) Explain how enzyme 1 in Fig. 1.1 is specific to the exact sequence of DNA bases.

...........................................................................................................................................

...........................................................................................................................................

...........................................................................................................................................

...........................................................................................................................................

..................................................................................................................................... [2]

© UCLES 2021 0610/43/O/N/21 [Turn over


Page 187 of 877
4

(c) Another enzyme, enzyme 2, is used later in the process of genetic engineering.

Fig. 1.2 is a diagram showing the action of enzyme 2.

plasmid DNA
strands

upper DNA strand


of the gene
lower DNA strand
step 6 of the gene

step 7 enzyme 2

step 8

Fig. 1.2

(i) Some organisms naturally contain DNA in the form of a plasmid.

State the name of the type of organism that naturally contains plasmids.

..................................................................................................................................... [1]

(ii) State the name of enzyme 2 in step 7 of Fig. 1.2.

..................................................................................................................................... [1]

(iii) State the name of the molecule formed in step 8.

..................................................................................................................................... [1]

© UCLES 2021 0610/43/O/N/21

Page 188 of 877


5

(d) Sketch a graph to describe how the activity of the enzymes used in genetic engineering would
change if the reaction occurred at a range of temperatures from very cold to very hot.

Label the axes with appropriate titles.

Do not use units or a numbered scale.

[3]

[Total: 14]

© UCLES 2021 0610/43/O/N/21 [Turn over


Page 189 of 877
6

2 Fig. 2.1 is a photomicrograph of the end of a plant root.

length / mm

root tip 0 5 10 15 20 25 30 35

Fig. 2.1

Fig. 2.2 shows the results of a study on the rate of uptake of nitrate ions at different points along
the root shown in Fig. 2.1.

30

25

20

rate of nitrate ion


uptake 15
2
/ pmol per cm per s

10

0
0 5 10 15 20 25 30 35
distance from root tip / mm

Fig. 2.2

© UCLES 2021 0610/43/O/N/21

Page 190 of 877


7

(a) (i) Describe the rate of uptake of nitrate ions along the root.

Use the information in Fig. 2.1 and Fig. 2.2 in your answer.

...........................................................................................................................................

...........................................................................................................................................

...........................................................................................................................................

...........................................................................................................................................

...........................................................................................................................................

...........................................................................................................................................

...........................................................................................................................................

...........................................................................................................................................

...........................................................................................................................................

...........................................................................................................................................

..................................................................................................................................... [5]

(ii) Explain how nitrate ions move from the soil into roots.

...........................................................................................................................................

...........................................................................................................................................

...........................................................................................................................................

...........................................................................................................................................

...........................................................................................................................................

...........................................................................................................................................

...........................................................................................................................................

...........................................................................................................................................

..................................................................................................................................... [4]

(iii) Explain why the uptake of ions, such as nitrate, is important for the uptake of water in
roots.

...........................................................................................................................................

...........................................................................................................................................

..................................................................................................................................... [1]

© UCLES 2021 0610/43/O/N/21 [Turn over


Page 191 of 877
8

(iv) Explain why plants need nitrate ions, other than for the uptake of water.

...........................................................................................................................................

...........................................................................................................................................

...........................................................................................................................................

...........................................................................................................................................

...........................................................................................................................................

...........................................................................................................................................

..................................................................................................................................... [3]

(v) Describe how nitrate ions are formed in the soil.

...........................................................................................................................................

...........................................................................................................................................

...........................................................................................................................................

...........................................................................................................................................

...........................................................................................................................................

...........................................................................................................................................

..................................................................................................................................... [3]

© UCLES 2021 0610/43/O/N/21

Page 192 of 877


9

(b) (i) Fig. 2.3 shows some of the events that occur when high concentrations of nitrate ions
flow into lakes.

A a decrease in the concentration of dissolved oxygen

B a decrease in the population of consumers

C a decrease in the population of producers

D a decrease in light intensity at the bottom of the lake

E an increase in the population of decomposers

F an increase in the population of producers

Fig. 2.3

Put the events shown in Fig. 2.3 into the correct sequence.

[2]

(ii) State the name of the process summarised in Fig. 2.3.

..................................................................................................................................... [1]

[Total: 19]

© UCLES 2021 0610/43/O/N/21 [Turn over


Page 193 of 877
10

3 A researcher investigated genetic variation in fruit flies, Drosophila melanogaster.

The bodies of fruit flies can be black or yellow. A yellow body colour is a recessive feature in fruit
flies.

(a) Two heterozygous fruit flies with black bodies were bred together.

Predict the phenotypes of the offspring and the phenotypic ratio for this cross.

............................................................................................................................................. [1]

(b) In another fruit fly breeding experiment, researchers counted 124 offspring with black bodies
and 121 offspring with yellow bodies.

Draw a genetic diagram to explain the results of this cross. Use the letter B to represent the
allele for black body colour and the letter b to represent the allele for yellow body colour.

parental phenotypes .................................. × ..................................

parental genotypes .................................. × ..................................

gametes , × ,
............. ............. ............. .............

offspring genotypes ..................................................................................................................

expected offspring phenotype ratio ...................... black : ...................... yellow

actual offspring phenotype ratio 124 black : 121 yellow


[5]

© UCLES 2021 0610/43/O/N/21

Page 194 of 877


11

(c) The crab, Cerberusa caeca, lives in dark caves and has no coloured pigment.

Fig. 3.1 is a photograph of C. caeca.

Fig. 3.1

(i) C. caeca and D. melanogaster are both arthropods.

State one feature present in all arthropods but not present in vertebrates.

..................................................................................................................................... [1]

(ii) C. caeca is a crustacean and D. melanogaster is an insect.

State one morphological feature of C. caeca that distinguishes it as a crustacean and


not as an insect.

..................................................................................................................................... [1]

© UCLES 2021 0610/43/O/N/21 [Turn over


Page 195 of 877
12

(d) The ancestors of C. caeca had pigmented bodies.

The lack of a coloured pigment in C. caeca is called albinism and was caused by a mutation
many thousands of years ago.

(i) Explain the mechanism that has resulted in the allele for albinism becoming common in
recent generations in populations of C. caeca.

...........................................................................................................................................

...........................................................................................................................................

...........................................................................................................................................

...........................................................................................................................................

...........................................................................................................................................

...........................................................................................................................................

..................................................................................................................................... [3]

(ii) State two factors that can cause mutations.

1 ........................................................................................................................................

2 ........................................................................................................................................
[2]

[Total: 13]

© UCLES 2021 0610/43/O/N/21

Page 196 of 877


13

4 HIV is a pathogen that can cause AIDS.

(a) Describe how HIV is transmitted from one person to another.

...................................................................................................................................................

...................................................................................................................................................

...................................................................................................................................................

...................................................................................................................................................

...................................................................................................................................................

...................................................................................................................................................

............................................................................................................................................. [3]

(b) All viruses contain genetic material. HIV contains genetic material called RNA.

State one other feature common to all viruses.

............................................................................................................................................. [1]

(c) (i) Describe the function of lymphocytes.

...........................................................................................................................................

...........................................................................................................................................

...........................................................................................................................................

...........................................................................................................................................

...........................................................................................................................................

...........................................................................................................................................

..................................................................................................................................... [3]

(ii) State how infection with HIV affects the lymphocytes if untreated.

...........................................................................................................................................

..................................................................................................................................... [1]

© UCLES 2021 0610/43/O/N/21 [Turn over


Page 197 of 877
14

(d) Doctors wanted to determine whether dietary supplements could help people infected
with HIV.

They randomly put volunteers with HIV into two groups:


• a treatment group, who received HIV medication and additional vitamin and mineral
supplements
• a control group, who received HIV medication but no additional supplements.

The details of the two groups are outlined in Table 4.1.

Table 4.1

treatment group control group


total number of volunteers 18 22
average age / years 45.6 46.6
average mass / kg 82.3 82.5

The dietary supplements were given to the treatment group twice a day for three months. The
nutrients in the supplements included:
• vitamin C
• vitamin D
• calcium
• iron
• other minerals and vitamins.

(i) Explain why vitamin C and iron are important in the human diet.

...........................................................................................................................................

...........................................................................................................................................

...........................................................................................................................................

...........................................................................................................................................

...........................................................................................................................................

...........................................................................................................................................

...........................................................................................................................................

...........................................................................................................................................

..................................................................................................................................... [4]

© UCLES 2021 0610/43/O/N/21

Page 198 of 877


15

Table 4.2 shows some of the results from the study.

Table 4.2

treatment group control group


after three after three
at the start at the start
months months
average number of
357 422 461 461
lymphocytes / cells per μg of blood
average number of copies of HIV
4291 897 2648 5935
RNA per cm3 of blood

(ii) Use the data for the treatment group, shown in Table 4.2, to calculate the percentage
decrease in the average number of copies of HIV RNA per cm3 of blood.

Space for working.

.............................................................%
[2]

(iii) Evaluate the effect of the dietary supplements on the lymphocytes.

Use the information in Table 4.2 in your answer.

...........................................................................................................................................

...........................................................................................................................................

...........................................................................................................................................

...........................................................................................................................................

..................................................................................................................................... [2]

[Total: 16]

© UCLES 2021 0610/43/O/N/21 [Turn over


Page 199 of 877
16

5 Fig. 5.1 shows people fishing on a large scale and a small scale.

large-scale fishing small-scale fishing

Fig. 5.1

All types of fishing can have a negative impact on fish stocks.

(a) Discuss how governments can regulate fishing to maintain fish stocks.

Use the word sustainable in your answer.

...................................................................................................................................................

...................................................................................................................................................

...................................................................................................................................................

...................................................................................................................................................

...................................................................................................................................................

...................................................................................................................................................

...................................................................................................................................................

...................................................................................................................................................

...................................................................................................................................................

...................................................................................................................................................

...................................................................................................................................................

...................................................................................................................................................

............................................................................................................................................. [6]

© UCLES 2021 0610/43/O/N/21

Page 200 of 877


17

(b) Fig. 5.2 shows the location of a chemical factory near a river.

site 1

direction of
water flow

site 2

Fig. 5.2

Fig. 5.3 shows the sex ratio of the fish, Catostomus commersonii, in the river at site 1 and
site 2. Intersex fish have both female and male reproductive organs.

Key:
female
site 1
intersex
male

site 2

0 10 20 30 40 50 60 70 80 90 100
percentage of fish

Fig. 5.3

State and explain what type of chemical the factory could be releasing into the river that
would cause the effects shown in Fig. 5.3.

type of chemical ........................................................................................................................

explanation ...............................................................................................................................

...................................................................................................................................................

...................................................................................................................................................
[2]

(c) State how sex is inherited in humans.

............................................................................................................................................. [1]

[Total: 9]
© UCLES 2021 0610/43/O/N/21 [Turn over
Page 201 of 877
18

6 The heart pumps blood around the body.

(a) Explain why the heart is an organ.

...................................................................................................................................................

...................................................................................................................................................

............................................................................................................................................. [1]

(b) Complete the sentences:

The ............................................... system includes the heart and blood vessels. Deoxygenated

blood from the body is transported to the heart in the ............................................... .

During a heart beat the ventricles contract. The right ventricle pumps deoxygenated blood to

the lungs. The right ventricle has a ............................................... muscular wall than the left

ventricle.

Gas exchange in the lungs occurs by ............................................... . Oxygenated blood

travels back to the heart where it enters the ............................................... of the heart.

The two sides of the heart are separated by the ............................................... . This

structure prevents the mixing of oxygenated and deoxygenated blood. Oxygenated blood is

then delivered to the rest of the body. Blood is supplied to the muscle of the heart in the

............................................... .
[7]

(c) Many people monitor their heart rate by counting their pulse.

State one other method of monitoring heart rate.

............................................................................................................................................. [1]

[Total: 9]

© UCLES 2021 0610/43/O/N/21

Page 202 of 877


Cambridge IGCSE™
* 4 7 8 0 1 4 8 1 2 4 *

BIOLOGY 0610/42
Paper 4 Theory (Extended) February/March 2021

1 hour 15 minutes

You must answer on the question paper.

No additional materials are needed.

INSTRUCTIONS
● Answer all questions.
● Use a black or dark blue pen. You may use an HB pencil for any diagrams or graphs.
● Write your name, centre number and candidate number in the boxes at the top of the page.
● Write your answer to each question in the space provided.
● Do not use an erasable pen or correction fluid.
● Do not write on any bar codes.
● You may use a calculator.
● You should show all your working and use appropriate units.

INFORMATION
● The total mark for this paper is 80.
● The number of marks for each question or part question is shown in brackets [ ].

This document has 20 pages. Any blank pages are indicated.

DC (NF/AR) 202933/3
© UCLES 2021 [Turn over

Page 205 of 877


2

BLANK PAGE

© UCLES 2021 0610/42/F/M/21

Page 206 of 877


3

1 (a) Fig. 1.1 is a diagram showing the position of some organs in the human body.

Fig. 1.1

Some of the organs shown in Fig. 1.1 are endocrine glands.

Table 1.1 shows the names of some of the endocrine glands, their identifying letters and the
hormones that they produce.

Complete Table 1.1.

Table 1.1

name of endocrine gland letter in Fig. 1.1 hormone produced

insulin

testes

[3]

© UCLES 2021 0610/42/F/M/21 [Turn over


Page 207 of 877
4

(b) Fig. 1.2 shows two graphs representing:

• the relative blood concentrations of two hormones, A and B, released by the ovaries
during the menstrual cycle
• the thickness of the lining of the uterus.

relative blood
concentration A B
of the
hormones

0 2 4 6 8 10 12 14 16 18 20 22 24 26 28
days

thickness
of the lining
of the uterus

0 2 4 6 8 10 12 14 16 18 20 22 24 26 28
days

Fig. 1.2

© UCLES 2021 0610/42/F/M/21

Page 208 of 877


5

(i) Describe the roles in the menstrual cycle of hormone A.

...........................................................................................................................................

...........................................................................................................................................

...........................................................................................................................................

...........................................................................................................................................

...........................................................................................................................................

...........................................................................................................................................

..................................................................................................................................... [3]

(ii) State the day in Fig. 1.2 when ovulation is most likely to occur.

..................................................................................................................................... [1]

(iii) State the days in Fig. 1.2 when the lining of the uterus is lost from the body.

..................................................................................................................................... [1]

© UCLES 2021 0610/42/F/M/21 [Turn over


Page 209 of 877
6

(c) The female contraceptive pill is a chemical method of birth control, which is available in many
countries.

(i) Describe the social implications of the increased availability of the female contraceptive
pill.

...........................................................................................................................................

...........................................................................................................................................

...........................................................................................................................................

...........................................................................................................................................

..................................................................................................................................... [2]

(ii) The hormones in the female contraceptive pill can enter rivers.

Describe the negative impacts of female contraceptive hormones entering rivers and
contaminating drinking water.

...........................................................................................................................................

...........................................................................................................................................

...........................................................................................................................................

...........................................................................................................................................

...........................................................................................................................................

...........................................................................................................................................

..................................................................................................................................... [3]

(iii) State two barrier methods of contraception.

1 ........................................................................................................................................

2 ........................................................................................................................................
[2]

[Total: 15]

© UCLES 2021 0610/42/F/M/21

Page 210 of 877


8

2 (a) Fig. 2.1 is a diagram of a prokaryotic cell.

not to scale

Fig. 2.1

(i) State one visible feature in Fig. 2.1 that identifies this cell as a prokaryotic cell.

..................................................................................................................................... [1]

(ii) State one cell structure that is present in the cells of all organisms.

..................................................................................................................................... [1]

(b) Prokaryotes, Animals and Plants are three of the five kingdoms of organisms.

State the names of the two other kingdoms.

1 ................................................................................................................................................

2 ................................................................................................................................................
[2]

(c) Fig. 2.2 shows part of the nitrogen cycle.

nitrogen in
atmosphere

proteins in proteins in
animals plants
animal waste B C

dead
organisms
A
nitrate ions
in soil

Fig. 2.2

© UCLES 2021 0610/42/F/M/21

Page 212 of 877


9

(i) Describe processes A, B and C in Fig. 2.2.

...........................................................................................................................................

...........................................................................................................................................

...........................................................................................................................................

...........................................................................................................................................

...........................................................................................................................................

...........................................................................................................................................

...........................................................................................................................................

...........................................................................................................................................

...........................................................................................................................................

...........................................................................................................................................

...........................................................................................................................................

...........................................................................................................................................

..................................................................................................................................... [6]

(ii) State the name of the process that plants use to absorb nitrate ions.

..................................................................................................................................... [1]

[Total: 11]

© UCLES 2021 0610/42/F/M/21 [Turn over


Page 213 of 877
10

3 (a) The activity of the heart can be monitored using different methods.

Fig. 3.1 shows two ECG traces. One trace was recorded when the person was at rest and the
second trace was recorded during exercise.

The length of time taken for one heart beat is indicated in Fig. 3.1 on the ECG trace recorded
at rest.

one heart beat

at rest

0.0 0.2 0.4 0.6 0.8 1.0 1.2 1.4 1.6 1.8 2.0 2.2 2.4 2.6 2.8
time / s

during
exercise

0.0 0.2 0.4 0.6 0.8 1.0 1.2 1.4 1.6 1.8 2.0 2.2 2.4 2.6 2.8
time / s

Fig. 3.1

(i) Estimate the resting heart rate of the person from their ECG trace in Fig. 3.1.

Space for working.

.................................... beats per minute


[2]

© UCLES 2021 0610/42/F/M/21

Page 214 of 877


11

(ii) Explain why the ECG trace recorded during exercise differs from the ECG trace recorded
at rest.

...........................................................................................................................................

...........................................................................................................................................

...........................................................................................................................................

...........................................................................................................................................

...........................................................................................................................................

...........................................................................................................................................

...........................................................................................................................................

...........................................................................................................................................

..................................................................................................................................... [4]

(iii) Suggest one other way of monitoring the activity of the heart.

...........................................................................................................................................

..................................................................................................................................... [1]

(b) If exercise is very intense an oxygen debt is formed.

(i) Complete the sentence.

An oxygen debt results in a build-up of ................................................ in the

................................................ during vigorous exercise.


[2]

(ii) Outline how the body removes an oxygen debt.

...........................................................................................................................................

...........................................................................................................................................

...........................................................................................................................................

...........................................................................................................................................

...........................................................................................................................................

...........................................................................................................................................

...........................................................................................................................................

...........................................................................................................................................

..................................................................................................................................... [4]
[Total: 13]
© UCLES 2021 0610/42/F/M/21 [Turn over
Page 215 of 877
12

4 (a) A leaf can be described as an organ.

Define the term organ.

...................................................................................................................................................

...................................................................................................................................................

............................................................................................................................................. [1]

(b) Fig. 4.1 is a photomicrograph of a cross section of part of a leaf.

Fig. 4.1

(i) Identify the tissue labelled A in Fig. 4.1.

..................................................................................................................................... [1]

(ii) Identify the structure labelled C in Fig. 4.1.

..................................................................................................................................... [1]

© UCLES 2021 0610/42/F/M/21

Page 216 of 877


13

(iii) Describe how the tissue labelled B is adapted to maximise photosynthesis.

...........................................................................................................................................

...........................................................................................................................................

...........................................................................................................................................

...........................................................................................................................................

...........................................................................................................................................

...........................................................................................................................................

..................................................................................................................................... [3]

(c) (i) State two substances that are transported only in the phloem.

..................................................................................................................................... [1]

(ii) Explain why some parts of a plant can act as both a source and a sink.

...........................................................................................................................................

...........................................................................................................................................

...........................................................................................................................................

...........................................................................................................................................

..................................................................................................................................... [2]

© UCLES 2021 0610/42/F/M/21 [Turn over


Page 217 of 877
14

(d) The effect of carbon dioxide concentration on the rate of oxygen production in an aquatic
plant was measured.

• A lamp was used to keep the light intensity constant.


• The oxygen gas released by the plant was collected in a gas syringe.
• The plant was placed in water that was kept constant at 20 °C.

Fig. 4.2 shows the results.

(i) The rate of oxygen production was assumed to be the same as the rate of photosynthesis.

Suggest why the rate of oxygen production was not the same as the rate of
photosynthesis.

...........................................................................................................................................

...........................................................................................................................................

..................................................................................................................................... [2]

0.30

0.25

rate of
oxygen 0.20
production
/ cm3 per m2
per s
0.15

0.10

0.05

0.00
0 200 400 600 800 1000 1200 1400 1600
concentration of carbon dioxide / μmol per dm3

Fig. 4.2

© UCLES 2021 0610/42/F/M/21

Page 218 of 877


15

(ii) Explain the results shown in Fig. 4.2.

...........................................................................................................................................

...........................................................................................................................................

...........................................................................................................................................

...........................................................................................................................................

...........................................................................................................................................

...........................................................................................................................................

..................................................................................................................................... [3]

(e) The investigation was repeated with the same type of aquatic plant at 10 °C.

Draw a line on Fig. 4.2 to predict the results at 10 °C. [2]

[Total: 16]

© UCLES 2021 0610/42/F/M/21 [Turn over


Page 219 of 877
16

5 (a) Measles is a transmissible disease.

The percentage of the population that were vaccinated against measles in a country was
determined.

The number of confirmed cases of measles in the country was also recorded.

Fig. 5.1 shows the data that were collected between 1975 and 2010.

160 000

100 140 000

90 120 000

80 100 000
number of
percentage
cases of
of the
measles
population 70 80 000
vaccinated
against
measles
60 60 000

50 40 000

40 20 000

30 0
1975 1980 1985 1990 1995 2000 2005 2010
year
Key:
percentage of population vaccinated
number of cases of measles

Fig 5.1

(i) Calculate the percentage change in the number of cases of measles between 1980 and
1990.

Space for working.

.............................................................%
[2]
© UCLES 2021 0610/42/F/M/21

Page 220 of 877


17

(ii) Describe the data shown in Fig. 5.1.

...........................................................................................................................................

...........................................................................................................................................

...........................................................................................................................................

...........................................................................................................................................

...........................................................................................................................................

...........................................................................................................................................

...........................................................................................................................................

...........................................................................................................................................

..................................................................................................................................... [4]

(iii) Explain how vaccination protects people against a transmissible disease such as
measles.

...........................................................................................................................................

...........................................................................................................................................

...........................................................................................................................................

...........................................................................................................................................

...........................................................................................................................................

...........................................................................................................................................

...........................................................................................................................................

...........................................................................................................................................

...........................................................................................................................................

...........................................................................................................................................

..................................................................................................................................... [5]

(b) The human body has several defences against pathogens.

(i) State two of the body’s chemical barriers to pathogens.

1 ........................................................................................................................................

2 ........................................................................................................................................
[2]

© UCLES 2021 0610/42/F/M/21 [Turn over


Page 221 of 877
18

(ii) Describe the process of blood clotting.

...........................................................................................................................................

...........................................................................................................................................

...........................................................................................................................................

...........................................................................................................................................

...........................................................................................................................................

...........................................................................................................................................

..................................................................................................................................... [3]

[Total: 16]

© UCLES 2021 0610/42/F/M/21

Page 222 of 877


19

6 (a) Fig. 6.1 shows a food web.

hawks

snakes

rabbits grasshoppers mice

grass corn

Fig. 6.1

(i) State the number of trophic levels in the food web in Fig. 6.1.

..................................................................................................................................... [1]

(ii) State the name of one organism that feeds at both the third and fourth trophic levels
from Fig. 6.1.

..................................................................................................................................... [1]

(iii) State the name of the type of energy that is transferred between trophic levels.

..................................................................................................................................... [1]

(b) The total biomass of the snakes is much less than the total biomass of the mice in the food
web shown in Fig. 6.1.

Explain why the total biomass of the snakes is less than the total biomass of the mice.

Use the term energy in your answer.

...................................................................................................................................................

...................................................................................................................................................

...................................................................................................................................................

...................................................................................................................................................

...................................................................................................................................................

...................................................................................................................................................

............................................................................................................................................. [3]

© UCLES 2021 0610/42/F/M/21 [Turn over


Page 223 of 877
20

(c) Food shortages that result in famine can be caused by many factors.

Describe how drought can contribute to famine.

...................................................................................................................................................

...................................................................................................................................................

...................................................................................................................................................

...................................................................................................................................................

...................................................................................................................................................

...................................................................................................................................................

............................................................................................................................................. [3]

[Total: 9]

Permission to reproduce items where third-party owned material protected by copyright is included has been sought and cleared where possible. Every
reasonable effort has been made by the publisher (UCLES) to trace copyright holders, but if any items requiring clearance have unwittingly been included, the
publisher will be pleased to make amends at the earliest possible opportunity.

To avoid the issue of disclosure of answer-related information to candidates, all copyright acknowledgements are reproduced online in the Cambridge
Assessment International Education Copyright Acknowledgements Booklet. This is produced for each series of examinations and is freely available to download
at www.cambridgeinternational.org after the live examination series.

Cambridge Assessment International Education is part of the Cambridge Assessment Group. Cambridge Assessment is the brand name of the University of
Cambridge Local Examinations Syndicate (UCLES), which itself is a department of the University of Cambridge.

© UCLES 2021 0610/42/F/M/21

Page 224 of 877


Cambridge IGCSE™
* 0 8 8 3 9 1 5 6 0 7 *

BIOLOGY 0610/41
Paper 4 Theory (Extended) May/June 2020

1 hour 15 minutes

You must answer on the question paper.

No additional materials are needed.

INSTRUCTIONS
● Answer all questions.
● Use a black or dark blue pen. You may use an HB pencil for any diagrams or graphs.
● Write your name, centre number and candidate number in the boxes at the top of the page.
● Write your answer to each question in the space provided.
● Do not use an erasable pen or correction fluid.
● Do not write on any bar codes.
● You may use a calculator.
● You should show all your working and use appropriate units.

INFORMATION
● The total mark for this paper is 80.
● The number of marks for each question or part question is shown in brackets [ ].

This document has 20 pages. Blank pages are indicated.

DC (ST/CT) 180481/3
© UCLES 2020 [Turn over

Page 225 of 877


2

1 The gas exchange system is one of the organ systems of the human body.

Fig. 1.1 shows parts of the gas exchange system during breathing in and breathing out.

breathing in breathing out

intercostal
muscles
vertebrae

sternum

diaphragm

Fig. 1.1

(a) Complete Table 1.1 to show:

• the functions of the diaphragm and the intercostal muscles during breathing in and
breathing out
• the pressure changes in the thorax.

Use these words:

contract
relax
increases
decreases.

Table 1.1

intercostal muscles pressure change


diaphragm
internal external in the thorax

breathing in

breathing out

[4]

© UCLES 2020 0610/41/M/J/20

Page 226 of 877


3

Fig. 1.2 shows part of the gas exchange surface of a human.

movement of air

magnification ×350

Fig. 1.2

(b) State two features of the gas exchange surface that are visible in Fig. 1.2.

1 ................................................................................................................................................

2 ................................................................................................................................................
[2]

(c) The cells labelled X on Fig. 1.2 form a tissue.

(i) Define the term tissue.

...........................................................................................................................................

...........................................................................................................................................

...........................................................................................................................................

...........................................................................................................................................

..................................................................................................................................... [2]

© UCLES 2020 0610/41/M/J/20 [Turn over


Page 227 of 877
4

(ii) Cartilage is another tissue found in the gas exchange system.

State the functions of cartilage in the gas exchange system.

...........................................................................................................................................

...........................................................................................................................................

...........................................................................................................................................

...........................................................................................................................................

..................................................................................................................................... [2]

[Total: 10]

2 Biological washing powders contain enzymes that break down food stains.

(a) Complete Table 2.1 by naming the enzymes that break down three substances in food stains
and by stating the product or products.

Table 2.1

substance enzyme product(s)

starch

fat

protein

[3]

Some students compared how effective biological and non-biological washing powders are at
removing stains at temperatures between 10 °C and 60 °C.

• Pieces of stained cloth were washed using two different washing powders.
• The degree of stain removal was measured by using a light meter to record the
percentage of light reflected from the cloth.
• A light meter gave a value of 100% when the cloth was completely clean.
• Any stain left on the cloth reduced the percentage of light reflected.

© UCLES 2020 0610/41/M/J/20

Page 228 of 877


5

The results of the students’ investigation are shown in Fig. 2.1.

110

100

90
percentage
of light 80
reflected
after 70
washing
60

50

40
10 20 30 40 50 60 70
temperature of washing / °C

Key:
non-biological washing powder
biological washing powder

Fig. 2.1

(b) Compare the effectiveness of the two washing powders at removing stains.

Use the information in Fig. 2.1 in your answer.

...................................................................................................................................................

...................................................................................................................................................

...................................................................................................................................................

...................................................................................................................................................

...................................................................................................................................................

...................................................................................................................................................

...................................................................................................................................................

...................................................................................................................................................

............................................................................................................................................. [4]

© UCLES 2020 0610/41/M/J/20 [Turn over


Page 229 of 877
6

(c) The students suggested that the enzymes in the biological washing powder were denatured
at high temperatures.

Explain why enzyme molecules do not function when they are denatured.

...................................................................................................................................................

...................................................................................................................................................

...................................................................................................................................................

...................................................................................................................................................

............................................................................................................................................. [2]

(d) Forensic scientists often try to find DNA on items of stained clothing. The DNA can be used to
identify individual people.

Suggest why DNA can be used to identify individual people.

...................................................................................................................................................

...................................................................................................................................................

...................................................................................................................................................

...................................................................................................................................................

............................................................................................................................................. [2]

[Total: 11]

3 (a) Dialysis tubing is an artificial membrane, which is similar to the lining of the intestine.

A student investigated the diffusion of glucose through dialysis tubing by using the apparatus
shown in Fig. 3.1.

rubber band
to secure the
dialysis tubing

glucose solution
water outside
the dialysis tubing
dialysis tubing

knot at the end of


the dialysis tubing

Fig. 3.1

© UCLES 2020 0610/41/M/J/20

Page 230 of 877


7

The student took samples of the water outside the dialysis tubing at 5 minute intervals and
tested the samples with Benedict’s solution.

The results are shown in Table 3.1.

Table 3.1

results of the Benedict’s tests on the


time / minutes
water outside the dialysis tubing
0 blue
5 green
10 yellow
15 red

(i) Describe and explain the results shown in Table 3.1.

...........................................................................................................................................

...........................................................................................................................................

...........................................................................................................................................

...........................................................................................................................................

...........................................................................................................................................

...........................................................................................................................................

..................................................................................................................................... [3]

(ii) The student repeated the investigation with a higher concentration of glucose in the
dialysis tubing.

Predict the results that the student would observe.

...........................................................................................................................................

...........................................................................................................................................

..................................................................................................................................... [1]

© UCLES 2020 0610/41/M/J/20 [Turn over


Page 231 of 877
8

(b) Fig. 3.2 shows a drawing of a cell from the lining of the small intestine. The lumen is the
space inside the intestine where food is digested.

lumen of the intestine

not to scale

Fig. 3.2

State the names of the three labelled structures in Fig. 3.2 and describe the role of each
structure in the intestinal cell.

...................................................................................................................................................

...................................................................................................................................................

...................................................................................................................................................

...................................................................................................................................................

...................................................................................................................................................

...................................................................................................................................................

...................................................................................................................................................

...................................................................................................................................................

...................................................................................................................................................

...................................................................................................................................................

...................................................................................................................................................

...................................................................................................................................................

............................................................................................................................................. [6]

© UCLES 2020 0610/41/M/J/20

Page 232 of 877


9

(c) The cholera bacterium can survive in the small intestine and the large intestine. The bacterium
releases a toxin that interacts with receptors on the surface of cells.

Fig. 3.3 shows the effect of the toxin. The arrows indicate the direction of movement.

lumen of the intestine


bacterium

Key:
toxin
ion X

not to scale

Fig. 3.3

The toxin stimulates the secretion of ion X out of the intestinal cell.

(i) State the name of ion X.

..................................................................................................................................... [1]

(ii) Describe the effects on the body of the secretion of ion X into the lumen of the intestine.

...........................................................................................................................................

...........................................................................................................................................

...........................................................................................................................................

...........................................................................................................................................

...........................................................................................................................................

...........................................................................................................................................

...........................................................................................................................................

...........................................................................................................................................

..................................................................................................................................... [4]

[Total: 15]

© UCLES 2020 0610/41/M/J/20 [Turn over


Page 233 of 877
11

4 Johnson grass, Sorghum halepense, is wind-pollinated.

(a) Fig. 4.1 shows some Johnson grass flowers.

Fig. 4.1

(i) State the genus of Johnson grass.

..................................................................................................................................... [1]

(ii) Describe two features visible in Fig. 4.1 that show that Johnson grass flowers are
adapted for wind-pollination.

1 ........................................................................................................................................

...........................................................................................................................................

2 ........................................................................................................................................

...........................................................................................................................................
[2]

© UCLES 2020 0610/41/M/J/20 [Turn over


Page 235 of 877
12

(b) Fig. 4.2 shows a section through a carpel shortly after pollination.

B
A

Fig. 4.2

(i) State the names of the parts of the carpel labelled C, D and E.

C ........................................................................................................................................

D ........................................................................................................................................

E ........................................................................................................................................
[3]

(ii) Complete the sentences:

Pollen grains are formed in anthers. During their formation the number of chromosomes

in the nuclei is halved by the process of .............................................. . This means the

male nucleus A in the pollen tube is described as a .............................................. nucleus.

When nucleus A .............................................. with nucleus B, the chromosome number

doubles to form a .............................................. nucleus. The name of this process is

.............................................. . Then the .............................................. divides by the

process of .............................................. to form an embryo.


[7]

© UCLES 2020 0610/41/M/J/20

Page 236 of 877


13

(c) Discuss the advantages of sexual reproduction to a wild population of flowering plants such
as Johnson grass.

...................................................................................................................................................

...................................................................................................................................................

...................................................................................................................................................

...................................................................................................................................................

...................................................................................................................................................

...................................................................................................................................................

...................................................................................................................................................

...................................................................................................................................................

...................................................................................................................................................

...................................................................................................................................................

............................................................................................................................................. [5]

(d) Sexual reproduction requires energy.

State three uses of energy in organisms other than in reproduction.

1 ................................................................................................................................................

2 ................................................................................................................................................

3 ................................................................................................................................................
[3]

[Total: 21]

© UCLES 2020 0610/41/M/J/20 [Turn over


Page 237 of 877
14

5 Ciliates are classified in the kingdom Protoctist. Bacteria are classified in the kingdom Prokaryote.

(a) State two structural features that distinguish the cells of a protoctist from a prokaryote.

1 ................................................................................................................................................

...................................................................................................................................................

2 ................................................................................................................................................

...................................................................................................................................................
[2]

(b) Fig. 5.1 shows five species of ciliate that are found in sewage treatment works.

cilia

rows of
cilia

A Chilodonella B Didinium
cilia
fused
together

C Euplotes D Paramecium E Vorticella

not to scale

Fig. 5.1

© UCLES 2020 0610/41/M/J/20

Page 238 of 877


15

Fig. 5.2 is a dichotomous key to identify the ciliates shown in Fig. 5.1.

START

yes
1 has a ring of cilia at yes 2 organism B
one end of the no
organism organism E
no

3 has star-like yes


structures inside the organism D
organism
no

4 yes
organism A

no

organism C

Fig. 5.2

Complete the key in Fig. 5.2 by writing suitable statements:

• for box 2 to distinguish species B and E


• for box 4 to distinguish species A and C.

text for box 2 .............................................................................................................................

...................................................................................................................................................

...................................................................................................................................................

text for box 4 .............................................................................................................................

...................................................................................................................................................

...................................................................................................................................................
[2]

© UCLES 2020 0610/41/M/J/20 [Turn over


Page 239 of 877
16

(c) Didinium is a predatory ciliate. A video recording was made of one Didinium feeding on a
Paramecium. Fig. 5.3 shows a sequence of still photographs taken from the video.

Didinium

Paramecium

Fig. 5.3

Complete the table by putting a tick (✓) by each characteristic of life that can be seen in the
still photographs from the video in Fig. 5.3.

excretion nutrition

growth reproduction

movement respiration

[1]

(d) Fig. 5.4 is a food web for some of the microorganisms in a sewage treatment works.

Didinium

rotifers Paramecium nematodes

Vorticella

photosynthetic bacteria decomposer


bacteria

Fig. 5.4

(i) Construct one food chain with three trophic levels that use energy derived from the
breakdown of sewage. Do not draw the organisms.

..................................................................................................................................... [1]
© UCLES 2020 0610/41/M/J/20

Page 240 of 877


17

(ii) The water that passed out of the sewage works was often cloudy with suspended matter.

Scientists discovered that ciliates reduce the cloudiness of water during sewage
treatment.

Suggest how the ciliates reduce the cloudiness of the water using the information in
Fig. 5.4.

...........................................................................................................................................

...........................................................................................................................................

...........................................................................................................................................

...........................................................................................................................................

..................................................................................................................................... [2]

(iii) Explain how sewage treatment reduces the spread of disease.

...........................................................................................................................................

...........................................................................................................................................

...........................................................................................................................................

...........................................................................................................................................

...........................................................................................................................................

...........................................................................................................................................

..................................................................................................................................... [3]

(iv) Nitrifying bacteria are found in sewage works.

Explain the importance of nitrifying bacteria in the nitrogen cycle.

...........................................................................................................................................

...........................................................................................................................................

...........................................................................................................................................

...........................................................................................................................................

...........................................................................................................................................

...........................................................................................................................................

..................................................................................................................................... [3]

[Total: 14]

© UCLES 2020 0610/41/M/J/20 [Turn over


Page 241 of 877
18

6 Colour blindness is a characteristic that is inherited. Colour blindness is more common in males
than in females.

Fig. 6.1 is a pedigree diagram showing the inheritance of colour blindness in a family.

Key:
male with normal
colour vision
male with colour
blindness
1 2
female with normal
colour vision
3 4

5 6 7 8

Fig. 6.1

(a) Define the term inheritance.

...................................................................................................................................................

...................................................................................................................................................

............................................................................................................................................. [1]

(b) (i) Using the symbols B and b, state the genotypes of individual 5 and individual 8 in the
pedigree diagram.

5 ........................................................................................................................................

8 ........................................................................................................................................
[3]

© UCLES 2020 0610/41/M/J/20

Page 242 of 877


19

(ii) Individual 3 is a carrier of colour blindness because she has one copy of the allele for
colour blindness but has normal colour vision.

Describe the evidence from Fig. 6.1 that shows that individual 3 is a carrier.

...........................................................................................................................................

...........................................................................................................................................

...........................................................................................................................................

...........................................................................................................................................

...........................................................................................................................................

...........................................................................................................................................

..................................................................................................................................... [3]

(iii) There was no history of colour blindness in the parents and grandparents of individuals 1
and 2.

Suggest how colour blindness first occurred in the family in Fig. 6.1.

...........................................................................................................................................

...........................................................................................................................................

...........................................................................................................................................

...........................................................................................................................................

..................................................................................................................................... [2]

[Total: 9]

© UCLES 2020 0610/41/M/J/20

Page 243 of 877


20

BLANK PAGE

Permission to reproduce items where third-party owned material protected by copyright is included has been sought and cleared where possible. Every
reasonable effort has been made by the publisher (UCLES) to trace copyright holders, but if any items requiring clearance have unwittingly been included, the
publisher will be pleased to make amends at the earliest possible opportunity.

To avoid the issue of disclosure of answer-related information to candidates, all copyright acknowledgements are reproduced online in the Cambridge
Assessment International Education Copyright Acknowledgements Booklet. This is produced for each series of examinations and is freely available to download
at www.cambridgeinternational.org after the live examination series.

Cambridge Assessment International Education is part of the Cambridge Assessment Group. Cambridge Assessment is the brand name of the University of
Cambridge Local Examinations Syndicate (UCLES), which itself is a department of the University of Cambridge.

© UCLES 2020 0610/41/M/J/20

Page 244 of 877


Cambridge IGCSE™
* 1 7 0 3 9 2 5 1 3 2 *

BIOLOGY 0610/42
Paper 4 Theory (Extended) May/June 2020

1 hour 15 minutes

You must answer on the question paper.

No additional materials are needed.

INSTRUCTIONS
● Answer all questions.
● Use a black or dark blue pen. You may use an HB pencil for any diagrams or graphs.
● Write your name, centre number and candidate number in the boxes at the top of the page.
● Write your answer to each question in the space provided.
● Do not use an erasable pen or correction fluid.
● Do not write on any bar codes.
● You may use a calculator.
● You should show all your working and use appropriate units.

INFORMATION
● The total mark for this paper is 80.
● The number of marks for each question or part question is shown in brackets [ ].

This document has 24 pages. Blank pages are indicated.

DC (JC/CT) 180482/4
© UCLES 2020 [Turn over

Page 245 of 877


2

1 Homeostasis is the maintenance of a constant internal environment.

(a) Human skin is involved in the maintenance of a constant internal body temperature.

(i) Skin is an organ.

State why the skin is an organ.

...........................................................................................................................................

...........................................................................................................................................

..................................................................................................................................... [1]

(ii) State the name of the organ that coordinates the control of body temperature.

..................................................................................................................................... [1]

Fig. 1.1 shows a diagram of a section through human skin.

A
F

Fig. 1.1

(iii) State the names of structures A, B and C in Fig. 1.1.

A ........................................................................................................................................

B ........................................................................................................................................

C ........................................................................................................................................
[3]

© UCLES 2020 0610/42/M/J/20

Page 246 of 877


3

(iv) Structure D is a shunt vessel and E is an arteriole.

Describe how these blood vessels are involved in maintaining a constant internal body
temperature in a cold environment.

...........................................................................................................................................

...........................................................................................................................................

...........................................................................................................................................

...........................................................................................................................................

...........................................................................................................................................

...........................................................................................................................................

..................................................................................................................................... [3]

(b) Energy is used to maintain body temperature.

State three other uses of energy in humans.

1 ................................................................................................................................................

2 ................................................................................................................................................

3 ................................................................................................................................................
[3]

[Total: 11]

© UCLES 2020 0610/42/M/J/20 [Turn over


Page 247 of 877
4

2 Pregnancy can occur after the fusion of a male gamete and a female gamete.

(a) State the name of the ball of cells that implants into the uterus after fertilisation.

............................................................................................................................................. [1]

(b) There are many changes that occur in a fetus during pregnancy.

Compare the development of a fetus in the early stages of pregnancy to its development in
the late stages of pregnancy.

...................................................................................................................................................

...................................................................................................................................................

...................................................................................................................................................

...................................................................................................................................................

............................................................................................................................................. [2]

(c) Describe the functions of amniotic fluid and the amniotic sac.

...................................................................................................................................................

...................................................................................................................................................

...................................................................................................................................................

...................................................................................................................................................

...................................................................................................................................................

...................................................................................................................................................

...................................................................................................................................................

...................................................................................................................................................

............................................................................................................................................. [4]

(d) The umbilical artery is found in the umbilical cord. This artery transports blood away from the
heart of the fetus.

The umbilical artery is unusual because it transports deoxygenated blood.

(i) State the name of one other artery in the mother that transports deoxygenated blood.

..................................................................................................................................... [1]

© UCLES 2020 0610/42/M/J/20

Page 248 of 877


5

(ii) State one excretory product that is transported from the fetus to the placenta.

..................................................................................................................................... [1]

(iii) State the name of the process that allows substances to move down a concentration
gradient across the placenta.

..................................................................................................................................... [1]

© UCLES 2020 0610/42/M/J/20 [Turn over


Page 249 of 877
6

(e) One of the functions of the placenta is to provide a barrier to toxins and pathogens.

A study was done on donated afterbirths. The afterbirth is a placenta with part of the umbilical
cord attached.

The purpose of the study was to find the maximum size of particles that can pass through the
placenta and enter the umbilical cord.

The researchers inserted beads with a diameter of 0.5 µm into blood vessels in the placenta.
Three hours later they recorded the percentage of beads found in the blood in the placenta
and in the umbilical cord.

They then repeated the tests using beads with diameters of 0.8 µm, 2.4 µm, 5.0 µm and
8.0 µm.

Their results are shown in Fig. 2.1.


100

80

percentage of 60
beads found in
the blood after
3 hours
40

20

0
0.5 μm 0.8 μm 2.4 μm 5.0 μm 8.0 μm
bead size
Key:
umbilical cord
placenta

Fig. 2.1

© UCLES 2020 0610/42/M/J/20

Page 250 of 877


7

(i) Convert the diameter of the 5.0 µm beads into millimetres (mm).

Space for working.

................................................... mm [1]

(ii) One million beads with a diameter of 2.4 µm were injected into the placenta.

Calculate the number of these beads in the umbilical cord after 3 hours.

Space for working.

..................................................... beads
[2]

(iii) Table 2.1 shows a range of substances and their diameters.

Table 2.1

toxins and pathogens diameter / µm


nicotine 2.0 × 10–2
drug X 3.0 × 10–2
rubella virus 5.0 × 10–2
Vibrio cholerae 8.0 × 10–1
Trypanosoma brucei 1.8 × 101

State the names of all the toxins and pathogens listed in Table 2.1 that could pass
through the placenta and enter the umbilical cord.

Use the data in Fig. 2.1 to make your choice.

...........................................................................................................................................

...........................................................................................................................................

..................................................................................................................................... [1]

© UCLES 2020 0610/42/M/J/20 [Turn over


Page 251 of 877
8

(f) Fig. 2.2 shows the junction between two neurones with drug X absent and two neurones with
drug X present, immediately after a painful stimulus.

B
Key:

electrical signal

C neurotransmitter
drug X
D

relay neurone

drug X absent drug X present

Fig. 2.2

(i) State the names of A, B, C and D in Fig. 2.2.

A ........................................................................................................................................

B ........................................................................................................................................

C ........................................................................................................................................

D ........................................................................................................................................
[4]

(ii) Describe and explain how drug X affects the function of the relay neurone shown in
Fig. 2.2.

...........................................................................................................................................

...........................................................................................................................................

...........................................................................................................................................

...........................................................................................................................................

...........................................................................................................................................

...........................................................................................................................................

..................................................................................................................................... [3]

© UCLES 2020 0610/42/M/J/20

Page 252 of 877


9

(g) Drug X can be injected into the body. This is one way that HIV can be transmitted.

Describe two other ways that HIV can be transmitted.

1 ................................................................................................................................................

...................................................................................................................................................

2 ................................................................................................................................................

...................................................................................................................................................
[2]

[Total: 23]

© UCLES 2020 0610/42/M/J/20 [Turn over


Page 253 of 877
11

3 Cheetahs, Acinonyx jubatus, are carnivores found in the dry grasslands and woodlands of
southern Africa. Cheetahs hunt for food during the day. They eat deer and antelope.

The cheetah is the fastest mammal on land but can only run at high speed (sprint) over a short
distance. Its hunting strategy is to creep up on prey and then sprint to catch them.

Fig. 3.1 is a photograph of a cheetah in its natural habitat.

Fig. 3.1

(a) Suggest how these adaptive features enable cheetahs to survive in their natural environment.

fur colouring ..............................................................................................................................

...................................................................................................................................................

...................................................................................................................................................

streamlined body shape ...........................................................................................................

...................................................................................................................................................

...................................................................................................................................................
[2]

© UCLES 2020 0610/42/M/J/20 [Turn over


Page 255 of 877
12

(b) The king cheetah is a rare variety of A. jubatus that has inherited striped fur markings.

Fig. 3.2 shows a cheetah with spots and a king cheetah.

cheetah with spots king cheetah

Fig. 3.2

(i) Define the term inheritance.

...........................................................................................................................................

...........................................................................................................................................

..................................................................................................................................... [1]

© UCLES 2020 0610/42/M/J/20

Page 256 of 877


13

Fig. 3.3 shows a pedigree diagram of a population of cheetahs.

1 2

3 4 5 6 7 8 9 10

11 12 13 14 15 16 17 18 19
?

Key:

female king cheetah female cheetah with spots

male king cheetah male cheetah with spots

Fig. 3.3

(ii) Deduce the genotype of cheetah 11.

..................................................................................................................................... [1]

(iii) Predict the probability of cheetah 14 being a king cheetah.

..................................................................................................................................... [1]

(iv) Describe how a breeder could determine the genotype of cheetah 17.

...........................................................................................................................................

...........................................................................................................................................

...........................................................................................................................................

...........................................................................................................................................

..................................................................................................................................... [2]

© UCLES 2020 0610/42/M/J/20 [Turn over


Page 257 of 877
14

(v) When the king cheetah was first discovered it was thought that it was a new species.

Pedigree diagrams of cheetahs proved it was not a new species.

Suggest one type of evidence, other than pedigree diagrams, that can be used to
determine how closely related organisms are.

...........................................................................................................................................

...........................................................................................................................................

..................................................................................................................................... [1]

(c) Cheetahs are at risk of becoming endangered.

(i) Suggest why the cheetah is at risk of becoming endangered.

...........................................................................................................................................

...........................................................................................................................................

...........................................................................................................................................

...........................................................................................................................................

...........................................................................................................................................

...........................................................................................................................................

..................................................................................................................................... [3]

(ii) Describe how species like the cheetah can be conserved.

...........................................................................................................................................

...........................................................................................................................................

...........................................................................................................................................

...........................................................................................................................................

...........................................................................................................................................

...........................................................................................................................................

..................................................................................................................................... [3]

[Total: 14]

© UCLES 2020 0610/42/M/J/20

Page 258 of 877


15

4 Some crop farmers use herbicides on their fields.

Fig. 4.1 shows a farmer spraying a rice crop with herbicides.

Fig. 4.1

(a) Herbicides kill weeds.

Explain why farmers use herbicides.

...................................................................................................................................................

...................................................................................................................................................

...................................................................................................................................................

...................................................................................................................................................

............................................................................................................................................. [2]

© UCLES 2020 0610/42/M/J/20 [Turn over


Page 259 of 877
16

(b) Fields of crop plants were sprayed with two herbicides. A farmer measured the concentration
of the two herbicides, A and B, in a lake near the fields.

The water in the lake was sampled at intervals for two weeks.

Fig. 4.2 shows the results.

0.20

0.16

0.12
concentration
of herbicides in
the lake water
samples / mg per kg
0.08

0.04

0.00
0 2 4 6 8 10 12 14 16
time / days

herbicides Key:
sprayed on herbicide A
the fields herbicide B

Fig. 4.2

(i) Compare the concentrations of herbicide A and herbicide B in the lake.

Use the information in Fig. 4.2 to support your answer.

...........................................................................................................................................

...........................................................................................................................................

...........................................................................................................................................

...........................................................................................................................................

...........................................................................................................................................

...........................................................................................................................................

..................................................................................................................................... [3]
© UCLES 2020 0610/42/M/J/20

Page 260 of 877


17

(ii) Suggest how herbicides damage ecosystems in a lake.

...........................................................................................................................................

...........................................................................................................................................

...........................................................................................................................................

...........................................................................................................................................

...........................................................................................................................................

...........................................................................................................................................

...........................................................................................................................................

...........................................................................................................................................

..................................................................................................................................... [4]

(c) Herbicide A is a synthetic plant hormone called 2,4-D that selectively kills dicotyledonous
plants only.

(i) State two features that distinguish leaves of dicotyledonous plants from leaves of
monocotyledonous plants.

1 ........................................................................................................................................

2 ........................................................................................................................................
[2]

(ii) State the name of a natural plant hormone that stimulates cell elongation.

..................................................................................................................................... [1]

(iii) Herbicide B is a chemical that prevents the uptake of magnesium ions.

Suggest how herbicide B kills plants.

...........................................................................................................................................

...........................................................................................................................................

...........................................................................................................................................

...........................................................................................................................................

...........................................................................................................................................

...........................................................................................................................................

..................................................................................................................................... [3]

[Total: 15]

© UCLES 2020 0610/42/M/J/20 [Turn over


Page 261 of 877
18

5 The Galápagos Islands are a group of small islands in the Pacific Ocean.

In 1839 Charles Darwin published a book that described differences in a family of birds called
finches.

Each species of Galápagos finch had:

• a different diet
• a different beak shape, as shown in Fig. 5.1.

Geospiza conirostris Geospiza scandens

Geospiza magnirostris Geospiza fortis

Fig. 5.1

(a) State the genus name for the Galápagos finches shown in Fig. 5.1.

............................................................................................................................................. [1]

© UCLES 2020 0610/42/M/J/20

Page 262 of 877


19

(b) Galápagos finches share a common ancestor.

Suggest how Galápagos finches have evolved different shaped beaks.

...................................................................................................................................................

...................................................................................................................................................

...................................................................................................................................................

...................................................................................................................................................

...................................................................................................................................................

...................................................................................................................................................

...................................................................................................................................................

...................................................................................................................................................

...................................................................................................................................................

...................................................................................................................................................

............................................................................................................................................. [5]

[Total: 6]

© UCLES 2020 0610/42/M/J/20 [Turn over


Page 263 of 877
20

6 (a) Fig. 6.1 is a diagram showing some parts of a plant. The circle shows a magnified cross-section
of part of the stem.

C
M D
E
L
F
K G

not to scale

Fig. 6.1

(i) Table 6.1 contains statements about the functions of some of the structures in Fig. 6.1.

Complete the table by:

• stating the name of the structure


• identifying the letter that labels that structure.

Table 6.1

function name of structure letter from Fig. 6.1

provides support to the stem

protects flower bud

produces glucose

produces pollen

delivers male nuclei to the site of fertilisation

[5]

© UCLES 2020 0610/42/M/J/20

Page 264 of 877


21

(ii) State one letter from Fig. 6.1 that identifies a structure that contains a haploid nucleus.

..................................................................................................................................... [1]

(iii) State the name of the process that describes the transport of sucrose in a plant.

..................................................................................................................................... [1]

(iv) State one letter from Fig. 6.1 that is a structure that is an example of a source for sucrose
transport.

..................................................................................................................................... [1]

(b) In addition to sucrose, amino acids are also transported in plants.

(i) State the name of a mineral ion that becomes part of an amino acid.

..................................................................................................................................... [1]

(ii) State the name of the structures inside cells that assemble amino acids into proteins.

..................................................................................................................................... [1]

(iii) State the name of the group of molecules that are made of proteins and act as catalysts.

..................................................................................................................................... [1]

[Total: 11]

© UCLES 2020 0610/42/M/J/20

Page 265 of 877


Cambridge IGCSE™
* 1 7 9 1 8 2 5 5 2 6 *

BIOLOGY 0610/43
Paper 4 Theory (Extended) May/June 2020

1 hour 15 minutes

You must answer on the question paper.

No additional materials are needed.

INSTRUCTIONS
● Answer all questions.
● Use a black or dark blue pen. You may use an HB pencil for any diagrams or graphs.
● Write your name, centre number and candidate number in the boxes at the top of the page.
● Write your answer to each question in the space provided.
● Do not use an erasable pen or correction fluid.
● Do not write on any bar codes.
● You may use a calculator.
● You should show all your working and use appropriate units.

INFORMATION
● The total mark for this paper is 80.
● The number of marks for each question or part question is shown in brackets [ ].

This document has 20 pages. Blank pages are indicated.

DC (ST/CT) 180793/3
© UCLES 2020 [Turn over

Page 269 of 877


2

1 (a) State three uses of energy in the human body.

1 ................................................................................................................................................

2 ................................................................................................................................................

3 ................................................................................................................................................
[3]

(b) Fig. 1.1 shows part of the digestive system of a human.

diaphragm

B
L

K
C
H

G
D

Fig. 1.1

© UCLES 2020 0610/43/M/J/20

Page 270 of 877


3

Complete Table 1.1. One row has been done for you.

Table 1.1

function name of structure letter from Fig. 1.1

pushes food to the stomach oesophagus A

assimilation of amino acids to produce


plasma proteins

storage of bile

secretion of insulin

absorption of fatty acids and glycerol

secretion of pepsin

digestion of starch

[6]

(c) Describe the role of the liver in the recovery from oxygen debt after strenuous exercise.

...................................................................................................................................................

...................................................................................................................................................

...................................................................................................................................................

...................................................................................................................................................

............................................................................................................................................. [2]

(d) Alcohol is a drug.

Define the term drug.

...................................................................................................................................................

...................................................................................................................................................

............................................................................................................................................. [2]

© UCLES 2020 0610/43/M/J/20 [Turn over


Page 271 of 877
4

(e) (i) State two immediate effects of excessive alcohol on the body.

1 ........................................................................................................................................

2 ........................................................................................................................................
[2]

(ii) State two long-term effects of excessive alcohol on the body.

1 ........................................................................................................................................

2 ........................................................................................................................................
[2]

(f) Pregnant women are advised not to drink alcohol as it may have harmful effects on the fetus.

(i) Outline these harmful effects.

...........................................................................................................................................

...........................................................................................................................................

...........................................................................................................................................

...........................................................................................................................................

..................................................................................................................................... [2]

(ii) State two harmful substances other than alcohol that can cross the placenta.

1 ........................................................................................................................................

2 ........................................................................................................................................
[2]

[Total: 21]

© UCLES 2020 0610/43/M/J/20

Page 272 of 877


5

2 (a) Fig. 2.1 shows the human population of a country between 1910 and 2020.

140

120

100
number of
people 80
/ million
60

40

20

0
1900 1920 1940 1960 1980 2000 2020
year

Fig. 2.1

(i) Calculate the percentage increase in the population of the country between 1940 and
2020.

Space for working.

.............................................................%
[3]

(ii) Describe the factors that could cause the change in the population size between 1940
and 2020, shown in Fig. 2.1.

...........................................................................................................................................

...........................................................................................................................................

...........................................................................................................................................

...........................................................................................................................................

...........................................................................................................................................

...........................................................................................................................................

..................................................................................................................................... [3]

© UCLES 2020 0610/43/M/J/20 [Turn over


Page 273 of 877
6

(b) Some countries have invested in biofuels such as ethanol, biomass and biodiesel.

(i) Describe how ethanol can be made by microorganisms.

...........................................................................................................................................

...........................................................................................................................................

...........................................................................................................................................

...........................................................................................................................................

..................................................................................................................................... [2]

(ii) Some countries use large areas of land to grow maize plants. This crop plant can be
used to produce biofuels.

Discuss the negative impact on the environment of growing large-scale monocultures


of crop plants such as maize.

...........................................................................................................................................

...........................................................................................................................................

...........................................................................................................................................

...........................................................................................................................................

...........................................................................................................................................

...........................................................................................................................................

...........................................................................................................................................

...........................................................................................................................................

..................................................................................................................................... [4]

[Total: 12]

© UCLES 2020 0610/43/M/J/20

Page 274 of 877


8

3 The American writer Ernest Hemingway lived on the island of Key West in Florida, USA in the
1930s. During this time he was given a male cat by a sea captain.

The cat had more toes than usual. This inherited condition is called polydactyly. The allele for
polydactyly is dominant.

(a) Define the term inheritance.

...................................................................................................................................................

...................................................................................................................................................

............................................................................................................................................. [1]

(b) Fig. 3.1 is part of a pedigree diagram for Hemingway’s cats.

1 2

3 4 5 6 7 8 9 10

11 12 13 14 15 16

Key:
17
female cat with normal female cat with
number of toes polydactyly

male cat with normal male cat with


number of toes polydactyly

Fig. 3.1

© UCLES 2020 0610/43/M/J/20

Page 276 of 877


9

(i) State the genotypes of cats 5, 6 and 14 in the pedigree diagram in Fig. 3.1.

Use the letters T and t.

cat 5 ..................................................................................................................................

cat 6 ..................................................................................................................................

cat 14 ................................................................................................................................
[3]

(ii) Explain why none of the offspring of cats 3 and 4 have inherited polydactyly.

Use the information in Fig. 3.1 in your answer.

...........................................................................................................................................

...........................................................................................................................................

..................................................................................................................................... [1]

© UCLES 2020 0610/43/M/J/20 [Turn over


Page 277 of 877
10

(c) Scientists published the results of an investigation into the DNA of cats with and without
polydactyly. They compared the base sequence from a particular region of DNA that controls
the development of the limbs.

Table 3.1 shows the base sequences.

Table 3.1

cats without polydactyly AGA CAC AGA AAT GAG


Hemingway’s cats with polydactyly AGA CAC GGA AAT GAG
cats with polydactyly from Oregon and Missouri in the USA AGA CAC GGA AAT GAG
cats with polydactyly from the UK AGA CAC AGT AAT GAG

(i) Describe how the base sequences of the cats with polydactyly differ from the base
sequence of cats without polydactyly.

...........................................................................................................................................

...........................................................................................................................................

...........................................................................................................................................

...........................................................................................................................................

..................................................................................................................................... [2]

(ii) State the name of the process by which base sequences in DNA are changed.

..................................................................................................................................... [1]

(iii) The base sequences in Table 3.1 provide evidence that indicates which country the male
cat given to Hemingway in the 1930s came from.

Suggest which country this cat came from and give a reason for your choice.

...........................................................................................................................................

...........................................................................................................................................

...........................................................................................................................................

...........................................................................................................................................

..................................................................................................................................... [2]

© UCLES 2020 0610/43/M/J/20

Page 278 of 877


11

(d) Fig. 3.2 shows part of a DNA molecule from a chromosome of a cat.

Complete Fig. 3.2 by writing the letters for the base sequence of the other strand of the DNA
molecule.

T ..........

A ..........

A ..........

T ..........

G ..........

C ..........

G ..........

T ..........

G ..........

Fig. 3.2
[1]

(e) Explain why polydactyly is an example of discontinuous variation.

...................................................................................................................................................

...................................................................................................................................................

...................................................................................................................................................

...................................................................................................................................................

............................................................................................................................................. [2]

[Total: 13]

© UCLES 2020 0610/43/M/J/20 [Turn over


Page 279 of 877
12

4 Xerophytes grow in habitats with low rainfall and soils that often have high concentrations of salts.

Fig. 4.1 shows the xerophyte Yucca treculeana growing on salt flats.

Y. treculeana

Fig. 4.1

(a) (i) Explain how xerophytes, such as Y. treculeana, are adapted to absorb sufficient water in
the conditions in which they live.

...........................................................................................................................................

...........................................................................................................................................

...........................................................................................................................................

...........................................................................................................................................

...........................................................................................................................................

...........................................................................................................................................

..................................................................................................................................... [4]

© UCLES 2020 0610/43/M/J/20

Page 280 of 877


13

(ii) Explain how xerophytes are adapted to reduce water loss to the atmosphere.

...........................................................................................................................................

...........................................................................................................................................

...........................................................................................................................................

...........................................................................................................................................

...........................................................................................................................................

...........................................................................................................................................

..................................................................................................................................... [3]

(iii) Xerophytes often have many defence mechanisms that reduce or prevent herbivores
eating them.

Suggest how xerophytes protect themselves against herbivores.

...........................................................................................................................................

...........................................................................................................................................

...........................................................................................................................................

...........................................................................................................................................

..................................................................................................................................... [2]

(b) Forest ecosystems can be affected by acid rain.

Describe how the production of acid rain and its effects on forest ecosystems can be reduced.

...................................................................................................................................................

...................................................................................................................................................

...................................................................................................................................................

...................................................................................................................................................

...................................................................................................................................................

...................................................................................................................................................

...................................................................................................................................................

...................................................................................................................................................

............................................................................................................................................. [4]

[Total: 13]

© UCLES 2020 0610/43/M/J/20 [Turn over


Page 281 of 877
15

5 Bacteria are classified in the Prokaryote kingdom.

(a) State two features of animal and plant cells that are not found in prokaryotes.

1 ................................................................................................................................................

2 ................................................................................................................................................
[2]

© UCLES 2020 0610/43/M/J/20 [Turn over


Page 283 of 877
16

(b) Methicillin-resistant Staphylococcus aureus (MRSA) is a type of bacterium that is resistant to


some antibiotics.

Fig. 5.1 shows how a population of bacteria may develop antibiotic resistance and how the
antibiotic resistance can be passed from one strain of bacterium to another.

population of pathogenic bacteria A in


patient 1

treatment with antibiotic Z started

population of pathogenic bacteria A in


patient 1 in the middle of treatment
with antibiotic Z

population of pathogenic bacteria A in


patient 1 when treatment with
antibiotic Z was stopped

transmission of bacteria A to patient 2

bacteria A and B in patient 2

not to scale

Key:
bacterium of strain A bacterium of strain B
resistant to antibiotic Z resistant to antibiotic Z
bacterium of strain A bacterium of strain B
not resistant to antibiotic Z not resistant to antibiotic Z
dying bacterium of strain A plasmid

Fig. 5.1

© UCLES 2020 0610/43/M/J/20

Page 284 of 877


17

Explain how resistance to an antibiotic develops in a population of bacteria and spreads in


the human population.

...................................................................................................................................................

...................................................................................................................................................

...................................................................................................................................................

...................................................................................................................................................

...................................................................................................................................................

...................................................................................................................................................

...................................................................................................................................................

...................................................................................................................................................

...................................................................................................................................................

...................................................................................................................................................

...................................................................................................................................................

...................................................................................................................................................

...................................................................................................................................................

...................................................................................................................................................

............................................................................................................................................. [6]

(c) Explain how the development of resistant bacteria, such as MRSA, can be minimised.

...................................................................................................................................................

...................................................................................................................................................

...................................................................................................................................................

...................................................................................................................................................

...................................................................................................................................................

...................................................................................................................................................

...................................................................................................................................................

............................................................................................................................................. [3]

[Total: 11]

© UCLES 2020 0610/43/M/J/20 [Turn over


Page 285 of 877
18

6 In many parts of the world dairy cattle are kept in large barns and reared intensively, as shown in
Fig. 6.1.

Fig. 6.1

(a) Food for cattle that are reared intensively includes cereals, such as maize and barley.

Ecologists have calculated that it is more energy efficient to grow crops for human consumption
than for food for livestock.

Explain why intensive rearing of livestock is not an efficient use of crops.

...................................................................................................................................................

...................................................................................................................................................

...................................................................................................................................................

...................................................................................................................................................

...................................................................................................................................................

...................................................................................................................................................

...................................................................................................................................................

............................................................................................................................................. [3]

© UCLES 2020 0610/43/M/J/20

Page 286 of 877


19

(b) The urine and faeces from cattle kept in barns is removed and treated in the same way as
human sewage to avoid polluting the aquatic environment.

Outline the effects of untreated waste from cattle on the aquatic environment.

...................................................................................................................................................

...................................................................................................................................................

...................................................................................................................................................

...................................................................................................................................................

...................................................................................................................................................

...................................................................................................................................................

...................................................................................................................................................

............................................................................................................................................. [4]

(c) Intensive livestock production could be one way of preventing famine.

Describe the causes of famine.

...................................................................................................................................................

...................................................................................................................................................

...................................................................................................................................................

...................................................................................................................................................

...................................................................................................................................................

...................................................................................................................................................

............................................................................................................................................. [3]

[Total: 10]

© UCLES 2020 0610/43/M/J/20

Page 287 of 877


Cambridge IGCSE™
* 3 3 1 0 5 8 2 8 9 5 *

BIOLOGY 0610/41
Paper 4 Theory (Extended) October/November 2020

1 hour 15 minutes

You must answer on the question paper.

No additional materials are needed.

INSTRUCTIONS
● Answer all questions.
● Use a black or dark blue pen. You may use an HB pencil for any diagrams or graphs.
● Write your name, centre number and candidate number in the boxes at the top of the page.
● Write your answer to each question in the space provided.
● Do not use an erasable pen or correction fluid.
● Do not write on any bar codes.
● You may use a calculator.
● You should show all your working and use appropriate units.

INFORMATION
● The total mark for this paper is 80.
● The number of marks for each question or part question is shown in brackets [ ].

This document has 16 pages. Blank pages are indicated.

DC (JC/CT) 187653/6
© UCLES 2020 [Turn over

Page 289 of 877


2

1 Water is an essential molecule for life.

(a) Complete the statements.

Water moves into and out of cells by ............................................... .

Water is known as a ............................................... because it can dissolve solutes.


[2]

(b) A leaf cell was put into a solution. The water potential of the solution was lower than the water
potential of the contents of the cell.

Fig. 1.1 is a sketch of the cell after three hours in the solution.

nucleus

Fig. 1.1

The leaf cell was transferred into pure water.

Sketch the expected appearance of the cell after it had been in the pure water for three
hours.

Draw one arrow on your sketch to show the direction of water movement.

[3]
© UCLES 2020 0610/41/O/N/20

Page 290 of 877


3

(c) A plant was not watered for one week.

Fig. 1.2 shows a series of photographs of the plant during the week.

day 0 day 3 day 7

Fig. 1.2

Explain how the lack of water has affected the support of the leaves of the plant shown in
Fig. 1.2.

Use the term turgor pressure in your answer.

...................................................................................................................................................

...................................................................................................................................................

...................................................................................................................................................

...................................................................................................................................................

...................................................................................................................................................

...................................................................................................................................................

............................................................................................................................................. [3]

[Total: 8]

© UCLES 2020 0610/41/O/N/20 [Turn over


Page 291 of 877
4

2 Pathogens cause disease.

(a) Fig. 2.1 shows some cells that are part of the human immune system. These cells are
responding to one type of pathogen.

R
T

Fig. 2.1

Explain how the immune system responds to an invasion of pathogens.

Use the letters in Fig. 2.1 in your answer.

...................................................................................................................................................

...................................................................................................................................................

...................................................................................................................................................

...................................................................................................................................................

...................................................................................................................................................

...................................................................................................................................................

...................................................................................................................................................

...................................................................................................................................................

...................................................................................................................................................

...................................................................................................................................................

...................................................................................................................................................

...................................................................................................................................................

...................................................................................................................................................

...................................................................................................................................................

............................................................................................................................................. [6]

© UCLES 2020 0610/41/O/N/20

Page 292 of 877


5

(b) A vaccine was introduced in 1942 for a particular disease.

Fig. 2.2 shows the effect of introducing the vaccine on the number of cases of the disease in
one country.

250
vaccine
introduced

200

150
number of cases
of disease
per million people
100

50

0
1920 1930 1940 1950 1960 1970 1980
year

Fig. 2.2

In 1946 the government of the country concluded that the vaccine was successful.

Discuss the evidence, shown in Fig. 2.2, for and against this conclusion.

...................................................................................................................................................

...................................................................................................................................................

...................................................................................................................................................

...................................................................................................................................................

...................................................................................................................................................

...................................................................................................................................................

...................................................................................................................................................

...................................................................................................................................................

............................................................................................................................................. [4]

[Total: 10]
© UCLES 2020 0610/41/O/N/20 [Turn over
Page 293 of 877
6

3 All living organisms excrete waste products.

(a) Fig. 3.1 is a photomicrograph of Naegleria fowleri, a single-celled protoctist that lives in watery
environments.

magnification ×4000

Fig. 3.1

(i) State a feature of N. fowleri, visible in Fig. 3.1, that distinguishes it from prokaryotes.

..................................................................................................................................... [1]

(ii) State the name and function of structure A.

structure ............................................................................................................................

function ..............................................................................................................................

...........................................................................................................................................
[2]

(iii) Suggest how N. fowleri excretes carbon dioxide.

...........................................................................................................................................

..................................................................................................................................... [1]

© UCLES 2020 0610/41/O/N/20

Page 294 of 877


7

(b) Urea is a toxin that is excreted by the kidneys in humans.

Describe how and where in the body urea is formed.

...................................................................................................................................................

...................................................................................................................................................

...................................................................................................................................................

...................................................................................................................................................

...................................................................................................................................................

...................................................................................................................................................

............................................................................................................................................. [3]

© UCLES 2020 0610/41/O/N/20 [Turn over


Page 295 of 877
8

(c) Fig. 3.2 shows part of the human excretory system and associated blood vessels.

The arrows indicate the direction of blood flow.

H
not to scale
Fig. 3.2

(i) Draw a label line and the letter X on Fig. 3.2 to show the location of the cortex in one of
the kidneys. [1]

(ii) Table 3.1 contains statements about the labelled structures in Fig. 3.2.

Complete the table by:

• stating the name of the structure


• identifying the letter that labels that structure.

Table 3.1

description name of structure letter from Fig. 3.2

organ that stores urine

tube that carries urine out of the


kidney
blood vessel with the lowest
concentration of urea
blood vessel with the lowest
concentration of carbon dioxide
tube that carries urine out of the
body
[5]
© UCLES 2020 0610/41/O/N/20

Page 296 of 877


9

(d) Doctors wanted to investigate the effect of exercise on the excretion of salts.

They collected urine from people before and after running a long distance on a hot day.

The results of their investigation are shown in Table 3.2.

Table 3.2

before running after running

average volume of urine / cm3 1156.0 569.0

average concentration of sodium in


85.6 78.2
urine / mmol per dm3

(i) Suggest why there is a difference in the volume of urine produced before running
compared with after running.

Use the information in Table 3.2 in your answer.

...........................................................................................................................................

...........................................................................................................................................

...........................................................................................................................................

...........................................................................................................................................

..................................................................................................................................... [2]

(ii) Calculate the percentage decrease in the average sodium concentration after running
compared with before running.

Give your answer to one significant figure.

Space for working.

............................................................ %
[3]

© UCLES 2020 0610/41/O/N/20 [Turn over


Page 297 of 877
10

(iii) Describe how the kidney tubules enable the excretion of salts.

...........................................................................................................................................

...........................................................................................................................................

...........................................................................................................................................

...........................................................................................................................................

...........................................................................................................................................

...........................................................................................................................................

..................................................................................................................................... [3]

(e) Large plasma proteins are usually prevented from entering the urine.

State the name of one protein found in blood plasma.

............................................................................................................................................. [1]

[Total: 22]

© UCLES 2020 0610/41/O/N/20

Page 298 of 877


11

4 (a) Fig. 4.1 shows a bee with pollen on its legs.

Fig. 4.1

Bees are insects that pollinate some flowering plants. They are attracted to the flowers by
their colour, scent and nectar.

(i) Describe other ways in which flowers and pollen grains are adapted for insect pollination.

...........................................................................................................................................

...........................................................................................................................................

...........................................................................................................................................

...........................................................................................................................................

...........................................................................................................................................

...........................................................................................................................................

..................................................................................................................................... [3]

(ii) State where pollen is produced in a flower.

..................................................................................................................................... [1]

(iii) State the name of the process that produces haploid pollen nuclei.

..................................................................................................................................... [1]

(iv) Explain why it is important that the pollen nuclei are haploid.

...........................................................................................................................................

..................................................................................................................................... [1]

© UCLES 2020 0610/41/O/N/20 [Turn over


Page 299 of 877
12

(b) (i) Describe how the pollen that is carried by an insect to another flower results in the
formation of a plant embryo.

...........................................................................................................................................

...........................................................................................................................................

...........................................................................................................................................

...........................................................................................................................................

...........................................................................................................................................

...........................................................................................................................................

...........................................................................................................................................

...........................................................................................................................................

...........................................................................................................................................

...........................................................................................................................................

..................................................................................................................................... [5]

(ii) Describe the advantages of cross-pollination.

...........................................................................................................................................

...........................................................................................................................................

...........................................................................................................................................

...........................................................................................................................................

..................................................................................................................................... [2]

(c) Some people are concerned that genetically modified plants might cross-pollinate with wild
varieties of plants.

(i) Suggest how farmers could prevent cross-pollination between genetically modified plants
and wild varieties of plants.

...........................................................................................................................................

...........................................................................................................................................

..................................................................................................................................... [1]

(ii) State two advantages of genetically modified crops.

1 ........................................................................................................................................

2 ........................................................................................................................................
[2]

[Total: 16]

© UCLES 2020 0610/41/O/N/20

Page 300 of 877


13

5 Milk is a source of some of the nutrients that are part of a balanced diet.

(a) Calcium and protein are two nutrients found in milk.

Describe the importance of calcium and protein in the diet.

calcium .....................................................................................................................................

...................................................................................................................................................

...................................................................................................................................................

protein .......................................................................................................................................

...................................................................................................................................................

...................................................................................................................................................
[4]

(b) Lactose is found in cows’ milk. Some people do not have the enzyme to digest lactose.

State the names of two organs, associated with the alimentary canal, that produce enzymes.

1 ................................................................................................................................................

2 ................................................................................................................................................
[2]

© UCLES 2020 0610/41/O/N/20 [Turn over


Page 301 of 877
14

(c) Fig. 5.1 shows a flow diagram for the production of lactose-free milk.

Step 1 collect cows’ milk from farm

diagram of Step 4
Step 2 heat milk to a high temperature
for 15 seconds
milk

Step 3 allow milk to cool to 20°C

beads
containing
Step 4 pour milk into a container of enzyme
beads containing enzymes

Step 5 collect lactose-free milk

lactose-free milk
Step 6 store lactose-free milk at 2°C in
sterile bottles

Fig. 5.1

(i) Explain how heating the milk in step 2 in Fig. 5.1 ensures the hygienic preparation of
lactose-free milk.

...........................................................................................................................................

...........................................................................................................................................

..................................................................................................................................... [1]

(ii) Explain why the milk must be cooled in step 3 before it makes contact with the enzymes.

...........................................................................................................................................

...........................................................................................................................................

...........................................................................................................................................

...........................................................................................................................................

..................................................................................................................................... [2]

(iii) State the name of the enzyme used to make lactose-free milk in step 4.

..................................................................................................................................... [1]

© UCLES 2020 0610/41/O/N/20

Page 302 of 877


15

(iv) Suggest why the enzymes are kept in the beads in step 4 rather than mixed as an
enzyme solution with the milk.

...........................................................................................................................................

...........................................................................................................................................

..................................................................................................................................... [1]

(d) Milk is produced by mammals.

(i) Explain the advantages to newborn mammals of breast milk.

...........................................................................................................................................

...........................................................................................................................................

...........................................................................................................................................

...........................................................................................................................................

...........................................................................................................................................

...........................................................................................................................................

...........................................................................................................................................

...........................................................................................................................................

..................................................................................................................................... [4]

(ii) Explain why breast-feeding mothers are advised to drink plenty of water and avoid
excessive alcohol consumption.

...........................................................................................................................................

...........................................................................................................................................

...........................................................................................................................................

...........................................................................................................................................

..................................................................................................................................... [2]

[Total: 17]

© UCLES 2020 0610/41/O/N/20 [Turn over


Page 303 of 877
16

6 Sensitivity is one of the characteristics of all living organisms.

(a) Define the term sensitivity.

...................................................................................................................................................

...................................................................................................................................................

............................................................................................................................................. [2]

(b) The eye is an example of a sense organ.

(i) Define the term sense organ.

...........................................................................................................................................

...........................................................................................................................................

..................................................................................................................................... [2]

(ii) Adrenaline is a hormone that is released in ‘fight or flight’ situations. It causes a change
in the eye.

Complete Table 6.1 by stating the parts of the eye that change when adrenaline is
released into the blood.

Table 6.1

action part of the eye

muscle that relaxes

muscle that contracts

widens

[3]

[Total: 7]

Permission to reproduce items where third-party owned material protected by copyright is included has been sought and cleared where possible. Every
reasonable effort has been made by the publisher (UCLES) to trace copyright holders, but if any items requiring clearance have unwittingly been included, the
publisher will be pleased to make amends at the earliest possible opportunity.

To avoid the issue of disclosure of answer-related information to candidates, all copyright acknowledgements are reproduced online in the Cambridge
Assessment International Education Copyright Acknowledgements Booklet. This is produced for each series of examinations and is freely available to download
at www.cambridgeinternational.org after the live examination series.

Cambridge Assessment International Education is part of the Cambridge Assessment Group. Cambridge Assessment is the brand name of the University of
Cambridge Local Examinations Syndicate (UCLES), which itself is a department of the University of Cambridge.

© UCLES 2020 0610/41/O/N/20

Page 304 of 877


Cambridge IGCSE™
* 4 5 4 7 9 8 8 2 3 2 *

BIOLOGY 0610/42
Paper 4 Theory (Extended) October/November 2020

1 hour 15 minutes

You must answer on the question paper.

No additional materials are needed.

INSTRUCTIONS
● Answer all questions.
● Use a black or dark blue pen. You may use an HB pencil for any diagrams or graphs.
● Write your name, centre number and candidate number in the boxes at the top of the page.
● Write your answer to each question in the space provided.
● Do not use an erasable pen or correction fluid.
● Do not write on any bar codes.
● You may use a calculator.
● You should show all your working and use appropriate units.

INFORMATION
● The total mark for this paper is 80.
● The number of marks for each question or part question is shown in brackets [ ].

This document has 20 pages. Blank pages are indicated.

DC (JC/CT) 187679/5
© UCLES 2020 [Turn over

Page 305 of 877


2

1 (a) Sensitivity is one of the characteristics of life. The eye is one of the major sense organs of
vertebrates.

(i) Define the term sensitivity.

...........................................................................................................................................

...........................................................................................................................................

...........................................................................................................................................

..................................................................................................................................... [2]

(ii) Define the term sense organ.

...........................................................................................................................................

...........................................................................................................................................

...........................................................................................................................................

..................................................................................................................................... [2]

(b) Accommodation (focusing) is one of the functions of the eye.

Fig. 1.1 is a diagram of an eye that is focusing on a distant object.

retina
ciliary muscle

suspensory ligaments

Fig. 1.1

(i) State the name of the part of the retina labelled X.

..................................................................................................................................... [1]

© UCLES 2020 0610/42/O/N/20

Page 306 of 877


3

Fig. 1.2 is an incomplete diagram of an eye that is focusing on a near object.

Fig. 1.2

(ii) Complete Fig. 1.2 by drawing the shape of the lens and the light rays from the object to
the retina. [3]

(iii) Describe the roles of the ciliary muscles and suspensory ligaments in focusing on a
distant object, as shown in Fig. 1.1.

...........................................................................................................................................

...........................................................................................................................................

...........................................................................................................................................

...........................................................................................................................................

...........................................................................................................................................

...........................................................................................................................................

...........................................................................................................................................

..................................................................................................................................... [3]

© UCLES 2020 0610/42/O/N/20 [Turn over


Page 307 of 877
4

(c) The eye also controls the amount of light that enters the pupil.

Fig. 1.3 shows an eye in low light and in bright light.

eye in low light eye in bright light

Fig. 1.3

Describe the changes that occur in the eye when the light becomes bright, as shown in
Fig. 1.3.

...................................................................................................................................................

...................................................................................................................................................

...................................................................................................................................................

...................................................................................................................................................

...................................................................................................................................................

...................................................................................................................................................

...................................................................................................................................................

............................................................................................................................................. [3]

(d) The change shown in Fig. 1.3 occurs automatically without thought.

State the name given to this type of action.

............................................................................................................................................. [1]

[Total: 15]

© UCLES 2020 0610/42/O/N/20

Page 308 of 877


5

2 Fig. 2.1 is a vertical section of a human molar tooth and surrounding structures.

B
C

Fig. 2.1

(a) State the names of the parts labelled A to D on Fig. 2.1.

A ...............................................................................................................................................

B ...............................................................................................................................................

C ...............................................................................................................................................

D ...............................................................................................................................................
[4]

(b) Describe and explain the function of molar teeth.

...................................................................................................................................................

...................................................................................................................................................

...................................................................................................................................................

...................................................................................................................................................

...................................................................................................................................................

...................................................................................................................................................

............................................................................................................................................. [3]

© UCLES 2020 0610/42/O/N/20 [Turn over


Page 309 of 877
6

(c) Fig. 2.2 is an X-ray of decay in a molar tooth.

decayed molar tooth

Fig. 2.2

Explain how tooth decay occurs.

...................................................................................................................................................

...................................................................................................................................................

...................................................................................................................................................

...................................................................................................................................................

...................................................................................................................................................

...................................................................................................................................................

...................................................................................................................................................

............................................................................................................................................. [4]

[Total: 11]

© UCLES 2020 0610/42/O/N/20

Page 310 of 877


7

3 Fig. 3.1 is a food web for a forested area in Central America.

great horned owl

jaguar
bobcat

long-tailed weasel nine-banded armadillo


hooded skunk

red harvester ants termites


collared peccary
lowland tapir

muhly grass Peruvian feather


grass

Fig. 3.1

(a) Complete Table 3.1 using information from Fig. 3.1.

Table 3.1

trophic level description example from Fig. 3.1

herbivore

producer

feeds on tertiary consumers

secondary consumer

[4]

© UCLES 2020 0610/42/O/N/20 [Turn over


Page 311 of 877
8

(b) Fig. 3.2 shows the flow of energy through a food chain. The size of each box represents the
energy available in each trophic level, numbered 1, 2, 3 and 4.

solar
energy

respiration trophic level 1

trophic
level 2

respiration trophic
level 3

Z respiration
trophic
level 4

Fig. 3.2

(i) State the term given to the group of organisms represented by Z in Fig. 3.2.

..................................................................................................................................... [1]

(ii) Explain, with reference to Fig. 3.2, why food chains usually have fewer than five trophic
levels.

...........................................................................................................................................

...........................................................................................................................................

...........................................................................................................................................

...........................................................................................................................................

...........................................................................................................................................

...........................................................................................................................................

...........................................................................................................................................

...........................................................................................................................................

..................................................................................................................................... [4]

© UCLES 2020 0610/42/O/N/20

Page 312 of 877


9

(c) Intensive livestock production can be damaging to natural ecosystems.

Fig. 3.3 shows intensive farming of chickens.

Fig. 3.3

(i) Describe the negative impact on an ecosystem of the intensive production of livestock,
such as chickens.

...........................................................................................................................................

...........................................................................................................................................

...........................................................................................................................................

...........................................................................................................................................

...........................................................................................................................................

...........................................................................................................................................

...........................................................................................................................................

..................................................................................................................................... [3]

© UCLES 2020 0610/42/O/N/20 [Turn over


Page 313 of 877
10

(ii) Forests are cleared for farmland. Deforestation can lead to a loss of soil (soil erosion).

Explain the effects of soil erosion on ecosystems.

...........................................................................................................................................

...........................................................................................................................................

...........................................................................................................................................

...........................................................................................................................................

...........................................................................................................................................

...........................................................................................................................................

..................................................................................................................................... [3]

[Total: 15]

© UCLES 2020 0610/42/O/N/20

Page 314 of 877


11

4 (a) Yeast cells have many structures in common with a plant cell.

Fig. 4.1 is a drawing of a yeast cell.

A B

cell wall

Fig. 4.1

(i) State the names of the cell structures labelled A and D on Fig. 4.1.

A ........................................................................................................................................

D ........................................................................................................................................
[2]

(ii) State the functions of the cell structures labelled B and C on Fig. 4.1.

B ........................................................................................................................................

C ........................................................................................................................................
[2]

(iii) State the name of one structure that is found in plant cells but is absent in yeast cells.

..................................................................................................................................... [1]

© UCLES 2020 0610/42/O/N/20 [Turn over


Page 315 of 877
12

(b) Yeast is used in the production of ethanol to manufacture a type of biofuel.

Fig. 4.2 is a flow chart of the process.

maize plants

extraction of starch from


maize

enzyme S

maltose

enzyme T

glucose
process U in anaerobic
conditions

ethanol carbon dioxide

Fig. 4.2

(i) State the names of enzymes S and T.

S ........................................................................................................................................

T ........................................................................................................................................
[2]

(ii) Yeast is used in process U. Complete the balanced chemical equation for anaerobic
respiration in yeast.

................................. ...........C2H5OH + ..................


[2]

(iii) Suggest the advantages of using biofuels instead of fossil fuels.

...........................................................................................................................................

...........................................................................................................................................

...........................................................................................................................................

...........................................................................................................................................

...........................................................................................................................................

...........................................................................................................................................

..................................................................................................................................... [3]

© UCLES 2020 0610/42/O/N/20

Page 316 of 877


13

(iv) Carbon dioxide may be collected from process U and sold for use in glasshouses.

Explain why carbon dioxide is used in glasshouses.

...........................................................................................................................................

...........................................................................................................................................

...........................................................................................................................................

...........................................................................................................................................

...........................................................................................................................................

...........................................................................................................................................

...........................................................................................................................................

..................................................................................................................................... [4]

[Total: 16]

© UCLES 2020 0610/42/O/N/20 [Turn over


Page 317 of 877
14

5 (a) The activities of the ovaries and the uterus are regulated by the hormones FSH, LH, oestrogen
and progesterone during the menstrual cycle.

Complete Table 5.1 to show the sites of production and the roles of these four hormones.

Table 5.1

target
hormone site of production role
organ

FSH pituitary gland ovary

LH pituitary gland ovary

oestrogen uterus stimulates growth of the lining of the uterus

progesterone uterus

[5]

© UCLES 2020 0610/42/O/N/20

Page 318 of 877


15

(b) Fig. 5.1 shows the changes in the lining of the uterus of a human female.

thickness of
the lining of
the uterus

0 7 14 21 28 7 14
day of menstrual cycle

Fig. 5.1

Describe the changes in the lining of the uterus between days 0 and 28 of the menstrual
cycle.

between days 0 and 7 ..............................................................................................................

...................................................................................................................................................

...................................................................................................................................................

...................................................................................................................................................

between days 7 and 28 ............................................................................................................

...................................................................................................................................................

...................................................................................................................................................

...................................................................................................................................................

...................................................................................................................................................
[3]

© UCLES 2020 0610/42/O/N/20 [Turn over


Page 319 of 877
16

(c) Some people are infertile.

Artificial insemination (AI) and in vitro fertilisation (IVF) are two methods of fertility treatment.

These two methods are outlined in Fig. 5.2.

artificial insemination

collection of
sperm

washed
sperm

in vitro fertilisation
collection of
eggs
collection of
sperm
washed
sperm

eggs and sperm mixed together

embryo

transfer of growth of embryos in freezing of


embryos culture dish spare embryos

not to scale

Fig. 5.2

© UCLES 2020 0610/42/O/N/20

Page 320 of 877


17

Describe the similarities and differences between the two processes of fertility treatment
shown in Fig. 5.2.

...................................................................................................................................................

...................................................................................................................................................

...................................................................................................................................................

...................................................................................................................................................

...................................................................................................................................................

...................................................................................................................................................

...................................................................................................................................................

...................................................................................................................................................

...................................................................................................................................................

...................................................................................................................................................

...................................................................................................................................................

...................................................................................................................................................

............................................................................................................................................. [6]

[Total: 14]

© UCLES 2020 0610/42/O/N/20 [Turn over


Page 321 of 877
18

6 Fig. 6.1 is a diagram of DNA.

Key:
base pair
base A

Fig. 6.1

(a) (i) State the letter of the base that pairs with A.

..................................................................................................................................... [1]

(ii) State the letters of the other bases in DNA.

..................................................................................................................................... [1]

(b) Outline the roles of DNA in a cell.

...................................................................................................................................................

...................................................................................................................................................

...................................................................................................................................................

...................................................................................................................................................

............................................................................................................................................. [2]

© UCLES 2020 0610/42/O/N/20

Page 322 of 877


19

(c) Fig. 6.2 shows a plant tissue in which cells are dividing by mitosis.

J
H

Fig. 6.2

(i) Cell H in Fig. 6.2 is about to divide by mitosis.

State what happens to the chromosomes in cell H before mitosis takes place and state
why it is necessary.

...........................................................................................................................................

...........................................................................................................................................

...........................................................................................................................................

...........................................................................................................................................

..................................................................................................................................... [2]

(ii) Cell K is about to divide into two cells.

State the structures that will form between the nuclei so that the cell divides into two
cells.

...........................................................................................................................................

..................................................................................................................................... [2]

© UCLES 2020 0610/42/O/N/20 [Turn over


Page 323 of 877
20

(iii) Cell J in Fig. 6.2 is an example of a diploid cell.

State what is meant by the term diploid.

...........................................................................................................................................

...........................................................................................................................................

..................................................................................................................................... [1]

[Total: 9]

Permission to reproduce items where third-party owned material protected by copyright is included has been sought and cleared where possible. Every
reasonable effort has been made by the publisher (UCLES) to trace copyright holders, but if any items requiring clearance have unwittingly been included, the
publisher will be pleased to make amends at the earliest possible opportunity.

To avoid the issue of disclosure of answer-related information to candidates, all copyright acknowledgements are reproduced online in the Cambridge
Assessment International Education Copyright Acknowledgements Booklet. This is produced for each series of examinations and is freely available to download
at www.cambridgeinternational.org after the live examination series.

Cambridge Assessment International Education is part of the Cambridge Assessment Group. Cambridge Assessment is the brand name of the University of
Cambridge Local Examinations Syndicate (UCLES), which itself is a department of the University of Cambridge.

© UCLES 2020 0610/42/O/N/20

Page 324 of 877


Cambridge IGCSE™
* 1 4 2 8 9 9 0 7 7 1 *

BIOLOGY 0610/43
Paper 4 Theory (Extended) October/November 2020

1 hour 15 minutes

You must answer on the question paper.

No additional materials are needed.

INSTRUCTIONS
● Answer all questions.
● Use a black or dark blue pen. You may use an HB pencil for any diagrams or graphs.
● Write your name, centre number and candidate number in the boxes at the top of the page.
● Write your answer to each question in the space provided.
● Do not use an erasable pen or correction fluid.
● Do not write on any bar codes.
● You may use a calculator.
● You should show all your working and use appropriate units.

INFORMATION
● The total mark for this paper is 80.
● The number of marks for each question or part question is shown in brackets [ ].

This document has 16 pages. Blank pages are indicated.

DC (JC/CT) 188259/4
© UCLES 2020 [Turn over

Page 325 of 877


2

1 Plants are sensitive to light.

(a) Fig. 1.1 shows a sequence of diagrams for two young plant seedlings. The seedlings were
first grown in pots with light from all directions and then placed into boxes.

• Seedling A was placed in a box with light entering from one side only.
• Seedling B was placed in a box with no light.

seedling

soil
water
A A

B B

Fig. 1.1

Complete Fig. 1.1 by drawing seedling A and seedling B, in the boxes, to show the expected
direction of their growth after one week. [2]

© UCLES 2020 0610/43/O/N/20

Page 326 of 877


3

(b) Plants are also sensitive to gravity.

(i) Define the term sensitivity.

...........................................................................................................................................

...........................................................................................................................................

..................................................................................................................................... [2]

(ii) Explain the role of plant hormones in gravitropism.

...........................................................................................................................................

...........................................................................................................................................

...........................................................................................................................................

...........................................................................................................................................

...........................................................................................................................................

...........................................................................................................................................

...........................................................................................................................................

...........................................................................................................................................

..................................................................................................................................... [4]

(iii) Gravitropism enables plants to survive in their environment.

Explain how gravitropism enables a plant to survive.

...........................................................................................................................................

...........................................................................................................................................

...........................................................................................................................................

...........................................................................................................................................

..................................................................................................................................... [2]

© UCLES 2020 0610/43/O/N/20 [Turn over


Page 327 of 877
4

(c) Reflexes in animals are also an example of sensitivity.

(i) Describe how reflexes in animals differ from sensitivity in plants.

...........................................................................................................................................

...........................................................................................................................................

...........................................................................................................................................

...........................................................................................................................................

..................................................................................................................................... [2]

(ii) The pupil reflex is an example of a reflex that occurs in the eye. The eye is a sense
organ.

Define the term sense organ.

...........................................................................................................................................

...........................................................................................................................................

...........................................................................................................................................

...........................................................................................................................................

..................................................................................................................................... [2]

[Total: 14]

© UCLES 2020 0610/43/O/N/20

Page 328 of 877


5

2 (a) A person ate a meal containing starch and fat.

Fig. 2.1 shows some events that occurred after ingesting this meal.

P absorption of nutrients in the villi

Q assimilation of fatty acids in the liver

R breakdown of large food particles by the teeth

S movement of small food particles through the oesophagus

T secretion of amylase from the salivary glands

Fig. 2.1

Put the events in Fig. 2.1 into the correct sequence. One has been done for you.

R
[2]

(b) The stomach lining contains cells that secrete proteins.

(i) State the names of two cell structures that are involved in making or secreting proteins.

1 ........................................................................................................................................

2 ........................................................................................................................................
[2]

(ii) State the name of one component of gastric juice and describe its functions.

...........................................................................................................................................

...........................................................................................................................................

...........................................................................................................................................

...........................................................................................................................................

...........................................................................................................................................

...........................................................................................................................................

...........................................................................................................................................
[3]

© UCLES 2020 0610/43/O/N/20 [Turn over


Page 329 of 877
6

(iii) There are goblet cells in many parts of the alimentary canal.

State the substance that goblet cells secrete.

..................................................................................................................................... [1]

(c) Emulsification of fats occurs in the alimentary canal.

(i) Describe the process of emulsification of fats.

...........................................................................................................................................

...........................................................................................................................................

...........................................................................................................................................

...........................................................................................................................................

..................................................................................................................................... [2]

(ii) State the name of the organ in the alimentary canal where fats are emulsified.

..................................................................................................................................... [1]

(iii) State the name of the structure in the villi where digested fats are absorbed.

..................................................................................................................................... [1]

[Total: 12]

3 (a) Fig. 3.1 is part of a food web in a rainforest.

anaconda
jaguar

iguana frog

monkey grasshopper fruit bat

coconut tree banana plant

Fig. 3.1

© UCLES 2020 0610/43/O/N/20

Page 330 of 877


7

(i) Complete Fig. 3.2 using the information in Fig. 3.1.

The first one has been done for you.

the number of different producers 2

the number of different secondary consumers

the number of different trophic levels in the food web

how many different trophic levels monkeys feed at

Fig. 3.2
[3]

(ii) Sometimes anacondas are able to kill and eat jaguars.

Using the information in Fig. 3.1, state the name of the highest possible trophic level of
an anaconda that eats a jaguar.

..................................................................................................................................... [1]

(iii) Explain why the population of anacondas is always smaller than the population of fruit
bats in the food web shown in Fig. 3.1.

Use the concept of energy flow in your answer.

...........................................................................................................................................

...........................................................................................................................................

...........................................................................................................................................

...........................................................................................................................................

...........................................................................................................................................

...........................................................................................................................................

..................................................................................................................................... [3]

© UCLES 2020 0610/43/O/N/20 [Turn over


Page 331 of 877
8

(b) (i) A giant rat was discovered in a natural rainforest on Vangunu Island. Scientists wanted
to determine if it was a new species.

Explain how scientists can use tissue samples to classify this rat.

...........................................................................................................................................

...........................................................................................................................................

...........................................................................................................................................

...........................................................................................................................................

..................................................................................................................................... [2]

(ii) Giant rats eat coconuts.

Coconuts are grown as a monoculture for human consumption.

Suggest how monocultures of crop plants can result in the extinction of some animals.

...........................................................................................................................................

...........................................................................................................................................

...........................................................................................................................................

...........................................................................................................................................

...........................................................................................................................................

...........................................................................................................................................

..................................................................................................................................... [3]

[Total: 12]

© UCLES 2020 0610/43/O/N/20

Page 332 of 877


9

4 Yeast is an example of a single-celled fungus.

(a) The population of yeast can be estimated by using a light microscope to view a small sample
of yeast cells in a modified microscope slide.

There is a square chamber, with a specific volume, cut into the microscope slide and a grid to
make it easier to count the number of cells.

Fig. 4.1 shows a diagram of the modified microscope slide. It also shows part of a light
microscope with the modified microscope slide filled to the top with a sample of yeast cells.

grid

chamber

Fig. 4.1

The length of the sides of each small square in the grid in the modified microscope slide is
200 μm. The depth of the chamber is 100 μm. There is a total of 25 small squares in the grid.

Each small square in the grid has an average of 52 yeast cells.

Calculate the concentration of yeast cells per mm3.

Space for working.

......................................... cells per mm3


[3]

© UCLES 2020 0610/43/O/N/20 [Turn over


Page 333 of 877
10

(b) A small sample of yeast cells and 19 g per dm3 of sugar were added to a fermenter. The
population size of live yeast cells and the concentration of the sugar in the fermenter were
calculated every day for 10 days.

The results are shown in Fig. 4.2 and Fig. 4.3.

20

20
18

18
16

16
14

concentration 12
14
of sugar
/concentration
g per dm3 12
10
of sugar
/ g per dm3 10
8

8
6

6
4

4
2

2
0
0 1 2 3 4 5 6 7 8 9 10
0 time / days
0 1 2 3 4 5 6 7 8 9 10
Fig. 4.2time / days

population size
of live yeast cells
population size
of live yeast cells

0 1 2 3 4 5 6 7 8 9 10
time / days
0 1 2 3 4 5 6 7 8 9 10
time / days

Fig. 4.3
© UCLES 2020 0610/43/O/N/20

Page 334 of 877


11

(i) Draw an X on Fig. 4.3 to indicate the position of the lag phase. [1]

(ii) Complete the line on Fig. 4.3 to show the expected change in the population size of live
yeast cells from day 3 to day 10. [2]

(iii) Describe and explain the changes in the concentration of sugar in the fermenter over the
10-day period.

...........................................................................................................................................

...........................................................................................................................................

...........................................................................................................................................

...........................................................................................................................................

...........................................................................................................................................

...........................................................................................................................................

...........................................................................................................................................

...........................................................................................................................................

...........................................................................................................................................

...........................................................................................................................................

..................................................................................................................................... [5]

(c) Explain why it is important that there is a tube in the fermenter that allows gases to escape.

...................................................................................................................................................

...................................................................................................................................................

...................................................................................................................................................

...................................................................................................................................................

............................................................................................................................................. [2]

[Total: 13]

© UCLES 2020 0610/43/O/N/20 [Turn over


Page 335 of 877
12

5 Fig. 5.1 shows a photomicrograph of a sperm cell reaching an egg cell.

Fig. 5.1

(a) (i) Complete the sentence:

A nucleus containing a single set of unpaired chromosomes in a sperm is called a

............................................... nucleus. [1]

(ii) State where, in the female reproductive system, the event shown in Fig. 5.1 occurs.

..................................................................................................................................... [1]

(iii) Describe what happens from the event shown in Fig. 5.1 until an embryo is formed.

...........................................................................................................................................

...........................................................................................................................................

...........................................................................................................................................

...........................................................................................................................................

...........................................................................................................................................

...........................................................................................................................................

...........................................................................................................................................

...........................................................................................................................................

..................................................................................................................................... [4]

© UCLES 2020 0610/43/O/N/20

Page 336 of 877


13

(b) Some people use fertility drugs and in vitro fertilisation (IVF) to have children.

Outline the use of fertility drugs and the process of IVF.

...................................................................................................................................................

...................................................................................................................................................

...................................................................................................................................................

...................................................................................................................................................

...................................................................................................................................................

...................................................................................................................................................

...................................................................................................................................................

...................................................................................................................................................

...................................................................................................................................................

...................................................................................................................................................

...................................................................................................................................................

...................................................................................................................................................

............................................................................................................................................. [6]

[Total: 12]

© UCLES 2020 0610/43/O/N/20 [Turn over


Page 337 of 877
14

6 (a) Researchers investigated four different insecticides to determine how effective they were at
removing three different species of insects, A, B and C, from crop plants.

They tested different concentrations of each insecticide to find the minimum dosage required
to remove 95% of each insect population in two hours.

Each insecticide had the same cost per gram.

The results are shown in Table 6.1.

Table 6.1

minimum dosage required


species of / mg per dm3
insect
insecticide 1 insecticide 2 insecticide 3 insecticide 4
A 3 51 5 58
B 31 27 2 75
C 10 2 3 65

(i) Use the information in Table 6.1 to decide which one of the four different insecticides and
which dosage would be the best choice for removing insects A, B and C from a field of
crop plants.

Explain your choices.

insecticide ............................................... dosage .............................................................

explanation ........................................................................................................................

...........................................................................................................................................

...........................................................................................................................................

...........................................................................................................................................

...........................................................................................................................................

...........................................................................................................................................
[3]

© UCLES 2020 0610/43/O/N/20

Page 338 of 877


15

(ii) Discuss factors, other than dosage, that should be considered to prevent the insecticide
damaging the environment.

...........................................................................................................................................

...........................................................................................................................................

...........................................................................................................................................

...........................................................................................................................................

...........................................................................................................................................

...........................................................................................................................................

..................................................................................................................................... [3]

(b) (i) Describe the advantages to farmers of using insecticides.

...........................................................................................................................................

...........................................................................................................................................

...........................................................................................................................................

...........................................................................................................................................

..................................................................................................................................... [2]

(ii) Farmers also use other chemicals on their crop plants.

State the names of chemicals, other than insecticides, that are used by farmers on crop
plants and explain their benefits.

...........................................................................................................................................

...........................................................................................................................................

...........................................................................................................................................

...........................................................................................................................................

...........................................................................................................................................

...........................................................................................................................................

..................................................................................................................................... [3]

© UCLES 2020 0610/43/O/N/20 [Turn over


Page 339 of 877
16

(c) Insecticides are often made from chemicals that occur naturally in plants. Tobacco plants are
one example of this.

(i) Suggest why having a natural insecticide is an adaptive feature of plants.

...........................................................................................................................................

...........................................................................................................................................

..................................................................................................................................... [1]

(ii) Describe why people find it difficult to stop smoking tobacco.

...........................................................................................................................................

...........................................................................................................................................

...........................................................................................................................................

...........................................................................................................................................

..................................................................................................................................... [2]

(iii) State the names of two diseases that can be caused by smoking tobacco.

1 ........................................................................................................................................

2 ........................................................................................................................................
[2]

(iv) State the name of the component of tobacco that can damage alveoli.

..................................................................................................................................... [1]

[Total: 17]

Permission to reproduce items where third-party owned material protected by copyright is included has been sought and cleared where possible. Every
reasonable effort has been made by the publisher (UCLES) to trace copyright holders, but if any items requiring clearance have unwittingly been included, the
publisher will be pleased to make amends at the earliest possible opportunity.

To avoid the issue of disclosure of answer-related information to candidates, all copyright acknowledgements are reproduced online in the Cambridge
Assessment International Education Copyright Acknowledgements Booklet. This is produced for each series of examinations and is freely available to download
at www.cambridgeinternational.org after the live examination series.

Cambridge Assessment International Education is part of the Cambridge Assessment Group. Cambridge Assessment is the brand name of the University of
Cambridge Local Examinations Syndicate (UCLES), which itself is a department of the University of Cambridge.

© UCLES 2020 0610/43/O/N/20

Page 340 of 877


Cambridge IGCSE™
* 3 0 7 6 1 6 8 2 5 0 *

BIOLOGY 0610/42
Paper 4 Theory (Extended) February/March 2020

1 hour 15 minutes

You must answer on the question paper.

No additional materials are needed.

INSTRUCTIONS
● Answer all questions.
● Use a black or dark blue pen. You may use an HB pencil for any diagrams or graphs.
● Write your name, centre number and candidate number in the boxes at the top of the page.
● Write your answer to each question in the space provided.
● Do not use an erasable pen or correction fluid.
● Do not write on any bar codes.
● You may use a calculator.
● You should show all your working and use appropriate units.

INFORMATION
● The total mark for this paper is 80.
● The number of marks for each question or part question is shown in brackets [ ].

This document has 20 pages. Blank pages are indicated.

DC (JC/CT) 180480/4
© UCLES 2020 [Turn over

Page 341 of 877


2

1 (a) Fig. 1.1 is a diagram of the human gas exchange system.

Fig. 1.1

(i) Identify the structures labelled A, B and C in Fig. 1.1.

A ........................................................................................................................................

B ........................................................................................................................................

C ........................................................................................................................................
[3]

(ii) Explain how the structures in the gas exchange system cause inspiration.

...........................................................................................................................................

...........................................................................................................................................

...........................................................................................................................................

...........................................................................................................................................

...........................................................................................................................................

...........................................................................................................................................

...........................................................................................................................................

...........................................................................................................................................

..................................................................................................................................... [4]

© UCLES 2020 0610/42/F/M/20

Page 342 of 877


3

(b) A person who does not smoke can be exposed to tobacco smoke from other people smoking.

Researchers studied the effect of exposure to tobacco smoke on the development of lung
cancer in three groups of women who did not smoke:

• group 1 – no exposure to tobacco smoke


• group 2 – low level exposure to tobacco smoke
• group 3 – high level exposure to tobacco smoke.

Their results are shown in Table 1.1.

Table 1.1

number of women percentage of


number of women
group who died from women who died
studied
lung cancer from lung cancer
1 21 895 32 0.15
2 44 184 86
3 25 461 56 0.22

(i) Calculate the percentage of women in group 2 who died from lung cancer.

Write your answer, to two significant figures, in Table 1.1.

[2]

(ii) Many countries have laws that ban smoking in public buildings.

Discuss the evidence from Table 1.1 that supports these laws.

...........................................................................................................................................

...........................................................................................................................................

...........................................................................................................................................

...........................................................................................................................................

...........................................................................................................................................

...........................................................................................................................................

..................................................................................................................................... [3]

© UCLES 2020 0610/42/F/M/20 [Turn over


Page 343 of 877
4

(iii) Smoking has been found to increase the risk of developing diseases other than cancer.

State two other diseases that can be caused by smoking.

1 ........................................................................................................................................

2 ........................................................................................................................................
[2]

[Total: 14]

© UCLES 2020 0610/42/F/M/20

Page 344 of 877


5

2 (a) Fig. 2.1 shows the transfer of materials between blood and tissues.

lymphatic
fluid A
vessel
fluid B

arteriole venule

Key:
flow of blood
transfer of materials

Fig. 2.1

(i) Complete Table 2.1 by:

• stating the names of the fluids


• writing yes if the fluid contains red blood cells or no if the fluid does not contain
red blood cells.

Table 2.1

letter on contains red


name of the fluid
Fig. 2.1 blood cells

[2]

(ii) State the name of the process by which oxygen is transferred from fluid B to the cells.

..................................................................................................................................... [1]

(iii) Explain why cells need oxygen.

...........................................................................................................................................

...........................................................................................................................................

...........................................................................................................................................

...........................................................................................................................................

..................................................................................................................................... [2]

© UCLES 2020 0610/42/F/M/20 [Turn over


Page 345 of 877
6

(b) Describe the functions of arterioles in the skin.

...................................................................................................................................................

...................................................................................................................................................

...................................................................................................................................................

...................................................................................................................................................

...................................................................................................................................................

...................................................................................................................................................

............................................................................................................................................. [3]

(c) Describe the functions of lymph nodes in the lymphatic system.

...................................................................................................................................................

...................................................................................................................................................

...................................................................................................................................................

...................................................................................................................................................

............................................................................................................................................. [2]

(d) Lacteals are part of the lymphatic system.

(i) State where in the body lacteals are found.

..................................................................................................................................... [1]

(ii) Describe the role of lacteals.

...........................................................................................................................................

...........................................................................................................................................

..................................................................................................................................... [1]

[Total: 12]

© UCLES 2020 0610/42/F/M/20

Page 346 of 877


7

3 (a) One of the characteristics of living organisms is sensitivity.

Define the term sensitivity.

...................................................................................................................................................

...................................................................................................................................................

...................................................................................................................................................

...................................................................................................................................................

............................................................................................................................................. [2]

(b) State the names of two sense organs.

1 ................................................................................................................................................

2 ................................................................................................................................................
[1]

© UCLES 2020 0610/42/F/M/20 [Turn over


Page 347 of 877
8

(c) Scientists investigated the effect of adrenaline on blood glucose concentration in rats.

The rats were put into two groups:

• group A was given an injection of adrenaline


• group B was given an injection that did not contain adrenaline.

The blood glucose concentrations of the rats in both groups were monitored for three hours
after the injections.

The rats did not eat for 12 hours before the investigation or while they were being monitored.

The results are shown in Fig. 3.1.

5.0
time of
injection

4.5

blood glucose
concentration
/ mmol per dm3

4.0

3.5
0.0 0.5 1.0 1.5 2.0 2.5 3.0 3.5
monitoring time / hours
Key:
group A (adrenaline)
group B (no adrenaline)

Fig. 3.1

© UCLES 2020 0610/42/F/M/20

Page 348 of 877


9

(i) Suggest why group B was given an injection that did not contain adrenaline.

...........................................................................................................................................

...........................................................................................................................................

...........................................................................................................................................

...........................................................................................................................................

..................................................................................................................................... [2]

(ii) Describe and explain the results shown in Fig. 3.1 for group A.

...........................................................................................................................................

...........................................................................................................................................

...........................................................................................................................................

...........................................................................................................................................

...........................................................................................................................................

...........................................................................................................................................

...........................................................................................................................................

...........................................................................................................................................

...........................................................................................................................................

...........................................................................................................................................

..................................................................................................................................... [5]

(d) Another group of rats was given an injection that did not contain adrenaline.

These rats were given food after 2 hours of monitoring.

Predict the changes to blood glucose concentration in this group of rats.

Sketch a line to show your prediction on the graph in Fig. 3.1. [2]

(e) Describe two effects of adrenaline on the body, other than a change in blood glucose
concentration.

1 ................................................................................................................................................

2 ................................................................................................................................................
[2]

[Total: 14]

© UCLES 2020 0610/42/F/M/20 [Turn over


Page 349 of 877
10

4 (a) Describe the similarities and differences between marasmus and kwashiorkor.

similarities .................................................................................................................................

...................................................................................................................................................

...................................................................................................................................................

...................................................................................................................................................

...................................................................................................................................................

differences ................................................................................................................................

...................................................................................................................................................

...................................................................................................................................................

...................................................................................................................................................

...................................................................................................................................................
[4]

(b) Researchers at a hospital recorded the total number of children admitted to the hospital
between 1984 and 2002.

A common reason for children being admitted to this hospital was severe diarrhoea.

Table 4.1 shows this information.

Table 4.1

total number of
year children admitted to
the hospital
1984 1386
1986 1604
1988 1955
1990 2054
1992 1726
1994 1143
1996 1422
1998 1419
2000 1580
2002 1161

© UCLES 2020 0610/42/F/M/20

Page 350 of 877


11

(i) Calculate the percentage decrease in the total number of children admitted to the hospital
between 1998 and 2002.

Give your answer to the nearest whole number.

Space for working.

............................................................ %
[3]

(ii) Health workers in the communities near the hospital were trained in the prevention and
treatment of diarrhoea. This affected the total number of children being admitted to the
hospital.

Suggest the year in which the training took place.

Give a reason for your answer.

year ...................................................................................................................................

reason ...............................................................................................................................

...........................................................................................................................................

...........................................................................................................................................
[2]

(iii) The health workers provided advice to the community on ways of preventing the spread
of the pathogens that cause diarrhoea.

Suggest the advice that was given to the community.

...........................................................................................................................................

...........................................................................................................................................

...........................................................................................................................................

...........................................................................................................................................

...........................................................................................................................................

...........................................................................................................................................

..................................................................................................................................... [3]

[Total: 12]

© UCLES 2020 0610/42/F/M/20 [Turn over


Page 351 of 877
12

5 (a) Mitosis is a type of nuclear division.

Fig. 5.1 is a series of photomicrographs showing a cell dividing by mitosis.

before mitosis 0 minutes 34 minutes

40 minutes 45 minutes 55 minutes

Fig. 5.1

(i) State the change that has occurred to the mass of DNA immediately before mitosis in
Fig. 5.1.

..................................................................................................................................... [1]

(ii) Estimate the time when the chromosomes shown in Fig. 5.1 begin to separate.

..................................................................................................................................... [1]

© UCLES 2020 0610/42/F/M/20

Page 352 of 877


13

(b) Meiosis is another type of nuclear division.

Describe how the nuclei in cells produced by meiosis differ from the nuclei in cells produced
by mitosis.

...................................................................................................................................................

...................................................................................................................................................

...................................................................................................................................................

...................................................................................................................................................

...................................................................................................................................................

...................................................................................................................................................

............................................................................................................................................. [3]

(c) Stem cells divide by mitosis during the growth of an embryo.

Describe the role of stem cells in the growth of an embryo.

...................................................................................................................................................

...................................................................................................................................................

...................................................................................................................................................

...................................................................................................................................................

............................................................................................................................................. [2]

© UCLES 2020 0610/42/F/M/20 [Turn over


Page 353 of 877
14

(d) An embryo develops into a fetus.

Outline the processes of labour and birth.

...................................................................................................................................................

...................................................................................................................................................

...................................................................................................................................................

...................................................................................................................................................

...................................................................................................................................................

...................................................................................................................................................

...................................................................................................................................................

...................................................................................................................................................

...................................................................................................................................................

...................................................................................................................................................

...................................................................................................................................................

...................................................................................................................................................

............................................................................................................................................. [6]

[Total: 13]

© UCLES 2020 0610/42/F/M/20

Page 354 of 877


15

6 (a) Atlantic cod, Gadhus morhua, is a type of fish that is an important resource for commercial
fishing.

Fig. 6.1 shows the estimated mass of Atlantic cod over 40 years.

6
mass of
Atlantic 5
cod
/ million 4
tonnes
3

0
1970 1975 1980 1985 1990 1995 2000 2005 2010
year

Fig. 6.1

(i) In 1970, the mass of Atlantic cod was 8 000 000 tonnes.

State one year when the mass of Atlantic cod was half this value.

..................................................................................................................................... [1]

(ii) State the years when there was a continuous increase in the mass of Atlantic cod for at
least five years.

..................................................................................................................................... [1]

(iii) Suggest reasons for the trend shown between 1990 and 1995.

...........................................................................................................................................

...........................................................................................................................................

...........................................................................................................................................

...........................................................................................................................................

...........................................................................................................................................

...........................................................................................................................................

..................................................................................................................................... [3]
© UCLES 2020 0610/42/F/M/20 [Turn over
Page 355 of 877
16

(iv) Explain how fish stocks can be conserved by restocking.

...........................................................................................................................................

...........................................................................................................................................

...........................................................................................................................................

...........................................................................................................................................

...........................................................................................................................................

...........................................................................................................................................

..................................................................................................................................... [3]

(b) Fish have adaptive features that enable them to live successfully in their environment.

Fig. 6.2 is a photograph of a great white shark, Carcharodon carcharias.

Great white sharks are efficient predators and have very good eyesight to see in poor light
conditions underwater.

Fig. 6.2

© UCLES 2020 0610/42/F/M/20

Page 356 of 877


17

(i) Describe two features, other than eyesight, visible in Fig. 6.2 that suggest that great
white sharks are efficient predators.

1 ........................................................................................................................................

2 ........................................................................................................................................
[2]

(ii) Describe how the ancestors of the great white shark developed adaptive features such
as good eyesight.

...........................................................................................................................................

...........................................................................................................................................

...........................................................................................................................................

...........................................................................................................................................

...........................................................................................................................................

...........................................................................................................................................

...........................................................................................................................................

...........................................................................................................................................

...........................................................................................................................................

...........................................................................................................................................

..................................................................................................................................... [5]

[Total: 15]

© UCLES 2020 0610/42/F/M/20

Page 357 of 877


Cambridge Assessment International Education
Cambridge International General Certificate of Secondary Education
* 5 9 1 3 2 7 4 2 5 9 *

BIOLOGY 0610/41
Paper 4 Theory (Extended) May/June 2019
1 hour 15 minutes
Candidates answer on the Question Paper.
No Additional Materials are required.

READ THESE INSTRUCTIONS FIRST

Write your centre number, candidate number and name on all the work you hand in.
Write in dark blue or black pen.
You may use an HB pencil for any diagrams or graphs.
Do not use staples, paper clips, glue or correction fluid.
DO NOT WRITE IN ANY BARCODES.

Answer all questions.

Electronic calculators may be used.


You may lose marks if you do not show your working or if you do not use appropriate units.

At the end of the examination, fasten all your work securely together.
The number of marks is given in brackets [ ] at the end of each question or part question.

This syllabus is regulated for use in England, Wales and Northern Ireland as a Cambridge International Level 1/Level 2 Certificate.

This document consists of 18 printed pages and 2 blank pages.

DC (ST/CT) 162740/4
© UCLES 2019 [Turn over

Page 361 of 877


3

1 All commercial breeds of sheep belong to the species Ovis aries.

(a) Define the term species.

...................................................................................................................................................

...................................................................................................................................................

...................................................................................................................................................

...................................................................................................................................................

............................................................................................................................................. [2]

The Merino is a breed of sheep that is farmed mainly for its wool. The wool is very thick and is
made of lots of very thin hairs.

Fig. 1.1 shows a female Merino sheep with her newborn lamb.

Fig. 1.1

(b) The presence of hair is a feature that is only found in mammals.

State two other features that distinguish mammals from all other vertebrates.

1 ................................................................................................................................................

2 ................................................................................................................................................
[2]

© UCLES 2019 0610/41/M/J/19 [Turn over


Page 363 of 877
4

(c) Merino sheep in South Africa have high quality wool with very thin hairs.

Breeders in New Zealand have used selective breeding programmes to improve the wool of
their sheep to match the quality of South African wool.

Describe the steps that breeders would take to breed sheep that have wool with very thin
hairs.

...................................................................................................................................................

...................................................................................................................................................

...................................................................................................................................................

...................................................................................................................................................

...................................................................................................................................................

...................................................................................................................................................

...................................................................................................................................................

...................................................................................................................................................

...................................................................................................................................................

...................................................................................................................................................

...................................................................................................................................................

............................................................................................................................................. [5]

(d) Explain how natural selection differs from selective breeding.

...................................................................................................................................................

...................................................................................................................................................

...................................................................................................................................................

...................................................................................................................................................

...................................................................................................................................................

...................................................................................................................................................

............................................................................................................................................. [3]

[Total: 12]

© UCLES 2019 0610/41/M/J/19

Page 364 of 877


5

2 The rate of photosynthesis of terrestrial plants can be determined by measuring the uptake of
carbon dioxide.

(a) Explain why plants take up carbon dioxide during photosynthesis.

...................................................................................................................................................

...................................................................................................................................................

...................................................................................................................................................

...................................................................................................................................................

............................................................................................................................................. [2]

(b) The rate of photosynthesis of parts of individual leaves can be measured using a hand-held
device as shown in Fig. 2.1.

transparent chamber
Fig. 2.1

This apparatus allows air to flow through the transparent chamber that encloses part of the
leaf. The apparatus measures the carbon dioxide concentration of the air entering and leaving
the chamber.

Explain how the results from the apparatus can be used to calculate the rate of photosynthesis.

...................................................................................................................................................

...................................................................................................................................................

...................................................................................................................................................

............................................................................................................................................. [2]

© UCLES 2019 0610/41/M/J/19 [Turn over


Page 365 of 877
6

(c) A student used the apparatus shown in Fig. 2.1 to investigate the effect of temperature on the
rate of photosynthesis of the leaves of Chinese plantain, Plantago asiatica, at two different
concentrations of carbon dioxide, A and B.

Fig. 2.2 shows the results of the investigation.

40

35
B
1000 ppm CO2

30

25
rate of photosynthesis
/ μmol per m2 per s A
370 ppm CO2
20

15

10

0
10 20 30 40

temperature of leaves / °C

Fig. 2.2

(i) State one environmental factor that should have been kept constant in this investigation.

..................................................................................................................................... [1]

© UCLES 2019 0610/41/M/J/19

Page 366 of 877


7

(ii) Describe the effect of temperature on the rate of photosynthesis when carbon dioxide
concentration A was supplied.

Use the data from Fig. 2.2 in your answer.

...........................................................................................................................................

...........................................................................................................................................

...........................................................................................................................................

...........................................................................................................................................

...........................................................................................................................................

...........................................................................................................................................

..................................................................................................................................... [3]

(iii) Calculate the percentage increase in the rate of photosynthesis at 30 °C when the carbon
dioxide concentration was increased from A to B as shown in Fig. 2.2.

Show your working and give your answer to the nearest whole number.

............................................................ %
[2]

(iv) Explain the effect of increasing temperature on the rate of photosynthesis for carbon
dioxide concentration B.

Use the term limiting factor in your answer.

...........................................................................................................................................

...........................................................................................................................................

...........................................................................................................................................

...........................................................................................................................................

...........................................................................................................................................

...........................................................................................................................................

..................................................................................................................................... [3]

© UCLES 2019 0610/41/M/J/19 [Turn over


Page 367 of 877
8

(v) The student concluded that carbon dioxide concentration is the factor limiting the rate of
photosynthesis between 30 °C and 35 °C for the results shown for A in Fig. 2.2.

State the evidence for this conclusion.

...........................................................................................................................................

...........................................................................................................................................

..................................................................................................................................... [1]

© UCLES 2019 0610/41/M/J/19

Page 368 of 877


9

(d) A similar investigation was carried out on Arizona honeysweet, Tidestromia oblongifolia, that
grows in Death Valley in California where the highest temperatures may be greater than
45 °C.

The results are shown in Fig. 2.3.

40

35

30

25

rate of photosynthesis
/ μmol per m2 per s 20

15

10

0
10 20 30 40 50 60 70
temperature of leaves / °C

Fig. 2.3

Predict and explain what would happen to the rate of photosynthesis if the investigation is
continued at temperatures higher than 45 °C.

...................................................................................................................................................

...................................................................................................................................................

...................................................................................................................................................

...................................................................................................................................................

............................................................................................................................................. [2]

[Total: 16]

© UCLES 2019 0610/41/M/J/19 [Turn over


Page 369 of 877
10

BLANK PAGE

© UCLES 2019 0610/41/M/J/19

Page 370 of 877


11

3 (a) Complete the five sentences about the eye and the nervous system.

Structures in the eye change the shape of the lens so that the eye can focus on near and

distant objects. This is called .............................................. .

The radial and circular muscles in the iris of the eye are a pair of ..............................................

muscles that work against each other.

Muscles in the eye are controlled by the nervous system. The ..............................................

nervous system contains only sensory and motor neurones.

The .............................................. nerve from the eye contains sensory neurones that conduct

impulses to the .............................................. .


[5]

(b) Transmission of impulses relies on the flow of ions through the cell membranes of neurones
down their concentration gradients. Active transport is responsible for maintaining the
concentration gradients of ions across the membranes of neurones.

Explain how ions are moved across membranes by active transport.

...................................................................................................................................................

...................................................................................................................................................

...................................................................................................................................................

...................................................................................................................................................

...................................................................................................................................................

...................................................................................................................................................

............................................................................................................................................. [3]

© UCLES 2019 0610/41/M/J/19 [Turn over


Page 371 of 877
12

(c) Fig. 3.1 shows the junction between two neurones.

presynaptic neurone

enzyme that
breaks down
neurotransmitter
molecules B

postsynaptic neurone

Fig. 3.1

Many drugs interfere with the action of neurotransmitters at the junctions between neurones.

Two drugs that influence the transmission of impulses between neurones are atropine and
eserine. The actions of these drugs are shown in Table 3.1.

Table 3.1

drug action at junctions between neurones

atropine blocks receptor molecules for neurotransmitters

eserine blocks the enzyme that breaks down neurotransmitters

© UCLES 2019 0610/41/M/J/19

Page 372 of 877


13

Explain the effects of these two drugs on the nervous system using the information in Fig. 3.1
and Table 3.1.

...................................................................................................................................................

...................................................................................................................................................

...................................................................................................................................................

...................................................................................................................................................

...................................................................................................................................................

...................................................................................................................................................

...................................................................................................................................................

...................................................................................................................................................

...................................................................................................................................................

...................................................................................................................................................

...................................................................................................................................................

...................................................................................................................................................

...................................................................................................................................................

............................................................................................................................................. [6]

(d) A scientific paper was published in 1997 that described the effects of anabolic steroids
on female athletes. Many of these athletes achieved great success in international sport
competitions during the 1960s and 1970s.

Discuss the arguments against the use of anabolic steroids in sport.

...................................................................................................................................................

...................................................................................................................................................

...................................................................................................................................................

...................................................................................................................................................

...................................................................................................................................................

...................................................................................................................................................

............................................................................................................................................. [3]

[Total: 17]

© UCLES 2019 0610/41/M/J/19 [Turn over


Page 373 of 877
14

4 (a) Table 4.1 shows four structures associated with the human male reproductive system.

Complete Table 4.1 by identifying the level of organisation of each structure.

Choose your answers from the list.

cell cell structure organ

organ system organism tissue

Table 4.1

structure level of organisation

epithelium

nucleus

sperm

testis

[4]

(b) Fig. 4.1 shows the male reproductive system.

C B

Fig. 4.1

© UCLES 2019 0610/41/M/J/19

Page 374 of 877


15

Table 4.2 shows information about the male reproductive system shown in Fig. 4.1.

Complete Table 4.2.

Table 4.2

name of structure function letter in Fig. 4.1

testis

transports sperm but not urine

tube for urine and seminal fluid


through the penis

prostate gland

contains the testes

[5]

(c) Draw an X on Fig. 4.1 on the structure where meiosis occurs. [1]

(d) Sperm and eggs each have a nucleus which is haploid.

(i) Define the term haploid nucleus.

...........................................................................................................................................

...........................................................................................................................................

..................................................................................................................................... [1]

(ii) State the number of chromosomes in a human haploid nucleus.

..................................................................................................................................... [1]

[Total: 12]

© UCLES 2019 0610/41/M/J/19 [Turn over


Page 375 of 877
16

5 (a) Tissue plasminogen activators (TPAs) are human proteins that are used as drugs to break
down blood clots.

TPAs break down blood clots by activating plasminogen. Plasminogen is a protein that is
always present in the blood.

When activated, plasminogen forms a protease that breaks down fibrin molecules.

(i) Plasminogen is found in the plasma.

State what is meant by the term plasma.

..................................................................................................................................... [1]

(ii) State the products of the action of protease on the protein fibrin.

..................................................................................................................................... [1]

TPAs can be produced by genetically-engineered bacteria.

Fig. 5.1 shows some of the stages involved in genetically engineering a bacterium to make a TPA.

DNA removed TPA gene


from a human A
cell
DNA removed
from a bacterial
cell

not to scale
Fig. 5.1

© UCLES 2019 0610/41/M/J/19

Page 376 of 877


17

(b) (i) State the name of structure A in Fig. 5.1.

..................................................................................................................................... [1]

(ii) In the flow chart, X represents the action of an enzyme on a molecule of DNA.

State the name of this enzyme.

..................................................................................................................................... [1]

(iii) The TPA gene is inserted into structure A.

Explain how the gene is inserted into structure A to form structure B as shown in Fig. 5.1.

...........................................................................................................................................

...........................................................................................................................................

...........................................................................................................................................

...........................................................................................................................................

...........................................................................................................................................

...........................................................................................................................................

..................................................................................................................................... [3]

(iv) Before TPA was made by genetically-engineered bacteria it was only available from
blood donated by people.

Suggest one advantage of producing TPA by genetically-engineered bacteria.

...........................................................................................................................................

..................................................................................................................................... [1]

(v) The genetically-engineered bacteria produce mRNA that is a copy of the human TPA
gene.

Explain the role of mRNA in the bacterium.

...........................................................................................................................................

...........................................................................................................................................

...........................................................................................................................................

...........................................................................................................................................

..................................................................................................................................... [2]

[Total: 10]

© UCLES 2019 0610/41/M/J/19 [Turn over


Page 377 of 877
18

6 Fig. 6.1 shows some cells from the shoot tip of an onion, Allium cepa.

cell A

Fig. 6.1

(a) (i) State the evidence visible in Fig. 6.1 that identifies the cells of A. cepa as plant cells.

..................................................................................................................................... [1]

(ii) Cell A is dividing by mitosis.

State the role of mitosis in a shoot tip.

...........................................................................................................................................

...........................................................................................................................................

..................................................................................................................................... [1]

© UCLES 2019 0610/41/M/J/19

Page 378 of 877


19

(b) The area labelled M is a mitochondrion.

Explain why mitochondria have an important role in dividing cells.

...................................................................................................................................................

...................................................................................................................................................

...................................................................................................................................................

...................................................................................................................................................

...................................................................................................................................................

...................................................................................................................................................

............................................................................................................................................. [3]

(c) Cells just behind a shoot tip absorb water and grow in length. A plant hormone stimulates cell
elongation and controls the response of stems to gravity.

(i) State the name of the plant hormone that stimulates cell elongation in stems.

..................................................................................................................................... [1]

(ii) Explain how the response of stems to gravity is controlled.

...........................................................................................................................................

...........................................................................................................................................

...........................................................................................................................................

...........................................................................................................................................

...........................................................................................................................................

...........................................................................................................................................

...........................................................................................................................................

...........................................................................................................................................

..................................................................................................................................... [4]

© UCLES 2019 0610/41/M/J/19 [Turn over


Page 379 of 877
20

(d) Some cells in shoot tips become leaf cells and others become cells in the stem or in flowers.

Explain why it is important that only some of the genes in cell A are expressed in these cells.

...................................................................................................................................................

...................................................................................................................................................

...................................................................................................................................................

...................................................................................................................................................

...................................................................................................................................................

...................................................................................................................................................

............................................................................................................................................. [3]

[Total: 13]

Permission to reproduce items where third-party owned material protected by copyright is included has been sought and cleared where possible. Every
reasonable effort has been made by the publisher (UCLES) to trace copyright holders, but if any items requiring clearance have unwittingly been included, the
publisher will be pleased to make amends at the earliest possible opportunity.

To avoid the issue of disclosure of answer-related information to candidates, all copyright acknowledgements are reproduced online in the Cambridge
Assessment International Education Copyright Acknowledgements Booklet. This is produced for each series of examinations and is freely available to download
at www.cambridgeinternational.org after the live examination series.

Cambridge Assessment International Education is part of the Cambridge Assessment Group. Cambridge Assessment is the brand name of the University of
Cambridge Local Examinations Syndicate (UCLES), which itself is a department of the University of Cambridge.

© UCLES 2019 0610/41/M/J/19

Page 380 of 877


Cambridge Assessment International Education
Cambridge International General Certificate of Secondary Education
* 7 7 4 2 0 4 5 0 3 7 *

BIOLOGY 0610/42
Paper 4 Theory (Extended) May/June 2019
1 hour 15 minutes
Candidates answer on the Question Paper.
No Additional Materials are required.

READ THESE INSTRUCTIONS FIRST

Write your centre number, candidate number and name on all the work you hand in.
Write in dark blue or black pen.
You may use an HB pencil for any diagrams or graphs.
Do not use staples, paper clips, glue or correction fluid.
DO NOT WRITE IN ANY BARCODES.

Answer all questions.

Electronic calculators may be used.


You may lose marks if you do not show your working or if you do not use appropriate units.

At the end of the examination, fasten all your work securely together.
The number of marks is given in brackets [ ] at the end of each question or part question.

This syllabus is regulated for use in England, Wales and Northern Ireland as a Cambridge International Level 1/Level 2 Certificate.

This document consists of 18 printed pages and 2 blank pages.

DC (ST/CT) 162741/5
© UCLES 2019 [Turn over

Page 381 of 877


2

1 Biotechnology is used in the process of bread-making.

Fig. 1.1 shows some of the steps in making bread.

flour, salt and water are put into a bowl

Step 1 mixing

organism A and an energy source are added

Step 2 mixing

dough forms

Step 3 dough left at 35 °C for 2 hours

gas bubbles form in the dough

Step 4 dough is shaped and left at room temperature

dough put into baking tins

Step 5 dough is baked for 30 minutes at 200 °C

bread is made

Fig. 1.1

(a) State the name of organism A in Fig. 1.1.

............................................................................................................................................. [1]

(b) (i) State the name of the source of energy used by organism A.

..................................................................................................................................... [1]

(ii) State the name of the process that occurs at step 3 that causes gas bubbles to form in
the dough.

..................................................................................................................................... [1]

(iii) State the name of the gas that forms to create the gas bubbles in the dough.

..................................................................................................................................... [1]

© UCLES 2019 0610/42/M/J/19

Page 382 of 877


3

(c) Explain the reasons for the different temperatures used in step 3 and step 5.

...................................................................................................................................................

...................................................................................................................................................

...................................................................................................................................................

...................................................................................................................................................

............................................................................................................................................. [2]

(d) State the name of two products of biotechnology, other than bread, that make use of
microorganisms.

1 ................................................................................................................................................

2 ................................................................................................................................................
[2]

[Total: 8]

© UCLES 2019 0610/42/M/J/19 [Turn over


Page 383 of 877
4

2 The concentration of atmospheric carbon dioxide has increased considerably in recent years.

(a) Describe the possible causes of increased atmospheric carbon dioxide.

...................................................................................................................................................

...................................................................................................................................................

...................................................................................................................................................

...................................................................................................................................................

...................................................................................................................................................

...................................................................................................................................................

............................................................................................................................................. [3]

(b) Soybean plants, Glycine max, were grown in two separate plots.

Each plot used a carbon dioxide enrichment system to control the atmospheric carbon dioxide
concentration.

The atmospheric carbon dioxide concentrations in the two plots were kept at:

• 370 ppm, which is similar to the current atmospheric carbon dioxide concentration
• 550 ppm, which is a possible future atmospheric carbon dioxide concentration.

When the soybean plants were fully grown, scientists calculated the average rates of
photosynthesis at regular intervals from 04:00 to 22:00 for both plots.

The results are shown in Fig. 2.1.

25
550 ppm CO2
20
average
rates of 15
photosynthesis 370 ppm CO2
/ μmol per m2 per s 10

0
04:00 06:00 08:00 10:00 12:00 14:00 16:00 18:00 20:00 22:00

time of day

Fig. 2.1

© UCLES 2019 0610/42/M/J/19

Page 384 of 877


5

Describe and explain the effect of carbon dioxide concentration on the average rates of
photosynthesis of the soybean plants from 04:00 to 22:00.

Use the data from Fig. 2.1 in your answer.

...................................................................................................................................................

...................................................................................................................................................

...................................................................................................................................................

...................................................................................................................................................

...................................................................................................................................................

...................................................................................................................................................

...................................................................................................................................................

...................................................................................................................................................

...................................................................................................................................................

...................................................................................................................................................

...................................................................................................................................................

...................................................................................................................................................

...................................................................................................................................................

...................................................................................................................................................

............................................................................................................................................. [6]

© UCLES 2019 0610/42/M/J/19 [Turn over


Page 385 of 877
6

(c) The scientists also made observations of the leaf structure of the soybean plants.

Epidermis and mesophyll tissues are adapted for photosynthesis.

Complete Table 2.1 by stating two structural features of each of these tissues and explain
how each feature is an adaptation for photosynthesis.

Table 2.1

how the feature is an adaptation


tissue feature
for photosynthesis

1 ............................................... .............................................................................

.................................................. .............................................................................

.................................................. .............................................................................
epidermis
2 ............................................... .............................................................................

.................................................. .............................................................................

.................................................. .............................................................................

1 ............................................... .............................................................................

.................................................. .............................................................................

.................................................. .............................................................................
mesophyll
2 ............................................... .............................................................................

.................................................. .............................................................................

.................................................. .............................................................................

[4]

(d) When the scientists were working in the plot with a carbon dioxide concentration of 550 ppm,
their breathing rates were higher than when they worked in the other plot.

Suggest why their breathing rates were higher.

...................................................................................................................................................

...................................................................................................................................................

...................................................................................................................................................

...................................................................................................................................................

............................................................................................................................................. [2]

[Total: 15]
© UCLES 2019 0610/42/M/J/19

Page 386 of 877


7

3 Very small pieces of plastic, called microplastics, are found in many products such as soaps and
toothpaste.

Fig. 3.1 shows toothpaste that contains microplastics.

Fig. 3.1

(a) (i) It is estimated that microplastics make up 5% of the mass of some toothpastes.

Each person uses approximately 2 g of toothpaste a day.

There were estimated to be 1.2 × 109 people using toothpaste that contained
microplastics in some countries in 2013.

Calculate the mass of microplastics contained in the toothpaste used on one day in 2013
for these countries.

Show your working and state appropriate units with your answer.

................................................................
[3]

(ii) State one recommendation, other than regular brushing, for the proper care of teeth.

..................................................................................................................................... [1]

© UCLES 2019 0610/42/M/J/19 [Turn over


Page 387 of 877
8

(b) Lugworms live in sand on coastal beaches and are eaten by wading birds. Lugworms feed on
diatoms. Diatoms are photosynthetic protoctists that require ammonium ions as a source of
nitrogen. Beach sand contains ammonium ions.

(i) Construct a food chain for these marine organisms.

[2]

(ii) There is some evidence that microplastics affect ammonium ions. Affected ammonium
ions cannot be used by diatoms. A group of researchers thought that this could affect
lugworms living in sand polluted by microplastics.

The researchers collected 30 healthy lugworms, all with the same initial mass.

They divided them into three groups, A, B and C. Each group contained 10 lugworms.

Each group of lugworms was placed in a bucket containing the same mass of beach
sand and ammonium ions and:

A biodegradable microplastics
B non-biodegradable microplastics
C no microplastics.

The measurements that were recorded at the end of the investigation are shown in
Table 3.1.

Table 3.1

group
variable measured
A B C
ammonium ion concentration in the
19.3 47.0 27.4
bucket / µmol per dm3
average respiration rate of lugworms
5.2 9.6 5.1
/ mg oxygen per hour per g mass
volume of lugworm faeces / cm3 60.0 25.0 40.0

average lugworm mass / g 9.1 7.0 9.1

© UCLES 2019 0610/42/M/J/19

Page 388 of 877


9

Describe and explain why the researchers concluded that non-biodegradable


microplastics are the most harmful to lugworms.

Use the information in Table 3.1 in your answer.

...........................................................................................................................................

...........................................................................................................................................

...........................................................................................................................................

...........................................................................................................................................

...........................................................................................................................................

...........................................................................................................................................

...........................................................................................................................................

...........................................................................................................................................

..................................................................................................................................... [4]

(c) Ammonium ions are an important part of the nitrogen cycle. They can be converted into
nitrate ions, which are used by plants and protoctists such as diatoms.

(i) State the name of the molecules that are converted into ammonium ions in the nitrogen
cycle.

..................................................................................................................................... [1]

(ii) State the name of the process of converting ammonium ions into nitrate ions.

..................................................................................................................................... [1]

(iii) Explain the effects of nitrate ion deficiency on plant growth.

...........................................................................................................................................

...........................................................................................................................................

...........................................................................................................................................

...........................................................................................................................................

...........................................................................................................................................

...........................................................................................................................................

..................................................................................................................................... [3]

© UCLES 2019 0610/42/M/J/19 [Turn over


Page 389 of 877
10

(d) Non-biodegradable plastics are also harmful to terrestrial ecosystems.

Discuss the effects of non-biodegradable plastics on terrestrial ecosystems.

...................................................................................................................................................

...................................................................................................................................................

...................................................................................................................................................

...................................................................................................................................................

...................................................................................................................................................

...................................................................................................................................................

...................................................................................................................................................

...................................................................................................................................................

...................................................................................................................................................

...................................................................................................................................................

............................................................................................................................................. [5]

[Total: 20]

© UCLES 2019 0610/42/M/J/19

Page 390 of 877


11

4 Neurones are part of the nervous system. Neurones are connected to each other by synapses.

(a) (i) Describe how the structure of a neurone is related to its function.

...........................................................................................................................................

...........................................................................................................................................

...........................................................................................................................................

...........................................................................................................................................

...........................................................................................................................................

...........................................................................................................................................

..................................................................................................................................... [3]

(ii) The nervous system is made up of the central nervous system and the peripheral
nervous system.

State the names of the organs that make up the central nervous system.

..................................................................................................................................... [1]

(b) Reflex actions allow the body to respond rapidly to changes in the external environment.

(i) Outline the pathway in a reflex arc in response to shining a bright light into the eye.

...........................................................................................................................................

...........................................................................................................................................

...........................................................................................................................................

...........................................................................................................................................

...........................................................................................................................................

...........................................................................................................................................

..................................................................................................................................... [3]

(ii) Doctors sometimes check the reflexes of people who are unconscious.

Suggest why reflexes occur in people who are unconscious.

..................................................................................................................................... [1]

© UCLES 2019 0610/42/M/J/19 [Turn over


Page 391 of 877
12

(c) Fig. 4.1 is a diagram of a synapse and parts of two neurones.

G F

Fig. 4.1

(i) State the names of the labelled parts in Fig. 4.1.

E ........................................................................................................................................

F ........................................................................................................................................

G .......................................................................................................................................
[3]

(ii) Draw an arrow on Fig. 4.1 to show the direction in which the signal travels across the
synapse. [1]

[Total: 12]

© UCLES 2019 0610/42/M/J/19

Page 392 of 877


13

5 (a) The testes are part of the endocrine system because they produce hormones.

(i) State the name of the hormone released from the testes.

..................................................................................................................................... [1]

(ii) The testes are also part of the reproductive system. This means that the testes are part
of two organ systems.

Complete Fig. 5.1 by stating two other organs that also belong to two organ systems.

One has been completed for you.

organ organ system

male reproductive system

testes
endocrine system

male reproductive system

endocrine system

Fig. 5.1
[4]

© UCLES 2019 0610/42/M/J/19 [Turn over


Page 393 of 877
14

Fig. 5.2 is a photomicrograph of part of a mammalian testis.

Fig. 5.2

(b) The cells labelled M in Fig. 5.2 are undergoing meiosis.

Explain why meiosis is necessary in the testes.

...................................................................................................................................................

...................................................................................................................................................

...................................................................................................................................................

...................................................................................................................................................

...................................................................................................................................................

...................................................................................................................................................

............................................................................................................................................. [3]

© UCLES 2019 0610/42/M/J/19

Page 394 of 877


15

(c) Fig. 5.3 is a photomicrograph of a section through a sperm.

Fig. 5.3

Table 5.1 shows information about the sperm shown in Fig. 5.3.

Complete Table 5.1.

Table 5.1

letter on Fig. 5.3 name of the structure function

haploid nucleus

releases energy

flagellum

[4]

© UCLES 2019 0610/42/M/J/19 [Turn over


Page 395 of 877
16

(d) Draw and label one human egg cell.

Include at least one labelled feature that is not found in a sperm cell.

[3]

(e) Describe what happens to a fertilised egg cell before implantation in the uterus.

...................................................................................................................................................

...................................................................................................................................................

...................................................................................................................................................

...................................................................................................................................................

...................................................................................................................................................

...................................................................................................................................................

............................................................................................................................................. [3]

[Total: 18]

© UCLES 2019 0610/42/M/J/19

Page 396 of 877


17

6 Fig. 6.1 shows some of the many different varieties of potato, Solanum tuberosum, that are
cultivated across the world for food.

Fig. 6.1

All varieties of S. tuberosum are classified as the same species.

(a) Define the term species.

...................................................................................................................................................

...................................................................................................................................................

............................................................................................................................................. [2]

© UCLES 2019 0610/42/M/J/19 [Turn over


Page 397 of 877
18

(b) Fig. 6.2 shows a method of reproduction that some potato farmers use to produce more
potato plants.

parent plant

offspring

potato tuber

plant pot
not to scale
Fig. 6.2

Describe the advantages of the type of reproduction shown in Fig. 6.2 in crop production.

...................................................................................................................................................

...................................................................................................................................................

...................................................................................................................................................

...................................................................................................................................................

...................................................................................................................................................

...................................................................................................................................................

............................................................................................................................................. [3]

(c) Potato tubers store starch.

Explain why plants store starch.

...................................................................................................................................................

...................................................................................................................................................

...................................................................................................................................................

...................................................................................................................................................

............................................................................................................................................. [2]

[Total: 7]
© UCLES 2019 0610/42/M/J/19

Page 398 of 877


Cambridge Assessment International Education
Cambridge International General Certificate of Secondary Education
* 1 1 8 0 0 3 8 1 3 2 *

BIOLOGY 0610/43
Paper 4 Theory (Extended) May/June 2019
1 hour 15 minutes
Candidates answer on the Question Paper.
No Additional Materials are required.

READ THESE INSTRUCTIONS FIRST

Write your centre number, candidate number and name on all the work you hand in.
Write in dark blue or black pen.
You may use an HB pencil for any diagrams or graphs.
Do not use staples, paper clips, glue or correction fluid.
DO NOT WRITE IN ANY BARCODES.

Answer all questions.

Electronic calculators may be used.


You may lose marks if you do not show your working or if you do not use appropriate units.

At the end of the examination, fasten all your work securely together.
The number of marks is given in brackets [ ] at the end of each question or part question.

This syllabus is regulated for use in England, Wales and Northern Ireland as a Cambridge International Level 1/Level 2 Certificate.

This document consists of 16 printed pages and 4 blank pages.

DC (ST/CT) 162742/4
© UCLES 2019 [Turn over

Page 401 of 877


2

1 Bacteria are classified in the Prokaryote kingdom.

(a) State two features of animal cells that are not found in bacteria.

1 ................................................................................................................................................

2 ................................................................................................................................................
[2]

(b) The bacterium Bacillus megaterium was grown in the laboratory fermenter shown in Fig. 1.1.

air lock water


sterile air

magnetic
stirrer

bacteria, source of
nitrogen and glucose

Fig. 1.1

(i) Explain why a source of nitrogen and glucose were added to the fermenter.

nitrogen .............................................................................................................................

...........................................................................................................................................

glucose ..............................................................................................................................

...........................................................................................................................................
[2]

(ii) Suggest why it is important to stir the contents of the fermenter continuously.

...........................................................................................................................................

...........................................................................................................................................

...........................................................................................................................................

...........................................................................................................................................

...........................................................................................................................................

...........................................................................................................................................

..................................................................................................................................... [3]

© UCLES 2019 0610/43/M/J/19

Page 402 of 877


3

(c) Samples were taken from the fermenter at frequent intervals and the number of live bacteria
was determined. The results are shown in Fig. 1.2.

800
C
700

600

500
number of
live bacteria D
400
/ million per
cm3 B
300

200

100
A
0
0 10 20 30 40 50 60

time / hours

Fig. 1.2

Describe and explain what happens to the number of live bacteria shown in the stages
labelled A, B, C and D in Fig. 1.2.

...................................................................................................................................................

...................................................................................................................................................

...................................................................................................................................................

...................................................................................................................................................

...................................................................................................................................................

...................................................................................................................................................

...................................................................................................................................................

...................................................................................................................................................

...................................................................................................................................................

...................................................................................................................................................

...................................................................................................................................................

............................................................................................................................................. [6]

[Total: 13]

© UCLES 2019 0610/43/M/J/19 [Turn over


Page 403 of 877
4

2 (a) State the word equation for photosynthesis.

............................................................................................................................................. [2]

(b) Scientists investigated the effect of light intensity on the rate of photosynthesis in the leaves
of eucalyptus trees at two different concentrations of carbon dioxide, A and B.

The results are shown in Fig. 2.1.

rate of photosynthesis
/ μmol per m2 per s

50

45 B

40

35

30

25
A
20

15

10

0
0 250 500 750 1000 1250 1500 1750 2000
light intensity / arbitrary units

Key:
A carbon dioxide concentration
140 ppm
B carbon dioxide concentration
1000 ppm

Fig. 2.1

© UCLES 2019 0610/43/M/J/19

Page 404 of 877


5

(i) Suggest and explain why the scientists kept the temperature of the leaves at 20 °C while
they recorded results.

...........................................................................................................................................

...........................................................................................................................................

...........................................................................................................................................

...........................................................................................................................................

..................................................................................................................................... [2]

(ii) Calculate the percentage increase in the rate of photosynthesis at a light intensity of
1250 arbitrary units when the carbon dioxide concentration was increased from 140 ppm
to 1000 ppm.

Show your working and give your answer to the nearest whole number.

............................................................ %
[3]

(iii) Describe the effect of increasing light intensity on the rate of photosynthesis when the
concentration of carbon dioxide was 140 ppm.

...........................................................................................................................................

...........................................................................................................................................

...........................................................................................................................................

...........................................................................................................................................

...........................................................................................................................................

...........................................................................................................................................

..................................................................................................................................... [3]

© UCLES 2019 0610/43/M/J/19 [Turn over


Page 405 of 877
6

(iv) Explain the effect of increasing light intensity on the rate of photosynthesis when the
concentration of carbon dioxide was 1000 ppm.

Use the term limiting factor in your answer.

...........................................................................................................................................

...........................................................................................................................................

...........................................................................................................................................

...........................................................................................................................................

...........................................................................................................................................

...........................................................................................................................................

...........................................................................................................................................

...........................................................................................................................................

..................................................................................................................................... [4]

[Total: 14]

© UCLES 2019 0610/43/M/J/19

Page 406 of 877


8

3 Cotton, Gossypium hirsutum, is grown for the fibres that form within the fruits after fertilisation, as
shown in Fig. 3.1.

Fibres from the fruits of cotton plants are used in the textile industry.

fibres

Fig. 3.1

Cotton plants have been genetically engineered to produce a protein that is toxic to the caterpillars
of several insect pests. This gives the cotton plants resistance to the pests.

The cry gene for pest resistance was isolated from the bacterium Bacillus thuringiensis and
inserted into the cells of cotton plants as shown in Fig. 3.2.

cry gene

A A
bacterial
DNA plasmid

cotton plant cell


not to scale
Fig. 3.2

© UCLES 2019 0610/43/M/J/19

Page 408 of 877


9

(a) An enzyme cuts the cry gene from the DNA of B. thuringiensis.

(i) State the name of the enzyme that cuts DNA.

..................................................................................................................................... [1]

(ii) State the name of the regions labelled A on Fig. 3.2.

..................................................................................................................................... [1]

(iii) Explain how the DNA is inserted into the plasmid.

...........................................................................................................................................

...........................................................................................................................................

...........................................................................................................................................

...........................................................................................................................................

...........................................................................................................................................

..................................................................................................................................... [3]

(b) The plasmids containing the cry gene are inserted into the cells of cotton plants.

Outline how the cells of cotton plants use the cry gene to make the toxic protein.

...................................................................................................................................................

...................................................................................................................................................

...................................................................................................................................................

...................................................................................................................................................

...................................................................................................................................................

...................................................................................................................................................

...................................................................................................................................................

............................................................................................................................................. [4]

© UCLES 2019 0610/43/M/J/19 [Turn over


Page 409 of 877
10

(c) Outline the advantages to farmers of growing genetically engineered cotton plants that
contain the toxic protein.

...................................................................................................................................................

...................................................................................................................................................

...................................................................................................................................................

...................................................................................................................................................

...................................................................................................................................................

...................................................................................................................................................

...................................................................................................................................................

...................................................................................................................................................

...................................................................................................................................................

...................................................................................................................................................

............................................................................................................................................. [3]

[Total: 12]

© UCLES 2019 0610/43/M/J/19

Page 410 of 877


11

4 Mammals have a double circulation.

(a) State what is meant by the term double circulation.

...................................................................................................................................................

...................................................................................................................................................

............................................................................................................................................. [1]

(b) Table 4.1 shows some information about the functions of the components of blood.

Complete Table 4.1.

Table 4.1

function type of cell

production of antibodies

phagocyte

promotes blood clotting

transports oxygen

[4]

(c) Blood is transported in arteries and veins.

Complete the drawings of the cross-sections of an artery and a vein to show the differences
between these two types of blood vessel. Label the lumen in each drawing.

artery vein

[2]

© UCLES 2019 0610/43/M/J/19 [Turn over


Page 411 of 877
12

(d) A diagram of a mammalian heart and associated blood vessels is shown in Fig. 4.1.

A
L

J D

Fig. 4.1

(i) Sketch arrows on Fig. 4.1 to show the pathway taken by deoxygenated blood from the
heart towards the lungs. [2]

© UCLES 2019 0610/43/M/J/19

Page 412 of 877


13

(ii) Table 4.2 contains statements about the structures visible in Fig. 4.1.

Complete Table 4.2 by:

• stating the name of each structure


• identifying the structure with the corresponding letter from Fig. 4.1.

Table 4.2

statement name of structure letter from Fig. 4.1


chamber that creates the highest
blood pressure
blood vessel containing blood with
the highest concentration of oxygen
structure that prevents blood going
from ventricle to atrium
structure that prevents backflow of
blood from artery to ventricle
chamber that receives blood from
vena cava
[5]

(e) Mammals also have a lymphatic system.

Outline the functions of the lymphatic system.

...................................................................................................................................................

...................................................................................................................................................

...................................................................................................................................................

...................................................................................................................................................

...................................................................................................................................................

...................................................................................................................................................

............................................................................................................................................. [3]

[Total: 17]

© UCLES 2019 0610/43/M/J/19 [Turn over


Page 413 of 877
14

5 Sickle-cell anaemia is an inherited disease.

Fig. 5.1 is a photomicrograph of some blood cells from a person who has sickle-cell anaemia.

a sickle-shaped red blood cell

Fig. 5.1

(a) Explain how red blood cells become sickle-shaped.

...................................................................................................................................................

...................................................................................................................................................

...................................................................................................................................................

...................................................................................................................................................

...................................................................................................................................................

...................................................................................................................................................

............................................................................................................................................. [3]

© UCLES 2019 0610/43/M/J/19

Page 414 of 877


15

(b) Some people who have sickle-cell anaemia have parents who do not have sickle-cell
anaemia.

Explain how people with sickle-cell anaemia inherit the disease.

...................................................................................................................................................

...................................................................................................................................................

...................................................................................................................................................

...................................................................................................................................................

...................................................................................................................................................

...................................................................................................................................................

...................................................................................................................................................

...................................................................................................................................................

...................................................................................................................................................

............................................................................................................................................. [4]

(c) Sickle-cell anaemia is most common in areas of the world where the infectious disease
malaria is found.

Some species of the genus Plasmodium cause malaria in humans.

(i) Define the term species.

...........................................................................................................................................

...........................................................................................................................................

...........................................................................................................................................

...........................................................................................................................................

..................................................................................................................................... [2]

© UCLES 2019 0610/43/M/J/19 [Turn over


Page 415 of 877
16

(ii) The distribution of sickle-cell anaemia is the result of natural selection.

Explain the distribution of the sickle-cell allele in human populations.

...........................................................................................................................................

...........................................................................................................................................

...........................................................................................................................................

...........................................................................................................................................

...........................................................................................................................................

...........................................................................................................................................

...........................................................................................................................................

...........................................................................................................................................

...........................................................................................................................................

..................................................................................................................................... [5]

[Total: 14]

6 Fig. 6.1 is a photomicrograph of part of a cell from the pancreas that produces enzymes that are
released into the small intestine.

×20 000

Fig. 6.1

© UCLES 2019 0610/43/M/J/19

Page 416 of 877


17

(a) (i) Structure Q is part of the nucleus of the cell.

State one function of a nucleus.

...........................................................................................................................................

...........................................................................................................................................

..................................................................................................................................... [1]

(ii) State the names of the structures labelled P and R in Fig. 6.1.

P ........................................................................................................................................

R ........................................................................................................................................
[2]

(b) The structure labelled S transports enzymes to the cell membrane for release into the
pancreatic duct. These structures contain molecules of amylase, trypsin and lipase.

Complete the sentences with the most appropriate words.

Enzymes are made of protein and act as .............................................. because they increase

the rate of chemical reactions, but are not changed in those reactions. Amylase speeds up the

digestion of .............................................. to .............................................. Trypsin continues

the chemical digestion of protein begun by the enzyme .............................................. in the

stomach.

The optimum pH for pancreatic enzymes is greater than pH 7. Bile is produced

by the .............................................. and enters the small intestine, where it

.............................................. stomach acid to provide the appropriate pH. Bile also breaks

down fat by .............................................. to increase the surface area for the action of lipase.

[7]

[Total: 10]

© UCLES 2019 0610/43/M/J/19

Page 417 of 877


Cambridge Assessment International Education
Cambridge International General Certificate of Secondary Education
* 4 0 8 1 1 6 6 6 2 9 *

BIOLOGY 0610/41
Paper 4 Theory (Extended) October/November 2019
1 hour 15 minutes
Candidates answer on the Question Paper.
No Additional Materials are required.

READ THESE INSTRUCTIONS FIRST

Write your centre number, candidate number and name on all the work you hand in.
Write in dark blue or black pen.
You may use an HB pencil for any diagrams or graphs.
Do not use staples, paper clips, glue or correction fluid.
DO NOT WRITE IN ANY BARCODES.

Answer all questions.

Electronic calculators may be used.


You may lose marks if you do not show your working or if you do not use appropriate units.

At the end of the examination, fasten all your work securely together.
The number of marks is given in brackets [ ] at the end of each question or part question.

This syllabus is regulated for use in England, Wales and Northern Ireland as a Cambridge International Level 1/Level 2 Certificate.

This document consists of 19 printed pages and 1 blank page.

DC (ST/CT) 173042/4
© UCLES 2019 [Turn over

Page 421 of 877


2

1 All living organisms are placed into groups according to their features.
Myriapods are one of the main groups of arthropods.

(a) State two features of myriapods that can be used to distinguish them from other arthropods.

1 ................................................................................................................................................

2 ................................................................................................................................................
[2]

Fig. 1.1 shows that there are four main groups of arthropods.

arthropods

group 1 group 2 group 3 group 4


myriapods

Fig. 1.1

(b) State the names of two of the other groups of arthropods in Fig. 1.1.

1 ................................................................................................................................................

2 ................................................................................................................................................
[2]

(c) Myriapods can be classified into four classes, 1, 2, 3 and 4.

Fig. 1.2 is a dichotomous key that can be used to distinguish the four classes of myriapods.

myriapods

no eyes eyes present

hairs on most no hairs on one leg on two legs on


body segments body segments each side of each each side of each
body segment body segment

class 1 class 2 class 3 class 4

Fig. 1.2

© UCLES 2019 0610/41/O/N/19

Page 422 of 877


3

species J

species K

species L

species M

species N

species O
not to scale

Fig. 1.3

Complete Table 1.1 by using the key in Fig. 1.2 to classify the six myriapods in Fig. 1.3 into
the four classes.

Table 1.1

letter(s) of species from


class
Fig. 1.3 in each class

[3]

© UCLES 2019 0610/41/O/N/19 [Turn over


Page 423 of 877
4

(d) Fig. 1.4 is a photograph of the myriapod, Apheloria virginiensis.

5 cm

Fig. 1.4

(i) State the genus name and kingdom name for the myriapod shown in Fig. 1.4.

genus ................................................................................................................................

kingdom .............................................................................................................................
[2]

(ii) A. virginiensis releases the poison cyanide when it is attacked by predators.


Cyanide stops enzymes in the mitochondria from functioning.

Suggest why cells die if the mitochondria do not function.

...........................................................................................................................................

...........................................................................................................................................

..................................................................................................................................... [1]

[Total: 10]

© UCLES 2019 0610/41/O/N/19

Page 424 of 877


5

2 Phloem is used to transport sucrose and amino acids in plants. Sucrose is a carbohydrate.

(a) Describe the uses of carbohydrates and amino acids in plants.

...................................................................................................................................................

...................................................................................................................................................

...................................................................................................................................................

...................................................................................................................................................

...................................................................................................................................................

...................................................................................................................................................

............................................................................................................................................. [4]

(b) Fig. 2.1 shows a diagram of a plant. The arrows point to circles containing magnified
cross-sections of those parts of the plant.

upper surface

not to scale

Fig. 2.1

Label the position of the phloem in each of the three magnified sections in Fig. 2.1.

Use a label line and the letter P for each section. [3]

© UCLES 2019 0610/41/O/N/19 [Turn over


Page 425 of 877
6

(c) Aphids are used by investigators to discover how plants transport sucrose.

Fig. 2.2 shows an aphid with its mouthparts inserted into a plant stem to feed on the liquid in
the phloem.

5 mm

Fig. 2.2

A plant was put in a dark cupboard for several days.


Four aphids, A, B, C and D, were then placed on the plant in the dark cupboard as shown in
Fig. 2.3.

dark cupboard

aphid B
aphid C
aphid A

aphid D

Fig. 2.3
© UCLES 2019 0610/41/O/N/19

Page 426 of 877


7

(i) Immediately after the aphids were placed on the plant it was observed that:

• all the aphids ingested the same volume of liquid from the phloem
• aphid D ingested the highest concentration of sucrose.

Explain why aphid D ingested the highest concentration of sucrose.

...........................................................................................................................................

...........................................................................................................................................

...........................................................................................................................................

...........................................................................................................................................

...........................................................................................................................................

...........................................................................................................................................

..................................................................................................................................... [3]

(ii) Many crop farmers try to prevent insects such as aphids from damaging their plants.

Describe how modern technology is used to reduce damage to crop plants by insects.

...........................................................................................................................................

...........................................................................................................................................

...........................................................................................................................................

...........................................................................................................................................

...........................................................................................................................................

...........................................................................................................................................

..................................................................................................................................... [3]

(iii) Other insects are useful to crop farmers.

Give one example of how insects are useful to farmers.

...........................................................................................................................................

..................................................................................................................................... [1]

[Total: 14]

© UCLES 2019 0610/41/O/N/19 [Turn over


Page 427 of 877
8

3 Kidneys are important for excretion.

(a) Explain the purpose of excretion.

...................................................................................................................................................

...................................................................................................................................................

............................................................................................................................................. [2]

(b) Sketch a simple diagram of a human kidney and its ureter.

Label the cortex, medulla and ureter on your diagram.

[2]

© UCLES 2019 0610/41/O/N/19

Page 428 of 877


9

(c) Fig. 3.1 is a photomicrograph of a glomerulus in a kidney.

Fig. 3.1

(i) Describe the structure of a glomerulus.

You may refer to Fig. 3.1 in your answer.

...........................................................................................................................................

...........................................................................................................................................

...........................................................................................................................................

...........................................................................................................................................

..................................................................................................................................... [2]

(ii) Describe the role of the glomerulus.

...........................................................................................................................................

...........................................................................................................................................

...........................................................................................................................................

...........................................................................................................................................

..................................................................................................................................... [2]

© UCLES 2019 0610/41/O/N/19 [Turn over


Page 429 of 877
10

(d) Table 3.1 shows some comparisons between a human and a mouse.

Table 3.1

feature human mouse


body mass / kg 60.0 0.025
kidney mass / g 320.0 0.310
water intake / dm3 per day 1.5 0.005
water reabsorption / dm3 per day 179.0 0.168
salt filtration rate / g per day 580.0 0.556
salt reabsorption / g per day 575.0 0.551

(i) Table 3.1 shows that salts are reabsorbed in the kidneys of both humans and mice.

Describe how salts are reabsorbed against a concentration gradient.

...........................................................................................................................................

...........................................................................................................................................

...........................................................................................................................................

...........................................................................................................................................

...........................................................................................................................................

...........................................................................................................................................

...........................................................................................................................................

...........................................................................................................................................

..................................................................................................................................... [4]

© UCLES 2019 0610/41/O/N/19

Page 430 of 877


11

(ii) Scientists stated a hypothesis:

“humans and mice reabsorb salt at almost the same rate in relation to the size of
their kidneys”

Determine whether the data in Table 3.1 supports this hypothesis.

Show your working.

human mouse

...........................................................................................................................................

...........................................................................................................................................

..................................................................................................................................... [4]

(iii) Table 3.1 also shows that water is reabsorbed in the kidneys of both humans and mice.

State the name of the process that mammals use to reabsorb water.

..................................................................................................................................... [1]

(iv) Water and salt are reabsorbed in the kidneys.

State the name of one other molecule that is also reabsorbed in kidney tubules.

..................................................................................................................................... [1]

[Total: 18]

© UCLES 2019 0610/41/O/N/19 [Turn over


Page 431 of 877
12

4 (a) The body has defence mechanisms to protect it from infection.

Outline the body’s defence mechanisms.

...................................................................................................................................................

...................................................................................................................................................

...................................................................................................................................................

...................................................................................................................................................

...................................................................................................................................................

...................................................................................................................................................

...................................................................................................................................................

...................................................................................................................................................

...................................................................................................................................................

...................................................................................................................................................

............................................................................................................................................. [5]

(b) State the name of a type of drug that can be used to treat bacterial infections.

............................................................................................................................................. [1]

[Total: 6]

© UCLES 2019 0610/41/O/N/19

Page 432 of 877


13

5 Fig. 5.1 shows an angiogram of a heart before and after treatment for coronary heart disease
(CHD). An angiogram is an image of the blood flow through the blood vessels of the heart.

before treatment after treatment

Fig. 5.1

(a) The arrow on Fig. 5.1 shows the position of a blockage in a blood vessel.

(i) State the name of the blocked blood vessel.

..................................................................................................................................... [1]

(ii) The blockage is caused by a blood clot.

Describe how a blood clot forms.

...........................................................................................................................................

...........................................................................................................................................

...........................................................................................................................................

...........................................................................................................................................

...........................................................................................................................................

...........................................................................................................................................

..................................................................................................................................... [3]

(iii) State the name of a drug that can be used to treat coronary heart disease.

..................................................................................................................................... [1]

© UCLES 2019 0610/41/O/N/19 [Turn over


Page 433 of 877
14

(b) Many health specialists think that the risk of coronary heart disease can be reduced by doing
regular exercise.

A long-term study of a large group of women was used to test this hypothesis.
The women were between 35 and 45 years old at the start of the study.
Every two years the same group of women were asked how much they were exercising.

After 28 years the researchers analysed their data:


• They calculated the average time spent exercising per week by each woman.
• They put the women into categories determined by how much exercise they had
done.
• For each category, they calculated the number of women who died from coronary
heart disease (CHD).

The results are shown in Fig. 5.2.

2.0

1.8

1.6

1.4
number of CHD
deaths per 1.2
10 000 women
1.0

0.8

0.6

0.4

0.2

0.0
0.0 0.1 – 2.0 2.1 – 4.0 4.1 – 10.0
exercise / hours per week

Fig. 5.2

© UCLES 2019 0610/41/O/N/19

Page 434 of 877


15

(i) Calculate the percentage decrease in the number of CHD deaths per 10 000 women
between those who did no exercise and those who exercised for 4.1 to 10.0 hours a
week, using the data in Fig. 5.2.

number of CHD deaths per 10 000 women who did no exercise .......................

number of CHD deaths per 10 000 women who did 4.1 to 10.0 hours per week of exercise

.......................

Give your answer to the nearest whole number.

Space for working.

........................................................... %
[3]

(ii) Health professionals wanted to use the results of this study to encourage the whole
population to take more exercise.

Discuss the arguments for and against health professionals using this study in this way.

Use the information about how the study was designed and the results in Fig. 5.2 in your
arguments.

...........................................................................................................................................

...........................................................................................................................................

...........................................................................................................................................

...........................................................................................................................................

...........................................................................................................................................

...........................................................................................................................................

...........................................................................................................................................

...........................................................................................................................................

...........................................................................................................................................

...........................................................................................................................................

..................................................................................................................................... [5]

© UCLES 2019 0610/41/O/N/19 [Turn over


Page 435 of 877
16

(c) Exercise causes heart rate to increase.

Explain why exercise causes an increase in heart rate.

...................................................................................................................................................

...................................................................................................................................................

...................................................................................................................................................

...................................................................................................................................................

...................................................................................................................................................

...................................................................................................................................................

............................................................................................................................................. [3]

[Total: 16]

6 Fish are an important part of a balanced diet for many people.

(a) Fish are a good source of vitamin D.

State one effect of a diet that is deficient in vitamin D.

...................................................................................................................................................

...................................................................................................................................................

............................................................................................................................................. [1]

(b) Many fish stocks have declined.

Marine ecologists considered the three topics shown in Fig. 6.1 when they discussed the
decline of fish stocks.

reasons why fish species risks to a fish species when


become endangered population size decreases

methods for maintaining


fish stocks

Fig. 6.1

© UCLES 2019 0610/41/O/N/19

Page 436 of 877


17

Describe the key points for each of the three topics shown in Fig. 6.1.

...................................................................................................................................................

...................................................................................................................................................

...................................................................................................................................................

...................................................................................................................................................

...................................................................................................................................................

...................................................................................................................................................

...................................................................................................................................................

...................................................................................................................................................

...................................................................................................................................................

...................................................................................................................................................

...................................................................................................................................................

...................................................................................................................................................

...................................................................................................................................................

...................................................................................................................................................

...................................................................................................................................................

............................................................................................................................................. [6]

[Total: 7]

© UCLES 2019 0610/41/O/N/19 [Turn over


Page 437 of 877
18

7 Fig. 7.1 shows part of the nitrogen cycle.

nitrogen in
process A atmosphere
nitrogen fixation

protein
synthesis proteins in deamination
plant and
amino acids animals and
animal waste
plants

compound X

process B

nitrate ions
in soil

Fig. 7.1

(a) (i) State two ways that nitrogen fixation can occur.

1 ........................................................................................................................................

2 ........................................................................................................................................
[2]

(ii) State the names of processes A and B in Fig. 7.1.

process A ..........................................................................................................................

process B ..........................................................................................................................
[2]

(iii) State the name of compound X in Fig. 7.1.

..................................................................................................................................... [1]

(iv) Define the term deamination.

...........................................................................................................................................

...........................................................................................................................................

...........................................................................................................................................

...........................................................................................................................................

..................................................................................................................................... [2]

© UCLES 2019 0610/41/O/N/19

Page 438 of 877


19

(b) State the name of the structure in a cell where protein synthesis occurs.

............................................................................................................................................. [1]

(c) State the name of an enzyme that digests proteins.

............................................................................................................................................. [1]

[Total: 9]

© UCLES 2019 0610/41/O/N/19

Page 439 of 877


Cambridge Assessment International Education
Cambridge International General Certificate of Secondary Education
* 1 0 7 2 4 3 2 1 9 0 *

BIOLOGY 0610/42
Paper 4 Theory (Extended) October/November 2019
1 hour 15 minutes
Candidates answer on the Question Paper.
No Additional Materials are required.

READ THESE INSTRUCTIONS FIRST

Write your centre number, candidate number and name on all the work you hand in.
Write in dark blue or black pen.
You may use a pencil for any diagrams or graphs.
Do not use staples, paper clips, glue or correction fluid.
DO NOT WRITE IN ANY BARCODES.

Answer all questions.

Electronic calculators may be used.


You may lose marks if you do not show your working or if you do not use appropriate units.

At the end of the examination, fasten all your work securely together.
The number of marks is given in brackets [ ] at the end of each question or part question.

This syllabus is regulated for use in England, Wales and Northern Ireland as a Cambridge International Level 1/Level 2 Certificate.

This document consists of 21 printed pages and 3 blank pages.

DC (ST/CT) 173043/4
© UCLES 2019 [Turn over

Page 441 of 877


2

1 (a) Fig. 1.1 shows four arthropods.

C D

not to scale

Fig. 1.1

(i) State two features, visible in Fig. 1.1, that are common to all arthropods.

1 ........................................................................................................................................

2 ........................................................................................................................................
[2]

© UCLES 2019 0610/42/O/N/19

Page 442 of 877


3

(ii) Fig. 1.2 is a dichotomous key for the arthropods shown in Fig. 1.1.

Complete Fig. 1.2 by writing suitable statements in:

• box 2 to identify species B

• box 3 to separate species C and A.

START

box 1 yes
animal has more than ten pairs of legs D

no

box 2 yes
B

no

box 3
yes
A

no

Fig. 1.2
[2]

© UCLES 2019 0610/42/O/N/19 [Turn over


Page 443 of 877
4

(b) The Hawaiian happy-face spider, Theridion grallator, is found on several of the Hawaiian
islands. Some of the spiders have a very distinctive pattern on their bodies as shown in
Fig. 1.3.

Fig. 1.3

(i) State one feature, visible in Fig. 1.3, that identifies T. grallator as an arachnid.

..................................................................................................................................... [1]

(ii) Scientists think that the pattern on the bodies of the spiders is an adaptive feature.

Explain the term adaptive feature with reference to this pattern.

...........................................................................................................................................

...........................................................................................................................................

...........................................................................................................................................

...........................................................................................................................................

...........................................................................................................................................

...........................................................................................................................................

..................................................................................................................................... [3]

© UCLES 2019 0610/42/O/N/19

Page 444 of 877


5

(c) DNA can be extracted from the webs of spiders. This DNA can be used to identify the species
of spider that made the web, and the species of prey caught in the web.

Explain how DNA extracted from spider webs can be used to identify different species.

...................................................................................................................................................

...................................................................................................................................................

...................................................................................................................................................

...................................................................................................................................................

............................................................................................................................................. [2]

(d) Many species of spider are thought to be endangered.

State three reasons why animal species such as spiders become endangered.

1 ................................................................................................................................................

...................................................................................................................................................

2 ................................................................................................................................................

...................................................................................................................................................

3 ................................................................................................................................................

...................................................................................................................................................
[3]

[Total: 13]

© UCLES 2019 0610/42/O/N/19 [Turn over


Page 445 of 877
7

2 Fig. 2.1 shows part of a cross-section of the stem of a young sunflower plant.

area Y

Fig. 2.1

(a) Draw a circle around one vascular bundle on Fig. 2.1.

Label the xylem in the vascular bundle with the letter X. [2]

(b) Explain how the cells in area Y are able to support the stem so that it stays upright.

...................................................................................................................................................

...................................................................................................................................................

...................................................................................................................................................

...................................................................................................................................................

...................................................................................................................................................

...................................................................................................................................................

............................................................................................................................................. [3]

© UCLES 2019 0610/42/O/N/19 [Turn over


Page 447 of 877
8

(c) Researchers used carbon dioxide that contained a traceable source of carbon (13C) to
investigate translocation of sucrose from the leaves of bean plants, Phaseolus vulgaris.

Fig. 2.2 shows that glucose produced in photosynthesis is converted to sucrose for
translocation.

carbon dioxide glucose sucrose

Fig. 2.2

Researchers selected four plants, Q, R, S and T, which had leaves that were of similar sizes.
The leaves on the four plants were supplied with 13CO2.

After the leaves had started to make sucrose, the researchers cut away a ring of tissue in
different places as shown in Fig. 2.3. The rings of tissue that were removed from plants R, S
and T contained the phloem.

Q R S T

13CO 13CO 13CO 13CO


2 2 2 2

Key: the positions on the stems where rings of tissue containing phloem were removed.

Fig. 2.3

© UCLES 2019 0610/42/O/N/19

Page 448 of 877


9

The quantities of sucrose containing 13C in the shoot tips and in the roots were determined.

The results are shown in Table 2.1.

Table 2.1

quantity of sucrose containing 13C / arbitrary units


plant
shoot tip root
Q 3.24 0.94
R 0.00 0.44
S 4.14 0.00
T 0.00 0.00

Describe and explain the effect of removing the phloem on the translocation of sucrose in
plants Q, R, S and T.

...................................................................................................................................................

...................................................................................................................................................

...................................................................................................................................................

...................................................................................................................................................

...................................................................................................................................................

...................................................................................................................................................

...................................................................................................................................................

...................................................................................................................................................

...................................................................................................................................................

...................................................................................................................................................

............................................................................................................................................. [5]

[Total: 10]

© UCLES 2019 0610/42/O/N/19 [Turn over


Page 449 of 877
10

3 Cells in the lining of the stomach secrete gastric juice, which contains hydrochloric acid and
pepsin.

(a) (i) State two roles of hydrochloric acid in the stomach.

1 ........................................................................................................................................

2 ........................................................................................................................................
[2]

(ii) Describe the function of pepsin.

...........................................................................................................................................

...........................................................................................................................................

...........................................................................................................................................

..................................................................................................................................... [2]

(b) Define the term assimilation.

...................................................................................................................................................

...................................................................................................................................................

...................................................................................................................................................

............................................................................................................................................. [2]

(c) There are stem cells in the epithelial tissue that forms the lining of the stomach.

Explain why these stem cells are necessary.

...................................................................................................................................................

...................................................................................................................................................

...................................................................................................................................................

...................................................................................................................................................

............................................................................................................................................. [2]

© UCLES 2019 0610/42/O/N/19

Page 450 of 877


11

(d) The epithelial cells of the small intestine have many microvilli.

State the role of the microvilli.

...................................................................................................................................................

...................................................................................................................................................

...................................................................................................................................................

............................................................................................................................................. [2]

© UCLES 2019 0610/42/O/N/19 [Turn over


Page 451 of 877
12

(e) Lactobacillus is a type of bacterium. A study was carried out to investigate the ability of
Lactobacillus to attach to the epithelial cells that form the lining of the small intestine.

Researchers added Lactobacillus bacteria to epithelial cells that had been grown in Petri
dishes.

Every 15 minutes, the researchers estimated the average number of bacteria that were
attached to the epithelial cells in the Petri dishes.

The results are shown in Fig. 3.1.

140

130

120

110

100

90

80
average number
of bacteria
70
attached to
epithelial cells
60

50

40

30

20

10

0
0 10 20 30 40 50 60 70 80 90 100 110 120 130 140
time / minutes

Fig. 3.1

© UCLES 2019 0610/42/O/N/19

Page 452 of 877


13

Calculate the percentage increase in the average number of bacteria attached to epithelial
cells from 45 minutes to 75 minutes.

average number of bacteria at 45 minutes .................. .

average number of bacteria at 75 minutes .................. .

Give your answer to the nearest whole number.

Space for working.

............................................................ %
[3]

[Total: 13]

© UCLES 2019 0610/42/O/N/19 [Turn over


Page 453 of 877
15

4 (a) Explain how the human gas exchange system is protected against pathogens.

...................................................................................................................................................

...................................................................................................................................................

...................................................................................................................................................

...................................................................................................................................................

...................................................................................................................................................

...................................................................................................................................................

...................................................................................................................................................

...................................................................................................................................................

............................................................................................................................................. [4]

(b) Tuberculosis (TB) is a transmissible disease caused by a bacterial pathogen. The spread of
this disease can be controlled by vaccination.

Explain how vaccination provides a defence against transmissible diseases.

...................................................................................................................................................

...................................................................................................................................................

...................................................................................................................................................

...................................................................................................................................................

...................................................................................................................................................

...................................................................................................................................................

...................................................................................................................................................

...................................................................................................................................................

............................................................................................................................................. [4]

(c) TB is a disease that can be treated with antibiotics. HIV infection cannot be treated with
antibiotics.

Explain why viral diseases cannot be treated with antibiotics.

...................................................................................................................................................

...................................................................................................................................................

...................................................................................................................................................

...................................................................................................................................................

............................................................................................................................................. [2]
© UCLES 2019 0610/42/O/N/19 [Turn over
Page 455 of 877
16

(d) Lung cancer is a disease that is strongly linked with smoking tobacco.

Fig. 4.1 shows some data about smoking and lung cancer in country A between 1900 and
2020 (2020 data has been estimated):

• percentage of the male population that smoke tobacco


• percentage of the female population that smoke tobacco
• number of deaths in males from lung cancer per 100 000 of the male population
• number of deaths in females from lung cancer per 100 000 of the female population.

percentage of deaths from


population that lung cancer per
smoke tobacco 100 000
100 100
90 90
80 80
70 70
60 60
50 50
40 40
30 30
20 20
10 10
0 0
1900 1920 1940 1960 1980 2000 2020
Key: year

percentage of population that smoke tobacco deaths from lung cancer per 100 000
male female male female

Fig. 4.1

(i) Describe the differences between the percentages of males and females in country A
that smoke as shown in Fig. 4.1.

...........................................................................................................................................

...........................................................................................................................................

...........................................................................................................................................

...........................................................................................................................................

...........................................................................................................................................

...........................................................................................................................................

..................................................................................................................................... [3]

(ii) Fig. 4.2 shows the same smoking statistics for country B between 1950 and 2020.

© UCLES 2019 0610/42/O/N/19

Page 456 of 877


17

percentage of deaths from


population that lung cancer per
smoke tobacco 100 000
100 100
90 90
80 80
70 70
60 60
50 50
40 40
30 30
20 20
10 10
0 0
1900 1920 1940 1960 1980 2000 2020
Key: year

percentage of population that smoke tobacco deaths from lung cancer per 100 000
male female male female

Fig. 4.2

Discuss the evidence for and against this statement:

‘The changes in the percentage of the population that smoke tobacco, and the deaths
from lung cancer per 100 000, follow the same trends in males and in females.’

Use the information for country A in Fig. 4.1 and country B in Fig. 4.2 in your answer.

...........................................................................................................................................

...........................................................................................................................................

...........................................................................................................................................

...........................................................................................................................................

...........................................................................................................................................

...........................................................................................................................................

...........................................................................................................................................

...........................................................................................................................................

...........................................................................................................................................

...........................................................................................................................................

...........................................................................................................................................

...........................................................................................................................................

..................................................................................................................................... [6]

[Total: 19]
© UCLES 2019 0610/42/O/N/19 [Turn over
Page 457 of 877
18

5 Grass plants are wind-pollinated. Fig. 5.1 shows a flower from a species of grass plant.

B
D

Fig. 5.1

(a) (i) Describe and explain how the features of the flower shown in Fig. 5.1 are adaptations for
wind-pollination.

...........................................................................................................................................

...........................................................................................................................................

...........................................................................................................................................

...........................................................................................................................................

...........................................................................................................................................

...........................................................................................................................................

..................................................................................................................................... [3]

(ii) State one letter in Fig. 5.1 that identifies a structure where meiosis occurs.

.................................... [1]

© UCLES 2019 0610/42/O/N/19

Page 458 of 877


19

(b) Wheat has flowers similar to those in Fig. 5.1, but reproduces by self-pollination.

Explain the consequences of self-pollination for a population of plants.

...................................................................................................................................................

...................................................................................................................................................

...................................................................................................................................................

...................................................................................................................................................

...................................................................................................................................................

...................................................................................................................................................

...................................................................................................................................................

...................................................................................................................................................

............................................................................................................................................. [4]

(c) Wheat is an important crop plant in many countries. Wheat can be infected by diseases
leading to total loss of the crop which results in famine.

Outline factors, other than plant diseases, that can cause famines.

...................................................................................................................................................

...................................................................................................................................................

...................................................................................................................................................

...................................................................................................................................................

...................................................................................................................................................

...................................................................................................................................................

............................................................................................................................................. [3]

© UCLES 2019 0610/42/O/N/19 [Turn over


Page 459 of 877
20

(d) Black stem rust is a disease of wheat that is caused by a fungus.

Plant breeders used two varieties of wheat to produce a variety of wheat that is both
rust-resistant and has a high yield.

Fig. 5.2 shows the breeding programme that was used.

step 1 rust-resistant variety of × variety of wheat with a high yield


wheat with a low yield

step 2 new variety 1 × variety of wheat with a high yield

step 3 selected rust-resistant offspring × variety of wheat with a high yield

repeat step 3 for several generations

rust-resistant variety with a high yield

Fig. 5.2

(i) Suggest how plant breeders make sure that the plants that they use for step 3 are
rust-resistant.

...........................................................................................................................................

...........................................................................................................................................

...........................................................................................................................................

...........................................................................................................................................

..................................................................................................................................... [2]

(ii) Suggest why step 3 is repeated for many generations before the new rust-resistant
variety is made available for farmers to grow.

...........................................................................................................................................

...........................................................................................................................................

..................................................................................................................................... [1]

© UCLES 2019 0610/42/O/N/19

Page 460 of 877


21

(e) Wheat plants are monocotyledons.

State one feature of monocotyledons that can be used to distinguish them from dicotyledons.

............................................................................................................................................. [1]

[Total: 15]

© UCLES 2019 0610/42/O/N/19 [Turn over


Page 461 of 877
22

6 Carbon dioxide forms approximately 0.04% of the atmosphere.

Fig. 6.1 shows part of the carbon cycle.

carbon dioxide
A B

feeding death
plants animals decomposers

death
D

coal, oil, gas

Fig. 6.1

(a) Complete Table 6.1 by naming the processes labelled A to D in Fig. 6.1.

Table 6.1

letter on
name of the process in the carbon cycle
Fig. 6.1

[4]

© UCLES 2019 0610/42/O/N/19

Page 462 of 877


23

(b) Complete the sentences by writing the missing words in the spaces.

Carbon dioxide is a greenhouse gas. Greenhouse gases trap .................................................

in the atmosphere. Water vapour is the most common greenhouse gas in the atmosphere.

Another greenhouse gas is methane which is released from .................................................

and ................................................. . Carbon dioxide and methane released from human

activities over the past 200 years have contributed to the .................................................

greenhouse effect.

Other atmospheric pollutants can cause acid rain. One of these is

................................................. which is a waste gas from some power stations.

One source of pollution in both aquatic and terrestrial ecosystems is

................................................. plastics. [6]

[Total: 10]

© UCLES 2019 0610/42/O/N/19

Page 463 of 877


Cambridge Assessment International Education
Cambridge International General Certificate of Secondary Education
* 9 8 4 9 3 1 5 0 1 1 *

BIOLOGY 0610/43
Paper 4 Theory (Extended) October/November 2019
1 hour 15 minutes
Candidates answer on the Question Paper.
No Additional Materials are required.

READ THESE INSTRUCTIONS FIRST

Write your centre number, candidate number and name on all the work you hand in.
Write in dark blue or black pen.
You may use an HB pencil for any diagrams or graphs.
Do not use staples, paper clips, glue or correction fluid.
DO NOT WRITE IN ANY BARCODES.

Answer all questions.

Electronic calculators may be used.


You may lose marks if you do not show your working or if you do not use appropriate units.

At the end of the examination, fasten all your work securely together.
The number of marks is given in brackets [ ] at the end of each question or part question.

This syllabus is regulated for use in England, Wales and Northern Ireland as a Cambridge International Level 1/Level 2 Certificate.

This document consists of 16 printed pages and 4 blank pages.

DC (ST/CT) 173044/4
© UCLES 2019 [Turn over

Page 465 of 877


2

1 (a) The ant-mimic jumping spider, Myrmarachne formicaria, is shown in Fig. 1.1.

The common name of this species describes its behaviour. It is an arachnid that tricks its prey
because it looks like the insects that it eats.

Fig. 1.1

(i) Suggest which trophic level in a food chain M. formicaria could belong to.

..................................................................................................................................... [1]

(ii) State the genus of the spider shown in Fig. 1.1.

..................................................................................................................................... [1]

(iii) Some keys use paired choices of features to identify species such as the ant-mimic
jumping spider.

State the name of this type of key.

..................................................................................................................................... [1]

© UCLES 2019 0610/43/O/N/19

Page 466 of 877


3

(b) Spiders are classified as arachnids. Arachnids are one of the main groups of arthropods.

Fig. 1.2 shows diagrams of six arthropods, four of which are arachnids.

A B C

D E F not to scale

Fig. 1.2

(i) State two common features of all the arthropods, visible in Fig. 1.2.

1 ........................................................................................................................................

2 ........................................................................................................................................
[2]

(ii) State two common features of all arachnids that can be used to distinguish them from
other arthropods.

1 ........................................................................................................................................

2 ........................................................................................................................................
[2]

(iii) State the letters of the four arachnids shown in Fig. 1.2.

..................................................................................................................................... [2]

(c) The features shown in Fig. 1.2 are morphological features. Many traditional methods of
classification used morphology.

State the name of one other type of feature that can also be used in classification.

............................................................................................................................................. [1]

[Total: 10]
© UCLES 2019 0610/43/O/N/19 [Turn over
Page 467 of 877
4

2 Plants produce glucose in leaves and convert some of it to sucrose.

(a) (i) Explain how glucose is produced in leaves.

...........................................................................................................................................

...........................................................................................................................................

...........................................................................................................................................

...........................................................................................................................................

...........................................................................................................................................

...........................................................................................................................................

..................................................................................................................................... [3]

(ii) State the name of the process that plants use to move sucrose from a source to a sink.

..................................................................................................................................... [1]

(iii) Roots can be an example of a sink.

Explain why sometimes roots act as a source rather than a sink.

...........................................................................................................................................

...........................................................................................................................................

...........................................................................................................................................

...........................................................................................................................................

..................................................................................................................................... [2]

© UCLES 2019 0610/43/O/N/19

Page 468 of 877


6

(b) The movement of sucrose in plants can be modelled using laboratory apparatus.

Fig. 2.1 shows the apparatus used to model the movement of sucrose in a plant:

• Partially permeable bags were attached tightly to the ends of tube Q.


• The bag representing a source was filled with a coloured sucrose solution.
• The bag representing a sink was filled with water.
• The containers and tube Q and tube S were filled with water.

tube Q

bag representing a
source containing a
coloured sucrose
solution bag representing
a sink containing
partially water
permeable
membrane P partially permeable
membrane R
water
tube S

Fig. 2.1

Fig. 2.2 shows the position of the coloured sucrose solution 30 minutes after the apparatus
was set up.

The arrows show the direction of the movement of the liquids.

movement of sucrose solution


tube Q

bag representing a
source
bag representing
a sink
partially
permeable
membrane P partially permeable
membrane R
water
tube S

movement of water

Fig. 2.2

© UCLES 2019 0610/43/O/N/19

Page 470 of 877


7

(i) State the name of the tissue represented by tube Q and the name of the tissue
represented by tube S in Fig. 2.2.

Q .......................................................................................................................................

S ........................................................................................................................................
[2]

(ii) Explain why the sucrose solution moves along tube Q in the model in Fig. 2.2.

...........................................................................................................................................

...........................................................................................................................................

...........................................................................................................................................

...........................................................................................................................................

...........................................................................................................................................

...........................................................................................................................................

...........................................................................................................................................

...........................................................................................................................................

..................................................................................................................................... [4]

(c) In plants the movement of sucrose is usually continuous. However, after 2 hours the
movement of sucrose in tube Q in Fig. 2.2 stopped.

Suggest why the movement of sucrose in tube Q stopped.

...................................................................................................................................................

...................................................................................................................................................

............................................................................................................................................. [1]

(d) Amino acids are also transported through plants.

State the name of the mineral ion that is used to make amino acids.

............................................................................................................................................. [1]

[Total: 14]

© UCLES 2019 0610/43/O/N/19 [Turn over


Page 471 of 877
8

3 Carbohydrates are an important component of a balanced diet.

The flow chart in Fig. 3.1 shows some of the processes that happen to carbohydrates in food that
is eaten.

ingestion of large biological molecules

chemical digestion

absorption into the blood via the alimentary canal

increase in blood glucose concentration

assimilation in the liver

Fig. 3.1

© UCLES 2019 0610/43/O/N/19

Page 472 of 877


9

(a) Describe what happens to carbohydrates in the human body between ingestion and
assimilation in the liver.

Use the information in Fig. 3.1 as a guide.

...................................................................................................................................................

...................................................................................................................................................

...................................................................................................................................................

...................................................................................................................................................

...................................................................................................................................................

...................................................................................................................................................

...................................................................................................................................................

...................................................................................................................................................

...................................................................................................................................................

...................................................................................................................................................

...................................................................................................................................................

...................................................................................................................................................

...................................................................................................................................................

...................................................................................................................................................

...................................................................................................................................................

...................................................................................................................................................

...................................................................................................................................................

...................................................................................................................................................

...................................................................................................................................................

............................................................................................................................................. [8]

© UCLES 2019 0610/43/O/N/19 [Turn over


Page 473 of 877
11

(b) Mineral salts are another important component of a balanced diet.

State the importance of calcium ions and iron ions in a balanced diet.

calcium ions ..............................................................................................................................

...................................................................................................................................................

...................................................................................................................................................

iron ions ....................................................................................................................................

...................................................................................................................................................

...................................................................................................................................................
[4]

(c) Consuming too much of some mineral salts, such as sodium chloride, increases the risk of
developing coronary heart disease (CHD).

Doctors studied the effect of diet on the risk of developing CHD.

The doctors first selected volunteers who had a high salt diet.

The doctors assessed the volunteers’ overall risk of developing CHD and monitored their
blood pressure.

(i) List two factors, other than diet and blood pressure, that the doctors considered when
assessing the overall risk of the volunteers developing CHD.

1 ........................................................................................................................................

2 ........................................................................................................................................
[2]

(ii) The doctors used urine tests to identify volunteers who had a high salt diet.

Explain why urine tests are a good indicator of how much salt has been consumed.

...........................................................................................................................................

...........................................................................................................................................

...........................................................................................................................................

...........................................................................................................................................

..................................................................................................................................... [2]

© UCLES 2019 0610/43/O/N/19 [Turn over


Page 475 of 877
12

(d) The volunteers were divided into two groups.

The mass of salt consumed by both groups was changed every 4 weeks:
• low salt intake for 4 weeks
• medium salt intake for 4 weeks
• high salt intake for 4 weeks.

In addition, group 2 was given other changes to their diet but group 1 was not.

(i) Suggest one component of the diet of group 2, other than salt, that the doctors changed
to further reduce the risk of developing CHD.

..................................................................................................................................... [1]

The systolic blood pressure of the volunteers was measured every 4 weeks.
These results are shown in Fig. 3.2.

18.0
Key:
17.5 group 1
group 2
average systolic 17.0
blood pressure
/ kPa 16.5

16.0

15.5

15.0
low medium high
salt intake

Fig. 3.2

(ii) Calculate the percentage increase in the average systolic blood pressure of the group 1
volunteers when they increased their salt intake from low to high.

low salt intake ............................... kPa

high salt intake ............................... kPa

Give your answer to the nearest whole number.

Space for working.

.......................................................... %
[3]

© UCLES 2019 0610/43/O/N/19

Page 476 of 877


13

(iii) The doctors concluded that some diets reduce the risk of CHD.

Give evidence from Fig. 3.2 to support this conclusion.

...........................................................................................................................................

...........................................................................................................................................

...........................................................................................................................................

...........................................................................................................................................

...........................................................................................................................................

...........................................................................................................................................

..................................................................................................................................... [3]

[Total: 23]

4 Forest ecosystems are threatened by many human activities.

(a) (i) Describe reasons why people cut down forests.

...........................................................................................................................................

...........................................................................................................................................

...........................................................................................................................................

...........................................................................................................................................

...........................................................................................................................................

...........................................................................................................................................

..................................................................................................................................... [3]

(ii) Describe how forests can be conserved.

...........................................................................................................................................

...........................................................................................................................................

...........................................................................................................................................

...........................................................................................................................................

...........................................................................................................................................

...........................................................................................................................................

..................................................................................................................................... [3]

© UCLES 2019 0610/43/O/N/19 [Turn over


Page 477 of 877
14

(b) Ecologists in one country were concerned that some mammals had been affected by
deforestation.

Fig. 4.1 is a diagram showing how deforestation affected one area of forest.

Key:
forest
cleared land

original forest

many trees were cut down

small forest area

remaining forest

large forest area

Fig. 4.1

The ecologists studied eight species of mammal. They recorded how many of the small and
large areas of the remaining forest contained each of the eight species.

Two species, the black rat and the house mouse, are not usually found in this forest
ecosystem. They were introduced to the area by humans many years ago before any trees
were cut down.

The other six species are known to live in this forest ecosystem.

© UCLES 2019 0610/43/O/N/19

Page 478 of 877


15

Table 4.1

percentage of small percentage of large


average body
species of mammal forest areas containing forest areas containing
mass / g
the species the species
house mouse* 18 46 42
brown antechinus 40 62 83
swamp rat 130 15 25
bush rat 160 85 100
black rat* 200 15 0
southern brown bandicoot 850 31 92
long-nosed potoroo 1100 8 17
long-nosed bandicoot 1300 8 25
*introduced species

(i) State which mammal in Table 4.1 showed almost no preference between small and large
areas of forest.

..................................................................................................................................... [1]

(ii) The ecologists made a hypothesis:

‘Larger areas of forest are better for the conservation of mammals.’

Discuss the evidence from Table 4.1 to support or reject this hypothesis.

...........................................................................................................................................

...........................................................................................................................................

...........................................................................................................................................

...........................................................................................................................................

...........................................................................................................................................

...........................................................................................................................................

...........................................................................................................................................

...........................................................................................................................................

..................................................................................................................................... [4]

[Total: 11]

© UCLES 2019 0610/43/O/N/19 [Turn over


Page 479 of 877
16

5 Genetic engineering is an example of an important biotechnology.

(a) Complete the passage below by filling in each space with a suitable word.

DNA is a biological molecule found in the ................................................. of an animal cell.

The sequence of the ................................................. in mRNA determines the order of the

amino acids that will be assembled into a ................................................. .

When carrying out genetic engineering, sections of human DNA called

................................................. are cut using restriction enzymes. Next bacterial plasmids

are cut with the ................................................. restriction enzymes to form complementary

................................................. ends.

The cut section of human DNA is inserted into the cut plasmid and they are joined together

to form a ................................................. plasmid. These plasmids are inserted into

................................................. and replication occurs. This process is used to produce

human ................................................. that is used to treat people with diabetes.


[9]

(b) In addition to genetic engineering, enzymes are also useful in other biotechnologies.

State two examples of how enzymes are used in another biotechnology.

1 ................................................................................................................................................

2 ................................................................................................................................................
[2]

[Total: 11]

© UCLES 2019 0610/43/O/N/19

Page 480 of 877


17

6 Fig. 6.1 is a photomicrograph of a blood clot.

L
J

Fig. 6.1

(a) Describe how a blood clot forms.

Use the letters in Fig. 6.1 in your answer.

...................................................................................................................................................

...................................................................................................................................................

...................................................................................................................................................

...................................................................................................................................................

...................................................................................................................................................

...................................................................................................................................................

...................................................................................................................................................

...................................................................................................................................................

...................................................................................................................................................

...................................................................................................................................................

............................................................................................................................................. [5]

© UCLES 2019 0610/43/O/N/19 [Turn over


Page 481 of 877
18

(b) State the importance of blood clotting.

...................................................................................................................................................

...................................................................................................................................................

...................................................................................................................................................

...................................................................................................................................................

............................................................................................................................................. [2]

(c) There are four blood group phenotypes A, B, AB and O in humans.

(i) Define the term phenotype.

...........................................................................................................................................

...........................................................................................................................................

..................................................................................................................................... [1]

(ii) State the name of the type of inheritance that is shown by blood groups.

..................................................................................................................................... [1]

(iii) State the two possible genotypes for a person who has the phenotype blood group A.

1 ........................................................................................................................................

2 ........................................................................................................................................
[2]

[Total: 11]

© UCLES 2019 0610/43/O/N/19

Page 482 of 877


Cambridge Assessment International Education
Cambridge International General Certificate of Secondary Education
* 7 8 6 9 8 4 1 0 1 9 *

BIOLOGY 0610/42
Paper 4 Theory (Extended) February/March 2019
1 hour 15 minutes
Candidates answer on the Question Paper.
No Additional Materials are required.

READ THESE INSTRUCTIONS FIRST

Write your centre number, candidate number and name on all the work you hand in.
Write in dark blue or black pen.
You may use an HB pencil for any diagrams or graphs.
Do not use staples, paper clips, glue or correction fluid.
DO NOT WRITE IN ANY BARCODES.

Answer all questions.

Electronic calculators may be used.


You may lose marks if you do not show your working or if you do not use appropriate units.

At the end of the examination, fasten all your work securely together.
The number of marks is given in brackets [ ] at the end of each question or part question.

This syllabus is regulated for use in England, Wales and Northern Ireland as a Cambridge International Level 1/Level 2 Certificate.

This document consists of 15 printed pages and 1 blank page.

DC (ST/SG) 165036/2
© UCLES 2019 [Turn over

Page 485 of 877


2

1 The kidney is one of the main excretory organs of the body.

(a) Define the term excretion.

...................................................................................................................................................

...................................................................................................................................................

...................................................................................................................................................

...................................................................................................................................................

............................................................................................................................................. [3]

(b) One of the roles of the kidney is to filter the blood.

Fig. 1.1 shows a section of a kidney.

Fig. 1.1

State the name of the parts labelled A, B and C on Fig. 1.1.

A ...............................................................................................................................................

B ...............................................................................................................................................

C ...............................................................................................................................................
[3]

© UCLES 2019 0610/42/F/M/19

Page 486 of 877


3

(c) Table 1.1 shows the concentrations of four solutes:

• in the blood in the renal artery


• in the fluid in the kidney tubule
• in the urine.

Table 1.1

solute concentration / g dm–3


solute blood in the fluid in the
urine
renal artery kidney tubule
glucose 0.9 0.9 0.0
protein 83.0 0.0 0.0
salts 8.0 8.0 16.5
urea 0.2 0.2 20.0

(i) Calculate the percentage increase in the concentration of urea between the blood in the
renal artery and the urine.

Show your working.

............................................................ %
[2]

(ii) Describe the results for the concentration of salts shown in Table 1.1.

...........................................................................................................................................

...........................................................................................................................................

...........................................................................................................................................

...........................................................................................................................................

..................................................................................................................................... [2]

(iii) State the reason for the difference in the concentration of protein between the blood in
the renal artery and the fluid in the kidney tubule.

...........................................................................................................................................

..................................................................................................................................... [1]

© UCLES 2019 0610/42/F/M/19 [Turn over


Page 487 of 877
4

(iv) State the reason for the difference in the concentration of glucose between the fluid in
the kidney tubule and the urine.

...........................................................................................................................................

..................................................................................................................................... [1]

(d) Dialysis is a treatment used for people with kidney failure.

Some people with kidney failure are given a kidney transplant.

State the advantages of having a kidney transplant instead of dialysis.

...................................................................................................................................................

...................................................................................................................................................

...................................................................................................................................................

...................................................................................................................................................

...................................................................................................................................................

...................................................................................................................................................

............................................................................................................................................. [3]

[Total: 15]

© UCLES 2019 0610/42/F/M/19

Page 488 of 877


5

2 (a) Fig. 2.1 shows some flowers of a snapdragon plant, Antirrhinum majus.

Snapdragons are insect-pollinated plants.

Fig. 2.1

(i) State one feature visible in Fig. 2.1 that suggests these flowers are insect-pollinated.

..................................................................................................................................... [1]

(ii) State how self-pollination differs from cross-pollination.

...........................................................................................................................................

...........................................................................................................................................

..................................................................................................................................... [1]

(iii) Suggest why self-pollination might be advantageous to a population of plants.

...........................................................................................................................................

...........................................................................................................................................

...........................................................................................................................................

...........................................................................................................................................

...........................................................................................................................................

...........................................................................................................................................

..................................................................................................................................... [3]

© UCLES 2019 0610/42/F/M/19 [Turn over


Page 489 of 877
6

(b) Petal colour in the flowers of snapdragon plants shows co-dominance.

The gene for petal colour has two co-dominant alleles:

• CR for red petals


• CW for white petals

Table 2.1 shows the genotypes and phenotypes of snapdragon plants with different petal
colours.

Table 2.1

genotype phenotype
C RC R red
CWCW white
C RC W pink

(i) Explain the term co-dominance.

...........................................................................................................................................

...........................................................................................................................................

...........................................................................................................................................

...........................................................................................................................................

..................................................................................................................................... [2]

© UCLES 2019 0610/42/F/M/19

Page 490 of 877


7

(ii) A botanist crossed two snapdragon plants with pink flowers.

Complete the genetic diagram to show the ratio of expected phenotypes in the offspring.

parental phenotypes pink flower × pink flower

parental genotypes CRCW × CRCW

gametes ........... ........... ........... ...........

offspring genotypes .................... .................... .................... ....................

offspring phenotypes .................... .................... .................... ....................

phenotypic ratio .................................................................................................................


[4]

(iii) The botanist wanted to produce a generation of snapdragons that all had pink flowers.

State the phenotypes of the parent plants that the botanist would need to cross.

Explain your answer.

parent phenotypes ............................................................................................................

explanation ........................................................................................................................

...........................................................................................................................................

...........................................................................................................................................
[2]

[Total: 13]

© UCLES 2019 0610/42/F/M/19 [Turn over


Page 491 of 877
8

3 Reflexes are simple responses that protect the body.

(a) The letters A to G show the components involved in a reflex action.

A stimulus
B motor neurone
C sensory neurone
D receptor cell
E response
F relay neurone
G effector

Put the letters into the correct sequence involved in a reflex action. Two have been done for
you.

A E
[1]

(b) Impulses travel along neurones.

Describe how impulses pass from one neurone to another neurone across a synapse.

...................................................................................................................................................

...................................................................................................................................................

...................................................................................................................................................

...................................................................................................................................................

...................................................................................................................................................

...................................................................................................................................................

...................................................................................................................................................

...................................................................................................................................................

............................................................................................................................................. [4]

© UCLES 2019 0610/42/F/M/19

Page 492 of 877


9

(c) Drugs such as heroin affect the nervous system. When users stop taking heroin they may
experience withdrawal symptoms.

(i) Outline the short-term effects of heroin on the body.

...........................................................................................................................................

...........................................................................................................................................

...........................................................................................................................................

...........................................................................................................................................

...........................................................................................................................................

...........................................................................................................................................

..................................................................................................................................... [3]

(ii) State two withdrawal symptoms that heroin users may experience.

...........................................................................................................................................

..................................................................................................................................... [2]

(iii) Suggest why heroin abuse may increase criminal activity.

...........................................................................................................................................

..................................................................................................................................... [1]

[Total: 11]

© UCLES 2019 0610/42/F/M/19 [Turn over


Page 493 of 877
10

4 Pollution is the harm done to the environment by the release of substances from human activities.

Table 4.1 shows the names of some pollutants, their sources and their effects on the environment.

Table 4.1

pollutant source effect on environment

acid rain

enhanced greenhouse
carbon dioxide
effect
enhanced greenhouse
cattle and rice farming
effect

fertilisers crop farming eutrophication

(a) Complete Table 4.1. [4]

(b) When fertiliser is applied to fields, it can lead to eutrophication in lakes and rivers.

(i) Describe and explain what happens in lakes when eutrophication occurs.

...........................................................................................................................................

...........................................................................................................................................

...........................................................................................................................................

...........................................................................................................................................

...........................................................................................................................................

...........................................................................................................................................

...........................................................................................................................................

...........................................................................................................................................

...........................................................................................................................................

...........................................................................................................................................

...........................................................................................................................................

...........................................................................................................................................

..................................................................................................................................... [6]

© UCLES 2019 0610/42/F/M/19

Page 494 of 877


11

(ii) Suggest ways in which a farmer could reduce the chances of eutrophication occurring
when applying fertiliser to crops.

...........................................................................................................................................

...........................................................................................................................................

...........................................................................................................................................

...........................................................................................................................................

..................................................................................................................................... [2]

[Total: 12]

© UCLES 2019 0610/42/F/M/19 [Turn over


Page 495 of 877
12

5 Scientists investigated the effect of cuticle thickness on water loss from the leaves of the balsam
fir tree, Abies balsamea.

The leaves were divided into three groups:

A – thick cuticle
B – medium cuticle
C – thin cuticle

Samples of leaves from each group were weighed. The leaves were placed on a tray in dry air at
20 °C. The samples of leaves were reweighed, at intervals, over 15 hours.

The scientists calculated the mass of each sample of leaves as a percentage of the initial mass.

Fig. 5.1 shows the results.

100

95

mass of the 90
leaves as a
percentage
of the initial
mass 85

80 B

75
0 1 2 3 4 5 6 7 8 9 10 11 12 13 14 15

time / hours

Fig. 5.1

© UCLES 2019 0610/42/F/M/19

Page 496 of 877


13

(a) (i) Describe and explain the results shown in Fig. 5.1.

...........................................................................................................................................

...........................................................................................................................................

...........................................................................................................................................

...........................................................................................................................................

...........................................................................................................................................

...........................................................................................................................................

...........................................................................................................................................

...........................................................................................................................................

...........................................................................................................................................

...........................................................................................................................................

..................................................................................................................................... [5]

(ii) The investigation was repeated on a day when the air humidity was higher.

Suggest and explain the effect that this would have on the results.

...........................................................................................................................................

...........................................................................................................................................

...........................................................................................................................................

...........................................................................................................................................

...........................................................................................................................................

...........................................................................................................................................

..................................................................................................................................... [3]

(b) The leaves of pine trees show xerophytic features. Stems and roots also show xerophytic
adaptations.

State one adaptation of the stem and one adaptation of the root in xerophytes.

stem ..........................................................................................................................................

root ...........................................................................................................................................
[2]

© UCLES 2019 0610/42/F/M/19 [Turn over


Page 497 of 877
14

(c) Water is one of the raw materials needed for the production of sugars in photosynthesis.

(i) State the name of the other raw material needed for photosynthesis.

..................................................................................................................................... [1]

(ii) State three ways a plant uses the sugars produced in photosynthesis.

1 ........................................................................................................................................

2 ........................................................................................................................................

3 ........................................................................................................................................
[3]

[Total: 14]

6 Fig. 6.1 is a diagram of the virus that causes measles.

Fig. 6.1

(a) (i) State the name of the parts of the virus shown in Fig. 6.1 labelled X and Y.

X ........................................................................................................................................

Y ........................................................................................................................................
[2]

(ii) Bacteria belong to the Prokaryote kingdom.

State two ways in which the structure of bacteria differs from the structure of viruses.

1 ........................................................................................................................................

2 ........................................................................................................................................
[2]

© UCLES 2019 0610/42/F/M/19

Page 498 of 877


15

(b) Viruses and some bacteria are pathogenic. Diseases caused by pathogens are transmissible.

(i) State two ways that a pathogen can be transmitted indirectly.

1 ........................................................................................................................................

2 ........................................................................................................................................
[2]

(ii) The body has barriers to defend itself against pathogens.

State two mechanical barriers of the body.

1 ........................................................................................................................................

2 ........................................................................................................................................
[2]

(c) Some white blood cells produce antibodies as part of the body’s defence against pathogens.

Describe the role of antibodies in defence of the body.

...................................................................................................................................................

...................................................................................................................................................

...................................................................................................................................................

...................................................................................................................................................

...................................................................................................................................................

...................................................................................................................................................

...................................................................................................................................................

...................................................................................................................................................

............................................................................................................................................. [4]

(d) The immunity gained after infection by a pathogen is active immunity.

Explain how active immunity differs from passive immunity.

...................................................................................................................................................

...................................................................................................................................................

...................................................................................................................................................

...................................................................................................................................................

...................................................................................................................................................

...................................................................................................................................................

............................................................................................................................................. [3]

[Total: 15]
© UCLES 2019 0610/42/F/M/19

Page 499 of 877


Cambridge International Examinations
Cambridge International General Certificate of Secondary Education
* 1 6 3 0 2 3 2 2 6 2 *

BIOLOGY 0610/41
Paper 4 Theory (Extended) May/June 2018
1 hour 15 minutes
Candidates answer on the Question Paper.
No Additional Materials are required.

READ THESE INSTRUCTIONS FIRST

Write your Centre number, candidate number and name on all the work you hand in.
Write in dark blue or black pen.
You may use an HB pencil for any diagrams or graphs.
Do not use staples, paper clips, glue or correction fluid.
DO NOT WRITE IN ANY BARCODES.

Answer all questions.

Electronic calculators may be used.


You may lose marks if you do not show your working or if you do not use appropriate units.

At the end of the examination, fasten all your work securely together.
The number of marks is given in brackets [ ] at the end of each question or part question.

This syllabus is approved for use in England, Wales and Northern Ireland as a Cambridge International Level 1/Level 2 Certificate.

This document consists of 17 printed pages and 3 blank pages.

DC (SC/AR) 145577/4
© UCLES 2018 [Turn over

Page 501 of 877


3

1 (a) The reactions of chemical digestion are catalysed by enzymes.

Fig. 1.1 shows the stages of an enzyme-catalysed reaction.

enzyme
A

Fig. 1.1

State the names of A to D in Fig. 1.1.

A ...............................................................................................................................................

B ...............................................................................................................................................

C ...............................................................................................................................................

D ...............................................................................................................................................
[4]

© UCLES 2018 0610/41/M/J/18 [Turn over


Page 503 of 877
4

(b) Explain the importance of chemical digestion.

...................................................................................................................................................

...................................................................................................................................................

...................................................................................................................................................

...................................................................................................................................................

...................................................................................................................................................

...............................................................................................................................................[2]

(c) Fig. 1.2 shows the human alimentary canal and associated organs.

The functions of some of these parts of the body are given in Table 1.1.

M
B

K D

H E

F
G

Fig. 1.2

© UCLES 2018 0610/41/M/J/18

Page 504 of 877


5

Complete Table 1.1. One row has been done for you.

Table 1.1

function letter from Fig. 1.2 name of structure

site of starch digestion

reabsorption of water

secretion of pepsin

site of maltose digestion

secretion of bile

storage of faeces F rectum

secretion of lipase and trypsin


[6]

[Total: 12]

© UCLES 2018 0610/41/M/J/18 [Turn over


Page 505 of 877
6

2 (a) Adaptive features are defined as the inherited features of an organism that increase its fitness.

State what is meant by fitness in this context.

...................................................................................................................................................

...................................................................................................................................................

...............................................................................................................................................[1]

(b) Rodents are the most common mammals in many hot deserts.

Fig. 2.1 shows the lesser Egyptian jerboa, Jaculus jaculus, which lives in North Africa and the
Middle East in areas that have high daytime temperatures and very little rainfall.

Fig. 2.1

Like many desert-living mammals, jerboas are active at night.

Suggest two features of J. jaculus that adapt it to each of the following challenges of living in
desert ecosystems:

(i) very high daytime temperatures

1 ........................................................................................................................................

2 ........................................................................................................................................
[2]

(ii) very little or no light at night

1 ........................................................................................................................................

2 ........................................................................................................................................
[2]

© UCLES 2018 0610/41/M/J/18

Page 506 of 877


7

(c) A scientist studied communities in different parts of a desert and estimated the biomass of the
organisms in each area.

He divided the organisms into four groups according to their roles in the food web as shown
in Table 2.1.

Detritivores are animals that eat dead organisms or parts of organisms.

Table 2.1

groups of organisms in the food web biomass / g per m2


producers 480

herbivores 220

detritivores 120

carnivores 40

Some of these results are shown as a pyramid of biomass in Fig. 2.2.

herbivores and detritivores

producers

Fig. 2.2

(i) Use the information in Table 2.1 to complete the pyramid of biomass in Fig. 2.2. [2]

(ii) The scientist observed the detritivores and decided to include them with herbivores in
this pyramid of biomass.

Suggest what the scientist discovered about the detritivores that made him make this
decision.

...........................................................................................................................................

...........................................................................................................................................

.......................................................................................................................................[1]

© UCLES 2018 0610/41/M/J/18 [Turn over


Page 507 of 877
8

(iii) Explain why there are rarely more than four or five trophic levels in ecosystems.

...........................................................................................................................................

...........................................................................................................................................

...........................................................................................................................................

...........................................................................................................................................

.......................................................................................................................................[2]

(iv) Explain the advantages of presenting information about food webs as a pyramid of
biomass and not as a pyramid of numbers.

...........................................................................................................................................

...........................................................................................................................................

...........................................................................................................................................

...........................................................................................................................................

...........................................................................................................................................

...........................................................................................................................................

...........................................................................................................................................

.......................................................................................................................................[3]

[Total: 13]

© UCLES 2018 0610/41/M/J/18

Page 508 of 877


9

3 A student cut a section of a root and made an outline drawing of the distribution of tissues as
shown in Fig. 3.1.

Fig. 3.1

(a) (i) Identify the position of the xylem tissue by drawing a label line and the letter X on Fig. 3.1.
[1]

(ii) State why xylem is a tissue.

...........................................................................................................................................

...........................................................................................................................................

...........................................................................................................................................

.......................................................................................................................................[2]

(b) Water absorbed by the roots moves through the stem and enters the leaves. Most of this
water is lost in transpiration.

Explain how the internal structure of leaves results in the loss of large quantities of water in
transpiration.

...................................................................................................................................................

...................................................................................................................................................

...................................................................................................................................................

...................................................................................................................................................

...................................................................................................................................................

...................................................................................................................................................

...............................................................................................................................................[3]

[Total: 6]

© UCLES 2018 0610/41/M/J/18 [Turn over


Page 509 of 877
11

4 The flow of blood through the skin can be investigated by using a flow-meter.

Fig. 4.1 shows a flow-meter above a section through the skin.

flow-meter

skin

Q
S
ring of
muscle

Not drawn to scale


Fig. 4.1

(a) (i) State the name of cell P.

.......................................................................................................................................[1]

(ii) State the types of blood vessel labelled Q, S and T.

Q .......................................................................................................................................

S ........................................................................................................................................

T ........................................................................................................................................
[3]

(iii) State the name of the tissue at R that provides insulation.

.......................................................................................................................................[1]
© UCLES 2018 0610/41/M/J/18 [Turn over
Page 511 of 877
12

(b) The blood flow through the skin of some volunteers was measured with a flow-meter when
their skin was exposed to different temperatures.

Capsaicin is a compound that gives people the sensation of feeling hot when it is put on the
skin. Researchers applied capsaicin to the skin of the volunteers and again measured the
blood flow through their skin at different temperatures.

Fig. 4.2 shows the results.

100

90

80
with capsaicin
average blood 70
flow as a
percentage 60
of maximum
blood flow 50

40 without
capsaicin
30

20

10

0
15 20 25 30 35 40 45
temperature of the skin surface / °C

Fig. 4.2

(i) Use the information in Fig. 4.2 to describe the effect of increasing the temperature of the
skin surface on blood flow to the skin without capsaicin.

...........................................................................................................................................

...........................................................................................................................................

...........................................................................................................................................

...........................................................................................................................................

...........................................................................................................................................

...........................................................................................................................................

...........................................................................................................................................

.......................................................................................................................................[3]

© UCLES 2018 0610/41/M/J/18

Page 512 of 877


13

(ii) Explain the mechanism that increases blood flow through the skin.

...........................................................................................................................................

...........................................................................................................................................

...........................................................................................................................................

...........................................................................................................................................

...........................................................................................................................................

...........................................................................................................................................

...........................................................................................................................................

.......................................................................................................................................[3]

(iii) State the difference between the average blood flow for the treatments (with and without
capsaicin) at 35 °C.

Space for working.

....................................................... % [1]

(iv) The researchers thought that capsaicin stimulated receptors in the skin.

Explain the process by which capsaicin could reach these receptors.

...........................................................................................................................................

...........................................................................................................................................

...........................................................................................................................................

...........................................................................................................................................

...........................................................................................................................................

...........................................................................................................................................

...........................................................................................................................................

.......................................................................................................................................[3]

© UCLES 2018 0610/41/M/J/18 [Turn over


Page 513 of 877
14

(c) Explain the importance of regulating body temperature in humans.

...................................................................................................................................................

...................................................................................................................................................

...................................................................................................................................................

...................................................................................................................................................

...................................................................................................................................................

...................................................................................................................................................

...................................................................................................................................................

...................................................................................................................................................

...............................................................................................................................................[4]

(d) Body temperature is controlled by both hormones and nerves.

Explain how co-ordination by hormones differs from co-ordination by nerves.

...................................................................................................................................................

...................................................................................................................................................

...................................................................................................................................................

...................................................................................................................................................

...................................................................................................................................................

...................................................................................................................................................

...................................................................................................................................................

...............................................................................................................................................[3]

[Total: 22]

© UCLES 2018 0610/41/M/J/18

Page 514 of 877


16

5 (a) State the balanced chemical equation for aerobic respiration.

...............................................................................................................................................[2]

(b) Researchers in the Czech Republic investigated oxygen consumption in horses. They
measured the oxygen consumption of the horses while they were exercising at four different
paces: walking, trotting, cantering and galloping.

The results are shown in Fig. 5.1.

120

110

100

90

average rate 80
of oxygen
consumption 70
/ cm3 kg–1 min–1
60

50

40

30

20

10

0
walking trotting cantering galloping
type of exercise

Fig. 5.1

© UCLES 2018 0610/41/M/J/18

Page 516 of 877


17

Calculate the percentage increase in the average rate of oxygen consumption as the horses
change from walking to trotting.

Show your working.

................................ %
[2]

(c) The researchers also calculated the oxygen debt for each type of exercise.

They found that the horses developed a larger oxygen debt when they exercised by galloping
and cantering rather than when they walked.

Explain why the horses developed an oxygen debt when they exercised.

...................................................................................................................................................

...................................................................................................................................................

...................................................................................................................................................

...................................................................................................................................................

...................................................................................................................................................

...................................................................................................................................................

...................................................................................................................................................

...............................................................................................................................................[3]

(d) Describe how the horses would recover from an oxygen debt when they stop exercising.

...................................................................................................................................................

...................................................................................................................................................

...................................................................................................................................................

...................................................................................................................................................

...................................................................................................................................................

...................................................................................................................................................

...................................................................................................................................................

...................................................................................................................................................

...................................................................................................................................................

...............................................................................................................................................[4]

[Total:11]
© UCLES 2018 0610/41/M/J/18 [Turn over
Page 517 of 877
18

6 (a) Fig. 6.1 is a diagram of the human female reproductive system.

T
S

W
R

X
Y

Fig. 6.1

(i) Complete Table 6.1 by stating the letter from Fig. 6.1 that identifies the structure where
each process occurs.

Table 6.1

process letter from Fig. 6.1

meiosis

fertilisation

implantation

[3]

(ii) State the name of the part of the female reproductive system labelled S in Fig. 6.1.

...................................................................................................................................... [1]

© UCLES 2018 0610/41/M/J/18

Page 518 of 877


19

(b) Fig. 6.2 is a diagram of a human sperm cell.

acrosome

nucleus
flagellum
mitochondria

Fig. 6.2

(i) Write the formula that would be used to calculate the magnification of the diagram.

[1]

(ii) The actual length of the sperm cell in Fig. 6.2 is 0.055 mm.

Convert this value to micrometres (μm).

Space for working.

.................................................... μm [1]

(c) Explain why the nuclei of sperm cells differ from those of other cells in the male.

...................................................................................................................................................

...................................................................................................................................................

...................................................................................................................................................

...............................................................................................................................................[2]

© UCLES 2018 0610/41/M/J/18 [Turn over


Page 519 of 877
20

(d) Explain the roles of the flagellum, the mitochondria and the acrosome in sperm cells.

...................................................................................................................................................

...................................................................................................................................................

...................................................................................................................................................

...................................................................................................................................................

...................................................................................................................................................

...................................................................................................................................................

...................................................................................................................................................

...................................................................................................................................................

...................................................................................................................................................

...................................................................................................................................................

...................................................................................................................................................

...................................................................................................................................................

...............................................................................................................................................[6]

(e) Explain why the sex of a child is determined by its father.

...................................................................................................................................................

...................................................................................................................................................

...................................................................................................................................................

...............................................................................................................................................[2]

[Total: 16]

Permission to reproduce items where third-party owned material protected by copyright is included has been sought and cleared where possible. Every
reasonable effort has been made by the publisher (UCLES) to trace copyright holders, but if any items requiring clearance have unwittingly been included, the
publisher will be pleased to make amends at the earliest possible opportunity.

To avoid the issue of disclosure of answer-related information to candidates, all copyright acknowledgements are reproduced online in the Cambridge International
Examinations Copyright Acknowledgements Booklet. This is produced for each series of examinations and is freely available to download at www.cie.org.uk after
the live examination series.

Cambridge International Examinations is part of the Cambridge Assessment Group. Cambridge Assessment is the brand name of University of Cambridge Local
Examinations Syndicate (UCLES), which is itself a department of the University of Cambridge.

© UCLES 2018 0610/41/M/J/18

Page 520 of 877


Cambridge International Examinations
Cambridge International General Certificate of Secondary Education
* 7 6 6 8 0 6 5 7 2 5 *

BIOLOGY 0610/42
Paper 4 Theory (Extended) May/June 2018
1 hour 15 minutes
Candidates answer on the Question Paper.
No Additional Materials are required.

READ THESE INSTRUCTIONS FIRST

Write your Centre number, candidate number and name on all the work you hand in.
Write in dark blue or black pen.
You may use an HB pencil for any diagrams or graphs.
Do not use staples, paper clips, glue or correction fluid.
DO NOT WRITE IN ANY BARCODES.

Answer all questions.

Electronic calculators may be used.


You may lose marks if you do not show your working or if you do not use appropriate units.

At the end of the examination, fasten all your work securely together.
The number of marks is given in brackets [ ] at the end of each question or part question.

This syllabus is approved for use in England, Wales and Northern Ireland as a Cambridge International Level 1/Level 2 Certificate.

This document consists of 16 printed pages and 4 blank pages.

DC (NF/SW) 145576/4
© UCLES 2018 [Turn over

Page 521 of 877


3

1 (a) Red pandas, Ailurus fulgens, and humans have a similar arrangement of teeth.

Fig. 1.1 shows a section through one tooth of a red panda. Fig. 1.2 shows the side view of the
lower jaw of a red panda.

C D E F

Fig. 1.1 Fig. 1.2

(i) State the names of the structures labelled A to F in Fig. 1.1 and Fig. 1.2.

A ........................................................................................................................................

B ........................................................................................................................................

C ........................................................................................................................................

D ........................................................................................................................................

E ........................................................................................................................................

F ........................................................................................................................................
[3]

(ii) State the type of digestion that breaks down large pieces of food.

...................................................................................................................................... [1]

(b) Food that sticks to the teeth can be used by bacteria for anaerobic respiration.

This type of respiration releases a substance that can cause tooth decay.

(i) State the type of substance released by the bacteria, during respiration, that causes
tooth decay.

...................................................................................................................................... [1]

© UCLES 2018 0610/42/M/J/18 [Turn over


Page 523 of 877
4

(ii) State the names of the two parts of a tooth that are dissolved by the substance released
by bacterial respiration.

1 .........................................................................................................................................

2 .........................................................................................................................................
[2]

(c) The teeth of red pandas do not decay as much as human teeth.

Suggest the component of a human diet that causes teeth to decay as a result of bacterial
respiration.

.............................................................................................................................................. [1]

[Total: 8]

© UCLES 2018 0610/42/M/J/18

Page 524 of 877


5

2 Mangrove trees are hydrophytes because they grow in water.

Fig. 2.1 shows a young mangrove tree.

Fig. 2.1

(a) An adaptive feature is a feature that increases the fitness of an organism.

(i) Define the term fitness.

...........................................................................................................................................

...........................................................................................................................................

...................................................................................................................................... [1]

(ii) Mangrove trees have many aerial roots and floating seeds.

Suggest how these adaptive features allow mangrove trees to survive in water.

many aerial roots ...............................................................................................................

...........................................................................................................................................

...........................................................................................................................................

floating seeds ....................................................................................................................

...........................................................................................................................................

...........................................................................................................................................
[2]

© UCLES 2018 0610/42/M/J/18 [Turn over


Page 525 of 877
6

(b) Fig. 2.2 shows a food chain in a mangrove forest.

mangrove tree fiddler crab seagull

Fig. 2.2

Table 2.1 gives the number of organisms and their biomass in a mangrove forest.

Table 2.1

organism number of organisms biomass of organisms / kg


mangrove trees 1 000 450 000
fiddler crabs 7 500 000 8 000
seagulls 150 000 1 200

(i) Estimate the biomass of one fiddler crab in grams.

Write your answer to two significant figures.

Show your working.

............................ g
[2]

(ii) Sketch a pyramid of numbers, using the information in Table 2.1, for the food chain
shown in Fig. 2.2.

Write the number of each trophic level on the appropriate part of your pyramid.

[3]
© UCLES 2018 0610/42/M/J/18

Page 526 of 877


7

(iii) Explain why the shape of a pyramid of biomass, for the information given in Table 2.1,
would be different from the shape of your pyramid of numbers.

...........................................................................................................................................

...........................................................................................................................................

...........................................................................................................................................

...........................................................................................................................................

...........................................................................................................................................

...........................................................................................................................................

...........................................................................................................................................

...........................................................................................................................................

...................................................................................................................................... [4]

[Total: 12]

© UCLES 2018 0610/42/M/J/18 [Turn over


Page 527 of 877
8

3 Aphids are insects that feed on the phloem sap in plants.

Fig. 3.1 shows a diagram of an aphid with its mouth parts inserted into the stem of a plant.

phloem xylem

Fig. 3.1

(a) The mouth parts of the aphid reach the phloem tissue of the stem.

(i) State the name of the foods the aphid could suck out of the phloem tissue.

1 .........................................................................................................................................

2 .........................................................................................................................................
[2]

(ii) Explain the role of phloem in plant transport. Use the words source and sink in your
answer.

...........................................................................................................................................

...........................................................................................................................................

...........................................................................................................................................

...........................................................................................................................................

...........................................................................................................................................

...........................................................................................................................................

...........................................................................................................................................

...........................................................................................................................................

...................................................................................................................................... [4]

© UCLES 2018 0610/42/M/J/18

Page 528 of 877


9

(b) Fig. 3.1 shows some of the features of xylem.

Describe how xylem is adapted for its functions.

...................................................................................................................................................

...................................................................................................................................................

...................................................................................................................................................

...................................................................................................................................................

...................................................................................................................................................

...................................................................................................................................................

...................................................................................................................................................

...................................................................................................................................................

...................................................................................................................................................

...................................................................................................................................................

...................................................................................................................................................

...................................................................................................................................................

.............................................................................................................................................. [6]

(c) Some farmers spray their crops with insecticides to kill pests such as aphids.

Explain the benefits of killing pests.

...................................................................................................................................................

...................................................................................................................................................

...................................................................................................................................................

...................................................................................................................................................

.............................................................................................................................................. [2]

[Total: 14]

© UCLES 2018 0610/42/M/J/18 [Turn over


Page 529 of 877
10

4 One of the roles of the Centers for Disease Control and Prevention (CDC) in Atlanta, US, is to try
to reduce the number of people who are infected with pathogens.

The CDC conducted a survey. They asked women which, if any, contraceptive methods they used.

(a) Suggest why the CDC collected data on contraceptive methods.

...................................................................................................................................................

...................................................................................................................................................

...................................................................................................................................................

...................................................................................................................................................

...................................................................................................................................................

...................................................................................................................................................

.............................................................................................................................................. [3]

Fig. 4.1 shows the results of the survey.

90

80

70

60
percentage
50
of women
in the survey
40

30

20

10

0
any condoms/ contraceptive other
method femidoms pills chemical
methods
contraceptive method

Fig. 4.1

© UCLES 2018 0610/42/M/J/18

Page 530 of 877


11

(b) (i) State two hormones that are used in contraceptive pills.

1 .........................................................................................................................................

2 .........................................................................................................................................
[2]

(ii) Suggest why contraceptive pills do not contain FSH.

...........................................................................................................................................

...........................................................................................................................................

...........................................................................................................................................

...........................................................................................................................................

...................................................................................................................................... [3]

(iii) Give one example of ‘other chemical methods’ (fourth bar) that could be included in the
bar in Fig. 4.1.

...................................................................................................................................... [1]

(iv) State two methods of birth control that were not listed in the survey.

1 .........................................................................................................................................

2 .........................................................................................................................................
[2]

(v) The percentage of the last three bars in Fig. 4.1 added together is 90%.

Suggest why the percentage of women who used any type of contraceptive method (first
bar) is not equal to the sum of the last three bars.

...........................................................................................................................................

...........................................................................................................................................

...................................................................................................................................... [1]

[Total: 12]

© UCLES 2018 0610/42/M/J/18 [Turn over


Page 531 of 877
12

5 Fig. 5.1 shows an adult fly, Chrysomya megacephala.

Fig. 5.1

(a) State three visible features from Fig. 5.1 that could be used to distinguish adult insects from
other arthropods.

1 ................................................................................................................................................

2 ................................................................................................................................................

3 ................................................................................................................................................
[3]

(b) Fly larvae are immature insects that are often used in experiments on respiration.

Give the balanced chemical equation for aerobic respiration.

.............................................................................................................................................. [2]

© UCLES 2018 0610/42/M/J/18

Page 532 of 877


13

(c) A respirometer is shown in Fig. 5.2. It can be used to estimate an organism’s rate of respiration.

8 7 6 5 4 3 2 1 0

droplet
of oil

water-bath

fly larvae wire mesh

soda lime

Fig. 5.2

(i) Complete the sentences:

A respirometer can be used to calculate the ............................. of oxygen used by the

fly larvae by measuring the ............................. the droplet of oil moves in one minute. A

water-bath is used to ............................. the temperature of the apparatus.


[3]

(ii) The soda lime in the respirometer absorbs carbon dioxide.

Explain why this is important in this investigation.

...........................................................................................................................................

...........................................................................................................................................

...................................................................................................................................... [1]

(iii) Fly larvae respire to release energy.

State two uses of energy in a fly larva.

1 .........................................................................................................................................

2 .........................................................................................................................................
[2]
© UCLES 2018 0610/42/M/J/18 [Turn over
Page 533 of 877
14

(d) A student used a respirometer to investigate the effect of temperature on the rate of respiration
of germinating seeds.

Predict the results of this investigation and explain your prediction.

prediction ..................................................................................................................................

...................................................................................................................................................

...................................................................................................................................................

explanation ...............................................................................................................................

...................................................................................................................................................

...................................................................................................................................................

...................................................................................................................................................

...................................................................................................................................................

...................................................................................................................................................
[4]

[Total: 15]

© UCLES 2018 0610/42/M/J/18

Page 534 of 877


15

6 Bacteria are useful in biotechnology and genetic engineering.

Fig. 6.1 shows a photomicrograph of a bacterium.

magnification ×27 000

Fig. 6.1

(a) State the name of the process that is taking place in Fig. 6.1.

.............................................................................................................................................. [1]

(b) (i) Write the formula that would be used to calculate the actual width of the bacterium.

[1]
(ii) The actual width of the bacterium is 0.0008 mm.

Convert this value to micrometres (μm).

Space for working.

................................. μm [1]

(c) Genetically modified bacteria can produce human insulin.

(i) State the name of the disease that can be treated with insulin injections.

...................................................................................................................................... [1]

© UCLES 2018 0610/42/M/J/18 [Turn over


Page 535 of 877
16

(ii) Insulin is a protein.

Describe the process of using bacteria in genetic engineering to produce human proteins.

...........................................................................................................................................

...........................................................................................................................................

...........................................................................................................................................

...........................................................................................................................................

...........................................................................................................................................

...........................................................................................................................................

...........................................................................................................................................

...........................................................................................................................................

...........................................................................................................................................

...........................................................................................................................................

...................................................................................................................................... [5]

(d) Genetically modified bacteria are often grown in fermenters.

(i) Suggest why steam is used to clean fermenters.

...........................................................................................................................................

...........................................................................................................................................

...................................................................................................................................... [2]

(ii) State three conditions inside a fermenter that are measured and controlled.

1 .........................................................................................................................................

2 .........................................................................................................................................

3 .........................................................................................................................................
[3]

(iii) State the name of one commercial product, other than insulin, that is made in fermenters.

...................................................................................................................................... [1]

© UCLES 2018 0610/42/M/J/18

Page 536 of 877


17

(e) Crop plants can also be genetically modified.

Describe the advantages of genetically modifying crops.

...................................................................................................................................................

...................................................................................................................................................

...................................................................................................................................................

...................................................................................................................................................

...................................................................................................................................................

...................................................................................................................................................

.............................................................................................................................................. [4]

[Total: 19]

© UCLES 2018 0610/42/M/J/18

Page 537 of 877


Cambridge International Examinations
Cambridge International General Certificate of Secondary Education
* 4 9 9 6 1 4 9 6 2 0 *

BIOLOGY 0610/43
Paper 4 Theory (Extended) May/June 2018
1 hour 15 minutes
Candidates answer on the Question Paper.
No Additional Materials are required.

READ THESE INSTRUCTIONS FIRST

Write your Centre number, candidate number and name on all the work you hand in.
Write in dark blue or black pen.
You may use an HB pencil for any diagrams or graphs.
Do not use staples, paper clips, glue or correction fluid.
DO NOT WRITE IN ANY BARCODES.

Answer all questions.

Electronic calculators may be used.


You may lose marks if you do not show your working or if you do not use appropriate units.

At the end of the examination, fasten all your work securely together.
The number of marks is given in brackets [ ] at the end of each question or part question.

This syllabus is approved for use in England, Wales and Northern Ireland as a Cambridge International Level 1/Level 2 Certificate.

This document consists of 17 printed pages and 3 blank pages.

DC (LEG/SW) 145575/4
© UCLES 2018 [Turn over

Page 541 of 877


2

1 Two functions of the alimentary canal are mechanical digestion and chemical digestion.

(a) Outline where and how mechanical digestion occurs in the alimentary canal.

...................................................................................................................................................

...................................................................................................................................................

...................................................................................................................................................

...................................................................................................................................................

...................................................................................................................................................

...................................................................................................................................................

...................................................................................................................................................

...................................................................................................................................................

...............................................................................................................................................[4]

(b) Enzymes catalyse the reactions of chemical digestion. Table 1.1 gives information about
chemical digestion in three parts of the alimentary canal.

Complete Table 1.1.

Table 1.1

part of the alimentary


enzyme substrate product(s)
canal

mouth starch

stomach peptides

fat fatty acids and glycerol

[3]

© UCLES 2018 0610/43/M/J/18

Page 542 of 877


3

(c) Substances that are absorbed from the alimentary canal may enter cells and become part of
the cells.

(i) State the storage carbohydrate made from glucose in liver cells.

.......................................................................................................................................[1]

(ii) State the type of protein used in the immune system that is produced from amino acids
by lymphocytes.

.......................................................................................................................................[1]

(iii) Fat is produced from fatty acids and glycerol by cells in the fatty tissue beneath the skin.

State one function of this layer of fat.

.......................................................................................................................................[1]

[Total: 10]

© UCLES 2018 0610/43/M/J/18 [Turn over


Page 543 of 877
4

2 Fig. 2.1 shows an Arctic wolf, Canis lupus. These wolves are one of the few mammals adapted to
the extreme cold of the tundra in the Canadian Arctic and in Alaska.

Fig. 2.1

(a) (i) State two features, visible in Fig. 2.1, that identify Arctic wolves as mammals.

1 ........................................................................................................................................

2 ........................................................................................................................................
[2]

(ii) Arctic wolves show many adaptive features to a cold environment.

Explain what is meant by the term adaptive feature.

...........................................................................................................................................

...........................................................................................................................................

...........................................................................................................................................

...........................................................................................................................................

...........................................................................................................................................

.......................................................................................................................................[3]

© UCLES 2018 0610/43/M/J/18

Page 544 of 877


5

(b) The food available to animals in the Arctic tundra is limited. There is a short growing season
for plants and the environmental conditions do not favour high rates of photosynthesis and
growth compared with temperate and tropical ecosystems.

State three conditions that limit plant growth rates.

1 ................................................................................................................................................

2 ................................................................................................................................................

3 ................................................................................................................................................
[3]

(c) Arctic wolves are the top carnivores in the food web in the tundra.

Explain why the number of Arctic wolves is so small in this ecosystem.

...................................................................................................................................................

...................................................................................................................................................

...................................................................................................................................................

...................................................................................................................................................

...................................................................................................................................................

...................................................................................................................................................

...................................................................................................................................................

...................................................................................................................................................

...................................................................................................................................................

...................................................................................................................................................

...................................................................................................................................................

...................................................................................................................................................

...................................................................................................................................................

...............................................................................................................................................[6]

[Total: 14]

© UCLES 2018 0610/43/M/J/18 [Turn over


Page 545 of 877
6

3 Fig. 3.1 is a scanning electron micrograph of a vertical section through part of the leaf of a broad
bean plant, Vicia faba.

air spaces

Fig. 3.1

(a) (i) State the names of the tissues labelled A and B.

A ........................................................................................................................................

B ........................................................................................................................................
[2]

(ii) The cells in regions B and C in Fig. 3.1 have a large surface area.

Explain why this is necessary for the functioning of the leaf cells.

...........................................................................................................................................

...........................................................................................................................................

...........................................................................................................................................

...........................................................................................................................................

...........................................................................................................................................

.......................................................................................................................................[3]

(iii) Explain why there are many interconnecting air spaces within the leaf.

...........................................................................................................................................

...........................................................................................................................................

...........................................................................................................................................

.......................................................................................................................................[2]

© UCLES 2018 0610/43/M/J/18

Page 546 of 877


7

(b) When water is in short supply, plants can wilt as shown in Fig. 3.2.

Fig. 3.2

(i) State two conditions that are likely to increase the chances of wilting.

1 ........................................................................................................................................

2 ........................................................................................................................................
[2]

(ii) Explain what happens to the cells of a leaf to cause wilting.

...........................................................................................................................................

...........................................................................................................................................

...........................................................................................................................................

...........................................................................................................................................

...........................................................................................................................................

...........................................................................................................................................

...........................................................................................................................................

...........................................................................................................................................

...........................................................................................................................................

.......................................................................................................................................[4]

© UCLES 2018 0610/43/M/J/18 [Turn over


Page 547 of 877
8

(iii) Wilting may look harmful, but it is often a strategy for survival.

Suggest the advantages to a plant of wilting.

...........................................................................................................................................

...........................................................................................................................................

...........................................................................................................................................

...........................................................................................................................................

...........................................................................................................................................

.......................................................................................................................................[2]

[Total: 15]

© UCLES 2018 0610/43/M/J/18

Page 548 of 877


10

4 (a) The endocrine system in mammals produces hormones.

Define the term hormone.

...................................................................................................................................................

...................................................................................................................................................

...................................................................................................................................................

...................................................................................................................................................

...............................................................................................................................................[2]

(b) The responses of the human body to danger are coordinated by the nervous and endocrine
systems.

Fig. 4.1 shows the sequence of events that occurs in response to a dangerous situation that
is detected by the eyes.

Key:
nerve impulses
hormone

Fig. 4.1

© UCLES 2018 0610/43/M/J/18

Page 550 of 877


11

(i) State the tissue in the eye that converts light energy into nerve impulses.

.......................................................................................................................................[1]

(ii) State the part of the eye that has the highest concentration of light-sensitive cells and
gives the most detailed image.

.......................................................................................................................................[1]

(iii) State the type of neurone that conducts impulses from the eye to the brain.

.......................................................................................................................................[1]

(iv) State the nerve that contains these neurones that conduct impulses from the eye to the
brain.

.......................................................................................................................................[1]

(v) Identify the organ labelled P.

.......................................................................................................................................[1]

(vi) Identify the gland labelled Q.

.......................................................................................................................................[1]

(c) Complete Table 4.1 to describe the effects of the hormone released when a person is in a
dangerous situation.

Table 4.1

organ effect of the hormone

heart

liver

lungs

eyes

[4]

© UCLES 2018 0610/43/M/J/18 [Turn over


Page 551 of 877
12

(d) Explain the advantages of coordinating the response to a dangerous situation using both the
nervous system and the endocrine system.

...................................................................................................................................................

...................................................................................................................................................

...................................................................................................................................................

...................................................................................................................................................

...................................................................................................................................................

...................................................................................................................................................

...............................................................................................................................................[4]

(e) (i) Plants also make hormones.

State the name of one hormone made by plants.

.......................................................................................................................................[1]

(ii) Some plant hormones are manufactured and applied to crops to alter aspects of plant
growth.

Describe how the synthetic plant hormone 2,4-D is used in agriculture.

...........................................................................................................................................

...........................................................................................................................................

...........................................................................................................................................

...........................................................................................................................................

.......................................................................................................................................[2]

[Total: 19]

© UCLES 2018 0610/43/M/J/18

Page 552 of 877


13

5 (a) State the balanced chemical equation for aerobic respiration.

...............................................................................................................................................[2]

(b) Students investigated the rate of respiration of crickets (a type of insect) using a carbon
dioxide sensor and laptop as shown in Fig. 5.1. The sensor was fitted inside an airtight glass
jar. The apparatus was set up in a room with a constant temperature of 17 °C.

Fig. 5.1

The students found that the concentration of carbon dioxide inside the jar increased by
50 ppm in 120 seconds.

Calculate the rate of carbon dioxide production as ppm per second.

Show your working and express your answer to two significant figures.

............................................ ppm s–1 [1]

© UCLES 2018 0610/43/M/J/18 [Turn over


Page 553 of 877
14

(c) After 10 minutes, the students opened the jar by removing the sensor. They left the jar open
for 5 minutes but made sure that the crickets remained in the jar. They then replaced the
sensor and took more readings for another 10 minutes.

State and explain one reason for opening the jar after 10 minutes.

...................................................................................................................................................

...................................................................................................................................................

...................................................................................................................................................

...................................................................................................................................................

...............................................................................................................................................[2]

(d) During the investigation the temperature inside the jar increased. The temperature outside
the jar remained constant.

Explain why the temperature inside the jar increased.

...................................................................................................................................................

...................................................................................................................................................

...................................................................................................................................................

...................................................................................................................................................

...............................................................................................................................................[2]

© UCLES 2018 0610/43/M/J/18

Page 554 of 877


15

(e) Researchers in Chile also investigated the rate of respiration in crickets.

They investigated the effect of temperature and body mass on the rate of respiration. They
measured the rate of oxygen consumption in crickets with different body masses, at different
temperatures.

The researchers’ results are shown in Fig. 5.2.

30
27 °C
25

20
rate of oxygen 17 °C
consumption 15
/ 10–3 cm3 h–1
10

7 °C
5

0
0 10 20 30 40 50 60 70 80
body mass / mg
Fig. 5.2

State two conclusions that can be made from the data in Fig. 5.2 and support each conclusion
with evidence from the graph.

...................................................................................................................................................

...................................................................................................................................................

...................................................................................................................................................

...................................................................................................................................................

...................................................................................................................................................

...................................................................................................................................................

...................................................................................................................................................

...................................................................................................................................................

...............................................................................................................................................[4]

[Total:11]

© UCLES 2018 0610/43/M/J/18 [Turn over


Page 555 of 877
16

6 (a) Fig. 6.1 is a half-flower drawing of pride of Barbados, Caesalpinia pulcherrima.

D
C

Fig. 6.1

Complete Table 6.1 by stating the letter from Fig. 6.1 that indicates the organ where each
function occurs and the name of the organ.

Table 6.1

function letter from Fig. 6.1 name of the organ

meiosis to produce pollen grains

pollination

development of seeds

protection of flower in the bud


[4]

© UCLES 2018 0610/43/M/J/18

Page 556 of 877


17

(b) Fig. 6.2 is a scanning electron micrograph of some pollen grains from wind-pollinated flowers
and insect-pollinated flowers.

magnification ×220

Fig. 6.2

(i) Write the formula that would be used to calculate the actual diameter of pollen grain H.

[1]

(ii) The actual diameter of pollen grain H is 0.082 mm.

Convert this value to micrometres (μm).

Space for working.

..................................................... μm [1]

(iii) Explain how the pollen grain labelled J is adapted for insect pollination.

...........................................................................................................................................

...........................................................................................................................................

...........................................................................................................................................

...........................................................................................................................................

.......................................................................................................................................[2]

© UCLES 2018 0610/43/M/J/18 [Turn over


Page 557 of 877
18

(c) Pollen grains grow tubes, which contain haploid male gamete nuclei.

(i) One of these male gamete nuclei fuses with the female gamete.

State the part of the flower that contains the female gamete.

.......................................................................................................................................[1]

(ii) Define the term haploid nucleus.

...........................................................................................................................................

...........................................................................................................................................

.......................................................................................................................................[1]

(iii) Explain why it is important for gametes to be haploid.

...........................................................................................................................................

...........................................................................................................................................

.......................................................................................................................................[1]

[Total: 11]

© UCLES 2018 0610/43/M/J/18

Page 558 of 877


Cambridge International Examinations
Cambridge International General Certificate of Secondary Education
* 2 2 0 3 4 7 9 9 5 2 *

BIOLOGY 0610/41
Paper 4 Theory (Extended) October/November 2018
1 hour 15 minutes
Candidates answer on the Question Paper.
No Additional Materials are required.

READ THESE INSTRUCTIONS FIRST

Write your Centre number, candidate number and name on all the work you hand in.
Write in dark blue or black pen.
You may use an HB pencil for any diagrams or graphs.
Do not use staples, paper clips, glue or correction fluid.
DO NOT WRITE IN ANY BARCODES.

Answer all questions.

Electronic calculators may be used.


You may lose marks if you do not show your working or if you do not use appropriate units.

At the end of the examination, fasten all your work securely together.
The number of marks is given in brackets [ ] at the end of each question or part question.

This syllabus is approved for use in England, Wales and Northern Ireland as a Cambridge International Level 1/Level 2 Certificate.

This document consists of 16 printed pages.

DC (ST/JG) 152888/3
© UCLES 2018 [Turn over

Page 561 of 877


2

1 Fig. 1.1 shows a pyramid of biomass and part of the carbon cycle.

CO2 in atmosphere

A
B
decomposers
C
D

fossil fuels

Fig. 1.1

(a) (i) State the principal source of energy required for trophic level D of the pyramid of biomass
in Fig. 1.1.

.......................................................................................................................................[1]

(ii) State the letter that represents the primary consumers in Fig. 1.1.

........................................... [1]

(iii) State how carbon is transferred from producers to primary consumers.

.......................................................................................................................................[1]

(iv) Explain why trophic level A is smaller than trophic level B in the pyramid of biomass in
Fig. 1.1.

...........................................................................................................................................

...........................................................................................................................................

...........................................................................................................................................

...........................................................................................................................................

...........................................................................................................................................

...........................................................................................................................................

.......................................................................................................................................[3]

© UCLES 2018 0610/41/O/N/18

Page 562 of 877


3

(b) Some fungi and bacteria are decomposers.

(i) Define the term decomposer.

...........................................................................................................................................

...........................................................................................................................................

.......................................................................................................................................[1]

(ii) Arrow X on Fig. 1.1 indicates the transfer of carbon from decomposers to the atmosphere.

State the name of process X.

.......................................................................................................................................[1]

(c) Describe how human activities are affecting the carbon cycle.

...................................................................................................................................................

...................................................................................................................................................

...................................................................................................................................................

...................................................................................................................................................

...................................................................................................................................................

...................................................................................................................................................

...................................................................................................................................................

...................................................................................................................................................

...................................................................................................................................................

...................................................................................................................................................

...............................................................................................................................................[5]

[Total: 13]

© UCLES 2018 0610/41/O/N/18 [Turn over


Page 563 of 877
4

2 Microbiologists test strains of bacteria for antibiotic resistance.

They do this by soaking paper discs in antibiotics and placing them on bacteria growing in Petri
dishes.

The paper discs in the centre of Petri dishes E and F in Fig. 2.1 have been soaked in penicillin.

zone where bacteria are growing

paper disc
soaked in
penicillin

E F
zone where bacteria are no longer growing

Fig. 2.1

(a) State the type of microorganism that produces penicillin.

...............................................................................................................................................[1]

(b) State and explain the evidence from Fig. 2.1 that suggests that the bacteria in dish F are
resistant to penicillin.

...................................................................................................................................................

...................................................................................................................................................

...................................................................................................................................................

...................................................................................................................................................

...............................................................................................................................................[2]

© UCLES 2018 0610/41/O/N/18

Page 564 of 877


5

(c) (i) Explain how bacteria become resistant to antibiotics and how humans can reduce the
problem of antibiotic resistance.

...........................................................................................................................................

...........................................................................................................................................

...........................................................................................................................................

...........................................................................................................................................

...........................................................................................................................................

...........................................................................................................................................

...........................................................................................................................................

...........................................................................................................................................

...........................................................................................................................................

...........................................................................................................................................

...........................................................................................................................................

...........................................................................................................................................

.......................................................................................................................................[6]

(ii) Explain why viral infections cannot be treated with antibiotics.

...........................................................................................................................................

.......................................................................................................................................[1]

(d) Some bacteria and viruses cause disease but many are useful to the biotechnology industry.

Explain why bacteria are useful in biotechnology.

...................................................................................................................................................

...................................................................................................................................................

...................................................................................................................................................

...................................................................................................................................................

...................................................................................................................................................

...................................................................................................................................................

...............................................................................................................................................[3]

[Total: 13]

© UCLES 2018 0610/41/O/N/18 [Turn over


Page 565 of 877
6

3 Fig. 3.1 shows a photomicrograph of a section of a root.

Fig. 3.1

(a) Structure J is a xylem vessel.

The xylem vessels conduct water from the roots to the stems.

State the features of xylem vessels that enable them to conduct water.

...................................................................................................................................................

...................................................................................................................................................

...................................................................................................................................................

...................................................................................................................................................

...................................................................................................................................................

...................................................................................................................................................

...............................................................................................................................................[3]

© UCLES 2018 0610/41/O/N/18

Page 566 of 877


7

(b) Describe the pathway of water from outside the root to the xylem vessels (J) at the centre of
the root. Use the letters in Fig. 3.1 in your answer.

...................................................................................................................................................

...................................................................................................................................................

...................................................................................................................................................

...................................................................................................................................................

...................................................................................................................................................

...................................................................................................................................................

...................................................................................................................................................

...................................................................................................................................................

...................................................................................................................................................

...................................................................................................................................................

...............................................................................................................................................[5]

© UCLES 2018 0610/41/O/N/18 [Turn over


Page 567 of 877
8

(c) Scientists wanted to determine the flow-rate of water in roots.

They measured the flow-rate in three zones of onion roots as shown in Fig. 3.2.

zone 3
glass jar containing
zone 2
water
zone 1

Fig. 3.2

They measured the flow-rate in healthy roots and roots that had been treated with a toxic
solution.

Their results are shown in Table 3.1.

Table 3.1

zone in average flow-rate of water / arbitrary units


Fig. 3.2 healthy roots treated roots
1 150 160
2 230 200
3 280 270

(i) Calculate the percentage increase in the average flow-rate between zone 1 and 3 for
healthy roots.

Give your answer to two significant figures.

Show your working.

............................................................ %
[2]
© UCLES 2018 0610/41/O/N/18

Page 568 of 877


9

(ii) The scientists observed that the xylem vessels nearer the root tip were narrower than
the xylem vessels higher up the root.

Describe how the width of xylem vessels in different zones of a root affects the average
flow-rate of water. Use the information in Table 3.1 in your answer.

...........................................................................................................................................

...........................................................................................................................................

...........................................................................................................................................

...........................................................................................................................................

...........................................................................................................................................

...........................................................................................................................................

.......................................................................................................................................[3]

(iii) Suggest why there was little difference in the flow-rate in healthy roots and in roots
treated with the toxic solution.

...........................................................................................................................................

...........................................................................................................................................

...........................................................................................................................................

...........................................................................................................................................

.......................................................................................................................................[2]

[Total: 15]

© UCLES 2018 0610/41/O/N/18 [Turn over


Page 569 of 877
10

4 The eye is a sense organ that responds to light.

Fig. 4.1 is a diagram of a section through the human eye.

R W

S
X

T
Y

Fig. 4.1

(a) Table 4.1 describes some of the functions of the parts of the eye.

Complete the table by:

• naming the parts of the eye


• using the letters on Fig. 4.1 to identify the parts of the eye.

Table 4.1

function name of part letter on Fig. 4.1

carries impulses to the brain

focuses light onto the back of the eye

controls the tension of the suspensory ligaments

tissue that detects light and colour

location of most of the cone cells

[5]

© UCLES 2018 0610/41/O/N/18

Page 570 of 877


11

(b) (i) A pair of muscles in the eye work in opposition to each other to adjust the amount of light
entering the pupil.

State the term that describes the action of a pair of muscles working in opposition to
each other.

.......................................................................................................................................[1]

(ii) A different pair of muscles in the eye work in opposition to each other to view objects at
different distances from the eye.

State the name of the process that allows the eye to view objects at different distances.

.......................................................................................................................................[1]

(c) Explain why the eye cannot easily identify different colours in low levels of light.

...................................................................................................................................................

...................................................................................................................................................

...................................................................................................................................................

...................................................................................................................................................

...............................................................................................................................................[2]

(d) Some people inherit colour blindness and cannot identify certain colours, even in bright light.

The gene responsible for colour vision is located on the X chromosome.

There are two alleles for this gene on the X chromosome:

• XB – normal colour vision


• Xb – colour blindness.

(i) People that are heterozygous for colour blindness are called carriers.

State the genotype of a heterozygous female carrier.

.......................................................................................................................................[1]

(ii) There is no gene for colour vision on the male sex chromosome.

State the genotype of a colour-blind male.

.......................................................................................................................................[1]

© UCLES 2018 0610/41/O/N/18 [Turn over


Page 571 of 877
12

Fig. 4.2 shows a pedigree diagram for colour blindness.

1 2 Key:
colour vision female

3 4 5 6 7 8 carrier female
colour-blind female
colour vision male
9 10 11 12 13........... carrier male
colour-blind male

Fig. 4.2

(iii) Person 13 in Fig. 4.2 is male. His parents are person 7 and person 8.

Use the key to complete Fig. 4.2 by drawing the correct symbol for person 13. [1]

(iv) Colour blindness is a sex-linked characteristic.

Explain why females 4 and 5 are carriers even though their mother is not a carrier.

...........................................................................................................................................

...........................................................................................................................................

...........................................................................................................................................

...........................................................................................................................................

.......................................................................................................................................[2]

[Total: 14]

© UCLES 2018 0610/41/O/N/18

Page 572 of 877


13

5 The liver is an important organ in many processes.

(a) The liver responds to changes in insulin concentration.

Insulin is a hormone.

(i) Define the term hormone.

...........................................................................................................................................

...........................................................................................................................................

...........................................................................................................................................

...........................................................................................................................................

...........................................................................................................................................

...........................................................................................................................................

.......................................................................................................................................[3]

(ii) Describe how the liver responds to an increase in insulin concentration.

...........................................................................................................................................

...........................................................................................................................................

...........................................................................................................................................

...........................................................................................................................................

.......................................................................................................................................[2]

(b) The liver is also involved in the processing of amino acids.

(i) Describe how excess amino acids are broken down.

...........................................................................................................................................

...........................................................................................................................................

.......................................................................................................................................[2]

(ii) State the name of the process that assembles amino acids to form proteins.

.......................................................................................................................................[1]

© UCLES 2018 0610/41/O/N/18 [Turn over


Page 573 of 877
14

(c) The liver is also involved in the processing of toxins.

(i) Lactic acid is an example of a toxin that is produced during vigorous exercise and
processed in the liver.
Describe how lactic acid is processed.

...........................................................................................................................................

...........................................................................................................................................

.......................................................................................................................................[2]

(ii) Alcohol is another toxin that is processed in the liver.


The effect of alcohol consumption on the risk of dying from liver disease was investigated
in men and women.
The results are shown in Fig. 5.1.

60 Key:
men
50 women

40
risk of dying
from liver
30
disease
/ arbitrary
units 20

10

0
0 20 40 60 80 100 120 140 160
alcohol consumption / g per day

Fig. 5.1

Describe the results shown in Fig. 5.1.

...........................................................................................................................................

...........................................................................................................................................

...........................................................................................................................................

...........................................................................................................................................

...........................................................................................................................................

...........................................................................................................................................

...........................................................................................................................................

...........................................................................................................................................

.......................................................................................................................................[4]
[Total: 14]
© UCLES 2018 0610/41/O/N/18

Page 574 of 877


15

6 Young mammals that are orphaned can be bottle-fed.

Fig. 6.1 shows a newborn tiger cub sucking on a bottle.

Fig. 6.1

(a) (i) Sucking is an example of an involuntary action observed in newborn mammals.

State the name given to involuntary actions.

.......................................................................................................................................[1]

(ii) Describe the advantages of breast-feeding compared with bottle-feeding.

...........................................................................................................................................

...........................................................................................................................................

...........................................................................................................................................

...........................................................................................................................................

...........................................................................................................................................

...........................................................................................................................................

...........................................................................................................................................

...........................................................................................................................................

...........................................................................................................................................

...........................................................................................................................................

.......................................................................................................................................[4]

© UCLES 2018 0610/41/O/N/18 [Turn over


Page 575 of 877
16

(b) The digestive systems of young mammals are not fully developed.

Enzymes such as amylase, maltase and protease are often added to baby food to aid
chemical digestion.

(i) Complete Table 6.1 by stating the substrate and product(s) for each enzyme reaction.

Table 6.1

enzyme substrate product(s)

amylase

maltase

protease

[3]

(ii) Suggest why the temperature of baby food must be controlled when the enzymes are
added.

...........................................................................................................................................

...........................................................................................................................................

.......................................................................................................................................[2]

(iii) State one other condition that must also be controlled to optimise enzyme activity.

.......................................................................................................................................[1]

[Total: 11]

Permission to reproduce items where third-party owned material protected by copyright is included has been sought and cleared where possible. Every
reasonable effort has been made by the publisher (UCLES) to trace copyright holders, but if any items requiring clearance have unwittingly been included, the
publisher will be pleased to make amends at the earliest possible opportunity.

To avoid the issue of disclosure of answer-related information to candidates, all copyright acknowledgements are reproduced online in the Cambridge International
Examinations Copyright Acknowledgements Booklet. This is produced for each series of examinations and is freely available to download at www.cie.org.uk after
the live examination series.

Cambridge International Examinations is part of the Cambridge Assessment Group. Cambridge Assessment is the brand name of University of Cambridge Local
Examinations Syndicate (UCLES), which is itself a department of the University of Cambridge.

© UCLES 2018 0610/41/O/N/18

Page 576 of 877


Cambridge International Examinations
Cambridge International General Certificate of Secondary Education
* 3 5 8 0 8 4 5 9 9 9 *

BIOLOGY 0610/42
Paper 4 Theory (Extended) October/November 2018
1 hour 15 minutes
Candidates answer on the Question Paper.
No Additional Materials are required.

READ THESE INSTRUCTIONS FIRST

Write your Centre number, candidate number and name on all the work you hand in.
Write in dark blue or black pen.
You may use a pencil for any diagrams or graphs.
Do not use staples, paper clips, glue or correction fluid.
DO NOT WRITE IN ANY BARCODES.

Answer all questions.

Electronic calculators may be used.


You may lose marks if you do not show your working or if you do not use appropriate units.

At the end of the examination, fasten all your work securely together.
The number of marks is given in brackets [ ] at the end of each question or part question.

This syllabus is approved for use in England, Wales and Northern Ireland as a Cambridge International Level 1/Level 2 Certificate.

This document consists of 18 printed pages and 2 blank pages.

DC (NF/CT) 191975/4
© UCLES 2018 [Turn over

Page 577 of 877


3

1 Wetlands are important ecosystems. Researchers studied the feeding relationships between the
organisms in an area of wetland on the coast of Texas.

Fig. 1.1 shows part of the food web that they studied.

bald eagle spotted


sandpiper

stone crab

marsh rice
blenny
rat
mycid shrimp

snow goose oyster pipe fish


water flea
muskrat

clover algae and


grass
phytoplankton

Fig. 1.1

(a) Complete Table 1.1 by giving the name of one organism from the food web in Fig. 1.1 for
each row.

Table 1.1

name of organism from Fig. 1.1

producer

secondary consumer

an animal that feeds


at two trophic levels
[3]

© UCLES 2018 0610/42/O/N/18 [Turn over


Page 579 of 877
4

The functioning of ecosystems relies on the cycling of nutrients.

Fig. 1.2 shows part of the nitrogen cycle.

nitrogen gas in
the air

C
amino acids
plants nitrate ions
in plants

D A

proteins proteins in dead plants ammonium ions


in plants

Fig. 1.2

(b) State the name of process A in Fig. 1.2 and give the type of organism that converts ammonium
ions to nitrate ions.

A ...............................................................................................................................................

type of organism .......................................................................................................................


[2]

(c) Describe how nitrate ions enter the roots of plants shown by arrow C on Fig. 1.2.

...................................................................................................................................................

...................................................................................................................................................

...................................................................................................................................................

...................................................................................................................................................

...................................................................................................................................................

...................................................................................................................................................

...................................................................................................................................................

.............................................................................................................................................. [3]

© UCLES 2018 0610/42/O/N/18

Page 580 of 877


5

(d) State the name of the structure in plant cells where process D occurs.

.............................................................................................................................................. [1]

(e) State the process that occurs at B.

.............................................................................................................................................. [1]

(f) A pyramid of numbers for the wetland ecosystem showed that there were very large numbers
of organisms at the base of the pyramid and very few at the top.

Explain why.

...................................................................................................................................................

...................................................................................................................................................

...................................................................................................................................................

...................................................................................................................................................

...................................................................................................................................................

...................................................................................................................................................

...................................................................................................................................................

.............................................................................................................................................. [3]

[Total: 13]

© UCLES 2018 0610/42/O/N/18 [Turn over


Page 581 of 877
7

2 Fig. 2.1 shows a dwarf sunflower and a tall sunflower, Helianthus annuus. The height of the dwarf
sunflower is 0.45 m and the height of the tall sunflower is 4.5 m.

dwarf tall not to scale

Fig. 2.1

Dwarf plants like the one in Fig. 2.1 have mutant alleles.

(a) Define the term allele.

...................................................................................................................................................

.............................................................................................................................................. [1]

(b) Shoot growth in plants is controlled by auxins. An enzyme in shoot tips converts molecules of
an amino acid into auxins as shown in Fig. 2.2.

enzyme
amino acid auxin

Fig. 2.2

Explain how a mutation in DNA results in an abnormal enzyme which does not catalyse the
reaction shown in Fig. 2.2.

...................................................................................................................................................

...................................................................................................................................................

...................................................................................................................................................

...................................................................................................................................................

...................................................................................................................................................

...................................................................................................................................................

...................................................................................................................................................

.............................................................................................................................................. [3]
© UCLES 2018 0610/42/O/N/18 [Turn over
Page 583 of 877
8

(c) Two tall sunflower plants were crossed. 25% of the offspring produced were dwarf.

Explain how it is possible for two tall parent plants to have this percentage of dwarf offspring.

...................................................................................................................................................

...................................................................................................................................................

...................................................................................................................................................

...................................................................................................................................................

.............................................................................................................................................. [2]

(d) Fig. 2.3 shows how several strawberry plants can be formed from one parent plant.

parent plant

offspring

Fig. 2.3

(i) Explain the type of reproduction that produces plants by the method shown in Fig. 2.3.

...........................................................................................................................................

...........................................................................................................................................

...........................................................................................................................................

...........................................................................................................................................

...........................................................................................................................................

...........................................................................................................................................

...................................................................................................................................... [3]
© UCLES 2018 0610/42/O/N/18

Page 584 of 877


9

(ii) Explain the disadvantages of the type of reproduction shown in Fig. 2.3.

...........................................................................................................................................

...........................................................................................................................................

...........................................................................................................................................

...........................................................................................................................................

...........................................................................................................................................

...........................................................................................................................................

...................................................................................................................................... [3]

[Total: 12]

© UCLES 2018 0610/42/O/N/18 [Turn over


Page 585 of 877
10

3 (a) Fig. 3.1 is a photomicrograph of some xylem vessels.

Fig. 3.1

(i) State one structural feature of xylem vessels and explain how this is related to the
function of water transport.

feature ...............................................................................................................................

...........................................................................................................................................

explanation ........................................................................................................................

...........................................................................................................................................

...........................................................................................................................................

...........................................................................................................................................
[2]

(ii) Explain the mechanism that is responsible for the movement of water in xylem vessels.

...........................................................................................................................................

...........................................................................................................................................

...........................................................................................................................................

...........................................................................................................................................

...........................................................................................................................................

...........................................................................................................................................

...........................................................................................................................................

...........................................................................................................................................

...................................................................................................................................... [4]

© UCLES 2018 0610/42/O/N/18

Page 586 of 877


11

(iii) State one role of xylem vessels other than transport.

...................................................................................................................................... [1]

(b) The rate of transpiration is affected by several factors including the temperature and the
humidity of the air.

State and explain the effect of an increase in temperature on the rate of transpiration.

...................................................................................................................................................

...................................................................................................................................................

...................................................................................................................................................

...................................................................................................................................................

...................................................................................................................................................

...................................................................................................................................................

...................................................................................................................................................

.............................................................................................................................................. [3]

[Total: 10]

© UCLES 2018 0610/42/O/N/18 [Turn over


Page 587 of 877
12

4 Insulin is a hormone that regulates the concentration of glucose in the blood.

(a) Define the term hormone.

...................................................................................................................................................

...................................................................................................................................................

...................................................................................................................................................

...................................................................................................................................................

...................................................................................................................................................

.............................................................................................................................................. [3]

(b) Two people, A and B, visited a doctor to discuss their similar symptoms. The doctor thought
that their blood glucose concentrations were not very well controlled. A glucose tolerance test
was carried out on both people.

A and B did not eat or drink anything other than water for eight hours before the test. They
then drank a glucose solution. Blood samples were taken at 30 minute intervals. The samples
were tested for glucose concentration.

The results are shown in Fig. 4.1.


blood glucose
concentration
/ mg per 100 cm3
220

200
A
180

160
glucose
solution
140
taken
120

100
B
80

60

40

20

0
0 30 60 90 120 150 180 210 240

time / minutes
Fig. 4.1
© UCLES 2018 0610/42/O/N/18

Page 588 of 877


13

(i) Use Fig. 4.1 to state the blood glucose concentrations of A and B at 180 minutes.

A .............................................................................................................. mg per 100 cm3

B .............................................................................................................. mg per 100 cm3


[1]

(ii) Calculate the percentage increase in the blood glucose concentration in person A
between 60 and 90 minutes.

Give your answer to the nearest whole number.

Show your working.

................ %
[2]

(iii) Describe how the response of person A differs from the response of person B in Fig. 4.1.

...........................................................................................................................................

...........................................................................................................................................

...........................................................................................................................................

...........................................................................................................................................

...................................................................................................................................... [2]

(iv) Explain the results of the glucose tolerance test shown by person B.

...........................................................................................................................................

...........................................................................................................................................

...........................................................................................................................................

...........................................................................................................................................

...........................................................................................................................................

...........................................................................................................................................

...........................................................................................................................................

...........................................................................................................................................

...................................................................................................................................... [4]

© UCLES 2018 0610/42/O/N/18 [Turn over


Page 589 of 877
14

(v) The doctor thought that person A had Type 1 diabetes.

Describe three symptoms of Type 1 diabetes.

...........................................................................................................................................

...........................................................................................................................................

...........................................................................................................................................

...........................................................................................................................................

...................................................................................................................................... [3]

[Total: 15]

© UCLES 2018 0610/42/O/N/18

Page 590 of 877


15

5 Fig. 5.1 shows a photomicrograph of human blood.

phagocyte

lymphocyte

red blood
cell

Fig. 5.1

(a) Describe the differences in appearance and the roles of the three cells labelled in Fig. 5.1.

...................................................................................................................................................

...................................................................................................................................................

...................................................................................................................................................

...................................................................................................................................................

...................................................................................................................................................

...................................................................................................................................................

...................................................................................................................................................

...................................................................................................................................................

...................................................................................................................................................

...................................................................................................................................................

...................................................................................................................................................

...................................................................................................................................................

.............................................................................................................................................. [6]

© UCLES 2018 0610/42/O/N/18 [Turn over


Page 591 of 877
16

(b) Fig. 5.2 shows some of the stages of blood clotting.

a blood vessel breaks

platelets collect at the


break in the blood vessel

platelets release an
enzyme

prothrombin thrombin

forms a mesh at the break


in the blood vessel

Fig. 5.2

(i) Complete Fig. 5.2 by filling in the two empty boxes. [1]

(ii) State two roles of blood clotting.

...........................................................................................................................................

...........................................................................................................................................

...........................................................................................................................................

...........................................................................................................................................

...................................................................................................................................... [2]

© UCLES 2018 0610/42/O/N/18

Page 592 of 877


17

(c) Haemophilia is a sex-linked blood disorder in which blood takes a long time to clot. Fig. 5.3 is
a pedigree diagram showing the inheritance of haemophilia.

Q R S T

male with normal clotting time

male with haemophilia

female with normal clotting time

Fig. 5.3

The normal allele is represented by XH and the mutant allele is represented by Xh.

(i) State the genotypes of the people identified as P, Q and R in Fig. 5.3.

P ........................................................................................................................................

Q .......................................................................................................................................

R ........................................................................................................................................
[3]

© UCLES 2018 0610/42/O/N/18 [Turn over


Page 593 of 877
18

(ii) The couple S and T are expecting another child.

What is the probability that the child will have haemophilia?

Space for working

...................................................................................................................................... [1]

(iii) Define the term sex-linked characteristic.

...........................................................................................................................................

...........................................................................................................................................

...........................................................................................................................................

...........................................................................................................................................

...................................................................................................................................... [2]

[Total: 15]

© UCLES 2018 0610/42/O/N/18

Page 594 of 877


19

6 Fig. 6.1 shows the Galapagos iguana, Amblyrhynchus cristatus.

Fig. 6.1

(a) (i) State two features that are used to classify animals, such as the Galapagos iguana, as
reptiles.

1 ........................................................................................................................................

2 ........................................................................................................................................
[2]

(ii) State two features that are present in plant cells that are not present in the cells of
reptiles.

1 ........................................................................................................................................

2 ........................................................................................................................................
[2]

(b) Galapagos iguanas feed on seaweed which contains starch and other carbohydrates.

(i) State the name of the enzyme that digests starch.

...................................................................................................................................... [1]

(ii) State the names of two parts of the alimentary canal where starch is digested.

1 ........................................................................................................................................

2 ........................................................................................................................................
[2]

© UCLES 2018 0610/42/O/N/18 [Turn over


Page 595 of 877
20

(c) There are many threats to wildlife in the Galapagos.

Describe ways in which endangered species can be conserved.

...................................................................................................................................................

...................................................................................................................................................

...................................................................................................................................................

...................................................................................................................................................

...................................................................................................................................................

...................................................................................................................................................

...................................................................................................................................................

...................................................................................................................................................

...................................................................................................................................................

...................................................................................................................................................

.............................................................................................................................................. [5]

(d) One aim of conservation is to maintain resources in natural ecosystems.

State three resources that natural ecosystems provide for humans.

1 ................................................................................................................................................

2 ................................................................................................................................................

3 ................................................................................................................................................
[3]

[Total: 15]

Permission to reproduce items where third-party owned material protected by copyright is included has been sought and cleared where possible. Every
reasonable effort has been made by the publisher (UCLES) to trace copyright holders, but if any items requiring clearance have unwittingly been included, the
publisher will be pleased to make amends at the earliest possible opportunity.

To avoid the issue of disclosure of answer-related information to candidates, all copyright acknowledgements are reproduced online in the Cambridge International
Examinations Copyright Acknowledgements Booklet. This is produced for each series of examinations and is freely available to download at www.cie.org.uk after
the live examination series.

Cambridge International Examinations is part of the Cambridge Assessment Group. Cambridge Assessment is the brand name of University of Cambridge Local
Examinations Syndicate (UCLES), which is itself a department of the University of Cambridge.

© UCLES 2018 0610/42/O/N/18

Page 596 of 877


Cambridge International Examinations
Cambridge International General Certificate of Secondary Education
* 2 4 8 0 3 9 0 3 5 7 *

BIOLOGY 0610/43
Paper 4 Theory (Extended) October/November 2018
1 hour 15 minutes
Candidates answer on the Question Paper.
No Additional Materials are required.

READ THESE INSTRUCTIONS FIRST

Write your Centre number, candidate number and name on all the work you hand in.
Write in dark blue or black pen.
You may use an HB pencil for any diagrams or graphs.
Do not use staples, paper clips, glue or correction fluid.
DO NOT WRITE IN ANY BARCODES.

Answer all questions.

Electronic calculators may be used.


You may lose marks if you do not show your working or if you do not use appropriate units.

At the end of the examination, fasten all your work securely together.
The number of marks is given in brackets [ ] at the end of each question or part question.

This syllabus is approved for use in England, Wales and Northern Ireland as a Cambridge International Level 1/Level 2 Certificate.

This document consists of 17 printed pages and 3 blank pages.

DC (CE/SW) 152729/4
© UCLES 2018 [Turn over

Page 597 of 877


2

1 Water is a very important molecule for all living organisms.

(a) (i) State the name of the organ in plants where most water is absorbed.

.......................................................................................................................................[1]

(ii) State the name of the organ in humans where most water is absorbed.

.......................................................................................................................................[1]

(iii) State one property of water that makes it useful to animals and plants.

.......................................................................................................................................[1]

(b) The flow diagram in Fig. 1.1 shows a town and part of the water cycle.

F G K L M N
O
J

Fig. 1.1

© UCLES 2018 0610/43/O/N/18

Page 598 of 877


3

Table 1.1 describes some of the processes in the water cycle.

Complete Table 1.1.

One row has been done for you.

Table 1.1

description name of the process letter in Fig. 1.1

nitrate ions are washed into rivers leaching F

an algal bloom in the water is caused by


leaching of nitrate ions

evaporation

conversion of water from a vapour to a


liquid

transpiration

[4]

(c) Polluted water can be purified at a sewage treatment works.

(i) State one reason why it is necessary to treat polluted water before it is used as drinking
water.

...........................................................................................................................................

.......................................................................................................................................[1]

(ii) Outline the process of sewage treatment. You may use the letters in Fig. 1.1 in your
answer.

...........................................................................................................................................

...........................................................................................................................................

...........................................................................................................................................

...........................................................................................................................................

...........................................................................................................................................

...........................................................................................................................................

...........................................................................................................................................

...........................................................................................................................................

.......................................................................................................................................[4]

[Total: 12]
© UCLES 2018 0610/43/O/N/18 [Turn over
Page 599 of 877
4

2 The Indian muntjac deer, Muntiacus muntjak, is recorded as the mammal with the lowest number
of chromosomes.

Fig. 2.1 is an image of the chromosomes in the nucleus of a diploid cell of a female muntjac deer.

sex chromosomes

Fig. 2.1

(a) State the diploid number of chromosomes for the female muntjac deer.

................................................................... [1]

(b) Fig. 2.2 is an image of the chromosomes in the nucleus of a diploid cell of a male muntjac
deer.

sex chromosomes

Fig. 2.2

Describe how the sex chromosomes of the male muntjac deer shown in Fig. 2.2 differ from
those of the female shown in Fig. 2.1.

...................................................................................................................................................

...................................................................................................................................................

...............................................................................................................................................[2]

© UCLES 2018 0610/43/O/N/18

Page 600 of 877


5

(c) Explain how meiosis can result in variation in a species.

Use the words chromosome and gametes in your answer.

...................................................................................................................................................

...................................................................................................................................................

...................................................................................................................................................

...................................................................................................................................................

...................................................................................................................................................

...................................................................................................................................................

...................................................................................................................................................

...................................................................................................................................................

...............................................................................................................................................[4]

(d) Another cause of variation is the formation of new alleles.

Describe how new alleles can be formed.

...................................................................................................................................................

...................................................................................................................................................

...................................................................................................................................................

...................................................................................................................................................

...................................................................................................................................................

...................................................................................................................................................

...............................................................................................................................................[3]

[Total: 10]

© UCLES 2018 0610/43/O/N/18 [Turn over


Page 601 of 877
7

3 (a) Fig. 3.1 is a photomicrograph of part of the upper surface of a broad bean leaf, Vicia faba.

Fig. 3.1

(i) On Fig. 3.1, identify and label two structures that are visible in cell P. [2]

(ii) State the name of the tissue shown in Fig. 3.1.

.......................................................................................................................................[1]

(iii) The tissue shown in Fig. 3.1 is transparent.

Explain why it is important to the plant that the tissue shown in Fig. 3.1 is transparent.

...........................................................................................................................................

...........................................................................................................................................

...........................................................................................................................................

...........................................................................................................................................

...........................................................................................................................................

...........................................................................................................................................

.......................................................................................................................................[3]

© UCLES 2018 0610/43/O/N/18 [Turn over


Page 603 of 877
8

(b) Stomata are found on the lower surface of broad bean leaves.

Describe the function of stomata.

...................................................................................................................................................

...................................................................................................................................................

...................................................................................................................................................

...................................................................................................................................................

...................................................................................................................................................

...................................................................................................................................................

...............................................................................................................................................[3]

(c) More than 40 years ago, botanists studied the leaves of broad bean plants and discovered
that guard cells control the opening and closing of stomata. They found that stomata were
open when the guard cells were turgid.

Table 3.1 shows some of their measurements.

Table 3.1

closed stomata open stomata


ion concentration in guard cells / pmol 0.3 2.5
guard cell volume / μm3 4000.0 6500.0
turgor pressure in the guard cells / MPa 2.0 4.8
width of stomatal opening / μm 0.0 8.0

(i) Ions move into guard cells by active transport.

Describe how the ions move into the guard cells.

...........................................................................................................................................

...........................................................................................................................................

.......................................................................................................................................[2]

© UCLES 2018 0610/43/O/N/18

Page 604 of 877


9

(ii) Describe and explain how the change in ion concentration causes the guard cell volume
to change. Use the information in Table 3.1 in your answer.

...........................................................................................................................................

...........................................................................................................................................

...........................................................................................................................................

...........................................................................................................................................

...........................................................................................................................................

...........................................................................................................................................

...........................................................................................................................................

...........................................................................................................................................

...........................................................................................................................................

...........................................................................................................................................

.......................................................................................................................................[5]

(iii) The botanists left the broad bean plants unattended for three days. During this time the
broad bean plants wilted.

Suggest two environmental factors that can cause plants to wilt.

1 ........................................................................................................................................

2 ........................................................................................................................................
[2]

[Total: 18]

© UCLES 2018 0610/43/O/N/18 [Turn over


Page 605 of 877
10

BLANK PAGE

© UCLES 2018 0610/43/O/N/18

Page 606 of 877


11

4 Glycogen is a storage carbohydrate in animals. Glycogen is made from glucose.

(a) (i) Cells that convert glucose to glycogen contain many mitochondria.

Suggest why these cells contain many mitochondria.

...........................................................................................................................................

...........................................................................................................................................

...........................................................................................................................................

.......................................................................................................................................[2]

(ii) State the type of biological molecule that catalyses reactions such as the conversion of
glycogen to glucose.

.......................................................................................................................................[1]

(b) A fetus needs glucose to make glycogen.

Describe how a fetus obtains glucose.

...................................................................................................................................................

...................................................................................................................................................

...................................................................................................................................................

...................................................................................................................................................

...................................................................................................................................................

...................................................................................................................................................

...............................................................................................................................................[3]

© UCLES 2018 0610/43/O/N/18 [Turn over


Page 607 of 877
12

(c) Fig. 4. 1 shows the concentration of glycogen in the fetus of a domestic cat during pregnancy
and immediately after birth.

birth

700

600

500
glycogen
concentration 400
/ μmol per g
300

200

100

0
0 10 20 30 40 50 60 70 80 90
time since fertilisation / days

Fig. 4.1

Hormones stimulate changes in the concentration of glycogen in the fetus.

(i) Define the term hormone.

...........................................................................................................................................

...........................................................................................................................................

...........................................................................................................................................

...........................................................................................................................................

.......................................................................................................................................[3]

© UCLES 2018 0610/43/O/N/18

Page 608 of 877


13

(ii) Calculate the percentage increase in the glycogen concentration in the fetus between
day 10 and birth in Fig. 4.1.

Give your answer to the nearest whole number.

Show your working.

............................................................ %
[2]

(iii) Describe the changes in glycogen concentration shown in Fig. 4.1 and explain how
hormones in the fetus cause these changes.

Use data from Fig. 4.1 to support your answer.

...........................................................................................................................................

...........................................................................................................................................

...........................................................................................................................................

...........................................................................................................................................

...........................................................................................................................................

...........................................................................................................................................

...........................................................................................................................................

...........................................................................................................................................

...........................................................................................................................................

...........................................................................................................................................

...........................................................................................................................................

...........................................................................................................................................

.......................................................................................................................................[6]

© UCLES 2018 0610/43/O/N/18 [Turn over


Page 609 of 877
14

(d) After birth, cats produce milk to feed their offspring.

Human babies can be breast-fed or bottle-fed with formula milk.

Outline three disadvantages of breast-feeding.

1 ................................................................................................................................................

...................................................................................................................................................

2 ................................................................................................................................................

...................................................................................................................................................

3 ................................................................................................................................................

...................................................................................................................................................
[3]

[Total: 20]

© UCLES 2018 0610/43/O/N/18

Page 610 of 877


15

5 An ecologist studied variation in a species of xerophyte.

(a) Xerophytes are adapted to a particular type of environment.

State this type of environment.

...............................................................................................................................................[1]

(b) The ecologist studied the features of the leaves in the species of xerophyte.

Fig. 5.1 shows the variation in the type of leaf spike.

number of 5
individuals
4

0
type 1 type 2 type 3
type of leaf spike

Fig. 5.1

(i) State the type of variation shown in Fig. 5.1.

.......................................................................................................................................[1]

(ii) Explain why the type of leaf spike is an example of the variation shown in Fig. 5.1.

...........................................................................................................................................

...........................................................................................................................................

.......................................................................................................................................[2]

© UCLES 2018 0610/43/O/N/18 [Turn over


Page 611 of 877
16

(c) The ecologist also measured other features of the leaves.

Fig. 5.2 shows the variation in leaf feature B.

number of 5
individuals
4

0
0–5 6–10 11–15 16–20 21–25
leaf feature B / arbitrary units

Fig. 5.2

State two named features of leaves that show the type of variation shown in Fig. 5.2.

1 ................................................................................................................................................

2 ................................................................................................................................................
[2]

© UCLES 2018 0610/43/O/N/18

Page 612 of 877


17

(d) After one year, the ecologist recorded the variation in leaf feature B again.

The results are shown in Fig. 5.3.

number of 5
individuals
4

0
0–5 6–10 11–15 16–20 21–25
leaf feature B / arbitrary units

Fig. 5.3

Suggest one reason for the difference in variation of leaf feature B after one year.

...................................................................................................................................................

...............................................................................................................................................[1]

[Total: 7]

© UCLES 2018 0610/43/O/N/18 [Turn over


Page 613 of 877
18

6 Fig. 6.1 is a diagram showing some body cells and parts of the human lymphatic and circulatory
systems.

arteriole

lymphatic vessel
B

Z capillary

venule

not to scale

Fig. 6.1

(a) Capillaries allow blood to reach most cells in the body.

(i) State the name of the process by which oxygen moves from A to Z as shown in Fig. 6.1.

.......................................................................................................................................[1]

(ii) Describe how some of the liquid in A moves to B in Fig. 6.1.

...........................................................................................................................................

...........................................................................................................................................

.......................................................................................................................................[2]

(iii) State one component of blood that remains inside the capillaries as the blood flows from
X to Y in Fig. 6.1.

.......................................................................................................................................[1]

© UCLES 2018 0610/43/O/N/18

Page 614 of 877


19

(b) Lymphatic vessels are similar in structure to veins.

(i) Describe the structure of veins.

...........................................................................................................................................

...........................................................................................................................................

...........................................................................................................................................

...........................................................................................................................................

.......................................................................................................................................[2]

(ii) Describe the role of the lymphatic vessel shown in Fig. 6.1.

...........................................................................................................................................

...........................................................................................................................................

...........................................................................................................................................

...........................................................................................................................................

.......................................................................................................................................[2]

(c) Lacteals are another part of the lymphatic system.

State where in the body lacteals are found and state their function.

location in the body ...................................................................................................................

function .....................................................................................................................................

...................................................................................................................................................
[2]

(d) In the lymphatic system, there are structures that contain large numbers of lymphocytes.

(i) State the name of these structures.

.......................................................................................................................................[1]

(ii) State the role of lymphocytes.

...........................................................................................................................................

...........................................................................................................................................

.......................................................................................................................................[2]

[Total: 13]

© UCLES 2018 0610/43/O/N/18

Page 615 of 877


Cambridge International Examinations
Cambridge International General Certificate of Secondary Education
* 8 8 5 9 8 1 2 6 6 1 *

BIOLOGY 0610/42
Paper 4 Theory (Extended) February/March 2018
1 hour 15 minutes
Candidates answer on the Question Paper.
No Additional Materials are required.

READ THESE INSTRUCTIONS FIRST

Write your Centre number, candidate number and name on all the work you hand in.
Write in dark blue or black pen.
You may use an HB pencil for any diagrams or graphs.
Do not use staples, paper clips, glue or correction fluid.
DO NOT WRITE IN ANY BARCODES.

Answer all questions.

Electronic calculators may be used.


You may lose marks if you do not show your working or if you do not use appropriate units.

At the end of the examination, fasten all your work securely together.
The number of marks is given in brackets [ ] at the end of each question or part question.

This syllabus is approved for use in England, Wales and Northern Ireland as a Cambridge International Level 1/Level 2 Certificate.

This document consists of 17 printed pages and 3 blank pages.

DC (SC/FC) 145585/4
© UCLES 2018 [Turn over

Page 617 of 877


2

1 (a) (i) Fig. 1.1 is a branching key used to identify different species of bacteria.

Do the bacteria have flagella?

Yes No

Do the bacteria have Do the bacteria have


more than one flagellum? a spiral shape?

Yes No Yes No

Do the bacteria have flagella A B Do the bacteria form


attached at one end only? a chain?

Yes No Yes No

C D E F

Fig. 1.1

Fig. 1.2 shows six different species of bacteria.

Use the key to identify the six different species of bacteria.

Write the letters on the lines in Fig. 1.2.

..................................... ..................................... .....................................

..................................... ..................................... .....................................


[5]
Fig. 1.2
© UCLES 2018 0610/42/F/M/18

Page 618 of 877


3

(ii) State the name of the kingdom that bacteria belong to.

...................................................................................................................................... [1]

(b) State one similarity between the structure of bacteria and the structure of viruses.

...................................................................................................................................................

...................................................................................................................................................

...............................................................................................................................................[1]

(c) Fig. 1.3 is a photomicrograph of Vibrio cholerae, the bacterium that causes cholera.

45 mm

magnification ×17 300

Fig. 1.3

(i) Write the formula that would be used to calculate the actual length of the bacterium (not
including the flagellum) in Fig. 1.3.

[1]

© UCLES 2018 0610/42/F/M/18 [Turn over


Page 619 of 877
4

(ii) The actual length of the bacterium shown in Fig. 1.3 is 0.0026 mm.

Convert this value to micrometres (µm).

Space for working.

..................................................... µm [1]

(d) (i) Describe and explain the effects of cholera bacteria on the gut.

...........................................................................................................................................

...........................................................................................................................................

...........................................................................................................................................

...........................................................................................................................................

...........................................................................................................................................

...........................................................................................................................................

...........................................................................................................................................

...........................................................................................................................................

.......................................................................................................................................[4]

(ii) Suggest one treatment for cholera.

.......................................................................................................................................[1]

[Total: 14]

© UCLES 2018 0610/42/F/M/18

Page 620 of 877


6

2 A study estimated the number of people with chronic obstructive pulmonary disease (COPD) in
India. Data were collected from two groups of people, those who lived in cities and those who
lived in villages.

Fig. 2.1 shows the results.

18 Key:
city
16
village
14

12
estimated numbers
of people with 10
COPD / million
8

0
1996 2001 2006 2011 2016
year

Fig. 2.1

© UCLES 2018 0610/42/F/M/18

Page 622 of 877


7

(a) Compare the number of people with COPD in cities with the number of people with COPD in
villages and suggest reasons for the differences.

Use the data in Fig. 2.1 to support your answer.

...................................................................................................................................................

...................................................................................................................................................

...................................................................................................................................................

...................................................................................................................................................

...................................................................................................................................................

...................................................................................................................................................

...................................................................................................................................................

...................................................................................................................................................

...................................................................................................................................................

...................................................................................................................................................

...................................................................................................................................................

...................................................................................................................................................

...............................................................................................................................................[6]

(b) (i) Explain how the body prevents particles in inspired air from reaching the gas exchange
surfaces.

...........................................................................................................................................

...........................................................................................................................................

...........................................................................................................................................

...........................................................................................................................................

...........................................................................................................................................

...........................................................................................................................................

...........................................................................................................................................

...........................................................................................................................................

.......................................................................................................................................[4]

© UCLES 2018 0610/42/F/M/18 [Turn over


Page 623 of 877
8

(ii) State two ways in which the composition of inspired air differs from the composition of
expired air.

1 ........................................................................................................................................

2 ........................................................................................................................................
[2]

(c) Alveoli are well-ventilated to provide efficient gas exchange.

(i) State the name of the muscles that cause the ribs to move during ventilation.

.......................................................................................................................................[1]

(ii) During inspiration the pressure and volume in the thorax changes.

State these changes.

pressure ............................................................................................................................

volume ...............................................................................................................................
[1]

[Total: 14]

© UCLES 2018 0610/42/F/M/18

Page 624 of 877


9

3 (a) Ecologists studied an area of woodland and estimated the biomass of each trophic level for
one of the food chains in the woodland.

Some students wanted to use the data to draw a pyramid of biomass for the food chain.

Table 3.1 shows the students’ table.

The students added a column to calculate the width of the bars they would need to draw.

Table 3.1

trophic level biomass / g m−2 width of bar / cm

1 producer 120 12.0

2 primary consumer 48 4.8

3 secondary consumer 16 1.6

4 tertiary consumer 2

(i) Complete Table 3.1 by calculating the missing value and writing it in the table. [1]

(ii) Using the information in Table 3.1, draw a pyramid of biomass.

Label each bar with the trophic level.

[3]

(b) A type of organism gains energy from waste organic material from all trophic levels.

State the name of this type of organism.

...............................................................................................................................................[1]

© UCLES 2018 0610/42/F/M/18 [Turn over


Page 625 of 877
10

(c) (i) Outline how organisms in the first trophic level of the woodland food chain produce
biomass using energy from the Sun.

...........................................................................................................................................

...........................................................................................................................................

...........................................................................................................................................

...........................................................................................................................................

...........................................................................................................................................

...........................................................................................................................................

.......................................................................................................................................[3]

(ii) Explain why the fourth trophic level has the least biomass in this food chain.

...........................................................................................................................................

...........................................................................................................................................

...........................................................................................................................................

...........................................................................................................................................

...........................................................................................................................................

...........................................................................................................................................

.......................................................................................................................................[3]

(d) The woodland is a conservation area.

Outline the possible benefits of conserving this specific area of woodland.

...................................................................................................................................................

...................................................................................................................................................

...................................................................................................................................................

...................................................................................................................................................

...................................................................................................................................................

...................................................................................................................................................

...............................................................................................................................................[3]

[Total: 14]

© UCLES 2018 0610/42/F/M/18

Page 626 of 877


11

4 Fig. 4.1 is a diagram of the human female reproductive system.

F
B
E

D C

Fig. 4.1

(a) Complete Table 4.1 to show the letter and the name of each of the structures that perform
these functions.

Table 4.1

function letter name

releases oestrogen

site of fertilisation

site of implantation

dilates during the process of birth

[4]

(b) Fertilisation is the fusion of the nuclei of a male gamete and a female gamete resulting in a
zygote.

State the number of chromosomes present in a human:

female gamete ...........................

zygote ........................................
[2]

© UCLES 2018 0610/42/F/M/18 [Turn over


Page 627 of 877
12

(c) Chlamydia is a sexually transmitted infection (STI).

Fig. 4.2 shows the number of reported cases of chlamydia in females in each age group in
one country.

400 000

350 000

300 000

250 000
number of
chlamydia 200 000
cases in 2008
150 000

100 000

50 000

0
10–14 15–19 20–24 25–29 30–34 35–39 40–44 45–54 55–64 65+
age group / years

Fig. 4.2

Describe the results shown by the data in Fig. 4.2.

...................................................................................................................................................

...................................................................................................................................................

...................................................................................................................................................

...................................................................................................................................................

...................................................................................................................................................

...................................................................................................................................................

...............................................................................................................................................[3]

(d) Chlamydia is caused by a bacterium.

(i) Suggest a treatment for chlamydia.

.......................................................................................................................................[1]

(ii) State the name of one other STI.

.......................................................................................................................................[1]

© UCLES 2018 0610/42/F/M/18

Page 628 of 877


13

(iii) Complete the sentences about the spread of STIs.

STIs are transmitted through the transfer of ............................................... during sexual

contact. One way individuals can avoid the spread of STIs is to use a type of

............................................... contraception. One example of this type of contraception

is ............................................... .
[3]

[Total: 14]

© UCLES 2018 0610/42/F/M/18 [Turn over


Page 629 of 877
14

5 2,4-D is a synthetic plant auxin that is used as a weedkiller.

Researchers investigated the effectiveness of different treatments of 2,4-D on the control of the
weed Conyza canadensis in fields of maize, Zea mays.

The results are shown in Table 5.1.

Table 5.1

treatment time of treatment mean dry mass of weed density / number


weeds / g per m2 of weeds per m2
day 7 day 23 day 33
A ✓ 7.40 6.20
B ✓ 3.90 4.90
C ✓ 0.50 1.20
D ✓ ✓ 0.66 1.90
E ✓ ✓ 0.18 0.98
F ✓ ✓ 0.07 0.29
G ✓ ✓ ✓ 0.08 0.51

(a) (i) Maize farmers that had been using treatment C were advised by the researchers to
change to treatment F.

Discuss the advantages and disadvantages of changing to treatment F.

...........................................................................................................................................

...........................................................................................................................................

...........................................................................................................................................

...........................................................................................................................................

...........................................................................................................................................

...........................................................................................................................................

.......................................................................................................................................[4]

(ii) Suggest two factors that could decrease the effectiveness of 2,4-D.

1 ........................................................................................................................................

2 ........................................................................................................................................
[2]

© UCLES 2018 0610/42/F/M/18

Page 630 of 877


15

(iii) Explain how 2,4-D acts as a weedkiller.

...........................................................................................................................................

...........................................................................................................................................

...........................................................................................................................................

...........................................................................................................................................

...........................................................................................................................................

...........................................................................................................................................

.......................................................................................................................................[3]

(b) Auxin causes the shoots of a plant to grow away from gravity.

State the name of this response.

...............................................................................................................................................[2]

[Total: 11]

© UCLES 2018 0610/42/F/M/18 [Turn over


Page 631 of 877
16

6 (a) Define the term chemical digestion.

...................................................................................................................................................

...................................................................................................................................................

...............................................................................................................................................[2]

(b) A student investigated the activity of the digestive enzyme pepsin.

Fig. 6.1 shows the apparatus used in the investigation.

test-tube 1 test-tube 2 test-tube 3 test-tube 4 stop-clock


egg white solution egg white solution, egg white solution, egg white solution
and pepsin pepsin and boiled pepsin and and hydrochloric
hydrochloric hydrochloric acid
acid acid

Fig. 6.1

The appearance of the four test-tubes was recorded at 0 and 5 minutes.

The protein in the egg white solution gives the solution a cloudy appearance.

The cloudy appearance clears when the protein in the egg white solution breaks down.

Table 6.1 shows the results.

Table 6.1

test-tube contents appearance at appearance after


0 mins 5 mins
1 egg white solution, cloudy less cloudy
pepsin
2 egg white solution, cloudy clear
pepsin, hydrochloric acid
3 egg white solution, boiled cloudy cloudy
pepsin, hydrochloric acid
4 egg white solution, cloudy cloudy
hydrochloric acid

© UCLES 2018 0610/42/F/M/18

Page 632 of 877


17

(i) Explain the results shown for test-tubes 1, 2 and 3 in Table 6.1.

...........................................................................................................................................

...........................................................................................................................................

...........................................................................................................................................

...........................................................................................................................................

...........................................................................................................................................

...........................................................................................................................................

...........................................................................................................................................

...........................................................................................................................................

...........................................................................................................................................

...........................................................................................................................................

.......................................................................................................................................[5]

(ii) Explain the purpose of test-tube 4.

...........................................................................................................................................

...........................................................................................................................................

...........................................................................................................................................

...........................................................................................................................................

.......................................................................................................................................[2]

(iii) State the name of the organ in the body that produces pepsin.

.......................................................................................................................................[1]

© UCLES 2018 0610/42/F/M/18 [Turn over


Page 633 of 877
18

(c) Maltase is another digestive enzyme.

Describe the action of maltase and state where it acts in the alimentary canal.

...................................................................................................................................................

...................................................................................................................................................

...................................................................................................................................................

...................................................................................................................................................

...................................................................................................................................................

...................................................................................................................................................

...............................................................................................................................................[3]

[Total: 13]

© UCLES 2018 0610/42/F/M/18

Page 634 of 877


0610/41 Cambridge IGCSE – Mark Scheme May/June 2022
PUBLISHED
Question Answer Marks Guidance

1(a)(i) Fungus / Fungi ; 1

1(a)(ii) aerobic respiration ; 1

1(b) any two from: 2


ref. to specificity (of enyme) ;
shape of active site is complementary to, substrate / sucrose ;
for, substrate / sucrose, to bind / fit, enzyme / sucrase / active site ;

1(c) total of six from: 6

max. four for description:


1 sucrase / enzyme, is active between pH2 and pH12 ;
2 activity, increases and decreases / reaches a peak ;
3 peak / maximum or 100% activity, at pH 6 ;
4 steeper increase between pH 5–6 / steeper decrease in activity
between pH 9–10 ;
5 minimum / 10%, activity at pH 12 ;
6 more activity in, acidic conditions / low pH, than, alkaline
conditions / high pH ;

explanation:
7 (change in) pH affects the shape of, sucrase / active site / enzyme ;
8 at pH 6, most enzyme-substrate complexes form / AW ;
9 at, low / high / extremes of, pH, enzyme is (partially) denatured ;
10 ref to substrate molecules can no longer bind with enzyme (at
low / high / extreme pH, so activity decreases) ;
11 AVP ;

© UCLES 2022 Page 5 of 12

Page 637 of 877


0610/41 Cambridge IGCSE – Mark Scheme May/June 2022
PUBLISHED
Question Answer Marks Guidance

2(a)(i) left ventricle ; 1

2(a)(ii) total of three from: 3

vena cava ;

max. two from:


has valves ;
wide, lumen / AW ;
thin wall ;
(wall) lined by single layer of cells ;
(wall) contains muscle (fibres) ;
(wall) contains elastic (fibres) ;

2(a)(iii) semilunar valve ; 2


prevents backflow of blood (correctly described) / ensures that blood
flows in one direction ;

2(b)(i) shading in any part of the pulmonary vein only ; 1

2(b)(ii) any two from: 2


heart has, two / left and right, sides / AW ;
blood must flow through the heart twice in one (complete) circuit / AW (of
the body) ;
pulmonary and systemic circuits / circuits from heart to lungs and from
heart to rest of body ;

© UCLES 2022 Page 6 of 12

Page 638 of 877


0610/41 Cambridge IGCSE – Mark Scheme May/June 2022
PUBLISHED
Question Answer Marks Guidance

2(b)(iii) any four from: 4


1 oxygenated and deoxygenated blood, are kept separate / do not
mix / separated by septum ;
2 ensures efficient supply of oxygen (to, body / AW) ;
3 ensures efficient supply of (named) nutrients (to, body / AW) ;
4 low(er) pressure in, pulmonary, artery / circuit / AW ;
5 to prevents damage to (capillaries in the) lungs ;
6 allows more time for gas exchange ;
7 allows high(er) pressure (in body) ;
8 to ensure efficient, blood supply to (rest of) body ;
9 to allow filtration in kidneys (for excretion) ;
10 to allow / maintain, a high, metabolic rate / rate of respiration ;
11 AVP ; e.g., larger diffusion gradient between
capillaries and respiring tissues

Question Answer Marks Guidance

3(a) J – liver ; 3
K – gall bladder ;
L – duodenum / small intestine ;

3(b) hormones: 5 enzymes and hormones can be in any order in


insulin ; each column
glucagon ;

enzymes:
amylase / carbohydrase ;
trypsin / protease ;
lipase ;

© UCLES 2022 Page 7 of 12

Page 639 of 877


0610/41 Cambridge IGCSE – Mark Scheme May/June 2022
PUBLISHED
Question Answer Marks Guidance

3(c) any three from: 3


active transport ;
against a concentration gradient / from low concentration to high
concentration ;
protein changes shape to move (chloride) ions ;
uses energy ;
AVP ; e.g., ref. to carrier proteins

3(d) any three from: 3


idea of presence of chloride ions (in duct) decreases water potential ;
(water moves by) osmosis ;
down water potential gradient / from high water potential to low water
potential ;
(movement of water) through partially permeable membrane ;

3(e)(i) parents of 5 and 7 / parents of people with cystic fibrosis / 2 and 3, do not 2
have cystic fibrosis ;
parents / 2 and 3 must be, heterozygous / carriers (of the mutant allele) ;

© UCLES 2022 Page 8 of 12

Page 640 of 877


0610/41 Cambridge IGCSE – Mark Scheme May/June 2022
PUBLISHED
Question Answer Marks Guidance

3(e)(ii) 5 MP1 and MP2 parents may be either way


round but following rows must match
1 correct parental genotype aa ;
2 correct parental genotype Aa ;
3 correct gametes from the parental genotypes ; ecf from MP1 and MP2
4 correct offspring genotypes from their gametes / parental genotypes ecf from MP3
(in any order) ;
5 correct offspring phenotypes and correct probability ; ecf from MP4

expected answer:

parent 7 man who is


heterozygous

parental genotypes aa  Aa

gametes a a + A a

offspring
genotypes Aa (Aa) aa (aa)

offspring
phenotypes without with
cystic fibrosis cystic fibrosis

probability 50% / 1 in 2 / 0.5

© UCLES 2022 Page 9 of 12

Page 641 of 877


0610/41 Cambridge IGCSE – Mark Scheme May/June 2022
PUBLISHED
Question Answer Marks Guidance

4(a) any five from: 5 points must be given in a correct context


1 protein, broken down / digested / decomposed, to amino acids ;
2 by decomposers / fungi / bacteria in terms (in context of breakdown of
manure / protein) ;
3 ref to use of proteases ;
4 deamination (of amino acids / proteins) / described ;
5 production of, ammonia / ammonium ions / NH3 / NH4+ ;
6 ammonium (ions), converted to, nitrite / nitrate (ions) / NO3-/ NO2-;
7 ref. to, nitrification / nitrifying bacteria ;
8 AVP ; e.g., nitrite to nitrate ions

4(b)(i) any one from: 1


farmers, do not have to add (nitrogen) fertiliser / use less fertiliser ;
idea of increased yield / more profit per hectare ;
(snap beans / beans / plants) grow, faster / better ;

4(b)(ii) 63 (%) ;;; 3 MP1 correct readings from graph: 96 and 156
MP2 correct answer calculated
MP3 answer correctly rounded to two
significant figures
ecf MP2 and MP3 from wrong
readings / calculation

4(b)(iii) any four from: 4


1 breakdown / decomposition, of manure / AW ;
2 increase in (availability of named), ions (in streams/rivers) ;
3 (increased) growth of, (named) plants / producers / algae ;
4 increased competition for, light / AW ;
5 no photosynthesis causing death of, producers / plants / algae ;
6 increase in, (number of) bacteria / decomposers ;
7 respiration (by decomposers) reduces (dissolved) oxygen (leads to
death of fish) ;

© UCLES 2022 Page 10 of 12

Page 642 of 877


0610/41 Cambridge IGCSE – Mark Scheme May/June 2022
PUBLISHED
Question Answer Marks Guidance

5(a) Widdringtonia ; 1

5(b) any three from: 3


1 no roots, to absorb water / bind soil / hold soil together ;
2 idea of: no canopy / AW, to protect (soil) from rainfall ;
3 increase in run off / AW ;
4 ref. to soil erosion ;
5 loss of, mineral content / ions / nutrients / AW, in the soil ;
6 flooding (in the valleys) ;
7 landslides / mudslides ;
8 ref. to visual pollution / dead tree stumps / bare ground / AW ;
9 loss of biodiversity / disruption of food chains / disruption of food
webs / (species) extinction ;
10 AVP ;

5(c) food source(s) ; 2


nesting / breeding, sites ;
shelter / shade / protection from predators ;
leaf litter for decomposers ;
ref. to nutrient cycling ;
(named) resources for humans ;
AVP ;

5(d) any two from: 2


ref. to genetic, diversity / variation ;

importance of genetic diversity:


plants (grown from seeds) may be adapted to changes in the
environment ;
plants (grown from seeds) may be resistant to, diseases / pests ;
(seeds collected) may not, be viable / germinate ;
(seeds collected) may, be diseased / have parasites / AW ;
(seeds collected) may have harmful, alleles / mutations ;
AVP ; e.g., increase in fitness

© UCLES 2022 Page 11 of 12

Page 643 of 877


0610/41 Cambridge IGCSE – Mark Scheme May/June 2022
PUBLISHED
Question Answer Marks Guidance

6(a) P – testis ; 6
S – zygote ;
Q – meiosis ;
R – fertilisation ;
T – mitosis ;
U – implantation ;

6(b)(i) idea of maintenance of the same number of chromosomes (from 1


generation to generation) / so (diploid) number of chromosomes does not
double at fertilisation (described) ;

6(b)(ii) to prevent more than one sperm fertilising the egg / stops other sperm 1
entering ;

6(c) 1 diffusion / exchange ; 7


2 amino / fatty ;
3&4 ;; max. two from:
glucose
(named) vitamins
(named) minerals / ions / salts
glycerol
fatty acids A fatty acids or amino acids only once, i.e. MP2
amino acids OR MP3/4
5 passive ;
6 antigens ;
7 pathogen / (micro)organism ;

© UCLES 2022 Page 12 of 12

Page 644 of 877


0610/42 Cambridge IGCSE – Mark Scheme May/June 2022
PUBLISHED

Question Answer Marks Guidance

1(a) 10 (m) ; 1

1(b) 5 one mark per correct row


letter in
function structure
Fig. 1.1

storage of DNA / controls


(named) activity of cell
nucleus C ;
/ produces RNA
/ makes ribosomes

photosynthesis
chloroplast E ;
/ absorb light / AW

mitochondrion
aerobic respiration D ;
/ mitochondria

contains cell sap and


vacuole B ;
stores water

supports (the cell)


/ resists turgor pressure cell wall A ;
/ prevents bursting / AW

1(c)(i) diffusion ; 1

1(c)(ii) (enters through) stoma(ta) / between guard cells ; 2


(moves through) interconnecting / intercellular / air, spaces ;

1(d)(i) limiting factor ; 1

© UCLES 2022 Page 5 of 12

Page 645 of 877


0610/42 Cambridge IGCSE – Mark Scheme May/June 2022
PUBLISHED

Question Answer Marks Guidance

1(d)(ii) (low / no) light intensity / (extreme air) temperature / (low / no) water 1
(availability) / (incorrect) (wavelength of light / (short) day length ;

1(e) any two from: 2


1 reduces, extraction / conserves, (named) fossil / non-renewable,
fuels ;
2 (temporarily) removes, carbon dioxide / greenhouse gases (from
atmosphere) / does not add more, carbon dioxide / greenhouse gas
(to the atmosphere) / idea that the process is carbon neutral ;
3 does not cause / prevents a further increase in, enhanced
greenhouse effect ;
4 does not cause / prevents further increase in global warming /
prevents further climate change ;
5 it is a sustainable process / uses renewable fuel source / described ;
6 AVP ; e.g. described example of less environmental impact caused
by use of non-renewable (fossil) fuels ;

Question Answer Marks Guidance

2(a)(i) glomerulus ; 1

2(a)(ii) any one from: 1


(vessel Q) is a (renal) artery / blood has not passed through any A it is an arteriole
capillaries ;
blood (in Q) comes (straight) from, the heart / an artery / aorta ;
(vessel Q) is narrow(er than R) ;
(vessel Q) has thick / elastic, walls ;
for (ultra)filtration ;

2(a)(iii) increase surface area ; 2


for faster / more, (re)absorption (of glucose / amino acids / minerals /
ions / salts / water / vitamins / nutrients / (other relevant) substances) ;

© UCLES 2022 Page 6 of 12

Page 646 of 877


0610/42 Cambridge IGCSE – Mark Scheme May/June 2022
PUBLISHED

Question Answer Marks Guidance

2(b) any six from: 6


1 no protein in, region 1 / (Bowman’s / renal) capsule / protein only in,
P / blood / plasma ;
2 all glucose / salts / urea, is filtered out, of P / blood plasma / into
region 1 / (Bowman’s / renal) capsule ;
3 (re)absorption of, all glucose, by region 3 / loop of Henle / in tubule /
in region 2 / after region 1 ; ora
4 (re)absorption of, some salts, by / at, region 3 / by loop of Henle / in
tubule / in region 2 / after region 1 ;
5 urea concentration is, increased / higher in, region 3 / loop of Henle ;
6 idea that size of the substance determines what is filtered ;
7 glucose / salts / urea, filtered out of blood / plasma OR proteins, stay
in blood / plasma / not filtered out, of blood / plasma ;
8 active transport, of glucose / salts (from tubule / back into blood
/ back into plasma) ;
9 movement of, glucose / salts, against a concentration gradient /
through proteins (in membranes) ;
10 (active transport) uses energy from, respiration / mitochondria ;
11 (most) water (re)absorbed by osmosis (in region 3 / loop of Henle) ;
12 urea concentration increases as a result of reabsorption of water ;
13 urea / excess salt, is, an excretory substance / waste product (of
metabolism) / toxic ;
14 urine contains salts and urea ;

2(c)(i) homeostasis ; 1

2(c)(ii) any three from: 3


1 (blood) glucose concentration, is low / decreases ;
2 (causing) glucagon, secretion / production ;
3 glucagon, released from / produced in, pancreas ;
4 (glucagon stimulates) breakdown of glycogen / release of glucose
(into the blood) ;
5 from liver / muscle ;
6 (blood) glucose concentration, goes (back) up / returns to normal
/ stays within limits ;

© UCLES 2022 Page 7 of 12

Page 647 of 877


0610/42 Cambridge IGCSE – Mark Scheme May/June 2022
PUBLISHED

Question Answer Marks Guidance

3(a)(i) refraction ; 1

3(a)(ii) any three from: 3


ciliary muscles contract ;
tension in suspensory ligaments decreases ;
lens becomes more convex ;
causing more refraction ;

3(b)(i) any four from: 4


rods / cones, are light receptors OR detect / respond / sensitive, to light ;

rods:
sensitive to / function in, light of low intensity OR used for night vision ;
provide black and white vision ;

cones:
sensitive to light of high intensity ;
ref. to three different types of cone ;
provide colour vision ;
AVP ;

3(b)(ii) 3 one mark for each correct column


distribution across the retina
receptor
peripheral retina blind spot fovea

rods many none none / few

cones few none many


; ; ;

3(c)(i) only males, are colour-blind / colour-blindness is more common in 1


males / no females are colour-blind / person 1 and 2 had two male
colour-blind children (but no female colour-blind children) ;

© UCLES 2022 Page 8 of 12

Page 648 of 877


0610/42 Cambridge IGCSE – Mark Scheme May/June 2022
PUBLISHED

Question Answer Marks Guidance

3(c)(ii) XAXa ;; 2 MP1 correct sex chromosomes


MP2 correct alleles

3(c)(iii) 1 correct genotype for person 3, i.e. (X)A(X)a ; 5


2 correct genotype for person 4, i.e. XAY ;
3 correct gametes from the parental genotypes ; MP3 ecf from MP1 and MP2
4 correct offspring genotypes from their gametes / parental genotypes MP4 ecf from MP3
(in any order) ;
5 correct offspring correct probability ; MP5 ecf from their offspring genotype

expected answer:
genotype (person 3) XAXa  (person 4) XAY

gametes XA, Xa + XA, Y

offspring genotypes XAXA , XAXa , XAY, XaY

offspring phenotypes
female with normal colour vision
female with normal colour vision
male with normal colour vision
male with colour blindness)

probability 0.25 / 25% / 1 in 4 / ¼

Question Answer Marks Guidance

4(a) N – protein coat ; 2


O – genetic, material / DNA / RNA ;

© UCLES 2022 Page 9 of 12

Page 649 of 877


0610/42 Cambridge IGCSE – Mark Scheme May/June 2022
PUBLISHED

Question Answer Marks Guidance

4(b)(i) –87(%) ;;; 3 Use ecf from each previous step throughout
MP1: both correct readings from graph 760
and 100
MP2: correct answer calculated
MP3: correct rounding to two significant
figures

4(b)(ii) any two from: 2


(overall) decrease in number (of lymphocytes during the 84 months) ;
rapid decrease from 10 months and then, less steep / (eventually) levels
off ;
suitable description from graph ;

4(b)(iii) any four from: 4


fewer antibodies (produced by lymphocytes) ;
decrease in immunity / inefficient immune system ;
fewer memory cells ;
any role of antibodies or lymphocytes (that will be impacted by fewer
lymphocytes) ;
develop AIDS ;
example of (secondary) infection / disease / pathogen that may result
from reduced number of lymphocytes ;

4(b)(iv) viruses are, acellular / non-living / do not have (named) cell structure ; 2 A antibiotics do not affect viruses / antibiotic
only affect, cells / bacteria
antibiotics target (named) cell structures / (named) process ;

© UCLES 2022 Page 10 of 12

Page 650 of 877


0610/42 Cambridge IGCSE – Mark Scheme May/June 2022
PUBLISHED

Question Answer Marks Guidance

5(a)(i) X – larynx ; 3
Y – trachea ;
Z – bronchus ;

5(a)(ii) cartilage ; 1

5(b) any four from: 4


1 external intercostal muscles contract (and internal intercostal
muscles relax) ;
2 lifts ribs, upwards / outwards;
3 diaphragm, contracts / flattens;
4 volume of, thorax, increases ;
5 pressure in, thorax, decreases ;
6 causing air to flow, down a pressure gradient / into the lungs / lungs
inflate ;
7 equalising pressure between atmosphere and lungs ;

5(c) alveoli / alveolus ; 1

Question Answer Marks Guidance

6(a) community / communities ; 2


environment(s) ;

6(b)(i) 4; 1

6(b)(ii) (reef) shark ; 1

6(b)(iii) 4; 1

6(b)(iv) no arrows pointing to phytoplankton / AW ; 1

© UCLES 2022 Page 11 of 12

Page 651 of 877


0610/42 Cambridge IGCSE – Mark Scheme May/June 2022
PUBLISHED

Question Answer Marks Guidance

6(b)(v) any three from: 3


1 idea that energy transfer along a food chain is inefficient ;
2 and 3 named examples of causes of inefficient energy flow between
trophic levels ;;
4 energy to, decomposers / decomposer food chain(s) ;
5 limited / not enough, energy to support another trophic level ;

6(c) any three from: 3 examples of named threats:


1 named threat ; (micro)plastics / litter / sewage / fertilisers /
2 and 3 detail of how it is a threat to aquatic ecosystems ;; effluent / (named) introduced / alien species
4 another named threat ; / escaped farmed fish / genetically modified
5 detail of how it is a threat to aquatic ecosystems ; fish / (over)fishing / hunting / cyanide fishing
/ bomb fishing / (named) chemical / nuclear,
pollutants / (named) development in
sea / coast / (named) development in
sea / coast / shipping / (named) tourism

examples of detail of threat:


habitat destruction / toxic to (aquatic)
organisms / chokes / strangles / traps / blocks
digestive systems (of aquatic animals)
/ accumulates in an organism
/ passes down a food chain / eutrophication
/ algal bloom / feminisation of fish / acidification
of water / sedimentation in water

© UCLES 2022 Page 12 of 12

Page 652 of 877


0610/43 Cambridge IGCSE – Mark Scheme May/June 2022
PUBLISHED
Question Answer Marks Guidance

1(a)(i) bacteria ; 1

1(a)(ii) antibiotic ; 1

1(b)(i) any two from: 2


nucleus ;
mitochondria ;
(r)ER ;
vacuole ;
AVP ; e.g., hyphae / structure of cell wall

1(b)(ii) Penicillium ; 1

1(b)(iii) any two from: 2


glucose ;
ammonia / amino acids ;
(named mineral) salts / ions ;;
water ;

1(b)(iv) stops the growth of microorganisms / AW ; 1 A prevent contamination

1(b)(v) any two from: 2


release of, waste gas / carbon dioxide ;
(carbon dioxide / gas) is produced during fermentation / respiration ;
to prevent fermenter exploding / to reduce pressure ;
carbon dioxide is toxic (to fungus if it builds up) ;

© UCLES 2022 Page 5 of 11

Page 653 of 877


0610/43 Cambridge IGCSE – Mark Scheme May/June 2022
PUBLISHED
Question Answer Marks Guidance

1(b)(vi) any four from: 4


1 (step 4) is reproduction (of A before adding to fermenter) ; A idea of adapting to environment
2 (step 5) inoculation in a fermenter / AW ;
3 (step 6) exponential growth (of organism A in fermenter) ;
4 sampling to check when penicillin is ready ;
5 penicillin is, a byproduct of maximum growth / a toxin
/ fermentation ;
6 (step 7) cells / product / penicillin, are removed ;
7 (step 8) purification / filtering / described ; A drying / AW
8 idea of packaging of penicillin ;
9 AVP ; e.g., batch cultures (described) / limited
resources / stationary phase triggers production of
penicillin

Question Answer Marks Guidance

2(a)(i) carbon and oxygen and hydrogen ; 2


nitrogen ;

2(a)(ii) binds to / releases / transports / AW, oxygen ; 1

2(b) any two from: 2


(it is) sickle-shaped / not biconcave ;
rigid / inflexible ;
get stuck in / block, blood vessels / capillaries ;
reference to (increased) clotting ;

© UCLES 2022 Page 6 of 11

Page 654 of 877


0610/43 Cambridge IGCSE – Mark Scheme May/June 2022
PUBLISHED
Question Answer Marks Guidance

2(c) parental phenotypes unaffected / sickle trait / (sickle) carrier / AW  5 ecf for subsequent rows
unaffected / sickle trait / (sickle) carrier ;

parental genotypes HbA HbS  HbA HbS ;

gametes HbA, HbS + HbA, HbS ;

offspring genotypes HbAHbA HbAHbS (HbAHbS ) HbSHbS ;

probability 0.25 / 25% / ¼ ;

2(d)(i) any five from: 5


1 A has, almost no / low(est), HbS (allele) frequency / AW, but
medium births ;
2 B has high(est) births and high(est) HbS (allele) frequency / AW ;
3 C has low(est) number of births and low / medium HbS (allele)
frequency / AW ;
4 B supports the statement / A does not support the statement / no
link between births and allele frequency / AW ;
5 data is estimated not actual ;
6 maps have different level of detail ;
7 ref. to effect of population density ;
8 AVP ;

2(d)(ii) look at, DNA base sequence / amino acid sequence / AW ; 1

2(e) any four from: 4


mutations are found in the DNA ;
asexual reproduction offspring are, genetically identical to / clones of,
the parent / AW ;
(so) any existing mutations (in parent DNA) will be inherited / AW ;
sexual reproduction (usually) involves two parents ;
mutation will only be in one of the parents ;
mutation will only be in some of the gametes ;
meiosis does not result in genetically identical cells ;

© UCLES 2022 Page 7 of 11

Page 655 of 877


0610/43 Cambridge IGCSE – Mark Scheme May/June 2022
PUBLISHED
Question Answer Marks Guidance

3(a) any three from: 3


lowers pH / acidifies soil ;
(acidified soil) causes leaching ;
burn / corrode / damage , leaves ;
affects ability of roots to absorb mineral ions ;
consequence of named mineral ion deficiency described ;
AVP ; e.g., described root damage / reduced ability to
take up water

3(b) any four from: 4


species become endangered / extinct ;
because there is a loss of habitat ;
knock on negative effect to, other organisms / food chains ;
(increases) soil erosion / nutrient leaching ;
causes flooding ;
(increases) landslides ;
idea of change in the pattern of rainfall ;
less photosynthesis ;
increase (atmospheric) carbon dioxide / less carbon dioxide removed ;

3(c) any two from: 2


adapted to a different pH ;
moist / permeable, skin ;
large surface to volume ratio ;
may have gills which are, fragile / thin / AW ;
lays (unshelled) eggs in water ;
part of the life cycle / larval stages, only exist in water ;

3(d) any three from: 3


scrubbers / flue gas desulfurisation, in power stations / chimneys ;
reduce use of coal-fired power stations ;
use alternative / renewable, energy sources ;
catalytic converters ;
low sulfur fuel / AW ;
recycling / reusing / reducing, plastic / less plastic waste burnt ;
AVP ; e.g., electric cars / carpool / public transport

© UCLES 2022 Page 8 of 11

Page 656 of 877


0610/43 Cambridge IGCSE – Mark Scheme May/June 2022
PUBLISHED
Question Answer Marks Guidance

4(a) 6 one mark for each correct row


letter or letters of all the
number from
name of the organ processes that occur in the
Fig. 4.1
organ

1 mouth I, C, M

2 stomach C, M (A)

3 duodenum A, C (M)

4 ileum A, C

5 colon A

6 anus E

;;;;;;

4(b) any six from: 6


1 P is a lacteal ;
2 (lacteals) absorb, fats / fatty acids / glycerol / (named) fat soluble
vitamin ;
3 T / lymphatic vessel, returns lymph / fats, to blood / to circulatory
system ;
4 Q / epithelial cell, has microvilli ;
5 (microvilli / villi) increase the surface area for absorption ;
6 Q is one cell thick for short diffusion distance / fast diffusion
7 Q is site of breakdown of maltose to glucose ;
8 glucose uptake by active transport ;
9 R / capillary, for absorption of, amino acids / glucose / salts
/ water / products of digestion / soluble nutrients ;
10 R has thin walls / R is one cell thick ;
11 blood flowing to maintain concentration gradient / AW ;
12 transports absorbed nutrients to, S / venule / vein ;

© UCLES 2022 Page 9 of 11

Page 657 of 877


0610/43 Cambridge IGCSE – Mark Scheme May/June 2022
PUBLISHED
Question Answer Marks Guidance

5(a)(i) carbon dioxide is needed for photosynthesis ; 3


(atmospheric) carbon dioxide concentration is a limiting factor / AW ;
glucose is, a product of photosynthesis / needed for growth ;

5(a)(ii) (named fossil fuel) burners / carbon dioxide gas cylinders / AVP ; 1

5(a)(iii) diffusion / high concentration to low concentration ; 2


through stomata / ref. to guard cells ;

5(b)(i) 65(%) ;; 2 MP1 correct calculation e.g.


(651 – 394) ÷ 394 = 65.228 to any number of
significant figures
MP2 correct rounding to two significant figures
ecf MP2 from incorrect MP1

5(b)(ii) sodium ; 1

5(b)(iii) any three from: 3


enzymes are involved (in, photosynthesis / respiration) ;
(too much heat) can, denature enzyme / change shape of active site ;
correct ref. to, transpiration / described / water vapour loss ;
(more likely to) wilt ;
decrease in yield / death ;
AVP ;

© UCLES 2022 Page 10 of 11

Page 658 of 877


0610/43 Cambridge IGCSE – Mark Scheme May/June 2022
PUBLISHED
Question Answer Marks Guidance

6(a) any one from 1


two cotyledons ;
(leaves have) branching / network / AW, veins ;
broad leaves ;
(multiples of) five petals ;
petioles ;

6(b) (made of) protein / biological ; 2


catalyst / described ;

6(c)(i) pectinase ; 1

6(c)(ii) 45 minutes ; 1

6(c)(iii) (vitamin) C / ascorbic acid ; 1

6(d)(i) genetic, engineering / modification ; 1

6(d)(ii) any three from: 3


reduced natural biodiversity ;
insect-resistant plants negatively affect pollinators ;
cross-pollinate into wild populations ;
(GM seeds are) expensive / AW ;
seeds need to be bought each season ;
ethical concerns of consumers ;
AVP ;

© UCLES 2022 Page 11 of 11

Page 659 of 877


0610/42 Cambridge IGCSE – Mark Scheme February/March 2022
PUBLISHED

Question Answer Marks Guidance

1(a)(i) alveoli / alveolus ; 1

1(a)(ii) any two from: 2


large surface area ;
thin surface / short diffusion distance ;
good blood supply ;
good ventilation (with air) ;
AVP ;

1(b)(i) correct label line with the label X to the external intercostal muscle ; 1

1(b)(ii) name cartilage ; 2


function support / prevent collapse / allows movement of air ;

1(b)(iii) increases in volume ; 2


decreases in pressure ;

1(c)(i) B oxygen ; 3
C carbon dioxide ;
D water vapour ;

1(c)(ii) ref. to (aerobic) respiration ; 3


B / oxygen, is a reactant of respiration / diffuses across alveoli / into
cells or blood ;
C / carbon dioxide, is a product of respiration / diffuses across
alveoli / out of cells or blood ;

© UCLES 2022 Page 5 of 12

Page 660 of 877


0610/42 Cambridge IGCSE – Mark Scheme February/March 2022
PUBLISHED

Question Answer Marks Guidance

2(a) sperm (max. four from): 6


1 presence of acrosome (containing enzymes) ;
2 idea of enzymes, digest / breakdown, egg membrane / jelly coat ;

3 many mitochondria ;
4 for respiration / to release energy (for swimming) ;

5 flagellum / streamlined shape ;


6 for, swimming / movement / locomotion ;

egg:
7 energy / (named) nutrient, stores ;
8 energy / (named) nutrients for, development of
embryo / mitosis / cell division ;

9 jelly coat ;
10 (hardens) to prevent other sperm from entering (after
fertilisation) ;

both:
11 haploid (nuclei) / half the normal number of chromosomes or 23
chromosomes in, sperm / egg ;
12 gametes are haploid so, correct number of chromosomes in
/ 46 chromosomes in / diploid, zygote ;

2(b)(i) any three from: 3


sample of, semen / sperm, collected / donated ;
sperm, washed / concentrated ;
(semen / sperm) inserted into vagina / uterus / near cervix ;
ref. to (sperm insertion) at the time of fertility ;
ref. to fertility drugs ;

© UCLES 2022 Page 6 of 12

Page 661 of 877


0610/42 Cambridge IGCSE – Mark Scheme February/March 2022
PUBLISHED

Question Answer Marks Guidance

2(b)(ii) any two from: 2


gametes collected from both parents / eggs harvested or collected ;
fertilisation occurs, outside the body / in a dish / not in oviduct ;
fertilisation can occur by injecting sperm into the egg ;
embryo inserted ;
more invasive / surgical procedure (in reference to egg collection) ;
AVP ; e.g. screening of embryos

2(b)(iii) any four from: 4


1 treatments are expensive ;
2 idea that there are questions over who will fund treatments ;
3 risk of multiple, births / children / pregnancies ;
4 poor success rates ;
5 ref to emotional stress / ref to not knowing heritage ;
6 ethical / religious / cultural, issues, with fertility treatment process ;

7 allows people who have lost their partners to have children ;


8 allows people undergoing cancer treatment to have children ;
9 lower chance of genetic diseases if embryos screened ;
10 allows infertile people to have children ;
11 allows older people to have children ;
12 spare embryos can be used for, medical
research / treatments / stem cells ;

13 AVP ; e.g screening for features in embryos

© UCLES 2022 Page 7 of 12

Page 662 of 877


0610/42 Cambridge IGCSE – Mark Scheme February/March 2022
PUBLISHED

Question Answer Marks Guidance

3(a)(i) any five from: 5


1 as temperature increases the mass (of leaves) decreases / AW ;
2 by (increased) transpiration ;
3 evaporation increases ;
4 from surface of the mesophyll cells ;
5 due to increased (kinetic) energy of water molecules ;
6 diffusion of water vapour increases ;
7 through stomata ;

3(a)(ii) humidity / AVP ; 1 e.g. windspeed

3(b)(i) xylem ; 1

3(b)(ii) any three from: 3


1 hollow / no cell contents, to reduce resistance / allow efficient
flow ;
2 no end walls, to reduce resistance / allow efficient flow ;
3 large cross-sectional area / wide, to allow transport of large
volume of water ;
4 lignin to, prevent collapse / provide support / provide strength ;
5 waterproof to prevent water loss ;
6 AVP ; e.g. bordered pits for lateral fluid transport

3(c) any three from: 3


into the root hair (cell) ;
by active transport ;
against a concentration gradient / AW ;
using energy ;
using carrier proteins (in the cell membrane) ;
(also / initially) by diffusion ;

© UCLES 2022 Page 8 of 12

Page 663 of 877


0610/42 Cambridge IGCSE – Mark Scheme February/March 2022
PUBLISHED

Question Answer Marks Guidance

4(a)(i) similarities max two from: 4


(presence of) cell wall ;
(presence of) cell membrane ;
(presence of) DNA / genetic material ;
(presence of) ribosomes ;
(presence of) cytoplasm ;

differences max two from:


no plasmids ;
no flagella ;
no capsule ;
no loop of DNA ;
presence of chloroplasts ;
presence of mitochondria ;
presence of nucleus ;
presence of large vacuole ;

AVP ;

4(a)(ii) any three from: 3


bacteria releases a toxin ;
causing chloride ion, secretion / AW, (into small intestine) ;
reducing water potential (in the gut) ;
ref. to osmosis / osmotic movement ;
water moves, into gut / into intestine / out of cells / AW ;

4(b)(i) largest clear area / has prevented most growth / killed the most 1
bacteria / AVP ; e.g. bacteria has no resistance (to antibiotic E)

© UCLES 2022 Page 9 of 12

Page 664 of 877


0610/42 Cambridge IGCSE – Mark Scheme February/March 2022
PUBLISHED

Question Answer Marks Guidance

4(b)(ii) any four from: 4


1 ref. to natural selection ;
2 ref. to mutation ;
3 bacteria without resistance die / bacteria with resistance survive ;
4 (resistant / surviving, bacteria) multiply ;
5 passing their, alleles / genes, (for resistance) on to their offspring ;
6 AVP ; e.g. ref. antibiotics destroy bacterial cell
walls / resistance passed through plasmids

4(b)(iii) only take antibiotics when needed / complete the course of 1


treatment / restrict the use of antibiotic B ;

Question Answer Marks Guidance

5(a) hair / fur ; 2


external ears / pinnae ;

5(b)(i) 247 ;;; 3 MP1 correct readings from graph of 75 and 260
MP2 correct calculation
(185÷75) × 100 = 246.66
MP3 correct rounding to 3 significant figures
ecf MP2 and MP3

5(b)(ii) any three from: 3


overall decrease in numbers / ref to increase then decrease ;
in 1980 there are 15 rhinoceros ;
(numbers of rhinoceros) increase to 32 in 2003 ;
(numbers of rhinoceros) decrease to 2 in 2020 ;
data manipulation ;
correct comparison of difference in rate of, increase / decrease ;

© UCLES 2022 Page 10 of 12

Page 665 of 877


0610/42 Cambridge IGCSE – Mark Scheme February/March 2022
PUBLISHED

Question Answer Marks Guidance

5(c) any three from: 3


providing, habitats / protected areas / AW ;
(laws to) prevent / limit, hunting / poaching ;
education / AW ;
(captive) breeding programmes / use of AI ;
AVP ;;; e.g. reintroduction programmes / legislation

5(d) any three from: 3


1 inbreeding ;
2 lack of genetic diversity ;
3 increased risk of genetic diseases / mutation ;
4 reduced ability to survive changes in the environment ;
5 increased risk of extinction ;
6 only small population, sex unknown / difficult to locate each
other / too old to breed ;
7 AVP ;

© UCLES 2022 Page 11 of 12

Page 666 of 877


0610/42 Cambridge IGCSE – Mark Scheme February/March 2022
PUBLISHED

Question Answer Marks Guidance

6(a) 3
name of
function site of action one mark for each correct row
enzyme

breaks down protein to,


pepsin stomach
amino acids / (poly)peptides

breaks down protein


trypsin / polypeptides, to amino acids small intestine
/ peptides

intestinal
maltase breaks down maltose to glucose epithelium
/ small intestine

;;;

6(b)(i) A gene / allele ; 2


C ribosome ;

6(b)(ii) mRNA / a copy of the, gene or A ; 2


moves out of, the nucleus (to the cytoplasm) ;

6(b)(iii) the sequence of bases in, mRNA / DNA / gene ; 1

6(c) ref. to specificity ; 3


(shape of) active site and substrate must be complementary ;
to enable, enzyme and substrate to bind / enzyme-substrate
complexes to be formed ;

© UCLES 2022 Page 12 of 12

Page 667 of 877


0610/41 Cambridge IGCSE – Mark Scheme May/June 2021
PUBLISHED
Question Answer Marks Guidance

1(a)(i) prokaryote / plant ; 1

1(a)(ii) 3 one mark per row


letter from
cell function cell structure
Fig. 1.1

storage of genes nucleus G ;

aerobic respiration mitochondria E ;

ribosome(s) H/F ;
amino acids are assembled OR
to make protein (rough) endoplasmic F
reticulum / (R)ER

1(b)(i) 0.002 (mm) ; 1

1(b)(ii) length of, drawing / image / Fig. 1.2 (in mm) ; 1

© UCLES 2021 Page 5 of 13

Page 668 of 877


0610/41 Cambridge IGCSE – Mark Scheme May/June 2021
PUBLISHED
Question Answer Marks Guidance

1(c) total of six from: 6

similarities, max four from:


single cell / unicellular / AW ;
(cell) wall / A ;
cytoplasm / D ;
ribosomes / H ;
cell membrane / B ;
DNA / genetic material ;

differences, max four from: ;;;;


(bacteria have) (yeast have)

no nucleus / no G / has nucleoid nucleus / G

loop / coil / AW, of DNA linear DNA / chromosome

no, (rough) endoplasmic (rough) endoplasmic reticulum


reticulum / ER

no mitochondria / no E mitochondria / E

no (large / permanent) (large / permanent) vacuole / C


vacuole / no C

plasmid(s) no plasmids

no membrane-bound, cell membrane-bound, cell


structures / organelles structures / organelles

© UCLES 2021 Page 6 of 13

Page 669 of 877


0610/41 Cambridge IGCSE – Mark Scheme May/June 2021
PUBLISHED
Question Answer Marks Guidance

1(d) 1 = nitrogen fixation ; 3


2 = deamination ;
3 = nitrification ;

Question Answer Marks Guidance

2(a) any four from: 4


quantity of plastic waste always higher inside GPGP than outside
it ; ora
quantity inside GPGP constant (from 1965) to, any year 1975 to 1994 ;
quantity increases in GPGP, steeply ;
smaller (overall) increase outside the GPGP ; ora
quantity of plastic waste outside the GPGP, fluctuates / AW ;
comparative data quote with years and unit ;

2(b)(i) any one from: 1


leathery / hard / scaly, skin ;
hard(er) / rubbery / leathery / AW, eggs ;
lay eggs on land (not in water) ;
internal fertilisation ;

© UCLES 2021 Page 7 of 13

Page 670 of 877


0610/41 Cambridge IGCSE – Mark Scheme May/June 2021
PUBLISHED
Question Answer Marks Guidance

2(b)(ii) any four from: 4

direct effects
(non-biodegradable plastic) does not break down ;
idea that ability to breathe affected ;
idea that ability to move affected ;
idea that ability to gain nutrition affected ;
damage / injury / infection / death ;
toxic / poisonous ;

indirect effects
blocks (sun)light, so algae / plants / producers, cannot photosynthesise ;
(so) less, food / energy, enters, food chains / food webs ;
loss of (named), habitat / feeding / breeding area ;
more likely to be predated ;
idea that (plastic) accumulates up the food chain / bioaccumulation ;

AVP ;

2(b)(iii) any two from: 2


use (named) alternatives for plastic ;
reduce, use of / manufacture, single-use plastic ;
use biodegradable plastics instead of non-biodegradable plastics ;
reduce unnecessary packaging ;
reuse plastic product(s) ;
recycle / described ;
education / awareness campaigns ;
AVP ; e.g. idea of fines / punishment, for unregulated disposal of plastic
waste / legislation / taxation / charging for using plastic / money-back
schemes / quotas or limits for production / international treaties

© UCLES 2021 Page 8 of 13

Page 671 of 877


0610/41 Cambridge IGCSE – Mark Scheme May/June 2021
PUBLISHED
Question Answer Marks Guidance

3(a) any three from: 3


(small so do) not need much space ;
rapid reproduction rate / AW ;
can make complex molecule(s) ;
no ethical concerns over use ;
idea that genetic code is universal ;
they have plasmids ;
idea that plasmids are used, as vectors / in genetic engineering ;
simple requirements / AW, so, easy / cheap, to grow ;
AVP ;

3(b)(i) pancreas ; 1

3(b)(ii) (type 1) diabetes ; 1

3(b)(iii) reduces blood sugar concentration ; 1

3(c)(i) 1 y-axis = number / population, of (living) bacteria per (1.0) mm3 1


and
2 x-axis = time / hours / days ;

3(c)(ii) P – lag 2 four correct = 2 marks


Q – exponential / log two or three correct = 1 mark
R – stationary one correct = 0
S – death / decline
;;

3(c)(iii) any three from: 3


limiting factor(s) / factors that limit ;
not enough space / overpopulation ;
competition ;
fewer / no, (named) nutrient(s) / food ;
not enough oxygen ;
change in pH / increase in acidity ;
increase in temperature ;
build-up of, (named) toxic / waste, substances ;

© UCLES 2021 Page 9 of 13

Page 672 of 877


0610/41 Cambridge IGCSE – Mark Scheme May/June 2021
PUBLISHED
Question Answer Marks Guidance

3(d) total of three from: 3

importance of iron to max 2


to make / for, haemoglobin ;
to make / for, red blood cells ;
for transport of oxygen ;
AVP ;

effects of iron deficiency


(iron-deficiency) anaemia ;
any symptom of anaemia ;

3(e)(i) restriction (enzyme) ; 1

3(e)(ii) any two from: 2


use same (restriction) enzyme to cut cassava DNA ;
sticky ends are, formed / joined ;
ref. to complementary, ends / base pairs ;
(joined by) ligase ;

3(e)(iii) any two from: 2


idea that need to check that offspring inherit ‘new’ gene / trait ;
ensure new (GM) variety can grow in field conditions / AW ;
make sure the ‘new’ gene does not cause any adverse effects ;
make sure (GM) plants have, the same / a high enough, yield ;
idea of building up a large quantity of, seed / plants, to sell to farmers ;
AVP ;

Question Answer Marks Guidance

4(a) gravitropism ; 1

4(b) idea of negating the effect of gravitational pull / gravity, on one side of the 2
plant (only) ;
(used as a control) to compare with (the response of) plant B ;

© UCLES 2021 Page 10 of 13

Page 673 of 877


0610/41 Cambridge IGCSE – Mark Scheme May/June 2021
PUBLISHED
Question Answer Marks Guidance

4(c)(i) auxin ; 1

4(c)(ii) any three from: 3


(auxin / hormone produced in the tip)
moves / diffuses, away from tip / down the, stem or shoot or plant ;
moves to / collects on, one / lower side (of stem / plant) ;
stimulates cell elongation ;
stem / plant, bends / grows / turns, upwards ;
AVP ;

4(d) any three from: 3


reach light for photosynthesis ;
(reach air for) carbon dioxide for photosynthesis ;
(reach air for) oxygen for respiration ;
idea that they grow tall so that flowers are exposed for, (wind / insect)
pollination ;
AVPs e.g. roots grow downwards / towards (direction of) gravity ;
to reach, water / ions or minerals ;
to anchor plant in the soil ;

Question Answer Marks Guidance

5(a) lumen or wall of left or right oviduct labelled Q ; 4


area of uterus with darkest shading labelled R ;
ovary labelled S ;
vagina labelled T ;

5(b)(i) any three from: 3


ref. to making new cells after fertilisation ;
making membranes ;
(protein for) making enzymes ;
making new, (named) cell structures / cytoplasm ;
(fat / protein) provide / source of, energy ;
energy for, cell division / mitosis / growth (of cell) / metabolism / AW ;

© UCLES 2021 Page 11 of 13

Page 674 of 877


0610/41 Cambridge IGCSE – Mark Scheme May/June 2021
PUBLISHED
Question Answer Marks Guidance

5(b)(ii) any two from: 2


hardens / changes, after fertilisation / entry of sperm ;
prevents more sperm entering ;
AVP ;

5(c) any four from: 4


chromosomes / DNA, duplicate(s) / replicate(s) ;
chromosomes separate ;
mitosis / nuclear division ;
(zygote / fertilised egg) divides / splits (into two) ;
(nuclear / cell) division / mitosis, repeated / AW ;
forming a, ball / cluster, of cells ;
cells are genetically identical ;
AVP ; e.g. cell, specialisation / differentiation / ref. to stem cells

Question Answer Marks Guidance

6(a)(i) any two from: 2


ref. to passive immunity ;
idea of immediate / fast, protection / response / AW ;
AVP ; e.g. idea that gives time for immune system to produce own,
antibodies / antitoxins

6(a)(ii) any three from: 3


ref. to active immunity ;
(more) memory cells are produced ;
long-term, immunity / protection ;
after second injection higher concentration of antibodies than passive
immunity ;
reduces the chance of catching the disease (again) / AW ;
response to second injection is, faster / greater, than first ;
AVP ;

© UCLES 2021 Page 12 of 13

Page 675 of 877


0610/41 Cambridge IGCSE – Mark Scheme May/June 2021
PUBLISHED
Question Answer Marks Guidance

6(b) any four from: 4


pathogens have antigens ;
antibodies, lock on to, antigens / pathogens ;
antibody is specific (to antigen / pathogen) ;
antibody has a complementary (shape) to antigen / AW ;
antibody marks pathogen for, destruction by phagocytes / phagocytosis ;
antibodies destroy pathogens / described ;
AVP ;

6(c) any two from: 2


absorbs / transports, fat / fatty acids (and glycerol from lacteals) ;
drains / AW, tissue fluid ;
returns, fluid / lymph, to, blood / plasma ;
AVP ;

© UCLES 2021 Page 13 of 13

Page 676 of 877


0610/42 Cambridge IGCSE – Mark Scheme May/June 2021
PUBLISHED
Question Answer Marks Guidance

1(a) central and peripheral (nervous system) ; 1

1(b)(i) motor ; 1

1(b)(ii) 4 one mark per row

letter in Fig.
part of the eye function
1.1

• controls / changes, shape /


size, thickness, of lens
suspensory • ref. to
G
ligament accommodation / focusing
• attachment of lens to ciliary
muscles

circular muscles contracts in response to a bright


E
/ iris light

cornea refracts / bends, light D

contains a high density of cones


fovea J
for colour vision
;;;;

1(c)(i) (ciliary muscles) contract / conduction of (nerve) impulses ; 1

1(c)(ii) haemoglobin ; 1

1(c)(iii) any two from: 2


diffusion (across cell membranes) ;
down concentration gradient / AW ;
passive process / (use of) kinetic energy / random movement of
molecules ;
through a thin capillary wall ;

© UCLES 2021 Page 5 of 12

Page 677 of 877


0610/42 Cambridge IGCSE – Mark Scheme May/June 2021
PUBLISHED
Question Answer Marks Guidance

1(d)(i) any two from: 2


hairs in the nose ;
skin ;
AVP ;

1(d)(ii) phagocytes ; 1

1(d)(iii) (different) pathogens / antigens, have different / unique shapes ; 3


pathogens have antigens ;
lymphocytes, produce (specific) antibodies ;
antibodies are, complementary / specific to, antigens ;
antibodies, mark / destroy, pathogens ;
ref. to memory cells ;

1(e) arthropods ; 1

Question Answer Marks Guidance

2(a) (dietary) fibre / carbohydrate / starch / (named) sugar / vitamin C ; 1

2(b)(i) external ears / pinnae ; 2


fur ;

© UCLES 2021 Page 6 of 12

Page 678 of 877


0610/42 Cambridge IGCSE – Mark Scheme May/June 2021
PUBLISHED
Question Answer Marks Guidance

2(b)(ii) any five from: 5

for (giant panda closer to red panda):


same diet / herbivores / AW ;
terrestrial / similar, ecosystems / habitats ;
fur markings described ;
ear position ;

against:
shorter (branch) distance between giant panda and polar bear (than to
red panda) ;
fewer, DNA (base sequence) / gene, differences between giant panda
and polar bear (than to red panda) ;
(red panda) first appeared, 40±3 million rather than 20±3 million, years
ago (giant and polar) ;
more time for, mutations / evolution (for red panda) ;
common, ancestor / branch / relationship, 20±3 million, (giant and
polar), than 47±3 million years ago (red) ;
DNA data is likely to be more, accurate / quantitative / not subjective ;

2(b)(iii) anatomy / ability to produce fertile offspring / AVP ; 1

Question Answer Marks Guidance

3(a)(i) 1 Accept
E A B D C ;
A E B D C

3(a)(ii) FSH ; 1

3(a)(iii) any two from: 2


(menstruating females need more) iron ; ORA
(iron is needed for) haemoglobin / red blood cells / transport of oxygen
/ to prevent (iron-deficiency) anaemia ;

© UCLES 2021 Page 7 of 12

Page 679 of 877


0610/42 Cambridge IGCSE – Mark Scheme May/June 2021
PUBLISHED
Question Answer Marks Guidance

3(b) 1 sperm / male gametes are, deposited in, S / vagina OR travel 6


through, R / cervix ;
2 egg / sperm / gametes, travel, to P / oviduct ;
3 enzymes (from acrosome / sperm) digest jelly coat (around egg) ;
4 fertilisation OR fusion of nuclei, of sperm / male gamete, and of
female gamete / egg (cell) ;
5 formation of diploid zygote ;
6 zygote / embryo, travels, down P / oviduct OR zygote / embryo,
travels to, T / uterus ;
7 mitosis / cell division / growth / develop (zygote / embryo / fetus) ;
8 ball of cells / embryo, implants in, T / uterus ;
9 U / placenta / V / umbilical cord (between mother, and fetus) ;
10 delivery / diffusion, of (named) resource / excretion of (named)
waste, via U / placenta / V / umbilical cord ;
11 AVP ; e.g.
• named sperm adaptation for swimming e.g. mitochondria
• Q / ovary / U / placenta, releases progesterone (to maintain
lining)
• ref to (cell) specialization / complexity, of fetus / (named)
organs (in fetus)

Question Answer Marks Guidance

4(a)(i) habitat destruction / named example ; 3


removing (too many) of a plant species ;
(named) pollutants (of plants) ;
additional named pollutant (of plant) ;
introduced species / disrupted food chains ;
disease ;
AVP ; e.g. extinction of other species / fewer, pollinators / (named)
pesticides killing pollinators / reduced genetic diversity / stated impact
of GM crops / war

© UCLES 2021 Page 8 of 12

Page 680 of 877


0610/42 Cambridge IGCSE – Mark Scheme May/June 2021
PUBLISHED
Question Answer Marks Guidance

4(a)(ii) any four from: 4


more carbon dioxide (is being released / in the atmosphere) ;
(carbon dioxide is produced by) burning / use of, fossil fuels ;
deforestation (leads to more carbon dioxide in atmosphere) ;
more methane (is being released / in the atmosphere) ;
(methane is produced by) farming ;
(methane produced by) landfill ;
increased / increasing human population ;
enhanced greenhouse effect ;
(greenhouse / named) gases trap, heat / (infra-red) radiation ;
AVP ;

4(b)(i) any three from: 3


ensure (genetic) variation / AW ;
represent many environments where the plant grows / able to survive
(in a changing environment) / AW ;
more chance of cross-fertilisation ;
not enough seeds in any one population ;
increases chances of successful germination ;
AVP ;

4(b)(ii) mutations (of DNA) / genetic change / change (base sequence of) 1
DNA ;

4(b)(iii) any two from: 2


low (kinetic) energy ;
for enzymes ;
low rate of (effective) collisions / AW ;
slow diffusion (of oxygen) ;

4(c)(i) any three from: 3


water ;
oxygen ;
suitable / optimum, temperature ;
AVP ;

© UCLES 2021 Page 9 of 12

Page 681 of 877


0610/42 Cambridge IGCSE – Mark Scheme May/June 2021
PUBLISHED
Question Answer Marks Guidance

4(c)(ii) obtain (named) resources from, larger / deeper, area ; 4


so that they have enough roots to survive (the reintroduction process) ;
ref. to large(r) surface area ;
more root hair (cells) ;
for (more) water and (named) minerals / ions / salts, uptake ;
anchorage / described ;
so that not pulled out (when animals grazing / wind etc) ;
reduced soil erosion ;
AVP ; e.g. named use of, water / mineral ion

4(c)(iii) lag ; 1 A death (phase)

Question Answer Marks Guidance

5(a) helix ; 3
one / the same / equal / identical / maintained / constant ;
identical ;

5(b)(i) 0.003 (mm) ;; 2 MP1 correct reading from graph i.e. 3 (μm)
MP2 correct conversion, ecf from wrong readings
in MP1

5(b)(ii) any two from: 2 units must be stated at least once


there are some (before division) cells that are less than 6 μm ;
the smallest that divide are 4.5±1 μm in length ;
some cells are less than 3 μm after division (assuming equal division) ;

5(c)(i) any two from: 2


(cut using) restriction enzyme ;
to form sticky ends ;
plasmid cut with same enzyme (as new gene) ;
to form complementary (sticky) ends ;

© UCLES 2021 Page 10 of 12

Page 682 of 877


0610/42 Cambridge IGCSE – Mark Scheme May/June 2021
PUBLISHED
Question Answer Marks Guidance

5(c)(ii) any two from: 2


rapid rate of reproduction ;
no ethical considerations ;
share same genetic code as other organisms ;
ability to make complex molecules ;
AVP ; do not take up much space / cheap to maintain cultures

Question Answer Marks Guidance

6(a)(i) prediction: 3 ORA throughout


set, A / in bag, will lose less, mass (than set B) ;

explanation:
because high(er) humidity (in A) ;
less steep diffusion gradient / AW (in A) ;
less transpiration (in A) ;

6(a)(ii) any three from: 3


water evaporates ;
from (surface of) mesophyll / into air spaces ;
water vapour, diffuses / described ;
through stomata (out of leaf) / AW ;

6(a)(iii) balance / scale / AW ; 2


stop-clock / timer / AW ;

© UCLES 2021 Page 11 of 12

Page 683 of 877


0610/42 Cambridge IGCSE – Mark Scheme May/June 2021
PUBLISHED
Question Answer Marks Guidance

6(b) 4 max 2 for diagram


4 correct labels on the diagram = 2 marks
2 or 3 correct labels = 1 mark
1 correct label = 0 marks

;;

letter description name of tissue

P a protective transparent layer that upper epidermis max 2 for table


allows light to reach inner tissues 4 correctly named tissues = 2 marks
2 or 3 correct tissues = 1 mark
Q conducts water from the stem xylem 1 correct tissue = 0 marks

R contains many interconnected air spongy mesophyll


spaces

S transports sucrose and amino acids phloem

T traps the most light energy to palisade mesophyll


synthesise carbohydrates
;;

© UCLES 2021 Page 12 of 12

Page 684 of 877


0610/43 Cambridge IGCSE – Mark Scheme May/June 2021
PUBLISHED
Question Answer Marks Guidance

1(a)(i) water ; 1

1(a)(ii) 4 elements for protein and fat for 1 mark


products of
enzyme and products for protein for 1 mark
nutrient elements enzyme digestion / products
enzyme and products for fat for 1 mark
absorbed into blood
enzyme for lactose for 1 mark
C, H, O, N protease /
protein (S) AND pepsin / amino acids ;
trypsin

fat C, H, O ; lipase fatty acids AND glycerol ;

lactose
galactose and glucose (simple
(milk C, H, O lactase ;
sugars)
sugar)

1(b)(i) any three from: 3


needed for (growth / development / strengthening of) bones ;
needed for (growth / development / strengthening of) teeth ;
prevents rickets ;
AVP ;

1(b)(ii) any two from: 2


required to make haemoglobin / red blood cells ;
for transport of oxygen ;
prevents (iron-deficiency) anaemia ;
AVP ;

1(c)(i) disease is caused by a pathogen ; 2


(pathogen is) passed from one host to another ;

© UCLES 2021 Page 5 of 12

Page 685 of 877


0610/43 Cambridge IGCSE – Mark Scheme May/June 2021
PUBLISHED
Question Answer Marks Guidance

1(c)(ii) any four from: 4


rubella is caused by a virus ;
(virus) can cross the placenta ;
(virus) can affect embryo / fetus ;
can cause serious, disease / damage / death, to baby / offspring ;
vaccination stimulates, long-lasting / active, immunity (in the mother) ;
(immunity gained) before they become pregnant ;
AVP ;

1(c)(iii) not enough protein (in the diet) ; 2


adequate energy (in the diet) ;
inadequate diet after breast feeding finishes / AW ;

Question Answer Marks Guidance

2(a)(i) mRNA / messenger RNA ; 1

2(a)(ii) ribosome ; 1

2(a)(iii) any three from: 3


molecule A / mRNA, is a copy of the gene (for insulin) ;
transfers instructions from, DNA / nucleus, to, cytoplasm / ribosome / B ;
passes through B ;
sequence of bases determines the order of amino acids ;

2(b) insulin is a hormone ; 4


stimulates liver (cells and muscle cells) ;
to, absorb / take up, glucose / sugar, from the blood ;
across cell membranes ;
stimulates production of glycogen (in cells) / AW ;
reduces concentration of, glucose / sugar, in the blood ;
homeostasis ;
AVP ;

© UCLES 2021 Page 6 of 12

Page 686 of 877


0610/43 Cambridge IGCSE – Mark Scheme May/June 2021
PUBLISHED
Question Answer Marks Guidance

2(c) any two from: 2


glucagon / hormone ;
amylase ;
trypsin / protease ;
lipase ;
AVP ;;

Question Answer Marks Guidance

3(a)(i) any four from: 4


(photosynthesis involves) enzymes / AW ;
increase in temperature increases rate (of photosynthesis) / ora ;
increases kinetic energy of molecules ;
increases effective collisions between enzyme and substrate
molecules / ora ;
reaction will have an optimum temperature ;
high temperature denatures enzymes ;
increases rate of diffusion ;
increased uptake of carbon dioxide molecules into leaf ;

3(a)(ii) any three from: 3


air becomes very dry ;
ref. to increasing the diffusion gradient for water vapour ;
from the air spaces in the leaf into the air surrounding the leaf ;
increases rate of transpiration ;
stomata close ;
carbon dioxide cannot enter ;

3(a)(iii) record the difference (in concentration of carbon dioxide) (between air in 2
and air out / between chamber 1 and 3 / before and after) ;
divide by, the time / 15 min ;

3(b)(i) CO2 uptake: 136 ; 2


CO2 release: 20 ;

© UCLES 2021 Page 7 of 12

Page 687 of 877


0610/43 Cambridge IGCSE – Mark Scheme May/June 2021
PUBLISHED
Question Answer Marks Guidance

3(b)(ii) any two from: 2


carbon dioxide produced in respiration ;
plants respire all the time ;
carbon dioxide released in the dark is produced by respiration ;
carbon dioxide released by cells is used in photosynthesis ;

3(b)(iii) any one from: 1


leaves have different surface areas ;
so, valid / AW, comparisons can be made ;

© UCLES 2021 Page 8 of 12

Page 688 of 877


0610/43 Cambridge IGCSE – Mark Scheme May/June 2021
PUBLISHED
Question Answer Marks Guidance

3(c) any six from: 6


1 glasshouses can maintain, optimum / AW, conditions for
photosynthesis ;
2 ref. to limiting factors for, photosynthesis / growth ;
3 to increase yield / continuous production / production independent of
seasons ;

ref. to control of light


4 (can, achieve / maintain) optimum, light / light intensity / wavelength ;
5 light duration / use artificial lights (and timers / light sensors) ;

ref. to control of temperature


6 (can, achieve / maintain) optimum temperature ;
7 heater / cooling / idea of ventilation ;

ref. to control of carbon dioxide


8 (can use) carbon dioxide enrichment / high carbon dioxide
concentration ;
9 burn (named) fuels / pump in carbon dioxide ;

ref. to control of water and nutrients


10 water supply / humidity ;
11 nutrients / minerals ;
12 irrigation / fertigation / misting / hydroponics / described ;

ref. to control of pests and weeds


13 pest control ;
14 any type described ; e.g. biological control / chemical control
15 herbicides not required ;

additional points
16 computer control / automation (described) ;
17 efficient use of land / use brownfield sites / local food near markets ;
18 reduction in pollution by, fertilisers / pesticides ;
19 control of pollinators for selective breeding ;

© UCLES 2021 Page 9 of 12

Page 689 of 877


0610/43 Cambridge IGCSE – Mark Scheme May/June 2021
PUBLISHED
Question Answer Marks Guidance

4(a) any five from: 5

1 habitat loss / deforestation ;


2 large mammals need large amounts of space ;
3 (named example of) climate change ; e.g. desertification

use of land for


4 land developed for agriculture ;
5 land developed for, housing / transport / factories / AW ;
6 land used for extraction of (named) material(s) ;
7 competing with humans for space ;

8 idea that populations are isolated ;


9 hunt / trophies / high value / poaching ;
10 laws are not enforced to protect from hunting ;
11 lack of education ;
12 poisoning / pollution ;
13 need a large quantity of food / lack of food sources / starvation ;
14 slow reproduction rate / hard to breed in captivity ;
15 AVP ;

4(b)(i) any two from: 2


inbreeding / AW ;
reduce (genetic) variation ;
(reduced variation) less able to respond to, environmental
changes / disease ;
increase in homozygosity / described in terms of alleles ;
increase chance of genetic disease(s) / birth defects ;
any example ; e.g. lameness
increase chance of infertility ;

© UCLES 2021 Page 10 of 12

Page 690 of 877


0610/43 Cambridge IGCSE – Mark Scheme May/June 2021
PUBLISHED
Question Answer Marks Guidance

4(b)(ii) any three from: 3


1 collect semen / sperm (implied by context) ;
2 freeze semen / sperm, / use sperm bank / use sperm collected, far
away/ long ago ;
3 selecting / washing / screening, of sperm / AW ;
4 use hormones for (super)ovulation ;
5 determine when female is about to ovulate ;
6 insert, semen / sperm, into female (vagina / uterus) ;

5(a) diaphragm / external intercostal muscles ; 4


trachea / bronchus ;
internal intercostal muscles ;
alveolus / alveoli ;

5(b) any four from: 4


1 (rate and depth of breathing) increase when carbon dioxide
concentration increases ;
2 depth of breathing levels off at 9–10 minutes while rate of breathing
still going up ;
3 larger increase in rate of breathing than the depth of breathing ;
4 both rate and depth of breathing decrease when the carbon dioxide
concentration decreases ;
5 rate and depth of breathing decrease below zero (percentage
change) ;
6 rate of breathing does not / depth of breathing starts to, return to zero
(percentage change);
7 use of comparative figures for rate of breathing ;
8 use of comparative figures for depth of breathing ;

5(c) respiration ; 5
mitochondria ;
brain ;
excreted / AW ;
toxic / poisonous / waste / acid(ic) ;

© UCLES 2021 Page 11 of 12

Page 691 of 877


0610/43 Cambridge IGCSE – Mark Scheme May/June 2021
PUBLISHED
Question Answer Marks Guidance

6(a) 4 one mark for four correct gametes in Punnett


IA IO square
one mark for four correct offspring genotypes
IAIB
IB IB IO in Punnett square

;;
IO IAIO IOIO

phenotypes of the children: AB, B, A, O ;


probability…blood group O: 0.25 / 25% / 1 in 4 ;

6(b) both / two, alleles are expressed in, heterozygote / phenotype / AW ; 2


ref. to, blood group AB / IAIB ;

6c) any two from: 2


phenotype is determined only by a gene ;
no effect of environment on phenotype ;
only, four / limited number of, phenotypes / (blood) groups / categories ;
no intermediates between categories ;

© UCLES 2021 Page 12 of 12

Page 692 of 877


0610/41 Cambridge IGCSE – Mark Scheme October/November 2021
PUBLISHED
Question Answer Marks Guidance

1(a)(i) reptiles and amphibians ; 1

1(a)(ii) one mark per row: 3


feature fish mammals birds

organ involved in gas


gills lungs lungs ;
exchange

circulatory system single double double ;

body covering scales fur feathers ;

presence of external ears


no yes no ;
(pinnae)

1(b)(i) (net) movement of particles, from a region of their higher 2


concentration to a region of their lower concentration / down a
concentration gradient ;
as a result of their random movement ;

1(b)(ii) any one from: 1


large surface (area) / AW ;
thin ;

1(c)(i) nitrate (ions) / sewage / fertilisers / AVP ; 1

1(c)(ii) 15 °C – 8.9 ± 0.1 (mg per dm3) and 25 °C – 7.3 ± 0.1 (mg per dm3) ; 3

2.35 ± 0.05 (μm) and 1.95 ± 0.05 (μm) ; ecf from readings in MP1 for MP2 and MP3

(–)0.4 (in μm) ;

© UCLES 2021 Page 5 of 12

Page 693 of 877


0610/41 Cambridge IGCSE – Mark Scheme October/November 2021
PUBLISHED
Question Answer Marks Guidance

2(a) selection of suitable letter and case for alleles, e.g. R and r ; 6

parental phenotypes: red fruit x red fruit

parental genotypes: Rr ; x Rr ;

gametes: R r x R r;

offspring genotypes: (1)RR and (2)Rr and (1)rr ;

expected ratio: 3 (red fruit) : 1 (yellow fruit) ;

2(b) (perform a) test cross ; 1

2(c)(i) any two from: 2


ref. to photosynthesis ;
transfers light energy into chemical energy ;
to make, carbohydrates / glucose / starch / AW ;

2(c)(ii) magnesium ; 1

2(c)(iii) any three from: 3


1 chlorophyll concentration decreases in both ;
2 lycopene concentration increases in both ;
3 (lycopene) increases from zero (in unripe fruit) ;
4 ref to lycopene change being much larger than chlorophyll
change / AW ;
5 comparative data quote to support observations (with units
used at least once) ;
6 AVP ;

2(d)(i) DNA ; 1

© UCLES 2021 Page 6 of 12

Page 694 of 877


0610/41 Cambridge IGCSE – Mark Scheme October/November 2021
PUBLISHED
Question Answer Marks Guidance

2(d)(ii) any two from: 2


cross-breeding with wild populations of plants ;
expensive ;
(named) unknown long-term effects (on the
environment / populations) ;
ethical considerations with manipulating ‘nature’ ;
AVP ;

Question Answer Marks Guidance

3(a)(i) any three from: 3


protease / trypsin / pepsin ;
chemical) digestion ;
(protein) digested to amino acids ;
insoluble to soluble molecules ;

3(a)(ii) any four from: 4


enzymes have, a specific shape / complementary shape to
substrate ;
correct ref. to active site ;
30°C is optimum temperature ;
above 50 °C (these) enzymes denature ;
ref. to active site changing (shape) ;
(below 30 °C,) less kinetic energy / lower frequency of effective
collisions (between substrate and enzyme) ; ora

© UCLES 2021 Page 7 of 12

Page 695 of 877


0610/41 Cambridge IGCSE – Mark Scheme October/November 2021
PUBLISHED
Question Answer Marks Guidance

3(b) any four from: 4


emulsification ;
increased surface area of fat (globules) ;
faster, digestion / breakdown (of fat) ;
by lipase / to fatty acids and glycerol ;
neutralises, (stomach) acid / chyme / gastric juice ;
alters / increases, pH for (pancreatic / intestinal) enzymes
/ AW ;
denatures, pepsin / stomach, enzymes ;
AVP ;

Question Answer Marks Guidance

4(a) bacteria ; 1

4(b)(i) any one from: 1


weakened / dead / AW, bacteria / pathogen ;
antigen(s) (of the pathogen) ;
AVP ;

4(b)(ii) any three from: 3


idea that the immune response takes time to occur ;
lymphocytes release antibodies ;
ref. to (lymphocytes) produce specific antibodies to the (cholera)
antigens / AW ;
memory cells (form) ;
long-term immunity ;

4(b)(iii) idea that they did not have (active) immunity / memory cells before 1
the start of the study / AW ;

© UCLES 2021 Page 8 of 12

Page 696 of 877


0610/41 Cambridge IGCSE – Mark Scheme October/November 2021
PUBLISHED
Question Answer Marks Guidance

4(b)(iv) any two from: 2


more, pathogens / diarrhoea / fever, in non-vaccinated group ; ora
some vaccinated people did get symptoms so vaccine not 100%
effective ; ora
comparative data quote between vaccinated and not vaccinated ;
AVP ;

4(c)(i) any four from: 4


cholera / pathogen, releases toxin ;
(toxin) causes (more) chloride released (into small intestine) ;
lowering water potential (in lumen) ;
ref. to, osmosis / movement of water (into the lumen) ;
(diarrhoea is) loss of watery faeces ;
loss of salts / loss of minerals / dehydration ;

4(c)(ii) any two from: 2


oral rehydration therapy ;
drink mixture of, sugar / nutrients and, salt / ions ;
replace lost, water / fluids ;
AVP ;

Question Answer Marks Guidance

5(a) any four from: 4


valves in, heart / (main) veins ;
correct sequence, of open / closing, of valves ;
prevention of backflow / description ;
semi-lunar valves ;
atrioventricular valves ;
heart, pumping / contracting ;
AVP ;

© UCLES 2021 Page 9 of 12

Page 697 of 877


0610/41 Cambridge IGCSE – Mark Scheme October/November 2021
PUBLISHED
Question Answer Marks Guidance

5(b) one mark per row: 6

type of letter on
function
blood vessel Fig. 5.1

regulates blood flow by arteriole P ;


constricting and dilating / artery N

collects blood from a network ;


venule K
of the narrowest blood vessels

withstands the highest blood artery ;


N
pressure

allows the transfer of ;


substances to and from tissue capillary J
fluid

transports blood towards the vein ;


M
heart / venule
K

redirects blood under the ;


shunt vessel L
surface of the skin

5(c) renal artery ; 2


coronary artery ;

© UCLES 2021 Page 10 of 12

Page 698 of 877


0610/41 Cambridge IGCSE – Mark Scheme October/November 2021
PUBLISHED
Question Answer Marks Guidance

5(d) any four from: 4 A blood vessel for artery throughout


stent ;
(small) mesh / gauze, tube inserted in artery ;
opens / supports, (narrow / weak) artery ;
(balloon) angioplasty / dilatation ;
(tube / catheter with) balloon inserted into artery ;
inflate balloon to widen artery ;
by-pass ;
(another / shunt) blood vessel, joined to / grafted to / replaces,
artery ;
AVP ; e.g. aspirin / warfarin / ref to treatment of clots

Question Answer Marks Guidance

6(a) any three from: 3


loss of biodiversity / AW ;
habitat destruction ;
damage to, food chains / food webs ;
soil erosion ;
by, water / wind ;
infertility of soil ;
competition for resources ;
ref to pollution ;
AVP ;; e.g. desertification / flooding / diseases or pests spreading to
wild-varieties / migration (of species)

6(b) any two from: 2


more / number / amount (of), fruits / seeds / kernels ;
size of, fruits / seeds / kernels ;
ref. to colour (of, fruits / seeds / kernels / cobs) ;
length / size / width, of cobs ;
ref. to arrangement of, fruits / seeds / kernels ;

6(c)(i) to prevent (natural) pollination / AW ; 1

© UCLES 2021 Page 11 of 12

Page 699 of 877


0610/41 Cambridge IGCSE – Mark Scheme October/November 2021
PUBLISHED
Question Answer Marks Guidance

6(c)(ii) any two from: 2


aerobic respiration ;
for energy ;
for growth ;

6(c)(iii) the plants may be heterozygous ; 1


recessive traits only visible if plants are homozygous ;

6(c)(iv) any two from: 2


reduce variation (in a population) / create uniformity ;
become more homozygous ;
to, improve / maintain, the desirable feature ;
produce more seeds ;
AVP ; e.g. ensure trait continues on subsequent generations / to
ensure no other alleles are present / does not result in unforeseen
effects

6(c)(v) mutation(s) ; 1

© UCLES 2021 Page 12 of 12

Page 700 of 877


0610/42 Cambridge IGCSE – Mark Scheme October/November 2021
PUBLISHED
Question Answer Marks Guidance

1(a) duodenum ; 1

1(b)(i) any three from: 3


ref. to membrane for active transport ;
uses (chemical) energy ;
energy from, the cell / respiration / mitochondria ;
(substances move) against concentration gradient / low to high concentration ;
using / AW, proteins / carriers (in membranes) ;

1(b)(ii) idea of more (cell) membrane / large surface (area) ; 2


allow, increased rate of / fast, absorption (of nutrients) ;

1(c)(i) mucus ; 1

1(c)(ii) any two from: 2

(to protect the intestine from)


bacteria / viruses / microorganisms / parasites / pathogens ;
(named) toxin(s) ;
(named) enzymes / prevents self digestion ;
(hydrochloric) acid / alkali / base / extreme pH ;
physical damage by food passing through intestine / AW ;

1(d) absorbs, fats / fatty acids (and glycerol) ; 2


transports, fats / fatty acids, into, lymph(atic) vessels / lymph(atic)
system / lymph ;

1(e) organ ; 4
cell structure ;
tissue ;
organ ;

© UCLES 2021 Page 5 of 13

Page 701 of 877


0610/42 Cambridge IGCSE – Mark Scheme October/November 2021
PUBLISHED
Question Answer Marks Guidance

1(f)(i) conclusions 5
1 fungal extract contains amylase and pectinase ;
2 clear zone is where substrate has been, digested / broken down ;
3 extract has a higher concentration than 1% of amylase ; MP3 ignore more effective
4 extract has lower concentration than 1% of pectinase ; If awarding MP3 and MP4, then award
5 extract has, higher concentration of / more, amylase than pectinase ; MP1 as well; if awarding MP5 also
award MP1
evidence - can refer to what is in the wells (A to D)
6 D / fungal extract, in both Petri dishes has clear zones (MP1) ;
7 no staining in clear zones shows that enzyme was active (MP2) ;
8 size of clear zone, D / with fungal extract, is bigger than A / amylase
solution (MP3) ; MP8 A more digested
9 size of clear zone, D / with fungal extract, is smaller than B / pectinase
solution (MP4) ; MP9 A less digested
10 clear zone in, D / with fungal extract, is larger in the plate containing starch
than in the plate containing pectin (MP5) ;

11 enzymes, diffuse / move, through the agar ;


12 greater the clear area the greater, the activity / concentration, of the
enzyme ;
13 no clearance / no breakdown / no change, with water ;

1(f)(ii) (same pattern but) smaller / no, clear zones ; 1

© UCLES 2021 Page 6 of 13

Page 702 of 877


0610/42 Cambridge IGCSE – Mark Scheme October/November 2021
PUBLISHED
Question Answer Marks Guidance

2(a)(i) (thread-like structure) made / length / strand / composed / molecule, of DNA ; 2


carries / contains, genes / alleles / genetic information or material ;

2(a)(ii) prevent pollination (from another flower or plant) / to prevent (other) pollen 1
landing on stigma ;

2(a)(iii) any two from: 2


reference to meiosis ;
gametes contain different alleles ;
idea of new / different, combinations of, maternal and paternal
chromosomes / alleles ;
idea of random fertilisation / fusion of gametes with different, genotypes or
alleles ;
AVP ;

2(a)(iv) either: 2
1 meiosis halves the number of chromosomes A diploid to haploid in meiosis
or
gametes are haploid A sperm and eggs / ova / pollen
or
gametes, have one of each type of chromosome / have one set of
chromosomes ;
and
2 idea that parental / diploid, number is restored at fertilisation A haploid gametes fuse to form diploid,
or zygote / offspring = 2 marks
cells have 23 chromosomes and fuse together form a cell with 23 pairs ;

© UCLES 2021 Page 7 of 13

Page 703 of 877


0610/42 Cambridge IGCSE – Mark Scheme October/November 2021
PUBLISHED
Question Answer Marks Guidance

2(b)(i) PRPR ; 3
PRPW ;
PRPR, PRPW, PWPW ;

2(b)(ii) codominance ; 1

Question Answer Marks Guidance

3(a)(i) ovary ; 1

3(a)(ii) plasma / blood (vessels) ; 1

3(a)(iii) any two from: 2


1 both hormones, follow the same pattern / increase and decrease ;
2 both hormones reach a peak at, day 14 / the same time / the same day ;
3 LH starts increasing before FSH ; ora
4 LH has a higher concentration than FSH at, the peak / day 14 ;
5 LH concentrations are lower than FSH before the peak ; ora
6 LH concentrations are higher than FSH after the peak; ora
7 comparative data quote between concentrations of LH and FSH ;

3(b) any four from: 4


1 FSH stimulates, development / growth / maturation, of follicles (in ovary) ;
2 FSH stimulates, development / growth / maturation, of, egg
(cell) / ova / ovum / (female) gamete(s) ;
3 FSH stimulates, production / release / secretion, of oestrogen (from ovary) ;
4 (FSH and) LH (surge) stimulates, ovulation / release of egg from ovary ;
5 LH stimulates (corpus luteum / yellow body / empty follicle),
secretion / production / release, of progesterone ;
6 decrease in LH decreases secretion of progesterone (by corpus luteum) ;
7 AVP ;

© UCLES 2021 Page 8 of 13

Page 704 of 877


0610/42 Cambridge IGCSE – Mark Scheme October/November 2021
PUBLISHED
Question Answer Marks Guidance

3(c) any three from: 3


1 loss / thinning, of lining (of uterus), at the beginning of the cycle / during first
week / between day 1–7 ;
2 regrowth / thickening of, lining (of uterus), during second week / after loss
(of lining) / before ovulation ;
3 thickness of lining remains constant, in the last two weeks / at the end of
the cycle / after ovulation / if no fertilisation ;
4 ref. to, blood vessels / glands ;

3(d) any three from: 3


1 (contraceptives contain) progesterone (with oestrogen) ;
2 decreases secretion / prevents release, of FSH / LH ;
3 prevents, development / maturation of, follicles / eggs / ova ;
4 prevents ovulation / described ;
5 AVP ; e.g. changes / thickens, cervical mucus to prevent entry of sperm
OR
prevents, regrowth / thickening, of uterus lining (reducing chance of
implantation)

© UCLES 2021 Page 9 of 13

Page 705 of 877


0610/42 Cambridge IGCSE – Mark Scheme October/November 2021
PUBLISHED
Question Answer Marks Guidance

4(a) one mark for each row: 5

letter
function name on Fig.
4.1
A nerve cell / nerve throughout
conducts impulses to
sensory / afferent, neurone V ;
CNS
A interneurone
conducts impulses to motor / effector / efferent,
Y ; A receive
an effector neurone
A change in,
conducts impulses relay / connector environment / surroundings
X ; A heat / hot object
within the CNS / intermediate, neurone

detects / senses, A carry out an action


stimulus / change in receptor U ; ignore triceps
temperature

contracts / causes
biceps / muscle /
movement / carry out Z ;
effector
response

4(b) any four from: 4


1 vesicles move towards end of, pre-synaptic neurone / axon / AW ;
2 vesicles fuse with (pre-synaptic) membrane ;
3 (named) neurotransmitter released ;
4 neurotransmitter diffuses across, synapse / gap / synaptic cleft ;
5 neurotransmitter binds to receptor, on post-synaptic neurone ;
6 impulse (generated) in post-synaptic neurone ;
7 AVP ;

4(c) blinking / pupil reflex / iris reflex / accommodation / corneal reflex / tear 1
reflex / AVP ;

© UCLES 2021 Page 10 of 13

Page 706 of 877


0610/42 Cambridge IGCSE – Mark Scheme October/November 2021
PUBLISHED
Question Answer Marks Guidance

5(a) any two from: 2


water / fluid / liquid, intake ;
exercise / activity level ;
sweating ;
temperature ;
salt / salty food, intake ;
quantity of urea produced / concentration of urea in blood ;
(named) drug ;
(named) medical condition ;
AVP ; e.g. water potential of blood / AW

5(b) any five from: 5


1 protein, broken down / digested / decomposed, to amino acids ;
2 ammonification / mineralisation ;
3 amino acids converted to, ammonium (ions) / NH4+ ;
4 by, decomposers / fungi / bacteria ; if linked to MP1, MP2 or MP3
5 ammonium (ions) / NH4+, converted / oxidised to, nitrate (ions) / NO3– ; MP5 A nitrite converted to nitrate if
ammonium not given
6 nitrification / nitrifying bacteria ;
7 plants, absorb / use, nitrate (ions) (and ammonium ions) ;
8 AVP ; e.g. deamination of amino acids

© UCLES 2021 Page 11 of 13

Page 707 of 877


0610/42 Cambridge IGCSE – Mark Scheme October/November 2021
PUBLISHED
Question Answer Marks Guidance

5(c) any six from: 6


1 eutrophication ;
2 increased growth of, plants / producers / algae ; MP2 A algal bloom
3 plants / producers, compete with each other for light / AW , MP3 A no photosynthesis as no light
4 plants / producers, die or plants / producers, are decomposed ;
5 increase in, decomposers / bacteria ;
6 bacteria / decomposers, use / absorb, (dissolved) oxygen in water ;
7 decrease in concentration of (dissolved) oxygen ;

reasons for reduction in biodiversity:


8 death of, (named) animals / fish / invertebrates (due to lack of dissolved MP8 A if linked to MP7
oxygen) ;
9 decrease in, food / energy, available in, ecosystem / AW ; MP9 A disruption of food, chains / webs
10 ref to, pathogens / disease / toxins / poisons ;

11 ref to aerobic respiration (in either section)


12 AVP ;

© UCLES 2021 Page 12 of 13

Page 708 of 877


0610/42 Cambridge IGCSE – Mark Scheme October/November 2021
PUBLISHED
Question Answer Marks Guidance

6(a) (substance) that increases the rate of (chemical) reactions ; 2


not changed, during / by, the reaction ;

6(b)(i) T; 1

6(b)(ii) (T) fits into / binds to, enzyme / active site ; 2 A T and enzyme form an, enzyme-
active site has a complementary shape (to T) / active site and T are substrate complex / ESC
complementary ;

6(c) glucose ; 6
pepsin / trypsin / protease ; A other named proteases
fatty acids and glycerol ;
lactase ;
(DNA) ligase ;
cuts / breaks / digests, DNA (molecule / strand(s)) / gene / plasmid ; A makes sticky ends

© UCLES 2021 Page 13 of 13

Page 709 of 877


0610/43 Cambridge IGCSE – Mark Scheme October/November 2021
PUBLISHED
Question Answer Marks Guidance

1(a) (made of) protein / biological ; 2


catalyst / described ;

1(b)(i) CTTAAG ; 1

1(b)(ii) restriction (enzyme) ; 1

1(b)(iii) any two from: 2


cuts (DNA, molecule / strands) ;
to form sticky ends / AW ;
the sticky ends are complementary / described ;

1(b)(iv) any two from: 2


DNA / bases sequence / substrate, and, enzyme / active site, are
complementary ;
ref. to active site ;
AVP ;

1(c)(i) bacteria ; 1

1(c)(ii) (DNA) ligase ; 1

1(c)(iii) recombinant (DNA / plasmid) ; 1

1(d) temperature (on x axis) and enzyme activity or rate of enzyme 3


reaction (on y axis) ;
plot showing one peak ;
gradient to the right of the peak is steeper than gradient to the left
of the peak (only consider gradients in the top half of the ‘peak’) ;

© UCLES 2021 Page 5 of 12

Page 710 of 877


0610/43 Cambridge IGCSE – Mark Scheme October/November 2021
PUBLISHED
Question Answer Marks Guidance

2(a)(i) any five from: 5


1 (nitrate ion) uptake peaks / uptake increases and then
decreases ;
2 maximum uptake / peak, is at 27 mm (from root tip) / of 28 pmol
per cm2 per s ;
3 no data before 2.75 mm / no (nitrate ion) uptake / uptake starts,
at 2.75 mm (from tip) ;
4 (nitrate ion) uptake levels off at about, 5 / 10 mm to 15 mm
(from tip) ;
5 comparative data quote ;
6 no root hair (cells), at tip / root cap / below 10 mm ;
7 between 25 ± 5 mm to 30 ± 5 mm (from root tip) is where many
root hair (cells) are found ;

2(a)(ii) any four from: 4


1 active transport ;
2 using energy (from respiration) ;
3 movement of (ions), across a cell membrane / into root hair
(cells) ;
4 from low concentration to high concentration / against a
concentration gradient (of nitrate ions) ;
5 through protein (molecules / pumps / carriers / channels) ;
6 nitrate ions are dissolved in water (before they move into
root) ;

2(a)(iii) lowers the water potential (in roots) / maintains the water potential 1
gradient (into the roots) ;

2(a)(iv) (nitrates) used to make, amino acids / proteins ; 3 MP1 A protein synthesis
proteins are composed of amino acids ;
named use of protein in plants (except water uptake or protein MP3 any named cell component or cell structure that
synthesis) ; contains protein or process that uses proteins in
plants, e.g. enzymes

© UCLES 2021 Page 6 of 12

Page 711 of 877


0610/43 Cambridge IGCSE – Mark Scheme October/November 2021
PUBLISHED
Question Answer Marks Guidance

2(a)(v) any three from 3


1 proteins / amino acids / urea / organic material / AW, are
decomposed ;
2 ammonium (ions) → (nitrite) → nitrate (by bacteria) ;
3 ref. to nitrification ;
4 nitrogen (gas) → ammonium (ions) / nitrate (by bacteria /
lightning) ; ‘nitrifying bacteria’ = MP3 and MP6
5 ref. to nitrogen fixation ; ‘nitrogen fixing bacteria’ = MP5 and MP6
6 any ref. to (nitrifying / nitrogen fixing) bacteria / lightning / root ‘decomposers breakdown dead organisms’ = MP1
nodules / fungi / decomposers ; and MP6

2(b)(i) FDCEAB ;; 2 MP1 for F at the beginning and B at the end of the
sequence
MP2 remaining letters in the correct position

2(b)(ii) eutrophication ; 1

Question Answer Marks Guidance

3(a) 3 black (bodies) : 1 yellow (bodies) ; 1

3(b) parental phenotypes: black x yellow 5

parental genotypes: Bb ; x bb ;

gametes: B b x b (b) ;

offspring genotype: Bb and bb ;

offspring phenotype: 1 black : 1 yellow ;

3(c)(i) exoskeleton / open circulatory system / compound eyes / segmented 1


body ;

© UCLES 2021 Page 7 of 12

Page 712 of 877


0610/43 Cambridge IGCSE – Mark Scheme October/November 2021
PUBLISHED
Question Answer Marks Guidance

3(c)(ii) any one from: 1


number of legs or 10 legs or 5 pairs of legs / limbs (in crab) or 8
legs or 4 pairs of legs (and 2 claws) ;
number of antennae / two pairs of antennae ;
gills ;
AVP ;

3(d)(i) any three from: 3


natural selection ;
albino crabs / crabs with no pigment, are (better) adapted / AW ; MP2 A no advantage of having pigment (in cave /
(albino crabs / AW) more likely to survive ; dark) / favourable trait / ora
MP2 A no pigment, is an advantage / increases
fitness
reproduction of albino crabs will pass on, mutation / allele (to MP3 A struggle for survival in context
offspring / next generation) ;
change over time is evolution ; MP4 A albino crabs will pass on, mutation / allele, to
offspring

3(d)(ii) any two from: 2


(ionising) radiation ;
chemicals / mutagens / (named) carcinogens ;
viruses ;

© UCLES 2021 Page 8 of 12

Page 713 of 877


0610/43 Cambridge IGCSE – Mark Scheme October/November 2021
PUBLISHED
Question Answer Marks Guidance

4(a) (exchange of) contaminated / infected (named) / AW, body fluids ; 3


AND
any two examples of mechanisms of transmission ;;
using (contaminated) needles (ignore ref to drugs unqualified)
sexual contact (with infected people)
(contaminated) blood transfusion
(tissue / organ) transplants
blood to blood contact
child birth
breast-feeding (if mother is infected)

4(b) protein coat ; 1

4(c)(i) any three from: 3


produce antibodies ;
named function of antibodies ;
antibodies are specific to, antigen / pathogen ; A complementary to antigen
ref to active immunity / described ;
ref to memory cells ;

4(c)(ii) (HIV) destroys lymphocytes / decrease in lymphocyte count ; 1

© UCLES 2021 Page 9 of 12

Page 714 of 877


0610/43 Cambridge IGCSE – Mark Scheme October/November 2021
PUBLISHED
Question Answer Marks Guidance

4(d)(i) any four from: 4

vitamin C
helps with, cell / tissue, repair / healing ; A prevents, scurvy / bleeding gums
helps to reduce risk of (named) infection / AW ;
AVP ; e.g. formation of, collagen / hair / nails
anti-ageing of skin
improves (named) mineral ion absorption
required for protein synthesis
antioxidant

iron:
involved in the production of haemoglobin ;
to transport oxygen (in red blood cells) ;
to reduce fatigue / to prevent anaemia ;

4(d)(ii) 79(%) ;; 2 MP1 selection of correct data (4291 and 897)


MP2 correct calculation
A answers to any number of correctly rounded
decimal places, e.g. 79.1 (%)

4(d)(iii) any two from: 2


1 increase (in average number of) lymphocytes in the treatment MP1 A (supplements are associated with) increase in
group (per μg blood after 3 months) ; the number of lymphocytes
2 (but) no change (in lymphocytes) in control group ; MP2 A no increase (in the number of lymphocytes) in
the control group
3 cannot come to a conclusion ;
4 (because) more, information / data, is required ;
5 (because) sample size is too small ;
6 (because) starting numbers of lymphocytes in the two groups
is not similar ;

© UCLES 2021 Page 10 of 12

Page 715 of 877


0610/43 Cambridge IGCSE – Mark Scheme October/November 2021
PUBLISHED
Question Answer Marks Guidance

5(a) any six from: 6


1 idea that a sustainable resource is one which is produced as MP1 A birth rate of fish must be as least as many fish
rapidly as it is removed (from the environment) so that it does as are being removed
not run out / idea that sustainable development is providing for
the needs of an increasing human population without harming
the environment ;
2 idea that sustainable development requires management of
conflicting demands ;
3 monitoring of, fish populations / factors affecting fish survival ;
4 education / awareness (of people who fish / people who eat MP4 A campaigns
fish) ;
5 (legal) quotas / licenses / restricted catch (of weight / length) ; MP5 A regulation of what can be done with by-
catch / ban overfishing
6 laws / guidelines / AW, on fishing equipment ;
e.g. mesh / net, size / bottom trawlers
7 support, small scale / subsistence / AW, fishing ;
8 to ensure sustainability of fish stock / described ;
9 conservation / protected, areas / no-catch zones ;
10 limit the fishing season / nursery zones ;
11 fines / law enforcement (of illegal fishing) ;
12 to discourage poor practice ;
13 AVP ;

5(b) (female contraceptive) hormones / oestrogen / progesterone ; 2


lower percentage of male / presence of intersex fish MP2 A male fish have become feminised
(downstream / site 2) ; ora (downstream) / AW

5(c) ref. to sex / X and Y, chromosomes ; 1

© UCLES 2021 Page 11 of 12

Page 716 of 877


0610/43 Cambridge IGCSE – Mark Scheme October/November 2021
PUBLISHED
Question Answer Marks Guidance

6(a) (group of) tissues working together, to pump blood around the 1
body / AW / performing the (same) function ;

6(b) 1 circulatory ; 7 MP1 A cardiovascular / circulation


2 vena cava ; MP2 A veins
3 thinner ; MP3 A less
4 diffusion ;
5 left atrium ;
6 septum ;
7 coronary, artery / arteries ;

6(c) ECG / electrocardiogram / listening to valves close / AVP ; 1

© UCLES 2021 Page 12 of 12

Page 717 of 877


0610/42 Cambridge IGCSE – Mark Scheme March 2021
PUBLISHED
Question Answer Marks Guidance

1(a) 3 one mark for each correct row


name of gland letter in Fig. 1.1 hormone produced

adrenal C adrenaline

pancreas E insulin

testis D testosterone
;;;

1(b)(i) hormone A is oestrogen ; 3


stimulates repair / thickening, of uterus lining ;
inhibits FSH secretion ;
stimulates secretion of LH ;

1(b)(ii) 12 / 13 / 14 / 15 / 16 ; 1 A 12–16

1(b)(iii) days 0–6 ; 1

1(c)(i) 1 lowers / controls, birth rate ; 2


2 more choice over when to start a family ;
3 fewer abortions / unwanted pregnancies / fewer adoptions ;
4 more women in the workforce ;
5 increased women in education ;
6 increase in (named) STIs ;
7 ref. to religious views ;
8 AVP ;

1(c)(ii) (named example of) feminisation of aquatic organisms ; 3


reduction in fertility in humans / reduced sperm count in men ;
reduction, in fertility / sperm count, in fish / aquatic organisms ;
reduction in population in, fish / aquatic organisms ;
AVP ;

© UCLES 2021 Page 5 of 10

Page 718 of 877


0610/42 Cambridge IGCSE – Mark Scheme March 2021
PUBLISHED
Question Answer Marks Guidance

1(c)(iii) condom ; 2
femidom ;
diaphragm ;
AVP ;

Question Answer Marks Guidance

2(a)(i) (bacterial) plasmid / loop of DNA / circular chromosome ; 1

2(a)(ii) ribosomes / (cell) membrane / cytoplasm ; 1

2(b) fungus ; 2
protoctist ;

2(c)(i) 1 idea of involvement of (named) bacteria ; 6

2 idea that A is decomposition / decay / deamination ;


3 releasing ammonium (ions) (to the soil) ;

4 idea that A is (also) nitrification / ref to nitrifying ;


5 convert ammonium (ions) to, nitrite / nitrate, (ions) ;

6 B is nitrogen fixation / ref to nitrogen fixing ;


7 by lightning ;
8 in, root nodules / leguminous plants ;

9 idea that C is denitrification / ref to denitrifying ;


10 in anaerobic conditions (described) ;

2(c)(ii) active transport ; 1 A diffusion

3(a)(i) (17 × 0.04(s)) = 0.68 (s) OR (2.8 ÷ 4) = 0.7 OR (4 ÷ 2.8) = 1.43 (beats 2 A unrounded answers
per s) ;
(60 ÷ 0.68) = 88 (bpm) OR (60 ÷ 0.7) = 86 (bpm) OR (1.43 × 60) = 86
(bpm) ;

© UCLES 2021 Page 6 of 10

Page 719 of 877


0610/42 Cambridge IGCSE – Mark Scheme March 2021
PUBLISHED
Question Answer Marks Guidance

3(a)(ii) ECG during exercise: 4


1 increased, frequency of heart beats / pulse rate ;
2 exercising muscles require more energy ;
3 for muscle contraction ;
4 increase in respiration ;
5 faster blood flow (to muscles) ;
6 to supply more, oxygen / glucose ;
7 to remove more carbon dioxide ;

3(a)(iii) listening to the valves of the heart shutting / count pulse rate / AW ; 1

3(b)(i) lactic acid ; 2


muscles / blood ;

3(b)(ii) 1 breathing rate, is / remains, high ; 4


2 heart rate, is / remains, high ;
3 to provide oxygen ;
4 to breakdown / to remove, lactic acid ;
5 lactic acid removed by aerobic respiration ;
6 to carbon dioxide (and water) ;
7 lactic acid is broken down in the liver ;

© UCLES 2021 Page 7 of 10

Page 720 of 877


0610/42 Cambridge IGCSE – Mark Scheme March 2021
PUBLISHED
Question Answer Marks Guidance

4(a) structure made up of a group of tissues, working together to perform a 1


specific function(s) ;

4(b)(i) A – (upper) epidermis ; 1 R lower epidermis

4(b)(ii) C – vascular bundle ; 1 A xylem and phloem / vein

4(b)(iii) B is the palisade, mesophyll / tissue / layer / cells ; 3


cells are tightly packed / AW ;
ref. to many chloroplasts / lots of chlorophyll ;
(cells) positioned at the top of the leaf ;
(large vacuole) ensures chloroplasts are at the edge of cells ;
AVP ;

4(c)(i) sucrose and amino acids ; 1

4(c)(ii) (acts as a ) source when it is (moving sucrose from) a region of 2


production / photosynthesising ;
(acts as a ) sink when it is, growing / storing / respiring / a region of
utilisation ;

4(d)(i) not all the oxygen is, released / collected / measured (from the plant) ; 2
some is used in respiration ;
some is dissolved in the water ;
(therefore) the rate of photosynthesis may appear lower than it
actually is ;

4(d)(ii) 1 carbon dioxide is required for photosynthesis ; 3


2 initially carbon dioxide concentration is limiting the rate of
photosynthesis ;
3 (after 500 μmol per dm3) carbon dioxide concentration is not
limiting the rate of photosynthesis / other factors are limiting the
rate of photosynthesis ;
4 ref. to named limiting factor ;

© UCLES 2021 Page 8 of 10

Page 721 of 877


0610/42 Cambridge IGCSE – Mark Scheme March 2021
PUBLISHED
Question Answer Marks Guidance

4(e) initial gradient is less steep ; 2


plateau below original line ;

Question Answer Marks Guidance

5(a)(i) (140 000 – 14 000) / 140 000 × 100 ; 2


90 (%) ;

5(a)(ii) 1 decrease in number of cases of measles as the percentage of the 4


population vaccinated against measles increases ;
2 percentage of the population vaccinated increases (until 1995)
and then fluctuates ;
3 measles cases decrease, then remain constant ;
4 ref. to rapid change before 1995 in percentage vaccinated and
number of cases ;
5 number of cases remains the same from, 2000 / 2005, while
percentage vaccinated fluctuates / decreases ;
6 correct data quote ;

5(a)(iii) 1 harmless / dead / attenuated, pathogen is given ; 5


2 antigens are present on pathogens ;
3 (antigens) trigger an immune response ;
4 lymphocytes / memory cells produce antibodies ;
5 ref. to specificity ;
6 memory cells are produced ;
7 antibodies destroy pathogen ;
8 providing long term immunity / recognise pathogen if it
returns / ref. to active immunity / AW ;

5(b)(i) mucus ; 2
hydrochloric / stomach, acid ;
AVP ;;

© UCLES 2021 Page 9 of 10

Page 722 of 877


0610/42 Cambridge IGCSE – Mark Scheme March 2021
PUBLISHED
Question Answer Marks Guidance

5(b)(ii) ref. to involvement of platelets ; 3


fibrinogen converted to fibrin ;
forms a mesh (trapping red blood cells) / forms scab ;
AVP ;

Question Answer Marks Guidance

6(a)(i) 4; 1

6(a)(ii) hawks ; 1

6(a)(iii) chemical ; 1

6(b) 1 snakes occupy a higher trophic level than mice ; 3


2 (most of the) energy is lost between the trophic levels ;
3,4 examples of energy loss ;;
5 not enough energy to sustain larger biomass (of snakes) at higher
trophic levels / AW ;

6(c) 1 drought is a lack of, water / rainfall ; 3


2 destruction of, crops ;
3 seeds / plants, can’t germinate ;
4 soil erosion / desertification ;
5 plants need water for photosynthesis ;
6 plants wilt ;
7 less, food / water, for livestock ;
8 AVP ;

© UCLES 2021 Page 10 of 10

Page 723 of 877


0610/41 Cambridge IGCSE – Mark Scheme May/June 2020
PUBLISHED
Question Answer Marks

1(a) one mark for each column: 4

intercostal muscles
diaphragm pressure change in the thorax
internal external

breathing in contract relax contract decreases (A increases)

breathing out relax contract / relax relax increases (A decreases)

;;;;

1(b) any two from: 2


thin / short distance (for diffusion) ;
well supplied by blood / surrounded by capillaries / AW ;
good ventilation with air ;

1(c)(i) a group of cells with similar structures ; 2


working together to perform a shared function ;

1(c)(ii) any two from: 2


forms incomplete rings around, trachea / bronchi ;
keeps (named) airways open ;
reduces resistance to movement of air ;
protects (named) airways ;
sound production in larynx ;

© UCLES 2020 Page 6 of 13

Page 724 of 877


0610/41 Cambridge IGCSE – Mark Scheme May/June 2020
PUBLISHED
Question Answer Marks

2(a) one mark per row: 3

substance enzyme product(s)

starch amylase maltose / glucose / (simple) sugar(s)

fat lipase fatty acid(s) and glycerol

protein protease / pepsin / trypsin amino acids


;;;

© UCLES 2020 Page 7 of 13

Page 725 of 877


0610/41 Cambridge IGCSE – Mark Scheme May/June 2020
PUBLISHED
Question Answer Marks

2(b) any four from: 4


biological washing powder is more effective, at lower temperatures / between 10 °C and 40 °C / 10 °C and 43 °C ;
comparative data quote for the difference at a stated temperature ;
biological washing powder removes all stain between 30 °C and 40 °C ;
non-biological removes all stain only at 60 °C ;
effectiveness is similar, at high temperatures / between 50 and 60 °C ;
same trend, below 30 °C / at low temperatures / from 50 °C ;
idea of effectiveness of biological washing powder decreases between 40 °C and 44 °C, no such decrease for non-
biological washing powder ;

2(c) any two from: 2


active site changes shape ;
substrate no longer fits into, enzyme / active site ;
no enzyme-substrate complex / no successful collisions ;

© UCLES 2020 Page 8 of 13

Page 726 of 877


0610/41 Cambridge IGCSE – Mark Scheme May/June 2020
PUBLISHED
Question Answer Marks

2(d) any two from: 2


individual people have, different / unique, DNA ;
DNA has genes or alleles have, sequences of bases ;
AVP ;

Question Answer Marks

3(a)(i) any three from: 3


blue at time 0 indicates no glucose present ;
ensures that no glucose on outer surface of dialysis tubing / in water, as a result of an error ;
green / yellow / red, indicates presence of glucose ;
glucose, diffuses / moves, out of dialysis tubing / into water ;
(movement is) down the concentration gradient / high to low concentration ;
dialysis tubing is permeable to glucose ;
AVP ;

3(a)(ii) idea that (Benedict’s solution) changes colour quicker / gives more intense colour / AW ; 1

3(b) A are microvilli ; 6


function: allow movement of substances into the cell / increase surface area for absorption by diffusion OR active
transport / have proteins in the membrane for active transport ;

B is the (rough) endoplasmic reticulum / (R)ER ;


function: site of protein synthesis / modify proteins / assemble amino acids in a specific sequence to make (named)
protein ;

C is a mitochondrion ;
function: aerobic respiration / provides energy for (named) cell process(es) ;

3(c)(i) chloride ; 1

© UCLES 2020 Page 9 of 13

Page 727 of 877


0610/41 Cambridge IGCSE – Mark Scheme May/June 2020
PUBLISHED
Question Answer Marks

3(c)(ii) any four from: 4


loss of water ;
by osmosis / down water potential gradient ;
diarrhoea ;
dehydration ;
loss of other, (named) ions / salt(s) ;
AVP ;

Question Answer Marks

4(a)(i) Sorghum ; 1

4(a)(ii) feathery stigma / stigma with large surface area ; 2


stigma / anthers, hang outside the flower(s) ;

4(b)(i) C ovary (wall) ; 3


D ovule ;
E style ;

4(b)(ii) meiosis / reduction division ; 7


haploid ;
fuses / joins / combines ;
diploid ;
fertilisation ;
zygote ;
mitosis ;

© UCLES 2020 Page 10 of 13

Page 728 of 877


0610/41 Cambridge IGCSE – Mark Scheme May/June 2020
PUBLISHED
Question Answer Marks

4(c) any five from: 5


(gives) genetic variation / diversity ;
ref to, alleles / genes / DNA, from different, plants / parents ;
allows mutations to be, expressed / AW ;
allows adaptation to, new conditions / changed environment / AW ;
(new species) can evolve / allows natural selection to occur ;
pollen exchanged between individuals / cross pollination ;
seeds are dispersed ;
can colonise new areas / AW ;
less competition (with parent plant / among offspring) ;
seeds may be dormant ;
survival through, harsh / adverse, conditions ;
AVP ;

4(d) any three from: 3


protein synthesis ;
transport in the phloem ;
cell division / mitosis / meiosis ;
active transport / absorption of ions (from the soil) ;
growth ;
movement / muscular contraction ;
sensitivity ;
nerve impulses ;
AVP ;;;

© UCLES 2020 Page 11 of 13

Page 729 of 877


0610/41 Cambridge IGCSE – Mark Scheme May/June 2020
PUBLISHED
Question Answer Marks

5(a) any two from: 2


assume features are of protoctists unless told otherwise
nucleus / nuclear membrane / nuclear envelope ;
(named) organelle(s) / internal membranes ;
cell walls (if present) have different composition ;
linear chromosomes ;
AVP ;

5(b) box 2: (organism) has two rings of cilia / (organism) stalk absent / AVP ; 2
box 4: (organism) has a covering of cilia / (organism) fused cilia absent / AVP ;

5(c) movement AND nutrition ticked ; 1

5(d)(i) bacteria Paramecium Didinium ; 1

5(d)(ii) any two from: 2


ciliates eat (many) bacteria ;
Didinium / predatory ciliates, eat other (named) ciliates ;
ciliates may eat, dead / decomposing, material ;

5(d)(iii) any three from: 3


removal of, harmful bacteria /pathogens, from sewage ;
e.g. cholera bacteria or any other water born disease / parasites ;
stop spread of pathogens via water ;
use of chlorination / chemical treatment ;

5(d)(iv) any three from: 3


conversion of ammonia / ammonium (ions), to nitrate (ions) ;
convert ammonium ions to nitrite ions ;
make nitrate ions available to plants ;
nitrate ions are absorbed by plants ;
nitrate ions are used to make, amino acids / proteins ;

© UCLES 2020 Page 12 of 13

Page 730 of 877


0610/41 Cambridge IGCSE – Mark Scheme May/June 2020
PUBLISHED
Question Answer Marks

6(a) transmission of genetic information from generation to generation ; 1

6(b)(i) 1 correct use of X and Y in responses for individual 5 and individual 8 ; 3


2 correct X allele given for individual 5: XbY / b ;
3 correct X allele given for individual 8: XBY / B ;

6(b)(ii) any three from: 3


colour blindness is a sex-linked characteristic ;
she is, heterozygous for the gene / Bb ;
she has, normal allele / B, so has normal colour vision ;
but has passed on the, recessive allele / b, to her sons / 5 and 7 ;
she has two X chromosomes which have the gene for colour vision ;
father / 4, passes on his Y chromosome ;

6(b)(iii) any two from: 2


mutation ;
to give, recessive allele / b ;
occurred in 3 or in one of her parents / 1 or 2 or her grandparents ;
AVP ; e.g. other reason such as donated gamete

© UCLES 2020 Page 13 of 13

Page 731 of 877


0610/42 Cambridge IGCSE – Mark Scheme May/June 2020
PUBLISHED
Question Answer Marks

1(a)(i) (because it is made of) a group of tissues working together to perform specific functions ; 1

1(a)(ii) brain ; 1

1(a)(iii) A (thermo)receptor ; 3
B sweat gland ;
F fatty tissue / fat cell(s) ;

1(a)(iv) any three from: 3


vasoconstriction (of arterioles / E) ;
shunt vessels / D, dilate / widen ;
less blood flow to skin (capillaries) / F ;
reduces heat loss from blood ;

1(b) any three from: 3


muscle contraction ;
protein synthesis ;
cell division ;
active transport ;
growth ;
passage of nerve impulses ;

Question Answer Marks

2(a) embryo ; 1

2(b) any two from: 2


growth in all stages ;
development during all stages ;
(more) increase in complexity in early stages ;
(more) increased in size in later stages ;

© UCLES 2020 Page 6 of 11

Page 732 of 877


0610/42 Cambridge IGCSE – Mark Scheme May/June 2020
PUBLISHED
Question Answer Marks

2(c) any four from: 4


maintains temperature ;
(mechanical) protection ;
provides support (of the fetus) ;
provides a sterile environment / prevents infections ;
allows movement (of the fetus) ;
(movement) allows for development of bones and muscles ;
ref. to swallowing (of fluid) ;
lubrication / AW ;
AVP ;

2(d)(i) pulmonary (artery) ; 1

2(d)(ii) carbon dioxide / urea / AVP ; 1

2(d)(iii) diffusion ; 1

2(e)(i) 0.005 (mm) ; 1

2(e)(ii) 130 000 ;; 2

2(e)(iii) nicotine, drug X, rubella virus, Vibrio cholerae ; 1

2(f)(i) A sensory neurone ; 4


B vesicle ;
C synapse / synaptic cleft ;
D receptor molecules ;

2(f)(ii) any three from: 3


drug X blocks, D / receptor (molecules) ;
neurotransmitters are not able to bind to, D / receptor (molecules) ;
drug X is similar in shape to neurotransmitter / complementary to shape of receptor (molecule) ;
drug X stops, impulse/electrical signal, being transmitted in relay neurone ;
(so) less / no, pain felt with drug X ;

© UCLES 2020 Page 7 of 11

Page 733 of 877


0610/42 Cambridge IGCSE – Mark Scheme May/June 2020
PUBLISHED
Question Answer Marks

2(g) any two from: 2


(contaminated) blood transfusion ;
sexual fluids ;
breast feeding ;
blood to blood contact ;
AVP ;;

Question Answer Marks

3(a) fur colouring 2


camouflaged so that not seen by prey (when stalking) / reflecting heat / AW ;

streamlined body shape


improved ability to run fast / reduced air resistance / AW / AVP ;

3(b)(i) transmission of genetic information from generation to generation ; 1

3(b)(ii) two of the same letter both lower case ; 1

3(b)(iii) 1 1
0.25 / 25% / ;
4

3(b)(iv) any two from: 2


perform a test cross ;
by breeding with, homozygous recessive / king cheetah ;
if any of the offspring of the test cross are king cheetahs it confirms 17 is heterozygous ;
DNA testing ;

3(b)(v) any one from: 1


compare, morphology / anatomy ;
compare, DNA / amino acid, sequences ;

© UCLES 2020 Page 8 of 11

Page 734 of 877


0610/42 Cambridge IGCSE – Mark Scheme May/June 2020
PUBLISHED
Question Answer Marks

3(c)(i) any three from: 3


hunting / poaching ;
disease ;
lack of, food / prey ;
loss of (natural) habitat / urbanization ;
pollution / poisoned carcasses ;
inbreeding / AW ;
climate change ;
AVP ;

3(c)(ii) any three from: 3


captive breeding programmes ;
local cooperation / education (of farmer / land users) ;
national parks / conservation areas / protect habitats ;
legislation / public pressure, against hunting ;
monitor numbers ;
AVP ;

Question Answer Marks

4(a) to increase crop, yield / production ; 2


to reduce competition with weeds ;
AVP ;

4(b)(i) concentration of both herbicides decreased (with time) / described ; 3


A higher concentration than B (throughout) ;
B reached zero concentration before A ;
comparative data quote with units stated ;
A steeper than B initially ;

© UCLES 2020 Page 9 of 11

Page 735 of 877


0610/42 Cambridge IGCSE – Mark Scheme May/June 2020
PUBLISHED
Question Answer Marks

4(b)(ii) kills, water plants / algae ; 4


lack of, producers / food for herbivores ;
bioaccumulation / described ;
reduced biodiversity ;
(lack of roots causes) erosion / silting / flooding ;
AVP ;;

4(c)(i) network / branched, veins ; 2


broad (leaves) ;
petiole ;
AVP ;

4(c)(ii) auxin ; 1

4(c)(iii) any three from: 3


no chlorophyll synthesis ;
cannot trap sunlight ;
cannot photosynthesise ;
AVP ;

Question Answer Marks

5(a)(i) Geospiza ; 1

5(a)(ii) any five from: 5


natural selection ;
variation (in beak shapes) ;
mutation / description ;
those birds with, selective advantage / unique beak shape, more likely to find food and survive ;
the birds that survive reproduce ;
pass on their alleles ;
continues over many generations ;
AVP ;

© UCLES 2020 Page 10 of 11

Page 736 of 877


0610/42 Cambridge IGCSE – Mark Scheme May/June 2020
PUBLISHED
Question Answer Marks

6(a)(i) one mark per correct row 5


function name of structure letter from Fig. 6.1

provides support to the stem xylem L

protects flower bud sepal G

produces glucose leaf H

produces pollen anther B

delivers male nuclei to the site of fertilisation pollen tube D


;;;;;

6(a)(ii) B/D/F ; 1

6(a)(iii) translocation ; 1

6(a)(iv) H; 1

6(b)(i) nitrate (ions) ; 1

6(b)(ii) ribosomes / (rough) endoplasmic reticulum ; 1

6(b)(iii) enzymes ; 1

© UCLES 2020 Page 11 of 11

Page 737 of 877


0610/43 Cambridge IGCSE – Mark Scheme May/June 2020
PUBLISHED
Question Answer Marks

1(a) any three from: 3


protein synthesis ;
transport in the phloem ;
cell division / mitosis / meiosis ;
active transport / absorption of ions (from the soil) ;
growth ;
movement / muscular contraction ;
sensitivity ;
nerve impulses ;
AVP ;;;

1(b) one mark per correct row 6

function name of structure letter from Fig. 2.1

pushes food through


oesophagus A
the stomach

assimilation of amino
acids to produce liver K
plasma proteins

storage of bile gall bladder L

secretion of insulin pancreas C

absorption of fatty acids


small intestine H/D
and glycerol

secretion of pepsin stomach B

digestion of starch small intestine H/D

;;;;;;

© UCLES 2020 Page 5 of 13

Page 738 of 877


0610/43 Cambridge IGCSE – Mark Scheme May/June 2020
PUBLISHED
Question Answer Marks

1(c) any two from: 2


lactic acid is produced, in muscles / during exercise ;
by anaerobic respiration ;
liver absorbs lactic acid from the blood ;
(aerobic) respiration / oxidation / breakdown, of lactic acid ;
to carbon dioxide and water ;

1(d) any substance taken into the body ; 2


that modifies or affects chemical reactions in the body ;

1(e)(i) any two from: 2


depressant ;
lengthens reaction time(s) ;
reduces self-control ;
any appropriate effect on the nervous system described ;
AVP ;

1(e)(ii) any two from: 2


addiction ;
liver damage ;
AVP ;;

1(f)(i) any two from: 2


miscarriage ;
premature birth ;
low birth weight ;
addiction / dependence ;
fetal alcohol syndrome (FAS) ;
AVP ;

© UCLES 2020 Page 6 of 13

Page 739 of 877


0610/43 Cambridge IGCSE – Mark Scheme May/June 2020
PUBLISHED
Question Answer Marks

1(f)(ii) any two from: 2


nicotine ;
pathogens / virus ;
any example ; e.g. HIV / rubella
(named) heavy metal(s) ; e.g. lead / mercury
carcinogen(s) ;
(named) toxin(s) ; e.g. pesticides
(named) medicinal drugs ;
(named) misused (illegal) drugs ; e.g. heroin

Question Answer Marks

2(a)(i) population in 1940 = 20 million, population in 2018 = 136 million ; 3


580% ;;

2(a)(ii) any three from: 3


increase in birth rate / decrease death rate ;
immigration ;
increased food supply ;
reduced poverty ; ora
better housing / sanitation / health care / vaccination / AW ;
AVP ;

2(b)(i) by yeast ; 2
(using) anaerobic respiration ;

© UCLES 2020 Page 7 of 13

Page 740 of 877


0610/43 Cambridge IGCSE – Mark Scheme May/June 2020
PUBLISHED
Question Answer Marks

2(b)(ii) any four from: 4


deforestation ;
loss in (variety) of, habitat / places where organisms live / described ;
disruption to food chains / described ;
loss of (bio)diversity / (local) extinction of species / species become endangered / AW ;
soil erosion / increased risk of landslides / flooding ;
disrupted nutrient cycling ;
decrease in (soil) water / desertification ;
outbreaks / spreading, of crop diseases ;
outbreaks / spreading, of (named) pests ;
overuse of herbicides ;
overuse of, pesticides / insecticides / AW ;
killing of non-target species ;
pollution of waterways by, plant nutrients / fertilisers ;
pollution of the atmosphere by NOx from fertilisers ;
pollution by use of fossil fuels in machinery ;
AVP ;

Question Answer Marks

3(a) transmission of genetic information from generation to generation ; 1

3(b)(i) Tt ; 3
tt ;
TT / Tt ;

3(b)(ii) cats 3 and 4 are homozygous recessive / do not have the allele for polydactyly ; 1

3(c)(i) any two from: 2


cats with normal number of toes have AGA for bases 7, 8 and 9 ;
cats with polydactyly have GGA or AGT ;
bases 7 and 9 are different / base 7 is G not A in the USA cats / base 9 is T not A in the UK cats ;

3(c)(ii) mutation ; 1

© UCLES 2020 Page 8 of 13

Page 741 of 877


0610/43 Cambridge IGCSE – Mark Scheme May/June 2020
PUBLISHED
Question Answer Marks

3(c)(iii) origin of the cat was USA ; 2


base sequence is the same as the other cats from the USA / they have the same, mutation/base sequence, as the
Oregon and Missouri cats ;

3(d) 1
T A

A T

A T

T A

G C

C G

G C

T A

G C
;

3(e) distinct, phenotypes / categories ; 2


no intermediates / phenotypes not on a continuous scale ;

© UCLES 2020 Page 9 of 13

Page 742 of 877


0610/43 Cambridge IGCSE – Mark Scheme May/June 2020
PUBLISHED
Question Answer Marks

4(a)(i) any four from: 4


deep roots / AW ;
to absorb water from the water table / AW ;
long and spread out below the surface ;
to absorb water when it rains ;
root cells have low water potential ;
to absorb water by osmosis ;
from (very) salty soils / AW ;
roots branch many times ;
have many roots hairs ;
to give a large surface area (for absorption of water) ;

4(a)(ii) any three from: 3


few stomata / low stomatal density ;
sunken stomata ;
stomata close during the day and open at night ;
rolled leaves ;
thick epidermis / thick cuticle ;
few / no / small, leaves ;
hairs on leaves ;
low rates of transpiration ;
AVP ;;

4(a)(iii) any two from: 2


make / store, toxins ;
make / store, foul-tasting substances / AW ;
spines / prickles / needles ;
resins (that trap insects) ;
thick (inedible) leaves ;
AVP ;;

© UCLES 2020 Page 10 of 13

Page 743 of 877


0610/43 Cambridge IGCSE – Mark Scheme May/June 2020
PUBLISHED
Question Answer Marks

4(b) any three from: 4


reduce air pollution ;
reduce emissions of, sulfur dioxide ;
use filters / ‘scrubbers’ on chimneys ;
catalytic converters ;
reduce use of (named) fossil fuel(s) ;
example of way to reduce demand for energy ;
use low-sulfur (fossil) fuels ;
use alternative sources of power ;
add lime to soils ;
to reduce mobilisation of aluminium in soils / AW ;
to raise pH of soils ;
AVP ;

Question Answer Marks

5(a) any two from: 2


nucleus / nuclear membrane / nuclear envelope ;
(linear) chromosomes ;
mitochondrion ;
endoplasmic reticulum ;
vacuoles / vesicles ;
AVP ;

© UCLES 2020 Page 11 of 13

Page 744 of 877


0610/43 Cambridge IGCSE – Mark Scheme May/June 2020
PUBLISHED
Question Answer Marks

5(b) any six from: 6


resistance arises by mutation ;
in small number of bacteria ;
ref. to a random event (not related to presence of antibiotic) ;
antibiotic kills bacteria that do not have the mutation / AW ;
resistant bacteria have no competition ;
resistant bacteria reproduce ;
pass on, gene / allele, for resistance ;
natural selection ;
method of transmission from one person to another described ;
gene transferred to other bacteria (of different type) in a plasmid ;
AVP ;

5(c) any three from: 3


prescribe / use, antibiotics less often ;
do not use for, viral / fungal, infections ;
make sure people complete the course of antibiotics / AW ;
develop new antibiotics ;
do not use the same antibiotics for too long / rotate antibiotics / AW ;
use combinations of antibiotics ;
AVP ;; e.g. isolation of patients with antibiotic-resistant infections / good hygiene to prevent spread of infection / reduce
use of antibiotics in farming

© UCLES 2020 Page 12 of 13

Page 745 of 877


0610/43 Cambridge IGCSE – Mark Scheme May/June 2020
PUBLISHED
Question Answer Marks

6(a) any three from: 3


if crops are used as food for humans fewer trophic levels in the food chain ;
energy lost at each trophic level in the food chain ;
90% energy lost / only 10% energy passed on ;
energy is lost from the cattle ;
any two examples of energy loss from cattle ;;
therefore less energy available to humans ;
AVP ;

6(b) any four from: 4


smell / visual pollution ;
increase risk of water-borne disease ;
increase organic content of, rivers / lakes ;
increase growth of, bacteria / decomposers ;
bacteria / decomposers, use up dissolved oxygen ;
death of (named) organisms that rely on dissolved oxygen ;
eutrophication ;
adds, urea / ammonia ;
increases plant growth ;
AVP ;;

6(c) any three from: 3


lack of food supply / unequal distribution of food ;
wars / sudden immigration, with inadequate resources for the population ;
drought / floods, destroy crops / kill livestock ;
disease in, food plants / animals ;
poverty ;
AVP ;;

© UCLES 2020 Page 13 of 13

Page 746 of 877


0610/41 Cambridge IGCSE – Mark Scheme October/November 2020
PUBLISHED

Question Answer Marks Guidance

1(a) osmosis ; 2
solvent ;

1(b) drawing with: 3


arrow showing water movement into cell ;

max. two from:


no space between cell membrane and cell wall ;
cell wall, slightly bent outwards / straight ;
vacuole larger in proportion than in Fig 1.1 ;

1(c) wilting ; 3
lack of turgor pressure (at the end of the week) ; ora
no longer a push against cell wall / AW ; ora
(mesophyll) cells not providing support / cell collapses / AW ;
(lack of water means cells become) flaccid / plasmolyse ;

© UCLES 2020 Page 5 of 13

Page 747 of 877


0610/41 Cambridge IGCSE – Mark Scheme October/November 2020
PUBLISHED
Question Answer Marks Guidance

2(a) Q / pathogen, are recognized as foreign ; 6


Q / pathogen, will have specific / unique / AW, antigen ;
S and R are white (blood) cells ;
S / lymphocytes, make antibodies ;
T are antibodies ;
T / antibodies are as specific shape / complementary to,
antigen / pathogen / Q ;
T / antibodies bind to, antigen / pathogen / Q ;
ref. to forming memory cells ;
ref. to, active / long-term, immunity ;
R / phagocytes, engulf, pathogens / antigens ;
R / phagocytes, have enzymes / digest pathogens OR antigens ;
AVP ;

2(b) support of conclusion: 4


general decrease, from 1942 / vaccination ;
cases do not return to pre-vaccine levels / AW ;
no cases from 1974 ;

against conclusion:
number of cases increased, (during the 2 years) after the vaccine was
introduced / until government made its conclusion ;
took 32 years after vaccine introduced before no cases of disease ;
but there are (small) peaks (in cases) / fluctuation (in cases) ;

comparative data quote ;

3(a)(i) any one from: 1


nucleus ;
membrane-bound (named) organelle / has internal membranes ;
vesicles ;
no cell wall ;

3(a)(ii) (cell) membrane ; 2


controls what, enters / leaves, the cell ;

© UCLES 2020 Page 6 of 13

Page 748 of 877


0610/41 Cambridge IGCSE – Mark Scheme October/November 2020
PUBLISHED
Question Answer Marks Guidance

3(a)(iii) diffusion (through cell membrane / A) ; 1

3(b) any three from: 3


breakdown of (excess) amino acids ;
(by) deamination ;
removal of nitrogen containing part (of amino acid) ;
in the liver ;

3(c)(i) X marked on either kidney in the outer / edge region ; 1

3(c)(ii) 5 one mark per correct row

letter from
function name of structure
Fig. 3.2

organ that stores urine bladder G;

tube that carries urine out of


ureter F;
the kidney

blood vessel with the lowest


renal vein D;
concentration of urea

blood vessel with the lowest


renal artery E;
concentration of carbon dioxide

tube that carries urine out of


urethra H;
the body

3(d)(i) any two from: 2


sweat more / lost more water (while running) ;
do not drink as much / reduced intake of water (while running) ;
ref. to homeostasis / negative feedback ;
AVP ;

© UCLES 2020 Page 7 of 13

Page 749 of 877


0610/41 Cambridge IGCSE – Mark Scheme October/November 2020
PUBLISHED
Question Answer Marks Guidance

3(d)(ii) 9(%) ;;; 3 MP1 correct values selected i.e. 78.2 and 85.6
MP2 correct calculation
MP3 correct rounding to one significant figure
ecf for MP2 and MP3 for incorrect MP1

3(d)(iii) any three from: 3


salts are in the blood / move from the blood into the tubule / AW ;
ref. to glomerulus ;
(ultra)filters / allows through ;
pores / gaps, in capillary wall / narrow capillaries ;
small molecules are filtered / large are not filtered / AW ;
(some salt) reabsorption ;
ref. to active transport / diffusion ;
excess (salt) remains in the, urine / filtrate ;
AVP ;

3(e) any one from: 1


fibrinogen / fibrin ;
(named) hormone ;
antibodies ;

© UCLES 2020 Page 8 of 13

Page 750 of 877


0610/41 Cambridge IGCSE – Mark Scheme October/November 2020
PUBLISHED
Question Answer Marks Guidance

4(a)(i) any three from: 3


large / obvious / AW, petals / sepals ;
anthers / stigmas, inside flower ;
filaments are stronger / thicker / AW ;
pollinators must touch anthers, to reach nectar / AW ;
sticky stigma ;
pollen, large ;
pollen, sticky / spiky ;
AVP ; honey guides / landing platforms / mimic insects

4(a)(ii) anther ; 1 A stamen

4(a)(iii) meiosis / reduction division ; 1

4(a)(iv) any one from: 1


so that diploid number restored (after fertilisation) / AW ;
to enable sexual reproduction ;
(so that the offspring) are genetically different / to allow variation ;

4(b)(i) any five from: 5


pollen transferred to stigma ;
ref to (pollen) tube ;
(pollen) tube, growth / germination ;
(pollen tube grows) down style ; MP4 A pollen nucleus moves down style
(pollen tube) enters ovule ;
(ovule is) in the ovary / carpel ;
pollen / male, nucleus fuses with ovule / female, nucleus ;
ref. to fertilisation ;
to form zygote ;
(zygote divides by) mitosis to form an embryo ;
AVP ; e.g. (fertilised) ovule becomes the seed

© UCLES 2020 Page 9 of 13

Page 751 of 877


0610/41 Cambridge IGCSE – Mark Scheme October/November 2020
PUBLISHED
Question Answer Marks Guidance

4(b)(ii) any two from: 2


allows, variation / genetic diversity ;
plant more likely to survive (named) environmental change ;
resistance to disease ;
(ability to) evolve ;
ref. to fitness ;
AVP ;

4(c)(i) any one from: 1 MP1 A isolate plants


grow, GM / wild varieties, in glasshouses ;
cover flowers ;
remove stamens ;
plant another species around the crop ;
make a large, gap / wall, around the field ;
use sterile GM plants ;
grow female plants (only) ;
AVP ;

4(c)(ii) any two from: 2


confer resistance, to a (named) factor ;
provide additional, nutrients / AW (to humans) ;
improved, shelf life / flavour / yield / AW ;
environmental protection idea A less use of pesticides / pollution ;
AVP ;

© UCLES 2020 Page 10 of 13

Page 752 of 877


0610/41 Cambridge IGCSE – Mark Scheme October/November 2020
PUBLISHED
Question Answer Marks Guidance

5(a) calcium: 4 max. three from either section


(formation of) bones ;
(formation of) teeth ;
prevents rickets ;
AVP ;

protein:
repair, cells / tissues ;
growth ;
used to make, muscle / enzyme / antibodies / protein, channels / carrier ;
prevents marasmus ;
AVP ;

5(b) any two from: 2


salivary glands ;
stomach ;
pancreas ;
small intestine / named part of small intestine ;

5(c)(i) kills, bacteria / microorganisms / pathogens ; 1

5(c)(ii) any two from: 2


(heat) denatures enzymes ;
so lactose, not broken down / not digested ; MP2 A not optimum temperature, for
digestion / AW
changes shape of active site ;
enzyme will not fit substrate ;

5(c)(iii) lactase ; 1

© UCLES 2020 Page 11 of 13

Page 753 of 877


0610/41 Cambridge IGCSE – Mark Scheme October/November 2020
PUBLISHED
Question Answer Marks Guidance

5(c)(iv) any one from: 1 ora throughout


can reuse the enzyme ;
cheaper ;
no enzymes left in milk ;
so milk does not need to be purified ;
AVP ; e.g. enzymes more stable / less likely to denature
/ affects taste / drinking the enzyme might trigger allergies

5(d)(i) any four from: 4


contains antibodies / ref. to colostrum / provides protection against,
pathogens / diseases / microorganisms ;
provides passive immunity ;
nutrient requirements met / change with age / change with development ;
easy to digest / AW ;
no additives / less risk of allergies ;
sterile / less risk of infection / AW ;
is at, body / correct, temperature ;
no preparation / always available / convenient ;
bonding with mother / AW ;
free / cheap ;
idea of volume is controlled / no over-feeding ;
AVP ;

5(d)(ii) any two from: 2


water needed to, produce breast milk / stay hydrated / AW ;
alcohol can pass to the baby in breast milk / AW ;
alcohol can harm / delay development of, baby / AW ;
AVP ;

6(a) (the ability to) detect stimuli (in the internal / external environment) ; 2
to make (appropriate) responses ;

6(b)(i) groups of receptor cells ; 2


responding to specific stimuli ;

© UCLES 2020 Page 12 of 13

Page 754 of 877


0610/41 Cambridge IGCSE – Mark Scheme October/November 2020
PUBLISHED
Question Answer Marks Guidance

6(b)(ii) 3
action structure

relaxes circular muscles (of the iris) ;

contracts radial muscles (of the iris) ;

widens pupil ;

© UCLES 2020 Page 13 of 13

Page 755 of 877


0610/42 Cambridge IGCSE – Mark Scheme October/November 2020
PUBLISHED
Question Answer Marks Guidance

1(a)(i) (the ability to) detect stimuli (in the internal / external environment) ; 2
to make (appropriate) responses ;

1(a)(ii) groups of receptor cells ; 2


responding to specific stimuli ;

1(b)(i) fovea ; 1

1(b)(ii) lens drawn with correct shape and position ; 3


light rays are shown refracted in, cornea / lens ;
light rays focused on fovea ;

1(b)(iii) (ciliary) muscles relax ; 3


suspensory ligaments are, taut / tight / tense / are pulled / AW ;
so ligaments pull on lens ;
lens is, thin(ner) / flatter / less convex / elliptical shape / stretched ;
light is refracted less ;

1(c) idea that size of pupil, decreases / constricts / gets smaller ; 3


iris in the correct context ;
circular muscles (of iris) contract ;
radial muscles (of iris) relax ;
ref to antagonistic muscles ;

1(d) reflex / involuntary ; 1

Question Answer Marks Guidance

2(a) A enamel ; 4
B dentine ;
C pulp / pulp cavity / nerve / sensory neurone / capillaries / blood
vessels ;
D gum ;

© UCLES 2020 Page 5 of 13

Page 756 of 877


0610/42 Cambridge IGCSE – Mark Scheme October/November 2020
PUBLISHED
Question Answer Marks Guidance

2(b) mechanical / physical, digestion ; 3


crushes food / chewing / grinds food / breaks food into smaller
pieces ;
increases surface area of food ;
for (named) enzyme action / chemical digestion ;
AVP ; e.g. mix food with saliva

2(c) (named) food remains on teeth ; 4


bacteria, use / breakdown, sugars / carbohydrate / sweet foods ;
respiration ;
acid is produced ;
acid, dissolves / erodes / destroys / wears away / AW, enamel ;
dentine is exposed / AW ;
dentine, softer / dissolves more rapidly (than enamel) ;
AVP ; e.g. decay reaches nerve endings leading to pain

© UCLES 2020 Page 6 of 13

Page 757 of 877


0610/42 Cambridge IGCSE – Mark Scheme October/November 2020
PUBLISHED
Question Answer Marks Guidance

3(a) 4 one mark per correct row


trophic level description example from Fig. 3.1

herbivore feeds on, (named parts of) collared peccary


plants / producers / / lowland tapir
/ autotrophs / red harvester ants
/ termites ;

producer makes own food muhly grass


/ photosynthesis / Peruvian feather
/ autotrophic grass ;

quaternary feeds on tertiary consumers great horned owl ;


/ 4° / fourth
/ 4th,
consumer

secondary gets energy from / feeds on, long-tailed weasel


consumer primary consumers / bobcat / jaguar
/ herbivores / nine-banded
armadillo
/ hooded skunk ;

3(b)(i) (named) decomposers ; 1

© UCLES 2020 Page 7 of 13

Page 758 of 877


0610/42 Cambridge IGCSE – Mark Scheme October/November 2020
PUBLISHED
Question Answer Marks Guidance

3(b)(ii) 1 idea that small percentage of energy from sun is ’fixed’ by 4


photosynthesis ;
2 most energy from sun not available / reference to wrong
wavelength / AW ;
3 energy is lost, between / within, trophic levels / along food chain ;
4 ref. to 10% energy transfer / ORA ;
5 ref. to material that is, inedible / not digestible / egested / not
absorbed / not consumed ;
6 energy lost, in respiration / heat / movement / (named) metabolic
process ;
7 ref. to energy loss to (named) decomposers ;
8 ref. to (small) total percentage reaching fourth trophic level ;
or
not enough energy (in fourth trophic level) to support, 5th / another,
level ;
9 would be very small population of predators in fifth trophic
level / (population of) predators in fifth trophic level unlikely to
survive ;
10 fifth trophic level may be parasites which are very small ;

3(c)(i) 1 effect of waste (faeces and urine) on, waterways / crops ; 3


2 disease spreading to, wild populations / humans ;
3 use of antibiotics and (spread of) antibiotic resistance ;
4 use of pesticides ;
5 loss of biodiversity ;
6 release of, (named) greenhouse gases / carbon emissions ;
7 named consequence ; e.g. climate change / global warming
8 providing food for livestock requires monocultures ;
9 deforestation / habitat loss ;
10 idea that livestock production is an inefficient use of, crop
plants / energy ;
11 AVP ; e.g. use of hormones and effect on wild populations
/ feminisation of fish

© UCLES 2020 Page 8 of 13

Page 759 of 877


0610/42 Cambridge IGCSE – Mark Scheme October/November 2020
PUBLISHED
Question Answer Marks Guidance

3(c)(ii) 1 silting of rivers ; 3


2 landslides / mudslides ;
3 soil does not absorb (rain)water / increased risk of flooding ;
4 increased rate of evaporation / land is exposed to drying ;
5 desertification / decreased soil water ;
6 reduction in transpiration ;
7 reduction in cloud formation ;
8 change in rainfall patterns ;
9 plants cannot grow (well) ;
10 idea that loss of anchorage for plants ;
11 idea of reduced (soil) fertility / increased leaching / AW ;
12 loss of, habitat / places where organisms live / described ;
13 disruption to food chain(s) / described ;
14 endangered / extinction / migration of, species ;
15 less, nutrient / nitrogen / carbon, recycling ;
16 AVP ; e.g. dust storms

Question Answer Marks Guidance

4(a)(i) A (rough) endoplasmic reticulum ; 2 A ribosomes


D (sap) vacuole ;

4(a)(ii) B / nucleus – one from: 2


store of, DNA / chromosomes / genetic information / genetic
material / genes ;
controls / regulates, (reactions / protein synthesis in) the cell ;

C / mitochondrion – one from:


(aerobic) respiration ;
releases / provides, energy ;

4(a)(iii) chloroplast ; 1 A starch grain

4(b)(i) S – amylase ; 2
T – maltase ;

© UCLES 2020 Page 9 of 13

Page 760 of 877


0610/42 Cambridge IGCSE – Mark Scheme October/November 2020
PUBLISHED
Question Answer Marks Guidance

4(b)(ii) C6H12O6 ; 2
2 (C2H5OH) + 2CO2 ;

4(b)(iii) 1 conserves fossil fuels ; 3


2 idea that fossil fuels are a finite resource / fossil fuels are not
renewable / biofuels are renewable ;
3 maize / plants, uses carbon dioxide (from the atmosphere) ;
4 ref to photosynthesis ;
5 idea that so balancing the carbon dioxide in, exhausts
/ emissions / AW ;
6 less likely to cause acid rain ;
7 ref to global warming / climate change / (enhanced) greenhouse
effect ;
8 uses waste from crops for energy ;
9 AVP ; e.g. reduces dependency on fossil fuels from other countries

4(b)(iv) 1 carbon dioxide is required for photosynthesis ; 4


2 (more carbon dioxide =) faster / maximum / optimum, rate of
photosynthesis ;
3 (more carbon dioxide) more, glucose / starch / organic molecules, is
produced ;
4 carbon dioxide (concentration) is a limiting factor ; ora
5 prevents concentration falling below that of atmosphere / AW ;
6 ref. to more, growth / yield / profit ;

© UCLES 2020 Page 10 of 13

Page 761 of 877


0610/42 Cambridge IGCSE – Mark Scheme October/November 2020
PUBLISHED
Question Answer Marks Guidance

5(a) 5
hormone site of target role
production organ

FSH pituitary ovary (stimulates) development


gland of egg or ovum / ref. to
secretion of oestrogen ;

LH pituitary ovary (stimulates) ovulation


gland / ref. to secretion of
progesterone (and
oestrogen) ;

oestrogen ovary ; uterus stimulates growth of the


A placenta lining of the uterus

progesterone ovary ; uterus maintains / AW, the lining


A placenta of the uterus ;

5(b) days 0–7: 3


decrease in thickness ;
decreases quickest between days 0 and 3 ;

days 7–28:
increase in thickness ;
remains at same thickness from day, 21 / 22 / 23 ;

© UCLES 2020 Page 11 of 13

Page 762 of 877


0610/42 Cambridge IGCSE – Mark Scheme October/November 2020
PUBLISHED
Question Answer Marks Guidance

5(c) 6
similarities differences A ora

1 treatment with fertility drugs ; 10 ovulation occurs in AI ;

2 (fertility drugs) encourage, 11 fertilisation in AI occurs


follicle production / egg inside the body / in IVF it
development ; occurs in, a Petri dish or
outside the body or AW ;

3 intercourse does not occur ; 12 eggs removed (from ovary)


in IVF ;

4 collection of sperm (from a 13 embryo starts development


donor) ; outside body in IVF / embryo
inserted into uterus in IVF ;

5 washing of sperm ; 14 excess / surplus, embryos


produced in IVF ;

6 fertilisation 15 embryo selection possible in


or fusion of, gametes / egg IVF ;
and sperm ;

7 lining of uterus must develop ; 16 frozen / stored, embryos in


IVF ;

8 implantation occurs
(naturally) ;

9 A as a similarity or as a way
IVF differs from AI
increase chances of multiple,
births / babies ;

© UCLES 2020 Page 12 of 13

Page 763 of 877


0610/42 Cambridge IGCSE – Mark Scheme October/November 2020
PUBLISHED
Question Answer Marks Guidance

6(a)(i) T; 1

6(a)(ii) C and G ; 1

6(b) 1 store of, genetic information / genetic make-up / genes / AW ; 2


2 DNA / genes / alleles / genetic information / base sequence, codes
for / makes, (named) proteins ;
3 sequence of bases in DNA codes for sequence of, amino acids ;
4 transfer of information to daughter cells (during mitosis) ;
5 DNA copied as mRNA ;
6 AVP ;

6(c)(i) each chromosome is, duplicated / replication / doubled / copied ; 2

to maintain the same number of chromosomes OR so all cells have the


same genetic information / cells are all genetically identical ;

6(c)(ii) cell membrane ; 2


cell wall ;
AVP ;

6(c)(iii) (a diploid cell has) two sets of chromosomes ; 1

© UCLES 2020 Page 13 of 13

Page 764 of 877


0610/43 Cambridge IGCSE – Mark Scheme October/November 2020
PUBLISHED

Question Answer Marks Guidance

1(a) seedling A bending towards the light ; 2


seedling B with taller shoot than original (growing straight up) ;

1(b)(i) the ability to detect stimuli, in the internal / external environment ; 2


to make appropriate, responses ;

1(b)(ii) any four from: 4


1 auxin (is a plant hormone) ;
2 auxin made in (root / shoot) tip (only) ;
3 auxin, diffuses (through the plant) ;
4 auxin, is unequally distributed / (more) auxin accumulates lower
parts ;
5 (auxin) stimulates cell elongation ;
6 shoots, grow away from / against, gravity / negative (tropic)
response OR roots, grow towards / with, gravity / positive (tropic)
response ;
7 AVP ;

1(b)(iii) any two from: 2


(roots / plants growing down) anchor the plant / AW ;
(roots / plants growing down are more likely) to reach, water / (named)
minerals ;
(shoots / plants) growing up are more likely) to reach light ;
(shoot or plant has better) access to (named) pollinators ;

1(c)(i) any two from: 2


(reflexes) respond faster ; ora
(reflexes) use, (electrical) impulses / neurons / nerves / nervous
system ;
(reflexes) are shorter lived ; ora
(reflexes) are not a growth response ; ora
AVP ;

1(c)(ii) groups of receptor cells ; 2


responding to specific stimuli ;

© UCLES 2020 Page 5 of 15

Page 765 of 877


0610/43 Cambridge IGCSE – Mark Scheme October/November 2020
PUBLISHED
Question Answer Marks Guidance

2(a) 2 one mark for T before S


R T S; P Q; one mark for P before Q

2(b)(i) any two from: 2


nucleus ;
ribosomes ;
rough endoplasmic reticulum ;
vesicles ;
mitochondria ;
cell membrane ;
AVP ;

© UCLES 2020 Page 6 of 15

Page 766 of 877


0610/43 Cambridge IGCSE – Mark Scheme October/November 2020
PUBLISHED
Question Answer Marks Guidance

2(b)(ii) component: 3
hydrochloric acid ;

functions:
(acid) creates optimum pH for, (stomach) enzymes / pepsin
/ protease ;
(acid) kills (harmful) microorganisms / denatures enzymes (of
microorganisms) ;

OR

component:
pepsin / protease ;

functions:
protein  amino acids ;
ref. to chemical digestion (by enzymes) or to allow soluble
molecules / amino acids, to be absorbed ;
pepsin digests, bacteria / pathogens ;

OR

component:
water ;

functions:
acts as a solvent / AW ;

OR

AVP ;;

2(b)(iii) mucus ; 1

© UCLES 2020 Page 7 of 15

Page 767 of 877


0610/43 Cambridge IGCSE – Mark Scheme October/November 2020
PUBLISHED
Question Answer Marks Guidance

2(c)(i) (mix with) bile ; 2


breaks up of large fat globules (into smaller fat globules) ;
increases surface area (of fat globules) ;
AVP ; e.g. water enables dispersion of (smaller) fat globules

2(c)(ii) small intestine / duodenum ; 1

2(c)(iii) lacteal ; 1

3(a)(i) 3
the number of different producers 2

the number of different secondary consumers 5;

the number of different trophic levels in the food web 4;

how many different trophic levels monkeys feed at 2;

3(a)(ii) fifth (trophic level) / (trophic level) 5 / quaternary consumer ; 1

3(a)(iii) any three from: 3


1 fruit bats are at a lower trophic level (than anaconda) ; ora
2 idea that energy transfer along a food chain is inefficient ; e.g. respiration / egestion / uneaten
3,4 named examples of causes of inefficient energy flow between parts / excretion / movement
trophic levels ;;

3(b)(i) any two from: 2


ref. to use of extracted DNA / protein (from tissue) ;
(determination of) DNA / base / amino acid, sequences ;
DNA / sequences / amino acids / genes, compared with, other (known)
species / organisms / ancestors / DNA databases ;

© UCLES 2020 Page 8 of 15

Page 768 of 877


0610/43 Cambridge IGCSE – Mark Scheme October/November 2020
PUBLISHED
Question Answer Marks Guidance

3(b)(ii) any three from: 3


loss of habitat / described ;
addition of (named polluting) chemicals ;
human animal conflict ;
(the crop is often an) introduced species ;
loss of biodiversity or loss of, suitable / variety of, food sources ;
outbreaks / spreading, of diseases / pests / plagues ;
AVP ;

4(a) 3 apply ecf from one step to the next

conversion from μm to mm ; MP1 e.g. (200 μm) = 0.2 (mm)


calculation of volume (of each grid square) ; MP2 e.g. 0.2 × 0.2 × 0.1 = 0.004 (mm3)
calculation of final answer =13 000 (cell per mm3) ; MP3 e.g. 52 yeast cells × 25 grid squares gives a
total of 1300 yeast cells in a volume of 0.1 mm3
(which is the total volume of grid (0.004 × 25 = 0.1
mm3)) multiple 1300 by 10 to convert to yeast
cells per mm3

4(b)(i) X on Fig. 4.3 on / near, solid line between 0 and up to 1 day ; 1

4(b)(ii) line drawn on Fig 4.3, increasing from day 3, more slowly / level ; 2
line drawn on Fig 4.3 showing any decrease (in population) at end of
candidate’s line ;

© UCLES 2020 Page 9 of 15

Page 769 of 877


0610/43 Cambridge IGCSE – Mark Scheme October/November 2020
PUBLISHED
Question Answer Marks Guidance

4(b)(iii) any five from: 5


sugar concentration decreases faster between day 6 and 9 ; ora
no change in sugar concentration / stops decreasing, from day 9 ;
the yeast population affects the sugar concentration ;
(yeast use sugar) in respiration ;
release energy ;
rate of sugar decrease is slow, at first / in lag phase, because there
are few yeast cells ; ora
ref. to dead yeast (after day 9) / death phase / AW ;
AVP ; e.g. ethanol build up / change in pH, could kill the yeast / some
sugar leftover so it is not a limiting factor

4(c) carbon dioxide (produced by respiration) ; 2


(gas) pressure will increase / AW ;

© UCLES 2020 Page 10 of 15

Page 770 of 877


0610/43 Cambridge IGCSE – Mark Scheme October/November 2020
PUBLISHED
Question Answer Marks Guidance

5(a)(i) haploid ; 1

5(a)(ii) oviduct ; 1

5(a)(iii) any four from: 4


ref. to acrosome / (digestive) enzymes, are released (from sperm) ;
jelly coat is digested (by enzymes) ;
fusion of nuclei (of gametes) / fertilisation ;
to form a (diploid) zygote ;
jelly coat hardens (after fertilization) ;
zygote moves, along oviduct / towards uterus ;
cell division / mitosis (after formation of zygote) ;
AVP ; e.g. use of energy stores from egg cell (for mitosis)

© UCLES 2020 Page 11 of 15

Page 771 of 877


0610/43 Cambridge IGCSE – Mark Scheme October/November 2020
PUBLISHED
Question Answer Marks Guidance

5(b) any six from: 6

use of fertility drugs:


1 FSH / LH (are used in fertility treatment) ;
2 taken / injected, during early stage of menstrual cycle ;
3 (FSH) stimulates follicles to, develop / mature ;
4 (fertility drugs) stimulate, the ovaries / production of oestrogen ;
5 (LH / FSH) stimulates, ovulation / described ;
6 AVP ;

process of IVF:
7 collect / extract, eggs ;
8 collect / extract, sperm ;
9 use of donor, eggs / sperm ;
10 fertilisation / described / IVF, outside the body / in a dish / in a
laboratory ;
11 embryo development in dish / AW ;
12 screening of embryos ;
13 inserting embryos into uterus ;
14 give mother progesterone after inserting embryos (to maintain
lining) ;
15 AVP ; appropriate ref. to surrogate mothers or multiple
embryos / zygotes, could lead to multiple births

6(a)(i) insecticide: 3 ; 3
dosage: 5 (mg per dm3) ;
explanation: insecticide 3 has lowest dosage required to remove all
insects species ;

© UCLES 2020 Page 12 of 15

Page 772 of 877


0610/43 Cambridge IGCSE – Mark Scheme October/November 2020
PUBLISHED
Question Answer Marks Guidance

6(a)(ii) any three from: 3

factor to consider discussion points


1 toxicity (of the 7 importance of (named) pollinators
insecticide) ; (in ecosystem) ;
8 potential creation of insecticide-
resistant ‘super’ bugs ;
2 persistence (of
9 harms other (named) non-target
insecticide) / how
species / damages food
quickly it breaks
chains / pollutes environment
down / biodegradable ;
/ described ;
3 presence of non-target
species in the
environment ;
4 prevailing 10 avoid spread into non-target
environmental areas ;
/ climatic conditions ;
5 method of application ;
6 other valid factor ; e.g. 11 relevant corresponding discussion
frequency of application point ;

6(b)(i) any two from: 2


fewer insects, eat / damage crops / AW ;
improved (quality / yield) of crops / AW ;
prevents spread of diseases (carried by insects) ;
AVP ; quick effect (to the treatment) / application is easy / not labour
intensive

© UCLES 2020 Page 13 of 15

Page 773 of 877


0610/43 Cambridge IGCSE – Mark Scheme October/November 2020
PUBLISHED
Question Answer Marks Guidance

6(b)(ii) 3 max 2 from chemical column


table rows indicate paired mark points

chemical benefit
magnesium ; for chlorophyll / photosynthesis ;
more, growth / yield ;
nitrates ; for, amino acids / (to synthesize) A fertilisers once as an alternative if no named
proteins ; ions given
more, growth / yield ;
carbon dioxide ; increased photosynthesis ;
more, growth / yield ;
water ; prevent wilting ; ref. to turgor / ref.
to dissolving solutes ;
herbicides ; reduced (named) competition with
weeds ;
2,4 D / (synthetic)
more, growth / yield ;
plant hormones ;
AVP ;; e.g. other relevant benefit ;;
valid chemicals or e.g. more , growth / yield
ions such as
phosphate
/ potassium
/ fungicide reduced competition with
fungi / prevent disease

6(c)(i) reduced predation / improve chance of survival / increased 1


fitness / AW ;

© UCLES 2020 Page 14 of 15

Page 774 of 877


0610/43 Cambridge IGCSE – Mark Scheme October/November 2020
PUBLISHED
Question Answer Marks Guidance

6(c)(ii) any two from: 2


(tobacco) contains nicotine ;
(nicotine) is addictive ;
ref. to withdrawal symptoms ;

6(c)(iii) any two from: 2


COPD ;
lung cancer ;
coronary heart disease / CHD ;
AVP ; e.g. gum disease / bronchitis

6(c)(iv) any one from: 1


tar ;
dust particles / AW ;

© UCLES 2020 Page 15 of 15

Page 775 of 877


0610/42 Cambridge IGCSE – Mark Scheme March 2020
PUBLISHED
Question Answer Marks Guidance

1(a)(i) A – bronchus ; 3
B – trachea ;
C – diaphragm ;

1(a)(ii) 1 diaphragm, contracts / flattens ; 4


2 external intercostal muscles contract ;
3 ribs move, upwards / outwards ;
4 volume, increases ;
5 pressure, decreases ;
6 air enters (the, mouth / trachea / lungs,) to equalise the
pressure ;

1(b)(i) 86 / 44184 × 100 = 0.194 ; 2


0.19 (%) ;

1(b)(ii) idea that non-smokers / passive smokers / AW, can die from / can 3
develop lung cancer ;
the greater the exposure to tobacco smoke the greater the risk (of
dying from lung cancer) ;
comparative data quote ;

1(b)(iii) COPD ; 2 apply list rule


CHD ;
AVP ;;

© UCLES 2020 Page 6 of 12

Page 776 of 877


0610/42 Cambridge IGCSE – Mark Scheme March 2020
PUBLISHED
Question Answer Marks Guidance

2(a)(i) 2 1 mark for each correct column


letter on contains red
name of the fluid
Fig. 2.1 blood cells

A lymph no

B tissue no
;;

2(a)(ii) diffusion ; 1

2(a)(iii) (aerobic) respiration ; 2


for the release of energy ;
for a (named) metabolic process ;

2(b) 1 deliver / supplies, blood / oxygen / glucose (from arteries) to, 3


capillaries / shunt vessels ;
2 controls blood flow, through capillaries / to the surface of the
skin ;
3 by vasoconstriction / vasodilation ;
4 ref to (arteriole) muscle contraction / relaxation ;
5 ref to maintenance of body temperature / homeostasis
/ description of ;

2(c) contains lymphocytes (which produce antibodies) ; 2 A stores I produces


filters the lymph ;
ref. to immunity / defends against disease / ref. to destruction of
(named) pathogens ;

2(d)(i) small intestine / villi ; 1

2(d)(ii) fat, absorption / transport ; 1

© UCLES 2020 Page 7 of 12

Page 777 of 877


0610/42 Cambridge IGCSE – Mark Scheme March 2020
PUBLISHED
Question Answer Marks Guidance

3(a) as the ability to detect or sense stimuli in the internal or external 2


environment ;
and to make appropriate responses ;

3(b) eye and skin ; 1 apply list rule


AVP ;

3(c)(i) as a control / for a comparison / AW ; 2


to see if blood glucose (concentration) was affected by an injection ;
so there is no effect, on water potential / osmotic effect ;

3(c)(ii) (before the injection) blood glucose was decreasing ; 5

after the injection:


peaks at 4.75 mmol per dm3 ;
increase is immediate / steep (after injection of adrenaline) ;
less steep / gradual, decrease after peak ;
decreases below initial starting value / below 4.4 or 4.5 mmol per
dm3 / decreases to 3.9 mmol per dm3 ;

blood glucose increases


converts glycogen to glucose ;
in the liver ;

blood glucose decreases


glucose used in respiration ;
AVP ;; e.g. insulin causes blood glucose concentration to
decrease / glucagon causes the blood glucose concentration to
increase

3(d) line sketched on the graph showing an increase ; 2


increase starts after 2 hours ;

© UCLES 2020 Page 8 of 12

Page 778 of 877


0610/42 Cambridge IGCSE – Mark Scheme March 2020
PUBLISHED
Question Answer Marks Guidance

3(e) any two from: 2 apply list rule


widened / dilated pupils ;
increased, pulse / heart, rate ;
increased breathing rate ;
increased blood pressure ;
AVP ;;

Question Answer Marks Guidance

4(a) similarities, max 3 from: 4


both caused by protein (energy) deficiency ;
both types of malnutrition / deficiency disease / caused by lack of a
balanced diet ;
primarily affects children ;
diarrhoea ;
low body mass / weight loss ;
poor growth ;
irritability / tiredness ;
wastage of muscles ;
dry skin / brittle nails ;
brittle hair / change of hair colour ;
fatty liver ;
anaemia ;
AVP ;

differences:
kwashiorkor has water retention / oedema / swelling of body parts
/ pot belly ;
people suffering marasmus also deficient in, carbohydrates / fats
/ calories / energy ;
AVP ;

© UCLES 2020 Page 9 of 12

Page 779 of 877


0610/42 Cambridge IGCSE – Mark Scheme March 2020
PUBLISHED
Question Answer Marks Guidance

4(b)(i) 18 (%) ;;; 3 MP1 selection of correct data i.e. 1419 and 1161
MP2 correct calculation
MP3 correct rounding to whole number

4(b)(ii) any year between 1990 and 1994 or from 2000 onwards ; 2
the number of children admitted to hospital decreases (after this
year) ;

4(b)(iii) 1 (described) hygienic food prep methods ; 3


2 washing hands / hand sanitisers ;
3 (description of use of) clean equipment ;
4 use of clean / boiled water / filtered water / pure water ;
5 (described examples of) disposal of waste correctly / idea of
where defaecation occurs / defaecation in a toilet ;
6 monitor or identify infective individuals;
7 isolate infected individuals ;
8 AVP ;

Question Answer Marks Guidance

5(a)(i) doubles ; 1 A increases

5(a)(ii) 34–39 minutes ; 1 A any value within this range

5(b) are genetically different (from each other and the parent cells) ; 3
(produce) haploid (nuclei) ;
ref. to reduction division / chromosome number being halved ;

5(c) ref. to unspecialised cells ; 2


that can become specialised ;
ref. to expression of genes in specialised cells ;
AVP ; e.g. continually divide

© UCLES 2020 Page 10 of 12

Page 780 of 877


0610/42 Cambridge IGCSE – Mark Scheme March 2020
PUBLISHED
Question Answer Marks Guidance

5(d) breaking of the amniotic sac ; 6


amniotic fluid is released ;
contraction of (the muscles in the) uterus (wall) ;
dilation of the cervix ;
passage through the vagina ;
(tying and) cutting the umbilical cord ;
delivery of the afterbirth / placenta ;
AVP ;

Question Answer Marks Guidance

6(a)(i) 1994 / 1995 / 1996 / 1997 / 2007 ; 1

6(a)(ii) at least a 5 year period between 1975 and 1983 ; 1

6(a)(iii) increased predation ; 3


introduced species / new predators ;
reduction in food supply ;
overfishing / increase in fishing ;
parasites / disease ;
named examples of pollution / eutrophication ;
global warming / climate change / ocean acidification ;
AVP ;

6(a)(iv) (captive) breeding programmes ; 3


release them (into the wild / into protected areas) ;
protected areas;
monitoring of numbers ;
limited fishing (so that wild stocks can recover) ;
using farmed fish / sustainable fish (so wild stocks can recover) ;
AVP ;

© UCLES 2020 Page 11 of 12

Page 781 of 877


0610/42 Cambridge IGCSE – Mark Scheme March 2020
PUBLISHED
Question Answer Marks Guidance

6(b)(i) sharp / large teeth ; 2 apply list rule


lots of teeth ;
big jaws ;
streamlined ;
camouflaged ;
AVP ;

6(b)(ii) 1 variation (in eyesight) ; 5


2 ref. to mutation ;
3 new alleles arise that increases fitness ;
4 (organisms with better eyesight) are better competitors ;
5 organisms with the good eyesight / feature, are more likely to
survive and reproduce ; ora
6 passing on their alleles (for good eyesight) ;
7 eventually all the organisms (in the population) will have the
feature / good eyesight ;
8 ref to natural selection / evolution ;

© UCLES 2020 Page 12 of 12

Page 782 of 877


0610/41 Cambridge IGCSE – Mark Scheme May/June 2019
PUBLISHED
Question Answer Marks Guidance

1(a) (group of) organisms that can reproduce ; 2


to produce fertile offspring ;

1(b) pinna(e) / external ears ; 2


mammary glands / milk glands / production of milk / lactating /
suckling / breast feeding / nipples / AW ;
diaphragm ;
(three) bones in the middle ear ;
(four) different types of teeth / two sets of teeth ;
sweat glands ;
enucleated red blood cells ;
uterus / placenta / navel / AW ;
AVP ;

1(c) select, parent(s) / sheep / AW, with, fine / thin, hairs (in wool) 5 max 4 if no reference to quality of wool
OR
use Merino sheep from South Africa and NZ sheep ;

cross them together / use artificial insemination / IVF / AW ;


measure / AW, the hairs in the wool of all the offspring ;
select offspring with, fine / thin, hairs (in wool) ;
cross / AW, offspring together ;
continue / repeat, selection and/or breeding ;
over many generations ;
AVP ;

1(d) features are, adaptive / adaptations (for environment) ; 3


caused by / AW, the, environment / surroundings ;
competition between individuals for (named) resource(s) ;
reference to named selective agent(s) ;
slow(er) ;
increase in fitness ;
explained: ability to survive AND reproduce (in natural environment) ;
maintains (genetic) variation / less (genetic) variation in
selective breeding ;
random mating ;

© UCLES 2019 Page 4 of 14

Page 783 of 877


0610/41 Cambridge IGCSE – Mark Scheme May/June 2019
PUBLISHED
Question Answer Marks Guidance

2(a) carbon dioxide is, raw material / substrate / reactant / AW ; 2


concentration of carbon dioxide is higher outside leaf than inside (so
carbon dioxide diffuses into the leaf) ;

2(b) subtract the concentration of carbon dioxide at the end from the 2
concentration at the start / AW ;
divide by the time (taken) / per unit time ;
ref. to taking (rate of) respiration into account ;

2(c)(i) light intensity ; 1


water (supply) ;
humidity ;

2(c)(ii) increases and, reaches a plateau / remains constant / ‘levels off’ ; 3


increases (between 10 °C) to 30 °C / levels off at 30 °C ;
any comparative use of figures for rate with units at least once ;

2(c)(iii) 36 ;; 2

© UCLES 2019 Page 5 of 14

Page 784 of 877


0610/41 Cambridge IGCSE – Mark Scheme May/June 2019
PUBLISHED
Question Answer Marks Guidance

2(c)(iv) temperature is the limiting factor (over whole range) ; 3


increased temperature increases, kinetic energy / KE, (of molecules) ;
increases rate of diffusion of carbon dioxide (into leaf) ;
temperature, influences / affects, (activity of) enzymes ;
idea of more (effective) collisions between substrate molecules and
enzymes (in plant) / more enzyme-substrate complexes formed ;
more carbon dioxide is, fixed / used in photosynthesis / converted into
sugar / AW ;
carbon dioxide (concentration) is not limiting ;

2(c)(v) B shows that: 1


rate of photosynthesis is, higher / continues to increase, if carbon
dioxide is increased (at all temperatures / AW) ;

2(d) prediction: 2
rate of photosynthesis, remains constant / decreases / slows ;

any explanation one from:


enzymes / active sites, are denatured (at high temperatures) ;
stomata close, so, little / no, carbon dioxide can enter leaves ;
plant is adapted to survive at high temperatures ;

© UCLES 2019 Page 6 of 14

Page 785 of 877


0610/41 Cambridge IGCSE – Mark Scheme May/June 2019
PUBLISHED
Question Answer Marks Guidance

3(a) accommodation ; 5
antagonistic ;
peripheral ;
optic ;
brain ;

3(b) involves, proteins / carriers / pumps (in neurone membrane) ; 3


(named) ion(s) bind to, proteins / carriers / pumps, to move ions / AW ;
move ions, against concentration gradient / from low to high
concentration ;
using energy ;
AVP ; e.g. change in shape of carrier (protein)

© UCLES 2019 Page 7 of 14

Page 786 of 877


0610/41 Cambridge IGCSE – Mark Scheme May/June 2019
PUBLISHED
Question Answer Marks Guidance

3(c) general marking point 6


neurotransmitters move across, synapse / gap / junction / AW ;

atropine
neurotransmitter cannot, bind to / enter / reach, receptors ;
therefore no impulses (along, next / postsynaptic, neurone) /
no impulses reach the CNS ;
no sensitivity to stimuli / feels no pain / painkiller ;
no, contraction of muscle / response ; A reaction time is longer / no reflex
depressant ;

eserine
neurotransmitter stays in, synapse / synaptic gap ;
neurotransmitter can bind to receptor (rather than stay in synapse) ;
continuously stimulates the, next / postsynaptic, neurone ;
(more) impulses are sent (in, next / postsynaptic, neurone) ;
repeated, contraction of muscle / response ;
stimulant ;

© UCLES 2019 Page 8 of 14

Page 787 of 877


0610/41 Cambridge IGCSE – Mark Scheme May/June 2019
PUBLISHED
Question Answer Marks Guidance

3(d) anabolic steroids increase, muscle mass / AW ; 3


gives athletes unfair advantage / ref. to cheating / unethical / immoral ;
(named), side effect / effect on health ;
can be banned from taking part in sport if found using them ;
ref. to illegality ;
AVP ; e.g. can lose sponsorship / loss of reputation / AW

© UCLES 2019 Page 9 of 14

Page 788 of 877


0610/41 Cambridge IGCSE – Mark Scheme May/June 2019
PUBLISHED
Question Answer Marks Guidance

4(a) tissue ; 4
cell structure ;
cell ;
organ ;

4(b) 5 one mark per row


name of structure function letter on Fig. 4.1

production of
testis sperm / produces or C;
releases testosterone

transports sperm but not


sperm duct D;
urine

passage for urine and


urethra seminal fluid through the A;
penis

secretes / produces,
prostate gland seminal fluid / nutrient-rich E;
fluid / alkaline fluid / AW

scrotum
contains testes B;
/ scrotal sac

© UCLES 2019 Page 10 of 14

Page 789 of 877


0610/41 Cambridge IGCSE – Mark Scheme May/June 2019
PUBLISHED
Question Answer Marks Guidance

4(c) 1
X on testis / label line on testis with X ;

4(d)(i) one set of chromosomes ; 1

4(d)(ii) 23 ; 1

© UCLES 2019 Page 11 of 14

Page 790 of 877


0610/41 Cambridge IGCSE – Mark Scheme May/June 2019
PUBLISHED
Question Answer Marks Guidance

5(a)(i) liquid / fluid / watery, part of blood ; 1

5(a)(ii) amino acid(s) ; 1

5(b)(i) plasmid ; 1

5(b)(ii) restriction (enzyme) ; 1

5(b)(iii) cutting / opening, A / the plasmid, with same (restriction) enzyme(s) ; 3


forming, sticky ends ;
idea that (sticky) ends of human DNA and plasmid DNA are
complementary ;
reference to, bases / base sequences (of sticky ends) ;
correct reference to (DNA) ligase ; e.g. inserting gene / sticky ends
joining / splicing
AVP ; e.g. B is a recombinant (plasmid / DNA)

5(b)(iv) reliable / constant, supply ; 1


produce, large(er) quantities / in a fermenter / bacteria reproduce
quickly (to make more genetically engineered bacteria) ;
not dependent on blood donations ;
idea that no (named) health risk(s) ;
higher quality of product ;
AVP ;

5(b)(v) mRNA moves through the cytoplasm ; 2


mRNA molecules, move to / through, ribosomes ;
sequence of bases in mRNA determines order of amino acids (in
TPA) ;
for protein synthesis / to make proteins ;
AVP ;

6(a)(i) cell wall ; 1


cells are a regular shape / described ;
vacuole(s) ;
AVP ;

© UCLES 2019 Page 12 of 14

Page 791 of 877


0610/41 Cambridge IGCSE – Mark Scheme May/June 2019
PUBLISHED
Question Answer Marks Guidance

6(a)(ii) growth ; 1
producing cells ;
increase length of shoot / elongation of shoot ;

6(b) dividing cell / cell division / mitosis, needs (lot of) energy ; 3
carry out aerobic respiration ;
provide / release, energy ;
(for) a named function in dividing cells ;
e.g. movement of chromosomes
making cell wall
making new (named) molecules (e.g. protein / DNA)
making (named) organelle(s)

© UCLES 2019 Page 13 of 14

Page 792 of 877


0610/41 Cambridge IGCSE – Mark Scheme May/June 2019
PUBLISHED
Question Answer Marks Guidance

6(c)(i) auxin ; 1

6(c)(ii) auxin / hormone: 4


made in the, shoot / stem, tip ;
moves away from the tip ;
moves to / collects on, lower side of stem ;
stimulates cell elongation ;
stem, bends / grows, upwards ;
AVP ;

6(d) plants have different, structures / parts / specialised cells ; 3


idea that different parts / specialised cells, have different,
functions / roles / features ;
idea that specific proteins are required in, parts / specialised cells ;
genes code for proteins ;
therefore some genes, are required / are not required ;
AVP ; e.g. idea that waste of (named) resource(s) if all genes
expressed

© UCLES 2019 Page 14 of 14

Page 793 of 877


0610/42 Cambridge IGCSE – Mark Scheme May/June 2019
PUBLISHED
Question Answer Marks Guidance

1(a) yeast ; 1

1(b)(i) flour / starch / sugar / glucose / sucrose / carbohydrate ; 1

1(b)(ii) respiration ; 1

1(b)(iii) carbon dioxide ; 1

1(c) step 3 / 35 °C / low(er) temperature: 2


optimum / best / suitable / AW, temperature for,
respiration / enzymes / gas production ;
to allow the dough to, rise / expand / AW ;

step 5 / 200 °C / high(er) temperature


organism A / microorganisms / yeast, killed / enzymes denature ;
to cook the dough ;
to allow ethanol to evaporate ;

1(d) biofuels ; 2
wine / beer, making ;
penicillin / antibiotic (production) ;
AVP ;;

© UCLES 2019 Page 4 of 12

Page 794 of 877


0610/42 Cambridge IGCSE – Mark Scheme May/June 2019
PUBLISHED
Question Answer Marks Guidance

2(a) burning / use, (named) fuels ; 3


deforestation / AW ;
increased human population ;
example of named relevant human activity ;
AVP ;

2(b) description: 6
rate (of photosynthesis) peaks at, 12:00 / midday / noon ;
photosynthesis starts at, 06:00 / stops at, 20:00 / 8 pm ;
rate (of photosynthesis) at 550 (ppm) / AW, is greater than at,
370 (ppm) / AW ;
both plots / 550 and 370 ppm, follow same trend / pattern ;
comparative data quote between two plots with units at least once ;

explanation:
maximum light at 12:00 / dark until 6:00 / after, 20:00 / 8 pm ;
reference to light intensity as a limiting factor ;
because light is required for photosynthesis ;

reference to CO2 as a limiting factor ;


(at high atmospheric CO2) the concentration gradient (to air
spaces) is steeper / diffusion is faster ;
effect of CO2 concentration is most at high light intensities ; ora

reference to temperature as a limiting factor ;


higher temperature causes increased rate of photosynthesis ; ora
AVP ;

© UCLES 2019 Page 5 of 12

Page 795 of 877


0610/42 Cambridge IGCSE – Mark Scheme May/June 2019
PUBLISHED
Question Answer Marks Guidance

2(c) epidermis 4 one mark per row, max two from each tissue
feature adaptation
transparent / clear / no chloroplasts allows light to pass through ;
thin / flat so less cytoplasm / more light, to
pass through / AW ;
guard cells / stomata allow gases to enter / leave the
leaf / gas exchange ;

mesophyll
feature adaptation
contains many chloroplasts trapping light energy ;
(palisade)
vertically / tightly, packed / column- maximise light received (by
shaped (palisade) cells) / reduce number of,
cross / cell, walls ;

contain (air) spaces / loosely for diffusion / movement of gases


packed (spongy) (within leaf) ;

2(d) more carbon dioxide in the blood ; 2


low pH / acid, in blood ;
(high) carbon dioxide detected by brain ;
increases impulses to (named) muscles used in breathing / AW ;
correct reference to negative feedback / homeostasis ;

© UCLES 2019 Page 6 of 12

Page 796 of 877


0610/42 Cambridge IGCSE – Mark Scheme May/June 2019
PUBLISHED
Question Answer Marks Guidance

3(a)(i) 1.2 × 108 (g) / 120 000 000 (g) / 1.2 × 105 (kg) / 120 000 (kg) ;; 3
kg or g (per day) ;

3(a)(ii) avoid too much (named) sugar in diet ; 1


flossing ;
regular visits to, dentist / hygienist / AW ;
AVP ;

3(b)(i) diatom (→) lugwom (→) (wading) bird ; 2


arrows in correct direction ;

3(b)(ii) description: 4
more ammonium ions remain in bucket / less ammonium, absorbed
(by diatoms) ;
less faeces ;
higher respiration rates ;
lower body mass ;

explanation:
less, diatoms / food / ammonium ions, for lugworms ;
(high respiration of lugworms) to, release more energy / for finding
food / stress etc. ;
slower growth rate of (lugworms) ;
(non-biodegradable) microplastics (negatively) affect digestion ;

3(c)(i) protein / urea / amino acid ; 1

3(c)(ii) nitrification ; 1

3(c)(iii) plants absorb (nitrogen as) nitrate (ions) ; 3


needed to make, amino acids / (named) proteins ;
to make DNA / RNA / nucleotides / bases ;
protein / DNA, is needed for, growth / cell division / mitosis ;

© UCLES 2019 Page 7 of 12

Page 797 of 877


0610/42 Cambridge IGCSE – Mark Scheme May/June 2019
PUBLISHED
Question Answer Marks Guidance

3(d) visual pollution ; 5


chokes / strangles / traps / blocks digestive systems / AW (of
animals) ;
reference to, chemical exposure / fumes / toxins ;
(plastic) accumulates in an organism / is passed down a food
chain ;
(described) habitat destruction ; e.g. plastic covers the habitats
(plastic) blocks (light / water for) photosynthesis (for land plants) ;
(plastic) block roots / prevents root growth ;
remain in the ecosystem (for a very long time) ;
AVP ;

© UCLES 2019 Page 8 of 12

Page 798 of 877


0610/42 Cambridge IGCSE – Mark Scheme May/June 2019
PUBLISHED
Question Answer Marks Guidance

4(a) long, to transmit (impulse), over (long) distance / faster / direct 3


connection ;
mitochondria to (release energy), for transmission impulse / protein
synthesis / active transport / making (neuro)transmitters ;
vesicles to, carry / hold / release, chemicals / (neuro)transmitters
(into synapse) ;
(neuro)transmitters are released, to allow connection to other
neurones / across a synapse ;
receptors / vesicles, allow unidirectional transmission ;
AVP ;;

4(a)(ii) brain and spinal cord (only) ; 1

4(b)(i) stimulus / light (detected by) retina / rod / cone / receptor ; 3


reference to (electrical) impulse / electrical signal ;
sensory neurone → relay / connector, neurone →
motor neurone ;
reference to synapses between neurones ;
effector / (circular) muscles (in iris), contract / respond ;

4(b)(ii) automatic / involuntary ; 1


receptors / neurones / nerve, still function ;

4(c)(i) E – vesicle ; 3
F – neurotransmitter ;
G – (neurotransmitter) receptor (molecule / site / protein) ;

4(c)(ii) arrow drawn from right to left, pointing left ; 1

© UCLES 2019 Page 9 of 12

Page 799 of 877


0610/42 Cambridge IGCSE – Mark Scheme May/June 2019
PUBLISHED
Question Answer Marks Guidance

5(a)(i) testosterone ; 1

5(a)(ii) one mark per box but organ system must match organ ;;;; 4

5(b) to produce, gametes / sperm ; 3


for sexual reproduction ;
to halve the number of chromosomes / produce haploid cells ;
so that when fertilisation occurs the number of chromosomes return
to the, same / diploid, number / AW ;
creates (genetic) variation / AW ;

© UCLES 2019 Page 10 of 12

Page 800 of 877


0610/42 Cambridge IGCSE – Mark Scheme May/June 2019
PUBLISHED
Question Answer Marks Guidance

5(c)(i) 4
one mark per row
letter name function

P acrosome contain enzymes / digests jelly


coat ;

Q haploid nucleus contains / AW, DNA / half


number / unpaired, single set
of / chromosomes / genes ;

R mitochondrion ; releases energy

S flagellum swimming / AW ;

5(d) drawing detail ; 3


additional drawing detail / any drawn and labelled common cell
structure e.g. nucleus, cytoplasm, cell membrane,
mitochondria / DNA / ribosome / (r)ER ;
drawn and labelled unique cell structure ; e.g. jelly (coat) / energy
store / protein-rich layer / yolk / large volume of cytoplasm

5(e) jelly coat (of fertilised egg) hardens ; 3


reference to zygote ;
mitosis / cell division ;
embryo forms ;
moves down oviduct ;
AVP ; e.g. use of nutrients in cytoplasm

© UCLES 2019 Page 11 of 12

Page 801 of 877


0610/42 Cambridge IGCSE – Mark Scheme May/June 2019
PUBLISHED
Question Answer Marks Guidance

6(a) (group of) organisms that can reproduce ; 2


produce fertile offspring ;

6(b) genetically identical ; 3


quick ;
can reproduce even if variety is sterile ;
described consequence of being genetically identical ;
AVP ; e.g. no pollinators required / reliable / no harmful variation

6(c) energy (store / sink) ; 2


example of use of starch in plant ;
as a reserve / source / store (of energy), when plant cannot
photosynthesise / dormancy / winter / no leaves / dark / night ;
AVP ; e.g. insoluble

© UCLES 2019 Page 12 of 12

Page 802 of 877


0610/43 Cambridge IGCSE – Mark Scheme May/June 2019
PUBLISHED
Question Answer Marks Guidance

1(a) nucleus ; 2
mitochondria ;
(rough) endoplasmic reticulum ;
AVP ;

1(b)(i) nitrogen 2
for making, amino acids / proteins ;
glucose
for respiration / source of energy ;

1(b)(ii) for even / uniform, distribution / concentration, of contents / AW ; 3


ensure more access of bacteria with, medium / AW ;
prevent settling / clumping of, contents / AW ;
so oxygen / glucose / nutrients, dissolves ;
so that growth is not limited ;
so bacteria can absorb substances (more easily) ;
bacteria need oxygen for respiration ;

© UCLES 2019 Page 4 of 12

Page 803 of 877


0610/43 Cambridge IGCSE – Mark Scheme May/June 2019
PUBLISHED
Question Answer Marks Guidance

1(c) description: 6
at least two correctly named phases ;
any two phases described correctly ; i.e. A: level, B: increasing,
C: level, D: decreasing (numbers of live bacteria)

correct relevant data quote for any phase ;;

A is the lag, phase / stage:


bacteria, growing / dividing, slowly ;
bacteria adjusting to conditions / absorbing resources ;

B is the exponential / log, phase / stage:


resources are, not limiting / AW ;
bacteria, growing / dividing, rapidly ;
growth / birth, rate greater than death rate ;

C is the stationary, phase / stage:


growth / birth, rate equal to death rate ;

D is the death, phase / stage:


death rate greater than growth, birth / rate ;

C or D:
many bacteria are dying ;
concentration of resources decreasing / resources become limiting ;
space becomes limiting / AW ;
(named) toxins building up ;

© UCLES 2019 Page 5 of 12

Page 804 of 877


0610/43 Cambridge IGCSE – Mark Scheme May/June 2019
PUBLISHED
Question Answer Marks Guidance

2(a) carbon dioxide + water ; 2


glucose + oxygen ;

2(b)(i) temperature is a factor that affects the rate of photosynthesis ; 2


reference to kinetic energy ;
idea of effect of temperature, on enzymes / diffusion rate (of carbon
dioxide) ;
idea that temperature is a variable that should be standardised ;
AVP ;

2(b)(ii) 74 ;;; 3

2(b)(iii) rate (of photosynthesis) increases and, reaches a plateau / AW ; 3


rate (of photosynthesis) increases until 1750 (a.u)
/ 25 µmol per m2 per s ;
any comparative use of figures for rate ;

2(b)(iv) light intensity is the limiting factor, at all light intensities used / AW ; 4
because rate of photosynthesis does not level off (even at
high light intensities) ;
carbon dioxide / temperature / chlorophyll / another factor, was not a
limiting factor ;
correct reference to (light) energy ;
light is absorbed by chlorophyll ;
AVP ;

© UCLES 2019 Page 6 of 12

Page 805 of 877


0610/43 Cambridge IGCSE – Mark Scheme May/June 2019
PUBLISHED
Question Answer Marks Guidance

3(a)(i) restriction (enzyme) ; 1

3(a)(ii) sticky ends ; 1

3(a)(iii) plasmid is cut with the same (restriction) enzyme (as DNA) ; 3
to form, sticky ends / region A, that are complementary to sticky
ends of, gene / cry ;
reference to base sequences (being complementary / AW) ;
(DNA) ligase used to join plasmid and, cry / gene ;
AVP ; e.g. formation of recombinant DNA / plasmid

3(b) gene / cry, codes for the sequence of amino acids in, (toxic) 4
protein ;
DNA / gene / cry, is copied / transcribed, to form mRNA ;
cry / gene / DNA, remains in nucleus ;
mRNA moves from nucleus to, cytoplasm / ribosome ;
mRNA passes through ribosomes / AW ;
ribosomes assemble amino acids (into protein molecules) ;
ribosomes make proteins ;
AVP ; e.g. order of amino acids determined by base sequence of,
mRNA / DNA / gene

3(c) to kill / harm / deter, caterpillars / insects / pests ; 3


higher yield / reduce losses due to, (certain) caterpillars /
insects / pests ;
higher quality of cotton ;
use less insecticide ;
(so) less pollution ;
(so) less money spent on, insecticides / spraying ;
(so) less risk to workers’ health (from using insecticides) ;
(so) less chance of insecticide resistance ;
AVP ;

© UCLES 2019 Page 7 of 12

Page 806 of 877


0610/43 Cambridge IGCSE – Mark Scheme May/June 2019
PUBLISHED
Question Answer Marks Guidance

4(a) one loop to lungs / pulmonary circulation, and one loop to rest of the 1
body / systemic circulation ;
blood flows through heart twice, for one (complete) circuit / to get
back to the same point ;

4(b) lymphocyte / AW ; 4
engulf/ digest / kill / destroy, bacteria / pathogens ;
platelet ;
red blood cell ;

4(c) wall of artery thicker than wall of vein ; 2


lumen labelled in both drawings ;

4(d)(i) arrow(s) start in right-hand side of heart in correct direction ; 2


arrow(s) point upwards inside pulmonary artery ;

© UCLES 2019 Page 8 of 12

Page 807 of 877


0610/43 Cambridge IGCSE – Mark Scheme May/June 2019
PUBLISHED
Question Answer Marks Guidance

4(d)(ii) 5 one mark per row

statement name of structure letter from Fig. 4.1

chamber that left ventricle F;


creates the highest
blood pressure

blood vessel pulmonary vein C


containing blood / aorta /A;
with the highest
concentration of
oxygen

structure that atrioventricular valve E;


prevents blood
going from ventricle
to atrium

structure that semilunar valve K;


prevents backflow
of blood from artery
to ventricle

chamber that right atrium J;


receives blood from
vena cava

© UCLES 2019 Page 9 of 12

Page 808 of 877


0610/43 Cambridge IGCSE – Mark Scheme May/June 2019
PUBLISHED
Question Answer Marks Guidance

4(e) tissue fluid drains (into lymphatic vessels) ; 3


transports tissue fluid ;
back into the blood / circulatory system ;
contains, lymphocytes / antibodies (in lymph nodes) ;
defence against infection / provide (active) immunity / AW ;
(lacteals) absorbs /transports, fats / fatty acids ;
from small intestine / duodenum / ileum ;
AVP ;

© UCLES 2019 Page 10 of 12

Page 809 of 877


0610/43 Cambridge IGCSE – Mark Scheme May/June 2019
PUBLISHED
Question Answer Marks Guidance

5(a) caused by a mutation ; 3


change in, DNA / base sequence ;
of gene for haemoglobin ;
(causes) a different sequence of amino acids ;
(so) abnormal haemoglobin produced ;
AVP ;

5(b) both parents carry the, recessive allele / allele for (sickle cell) 4
anaemia ;
both parents are heterozygous ;
half the gametes of both parents have the recessive allele ;
people / children, who are homozygous recessive have (sickle cell)
anaemia ;
there is a, 0.5 × 0.5 / 0.25, chance of, being homozygous
recessive / having (sickle cell) anaemia ;

5(c)(i) (group of) organisms that can reproduce ; 2


to produce fertile offspring ;

5(c)(ii) people with sickle cell anaemia / heterozygotes / carriers / are 5


resistant to, malaria / AW ;
people with (homozygous) sickle cell anaemia are, less likely to
survive / die of sickle cell disease ;
people who are heterozygous / have one copy of the sickle cell
allele, are more likely to survive / have selective advantage ;
idea that they are more likely to breed ;
pass on allele for sickle cell ;
so increase in frequency of sickle cell allele (in population) ;
selective advantage for sickle cell only exists where, (mosquitoes
carrying) malaria are present ;
AVP ; ref. to evolution / adaptation to (local) conditions

© UCLES 2019 Page 11 of 12

Page 810 of 877


0610/43 Cambridge IGCSE – Mark Scheme May/June 2019
PUBLISHED
Question Answer Marks Guidance

6(a)(i) stores / contains, DNA / chromosomes / genes ; 1


controls the cell ;
AVP ;

6(a)(ii) P – endoplasmic reticulum / (rough) ER / ribosome ; 2


R – mitochondrion / mitochondria ;

6(b) catalysts ; 7
starch ;
maltose / glucose ;
pepsin ;
liver ;
neutralises ;
emulsification ;

© UCLES 2019 Page 12 of 12

Page 811 of 877


0610/41 Cambridge IGCSE – Mark Scheme October/November 2019
PUBLISHED
Question Answer Marks Guidance Notes

1(a) many (body) segments ; 2


head and, body (segments) / AW ;
many legs / many pairs of legs;
elongated bodies ;

1(b) crustaceans ; 2
arachnids ;
insects ;

1(c) 3 4 rows correct = 3


class letter(s) of species from
2 or 3 rows correct = 2
Fig. 1.3 in each class
1 row correct = 1
1 J

2 L

3 M,

4 K,N,O
;;;

1(d)(i) (genus) Apheloria ; 2


(kingdom) animal ;

1(d)(ii) no (aerobic) respiration ; ora 1


cannot release energy ; ora

© UCLES 2019 Page 4 of 13

Page 812 of 877


0610/41 Cambridge IGCSE – Mark Scheme October/November 2019
PUBLISHED
Question Answer Marks Guidance Notes

2(a) carbohydrates 4
cellulose ;
for cell walls ;
starch ;
for energy/respiration ;
to attract insects to flowers / nectar / fruits ;

amino acids
to make (named) proteins ;
for enzymes ;
for growth ;

AVP ;

2(b) correct position labelled on the leaf ; 3


correct position labelled on the stem ;
correct position labelled on the root ;

2(c)(i) higher concentration in the stem / aphid D is nearer the root / is 3


before the branching of the plant ;
(sucrose moves by) translocation ;
sucrose moves up the plant ;
root / tuber, is a source ;
(leaves / stems / AW) are a sink ;
no photosynthesis (in the dark) ;
no / less, glucose/sucrose (made in the leaves) ;
plant uses stored starch (from root) / AW ;

2(c)(ii) insert gene / ref. to genetic engineering / ref. to genetic 3


modification ;
gene, for insect / aphid resistance ;
ref. to insecticide / described ;
AVP ; description of how insecticide applied / biological
control / grow in glasshouses / netting

© UCLES 2019 Page 5 of 13

Page 813 of 877


0610/41 Cambridge IGCSE – Mark Scheme October/November 2019
PUBLISHED
Question Answer Marks Guidance Notes

2(c)(iii) pollination ; 1
AVP ; e.g. biological control described / insect products e.g. honey

© UCLES 2019 Page 6 of 13

Page 814 of 877


0610/41 Cambridge IGCSE – Mark Scheme October/November 2019
PUBLISHED
Question Answer Marks Guidance Notes

3(a) remove from the, body / organism / cell ; 2


waste / poisons / toxins / harmful substances ;
(waste products) of metabolism / respiration ;
(named) substances in excess ;

3(b) the outline shape of a kidney, with one tube attached, drawn ; 2
tube labelled ureter, outer portion of kidney labelled as cortex,
medulla labelled inside the kidney ;

3(c)(i) ref. to capillaries ; 2


(capillaries are) one cell thick / thin / AW ;
idea of fenestrations / pores ;
network (of capillaries) / tangled / knotted / tightly packed tubes ;
description of shape e.g. round / ball-shaped ;

3(c)(ii) provides blood at high pressure ; 2


provides a large surface area ;
(ultra)filtration ;
ref. to small or soluble molecules / water / glucose / urea / salts, (are
filtered) out ;
ref. to (named) large OR insoluble (molecules) / blood cells, stay in
the glomerulus ;
AVP ;

3(d)(i) (by) active transport ; 4


from a low to a high concentration / AW ;
(through cell) membrane ;
ref. to proteins (pumps / channels / AW) ;
uses energy ;
from respiration ;

© UCLES 2019 Page 7 of 13

Page 815 of 877


0610/41 Cambridge IGCSE – Mark Scheme October/November 2019
PUBLISHED
Question Answer Marks Guidance Notes

3(d)(ii) 4
human mouse
575 ÷ 320 0.551 ÷ 0.31 ;

=1.797 or 1.8 =1.778 or 1.8 ;

g (salt) per day per g (kidney) ;

similar or the same, results / rates / ratios, so hypothesis is


supported ;

3(d)(iii) osmosis ; 1

3(d)(iv) glucose / AVP ; 1

© UCLES 2019 Page 8 of 13

Page 816 of 877


0610/41 Cambridge IGCSE – Mark Scheme October/November 2019
PUBLISHED
Question Answer Marks Guidance Notes

4(a) (named) mechanical (barriers) ; 5


(named) chemical barriers ;
ref. to active immunity ;
white blood cells / lymphocytes / phagocytes ;
(phagocytes) engulf (named) microorganisms / phagocytosis ;
lymphocytes produce antibodies ;
ref. to specific, antigens / pathogens ;
ref. to long term immunity / memory cells ;
AVP ;

4(b) antibiotics ; 1

© UCLES 2019 Page 9 of 13

Page 817 of 877


0610/41 Cambridge IGCSE – Mark Scheme October/November 2019
PUBLISHED
Question Answer Marks Guidance Notes

5(a)(i) coronary artery ; 1

5(a)(ii) ref. to platelets ; 3


fibrinogen converted to fibrin ;
soluble to insoluble ;
forms a mesh ;
traps, (red blood) cells ;

5(a)(iii) aspirin / AVP ; 1

5(b)(i) 98 (%) ;;; 3 one mark for correct readings from graph
one mark for correct calculation
one mark for correctly rounding to a whole number

© UCLES 2019 Page 10 of 13

Page 818 of 877


0610/41 Cambridge IGCSE – Mark Scheme October/November 2019
PUBLISHED
Question Answer Marks Guidance Notes

5(b)(ii) argument for: 5


as exercise increased CHD deaths decreased ; ora
comparative data quote with units ;
the same group of people were studied ;
regular measurements were taken ;
large benefit for doing only a small amount of exercise (therefore
easy to do) ;
even if there are some doubts about the benefits no harm will be
done / AW ;

argument against:
only women in the study ; ora
none younger than 35 (at the start of the study) ; ora
actual number of deaths per 10 000 is very small even for those
that do not exercise ;
other risk factors not considered ;
named examples of other risk factors ;;
e.g. diet / smoking / alcohol / genetics
some women may have forgotten / not answered correctly about
how much exercise they did / AW ;
some women may have been successfully treated for CHD / not
died from the condition / AW ;
other variables not considered ; e.g. pre-existing conditions
/ medication / type of exercise / length of exercise

5(c) more blood, to muscles ; 3


to deliver more, oxygen / glucose ;
for muscle contraction ;
for (aerobic) respiration ;
more energy required ;
ref. to adrenaline ;

© UCLES 2019 Page 11 of 13

Page 819 of 877


0610/41 Cambridge IGCSE – Mark Scheme October/November 2019
PUBLISHED
Question Answer Marks Guidance Notes

6(a) poor absorption of calcium / weak bones / weak 1


teeth / depression / fatigue / muscle pain / joint
pain / rickets / osteomalacia / AVP ;

6(b) reasons why endangered: 6


(described) overfishing / hunting ;
food chain disrupted (described);
overconsumption (by humans) ;
(named) pollution ;
introduced diseases / species ;
habitat destruction ;
climate change ;

risks if populations drop:


reduced variation ;
reproduction rate is lower / harder to find a mate ;
extinction ;
AVP ;

how to maintain fish stocks:


education ;
quotas ;
no-catch zones / nursery zones / seasonal fishing / protected
areas / MPAs / Marine Protected Areas ;
fines ;
restocking ;
fish farms ;
method of fishing (described) ;
AVP ;

© UCLES 2019 Page 12 of 13

Page 820 of 877


0610/41 Cambridge IGCSE – Mark Scheme October/November 2019
PUBLISHED
Question Answer Marks Guidance Notes

7(a)(i) (named) bacteria ; 2


lightning ;
AVP ;

7(a)(ii) process A 2
denitrification ;
process B
nitrification ;

7(a)(iii) ammonia / ammonium (ions) ; 1 A nitrite (ions)

7(a)(iv) removal of nitrogen containing part of amino acids ; 2


to form urea ;

7(b) ribosome / rough endoplasmic reticulum ; 1

7(c) protease / pepsin / trypsin ; 1

© UCLES 2019 Page 13 of 13

Page 821 of 877


0610/43 Cambridge IGCSE – Mark Scheme October/November 2019
PUBLISHED
Question Answer Marks Guidance

1(a)(i) (level) 3 / tertiary / secondary consumer ; 1

1(a)(ii) Myrmarachne ; 1

1(a)(iii) dichotomous (key) ; 1

1(b)(i) exoskeleton ; 2
jointed / segmented, legs / AW ;
(at least) 1 (or more up to 6) / (at least 1) (or more up to 3)
pair, of legs / AW ;
segmented (body) ;
bilateral body symmetry ;

1(b)(ii) two body parts ; 2


eight legs / four pairs of legs ;
AVP ;

1(b)(iii) B C D E (in any order) ;; 2

1(c) anatomy ; 1
DNA (sequences) / genes ;
sequences of amino acids (in proteins) ;
AVP ; e.g. behaviour

© UCLES 2019 Page 4 of 11

Page 822 of 877


0610/43 Cambridge IGCSE – Mark Scheme October/November 2019
PUBLISHED
Question Answer Marks Guidance

2(a)(i) (glucose is produced by) photosynthesis ; 3


light (energy) is, trapped / AW, by chlorophyll ;
light energy is converted to chemical energy ;
carbon dioxide and water, are used / react together / AW ;
to produce (glucose and) oxygen ;

2(a)(ii) translocation ; 1

2(a)(iii) (sometimes roots) release / AW, energy / glucose / sucrose 2


/ (named) sugar ;
for respiration ;
example of use of energy in a plant ; e.g. flowering / new,
leaves / growth / plant
sometimes leaves cannot produce enough, glucose
/ carbohydrates ;

2(b)(i) Q phloem ; 2
S xylem ;

2(b)(ii) 1 ref to osmosis (of water / across / bag / membrane) ; 4


2 water moves into, (source) bag / sucrose solution ;
3 from high water potential to low water potential ;
4 sucrose (molecules) cannot cross the (partially
permeable) membrane ;
5 sucrose is too large (to fit through partially permeable
membrane) ;
6 (water moving in) increases the pressure / volume, of
solution in (source) bag ;
7 (increased, volume / pressure / water moving in) forces
/ pushes, the solution up (tube Q) ;
8 volume of bags has not increased / water has moved
out of the (sink) bag ;
9 sucrose diffuses (along tube Q) ;
10 down a (sucrose) concentration gradient (between
source and sink) ;

© UCLES 2019 Page 5 of 11

Page 823 of 877


0610/43 Cambridge IGCSE – Mark Scheme October/November 2019
PUBLISHED
Question Answer Marks Guidance

2(c) the sucrose concentration / water potential (in the two bags) 1
is the same / AW ;

2(d) nitrate (ions) ; 1

© UCLES 2019 Page 6 of 11

Page 824 of 877


0610/43 Cambridge IGCSE – Mark Scheme October/November 2019
PUBLISHED
Question Answer Marks Guidance

3(a) 1 ingestion / digestion / described, occurs in mouth ; 8


2 chemical digestion / absorption / described, occurs in
small intestine / duodenum / ileum ;

ingestion of large biological molecules


3 mechanical / physical, digestion / breaking,
carbohydrate / food, into small pieces ;
4 ref to chew / grind / bite / by teeth / tongue / swallow /
moves through oesophagus / churned in stomach ;

chemical digestion
5 breakdown insoluble molecules into (smaller) soluble
molecules ;
6 salivary (glands) / pancreas, secrete amylase ;
7 amylase breaks down, starch, to, maltose / glucose /
sugar ;
8 maltase is on epithelium of the, small intestine /
duodenum / ileum ;
9 maltase breaks down maltose to glucose ;
10 bile neutralizes (stomach) acid ;
11 ref to neutral / 7 / 8 pH for, amylase / maltase ;

absorption into the blood via the alimentary canal


12 by diffusion / active transport, into villi / microvilli /
capillaries ;
13 microvilli / villi / folds, increase the surface area (for
absorption) ;

increased blood glucose concentration


14 insulin, secreted / produced / AW, from pancreas ;
15 insulin reduces blood glucose concentration ;
16 ref to negative feedback / homeostasis / described ;

© UCLES 2019 Page 7 of 11

Page 825 of 877


0610/43 Cambridge IGCSE – Mark Scheme October/November 2019
PUBLISHED
Question Answer Marks Guidance

3(a) assimilation in the liver


17 glucose, stored as / converted to, glycogen (in liver) ;
18 (assimilated) into, cell / tissues to become part of cell /
ref. to respiration / for release of energy ;

3(b) calcium ion: 4


for (the formation / maintenance, of healthy / strong)
bones ;
for (the formation / maintenance, of healthy / strong) teeth ;
AVP ;;

iron ions:
found in / AW, haemoglobin (molecule) / red blood cell ;
transport oxygen ;
prevent anaemia ;

3(c)(i) stress ; 2
smoking ;
genetic predisposition / family history ;
age ;
sex ;
activity level / AW ;
any pre-existing medical conditions / AW ; alcohol / drug /
medication ;
obesity / weight / mass / BMI / AW ;

3(c)(ii) (excess) salt is, excreted / removed from body (in urine) ; 2
some salt is (re)absorbed in the, kidney / tubules / into the
blood ;
people are not reliable in recording / remembering
/ measuring how much salt they eat ;

3(d)(i) (reduce) fat / cholesterol ; 1


(increase) fibre / roughage ;
(increase) water ;

© UCLES 2019 Page 8 of 11

Page 826 of 877


0610/43 Cambridge IGCSE – Mark Scheme October/November 2019
PUBLISHED
Question Answer Marks Guidance

3(d)(ii) low = 16.8 (kPa) and high = 17.7 (kPa) ; 3


5(%) ;;

3(d)(iii) low salt diets reduce (systolic) blood pressure / risk of 3


CHD ; ora
modified diets / group 2, reduce (systolic) blood pressure
/ risk of CHD ; ora
any description of an interaction between the salt diet and
modified diets together affecting the, blood pressure / risk of
CHD ;
comparative data quote with units ;

4(a)(i) use of trees 3


paper ;
(as building materials) for furniture / construction / poles
/ boats / AW ;
firewood / fuel ;
to sell ;

clearance of trees for


agriculture ;
urbanisation / roads / housing / factories / industry
/ developments ;
extraction of, minerals / natural resources ;

4(a)(ii) education ; 3
replanting / reforestation / afforestation ;
government policies / legal quota / penalties / controls
/ bans / rules / AW ;
management of conflicting demands ;
co-operation with local communities ;
protected area / national parks / wardens ;
AVP ; e.g. selective felling

4(b)(i) house mouse ; 1

© UCLES 2019 Page 9 of 11

Page 827 of 877


0610/43 Cambridge IGCSE – Mark Scheme October/November 2019
PUBLISHED
Question Answer Marks Guidance

4(b)(ii) for hypothesis 4


all native / nearly all / more / most, species prefer large
areas of forest ; ora
comparative data quote for one mammal, between both
areas as a percentage or last column ;
heavier / larger, mammals are more (negatively) affected by
the breakup of large areas (than smaller mammals) ; ora

against hypothesis
not true for, introduced species / black rat / house mouse ;
comparative data quote for black rat / mouse, between both
areas as a percentage ;
cannot make (broad) conclusion on, only study / one area ;
AVP ;

Question Answer Marks Guidance

5(a) 1 nucleus / chromosome(s) ; 9


2 bases ;
3 protein ;

4 genes / alleles ;
5 same ;
6 sticky ;

7 recombinant ;
8 bacteria / vectors / viruses / yeast / (prokaryotic / host)
cells ;
9 insulin ;

5(b) washing powders ; 2


(pectinase) for (fruit) juice production ;
(lactase) for lactose-free milk ;
AVP ;

© UCLES 2019 Page 10 of 11

Page 828 of 877


0610/43 Cambridge IGCSE – Mark Scheme October/November 2019
PUBLISHED
Question Answer Marks Guidance

6(a) ref. to platelets ; 5


fibrinogen is converted to fibrin / L ;
fibrinogen is soluble / fibrin is insoluble ;
(L / fibrin) forms a, mesh / AW ;
(L / fibrin) traps / AW, blood cells / J / M ;
J is a red (blood) cell ;
L is fibrin ;
M is a, white (blood) cell / lymphocyte / phagocyte ;

6(b) prevents blood loss / AW ; 2


prevent (named) pathogens entering a wound ;

6(c)(i) observable features (of an organism) ; 1

6(c)(ii) co-dominance ; 1

6(c)(iii) IAIA ; 2
IAIo ;

© UCLES 2019 Page 11 of 11

Page 829 of 877


0610/42 Cambridge IGCSE – Mark Scheme March 2019
PUBLISHED
Question Answer Marks Guidance

1(a) removal from the, body / organism / cell ; 3


toxic substances ;
waste product(s), of metabolism / respiration ;
(named) substances in excess (of requirements) ;

1(b) A – medulla ; 3
B – cortex ;
C – renal vein ;

1(c)(i) 9900 (%) ;; 2

1(c)(ii) concentration of salts is the same in the blood in the renal artery and the fluid 2
in the kidney tubule ;
concentration of salts, increases between the blood in the renal artery and
the fluid in the kidney tubule and the urine / (approximately) doubles ;

1(c)(iii) proteins are (too) big (to be filtered) / AW ; 1

1(c)(iv) (all) glucose is reabsorbed ; 1

1(d) idea of convenience: no need to visit hospital / time not taken up with dialysis ; 3
no need for a restricted diet ;
idea of improved quality of life / AW ;
cost effective in the long term ;
ref. to having a working kidney / long-lasting / ref. to cure / one-time treatment ;

© UCLES 2019 Page 4 of 9

Page 830 of 877


0610/42 Cambridge IGCSE – Mark Scheme March 2019
PUBLISHED
Question Answer Marks Guidance

2(a)(i) (large) petals ; 1

2(a)(ii) self-pollination is within the same, plant / flower ; 1


OR
cross-pollination is between different plants (of the same species) ;

2(a)(iii) prevents extinction / enables survival of species ; 3


more chances of fertilisation ;
more chances of pollination ;
no need for (named) pollinators ;
useful if plants are (geographically) isolated / on their own / AW ;
parent plants adapted to the environment, pass alleles to offspring / AW ;
idea of sexual reproduction better than asexual reproduction for variation ;

2(b)(i) both alleles are, expressed / neither allele is, dominant / recessive to the other ; 2

the phenotype (of heterozygote is), intermediate / new / different / AW ;

presence of multiple alleles, for one trait ;

2(b)(ii) gametes: CR , CW + CR , CW ; 4

offspring genotypes: CRCR, CRCW, CRCW, CWCW ;


offspring phenotypes: red pink white ;
ratio: 1 : 2 : 1;

2(b)(iii) (parents phenotypes must be) red and white ; 2


offspring must, inherit a CR and a CW allele / be heterozygous ;

© UCLES 2019 Page 5 of 9

Page 831 of 877


0610/42 Cambridge IGCSE – Mark Scheme March 2019
PUBLISHED
Question Answer Marks Guidance

3(a) (A) D C F B G (E) ; 1

3(b) correct ref. to neurotransmitter ; 4


released from vesicles (into synapse / synaptic gap) ;
ref. to movement (of neurotransmitter) by diffusion ;
(neurotransmitter) binds with receptor (molecules on neurone on the other side
of synapse) ;
causing impulse to continue / AW ;

3(c)(i) depressant ; 3
increases reaction times / slows down impulses / slows down the nervous
system ;
ref. to effect on synapse / AW ;
loss of self-control / mental function is affected / inability to walk or talk ;
AVP ;;;

3(c)(ii) sleeplessness / insomnia / restlessness ; 2


fatigue / yawning ;
diarrhoea ;
hallucinations ;
anxiety / depression / mood swings / aggression / irritation / confusion ;
muscle cramps / aches / pain ;
nausea / vomiting ;
headaches / dizziness ;
shivering / chills / fever ;
itching ;
runny nose / sweating / clammy skin / crying ;
dehydration / (extreme) thirst ;
rapid heart rate / hypotension ;
AVP ; cravings / double vision

3(c)(iii) (addicts) turn to crime to finance their addiction / AW ; 1

© UCLES 2019 Page 6 of 9

Page 832 of 877


0610/42 Cambridge IGCSE – Mark Scheme March 2019
PUBLISHED
Question Answer Marks Guidance

4(a) 4
pollutant source effect on environment

sulfur dioxide ; (burning) fossil fuels ; acid rain

enhanced greenhouse
carbon dioxide (burning fossil) fuels ;
effect

enhanced greenhouse
methane ; cattle and rice farming
effect

fertilisers arable agriculture eutrophication

4(b)(i) (named) fertiliser leached into, rivers / streams / lakes ; 6


producer growth / algal bloom / algae growth / plant growth ;
death of producers ;
increased decomposition / increased decomposers ;
increased respiration (aerobic) ;
decomposers use up the oxygen in the water ;
organisms / fish / creatures, die / suffocate / migrate, due to lack of oxygen ;
AVP ;

4(b)(ii) use, less / correct amount, of fertiliser / calculate how much fertiliser is needed ; 2
use slow-release fertilisers ;
do not apply, during / after, rain / when rain is forecast / limit watering ;
do not use near water / make channels between land and water body ;
only apply when crops, will take-up fertiliser / are growing ; ora

© UCLES 2019 Page 7 of 9

Page 833 of 877


0610/42 Cambridge IGCSE – Mark Scheme March 2019
PUBLISHED
Question Answer Marks Guidance

5(a)(i) 1 leaf A / thick cuticle, lost the least mass / water ; 5


2 leaf C / thin cuticle, lost the most mass / water ;
3 leaf B / medium cuticle, lost more mass or water than A / less mass or
water than C ;
4 loss of mass is due to the loss of water ;
5 the thicker the cuticle the, less water / mass, lost ;
6 loss of, mass / water, fastest initially (then slows) ;
7 ref. to transpiration / (reduced) evaporation ;
8 data manipulation with ref. to mass ; e.g. calculation of loss

5(a)(ii) less, water / mass, loss (from leaves A, B and C) ; 3


idea of the same pattern of results as the first experiment ;
reduced transpiration ;
increased water (concentration) in the air ;
ref. to a smaller concentration gradient ;
less diffusion of water vapour ;

5(b) stem – swollen / AW ; 2


root – extensive / widespread / shallow (root system) / AW ; A deep / branched

5(c)(i) carbon dioxide ; 1

5(c)(ii) respiration ; 3
starch ;
cellulose ;
chlorophyll ;
sucrose ;
nectar ;
amino acids / protein ;
fats / oils ;
nucleic acids / DNA / RNA ;
growth of (any named part) membrane, cell wall, cytoplasm ;

© UCLES 2019 Page 8 of 9

Page 834 of 877


0610/42 Cambridge IGCSE – Mark Scheme March 2019
PUBLISHED
Question Answer Marks Guidance

6(a)(i) X protein (coat) / AW ; 2


Y genetic material ;

6(a)(ii) cell wall ; 2


cell membrane ;
cytoplasm ;
loop of DNA ;
(slime) capsule ;
flagella ;
plasmid ;
ribosome ;

6(b)(i) food ; 2
contaminated surfaces ;
animals ;
air ;
water ;
soil ;

6(b)(ii) skin / epidermis ; 2


hairs in nose ;
mucous membrane ;
mucus ;
ear wax ;

6(c) a pathogen has antigens (on their surface) ; 4


ref. to specificity / antibodies have complementary shape (to antigen) ;
lock onto (specific) antigens / pathogen ;
destroy pathogens ;
marking for destruction / phagocytosis ;
AVP ;

6(d) long-term / ref. to permanent / protects against some recurring diseases ; 3


produces memory cells ;
antibodies are produced (by the body) ;
slower response ;

© UCLES 2019 Page 9 of 9

Page 835 of 877


0610/41 Cambridge IGCSE – Mark Scheme May/June 2018
PUBLISHED
Question Answer Marks Guidance

1(a) A substrate ; 4
B active site ;
C enzyme-substrate complex ;
D product(s) ;

1(b) production of, small(er) / soluble / simple(r), molecules ; 2

(small molecules can be) absorbed / ref. to absorption ; ora

(moves through) cell membranes / wall of intestine / into blood / into cells ;

1(c) 6 one mark per row


the letter must agree with the name
function letter from Fig. name of structure if more than one letter or name mark first one
2.1 only
site of starch digestion A mouth / buccal cavity
J/E small intestine
reabsorption of water J/E small intestine
H colon / large intestine
F rectum
secretion of pepsin C stomach
site of maltose digestion J/E small intestine
secretion of bile K liver
L gall bladder
storage of faeces F rectum
secretion of lipase and D pancreas A J/E small intestine
trypsin
;;;;;;

© UCLES 2018 Page 5 of 10

Page 836 of 877


0610/41 Cambridge IGCSE – Mark Scheme May/June 2018
PUBLISHED
Question Answer Marks Guidance

2(a) (the probability that an organism will) survive and, reproduce / AW ; 1

2(b)(i) (during the day they) remain in a burrow / stay in the shade / stay 2 A nocturnal (habit)
inactive / sleep / AW ;
light colour / pale / yellow / white (fur / hair) / AW ;
thin / long, tail / legs ;
thin / short / little, fur ;
no fur on legs ;
fur on feet ;
large, ears / pinna(e) ;
little / no, fat ;
large surface area: volume ratio ;
produces, little / concentrated, urine ; A ref. to long kidney tubules / AW
AVP ;

2(b)(ii) big eyes / large pupils / good eyesight ; 2


whiskers ;
lots of rods (in the retina / fovea) ; A eye has no cones
large ears / good sense of hearing / sensitive ears ;
good sense of smell ;

2(c)(i) block added to the top of the pyramid that is 4 small squares wide ; 2
labelled carnivores ;

2(c)(ii) (detritivores) eat (mainly), plants / producers ; 1


(detritivores) feed, at second trophic level / as primary consumers ;
detritivores are eaten by, third trophic level / secondary consumers ;

2(c)(iii) little energy is transferred from one trophic level to the next ; ora 2
not all of the organisms are, eaten / digested / absorbed ;
named example of energy loss ;
idea that not enough energy to support higher trophic levels ;

© UCLES 2018 Page 6 of 10

Page 837 of 877


0610/41 Cambridge IGCSE – Mark Scheme May/June 2018
PUBLISHED
Question Answer Marks Guidance

2(c)(iv) idea that 3


in a pyramid of numbers one large individual is shown in the same way as
one very tiny individual ; ora
biomass indicates how much food there is, available / left ;
biomass is an indicator of the energy available ;
pyramid of biomass is pyramid shaped whereas a pyramid of numbers is not
always ; ora
AVP ;

Question Answer Marks Guidance

3(a)(i) label line and X pointing to any part of the ‘star’ in the centre of the root 1
section ;

3(a)(ii) composed of (group of) cells with similar structures ; 2


working together to perform shared functions ;

3(b) xylem supplies water ; 3


air spaces ;
large (internal) surface area ;
water evaporates from surface of mesophyll cells ;
guard cells, open / close, stomata ;
water vapour, diffuses / moves, out through stomata ;

Question Answer Marks Guidance

4(a)(i) sensory neurone / (temperature / thermo-) receptor (neurone) ; 1

4(a)(ii) Q venule ; 3
S arteriole ;
T capillary ;

4(a)(iii) fat / fatty tissue ; 1

© UCLES 2018 Page 7 of 10

Page 838 of 877


0610/41 Cambridge IGCSE – Mark Scheme May/June 2018
PUBLISHED
Question Answer Marks Guidance

4(b)(i) blood flow remains constant and then increases / AW ; 3


blood flow remains at, 4 / 5% ;
increase in blood flow from 25 (±2) ºC ;
to, maximum / 100%, at 41 ºC ;

4(b)(ii) detection by, sensory neurone / receptor (in skin) ; 3


brain / hypothalamus, as control centre / AW ; A brain / hypothalamus, detecting temperature
impulses in, motor / effector, neurones ;
muscles in, shunt vessels contract / arterioles relax ;
so shunt vessels, constrict / close ;
arterioles dilate / vasodilation ;
increased / more, blood flow, into capillaries / near surface (of skin) ;

4(b)(iii) 46 (%) ; 1

4(b)(iv) diffusion ; 3
down concentration gradient / high to low concentration ;
active transport ;
through epidermis ;
between / into / through, cells ;
across cell membranes ;
AVP ;

4(c) (so that) enzymes do not denature / enzymes remain active / maintains 4
optimum temperature for enzymes ;
idea of maintaining a constant rate of, reactions / metabolism / respiration ;
avoids to damage to other named (type of) protein ;
avoids damage to cell membranes ;
avoids, heatstroke / hyperthermia / overheating / dehydration
/ freezing / chills / becoming too cold / hypothermia ;
at high temperature sperm production, reduced / harmed ;
AVP ; e.g. (permits) colonisation of different parts of
the world / different climates active in, both day
and night / different seasons

© UCLES 2018 Page 8 of 10

Page 839 of 877


0610/41 Cambridge IGCSE – Mark Scheme May/June 2018
PUBLISHED
Question Answer Marks Guidance

4(d) hormones are chemicals / hormonal coordination is only chemical ; 3


transported in the, blood / circulatory system ;
(effects are) slower (than nerves) ; ora
(effects are) longer lasting ; ora
each hormone may have more than one target, organ / tissue / cells ; ora

Question Answer Marks Guidance

5(a) C6H12O6 + 6O2 → 6CO2 + 6H2O (+ energy released) ;; 2 one mark for correct symbols
one mark for correct balancing

5(b) 150(%) ;; 2 one mark for correct working if answer wrong

5(c) demand for, energy / oxygen, increases ; 3


(rate of) respiration increases ;
limited supply of oxygen to muscle (tissue) ;
idea that heart / pulse / breathing, rate not increased enough ;
muscles respire anaerobically ;
lactic acid is produced ;

5(d) horses continue to breathe, at high rate / deeper ; 4


continue with a high, heart / pulse, rate ;
to provide, enough / AW, oxygen (to ‘pay-off’ the debt) ;
lactic acid, moves / diffuses / AW, (from muscle) into blood ;
lactic acid transported to the liver ;
(in the liver) lactic acid is, broken down / oxidised / respired (aerobically) ;

6(a)(i) process / event letter from Fig. 6.1 3

meiosis R;
fertilisation S;
implantation V;

© UCLES 2018 Page 9 of 10

Page 840 of 877


0610/41 Cambridge IGCSE – Mark Scheme May/June 2018
PUBLISHED
Question Answer Marks Guidance

6(a)(ii) oviduct ; 1

6(b)(i) image size ÷ actual size ; 1

6(b)(ii) 55 (µm) ; 1

6(c) haploid / n / one set of chromosomes / half the diploid number 2


/ 23 chromosomes ;
(produced by) meiosis ;
so number of chromosomes, remains the same / does not double at
fertilisation ; A so diploid number restored at fertilisation / so
zygote is diploid

6(d) flagellum 6
(flagellum) propels the sperm ; A flagellum allows sperm to swim
to, oviduct / site of fertilisation / egg (cell) / ovum ;

mitochondria
aerobic respiration ;
provides / releases / supplies, energy / ATP ;
R 'produces energy'
acrosome
(contains / has / releases) enzyme(s) ;
(enzymes) digest / break down / dissolve, jelly coat / protein layer ;
so sperm nucleus can enter the egg cell / so sperm and egg membranes
can fuse together ;

6(e) idea that sex is determined by X and Y chromosomes / males are XY and 2
females are XX ;
egg cells have X chromosome / females can only provide X chromosome ;
sperm cells have X or Y chromosome / only the males can provide X or Y
chromosome / only males can provide the Y chromosome ;

© UCLES 2018 Page 10 of 10

Page 841 of 877


0610/42 Cambridge IGCSE – Mark Scheme May/June 2018
PUBLISHED
Question Answer Marks Guidance

1(a)(i) A dentine 3 6 / 5 correct = 3 marks


B cement 3 / 4 = 2 marks
C incisors 1 / 2 = 1 mark
D canine(s)
E premolars
F molars
;;;

1(a)(ii) mechanical ; 1

1(b)(i) acid ; 1 A carbon dioxide

1(b)(ii) enamel ; 2
dentine ;

1(c) (named) sugar ; 1

Question Answer Marks Guidance

2(a)(i) the probability of an organism will surviving and reproducing (in the 1
environment in which it is found) / AW ;

2(a)(ii) aerial roots 2


for anchorage / stability (in flowing water) / (aerobic) respiration
/ gas exchange / oxygen absorption ;

floating seeds
for (seed) dispersal (carried on water) / reduce competition (from parent)
/ access to oxygen (to germinate / respire) ;

2(b)(i) 1.1 (g) ;; 2 one mark for correct working if answer wrong
(8000 / 7 500000) × 1000

© UCLES 2018 Page 5 of 11

Page 842 of 877


061
10/42 Cambridg
ge IGCSE – Marrk Scheme May/June 2018
PUBLISHED
Que
estion Answ
wer Marks Guidance
e

2((b)(ii) bottom
m bar is narrowest ; 3
middle
e bar is widest ;
correct numbering of trophic
t levels ;

2((b)(iii) pyramid of biomass is pyramid-shaped d (and pyramid of numbers is 4


not) ; o
ora
mangrrove trees have a larger biomasss (than crabs) ; ora
so one e tree provides fo
ood for many craabs / AW ;
(one) ccrab has a smaller biomass than n (one) seagull ; ora
each sseagull needs to o eat many crabss / AW ;
(total) crab biomass iss greater than (to
otal) seagull biom
mass ; ora
compa arative data to su
upport an argumment with units ;

Que
estion Answ
wer Marks Guidance
e

3(a)(i) sucrosse / sugar ; 2


amino acids ;

3((a)(ii) translo
ocation ; 4
(phloem) allows bidireectional moveme ent / AW ;
movem ment (of food / saap) from source to sink ;
sucrosse / amino acids / food, are produced / taken from m storage, at
a sourrce ;
region of respiration / storage
s / growth, is a sink ;
named d example of a, source
s / sink (in the correct conttext) ;
some o organs can be both
b a source or a sink at differe
ent times ;

© UCLES 2018 Page 6 of 11

Page 843 of 877


0610/42 Cambridge IGCSE – Mark Scheme May/June 2018
PUBLISHED
Question Answer Marks Guidance

3(b) functions 6 max 5 from one section


conduct / transport, water (and mineral ions) ;
ref to transpiration ;
reduced resistance to water flow / AW ;
structural) support (for plant) ;
prevents (inward) collapse (of xylem vessels) ;
(spirals) allows (some) flexibility / bending, of stems
(to prevent breaking) ;

adaptations
long / elongated (cells / vessels / tubes) ;
ref to lignin (in walls) ;
(cell walls) are water impermeable / waterproof / AW ;
(secondary) thickening of cell walls ;
hollow / no cytoplasm / no (named) organelles ; A rings / spirals / AW
no, end / cross, walls (between cells) ;
end plates to connect vessels (end to end) ;
pits in walls (for water movement between vessels) ;

3(c) reduced / no, damage to crops ; ora 2 A not / less, eaten by pests
increased, yield / quality (of the crop) ; ora
more, income / profit ; ora
because more, sugar / amino acids, available for growth ; ora
reduced disease transmission / AW ; ora

© UCLES 2018 Page 7 of 11

Page 844 of 877


0610/42 Cambridge IGCSE – Mark Scheme May/June 2018
PUBLISHED
Question Answer Marks Guidance

4(a) there are many, diseases / infections / pathogens / transmitted through 3


sexual contact ;
named example of STI ;
STIs / AW, can be prevented by the use of some (contraceptive)
methods ;
such as, condoms / femidoms ;
for education about STI prevention / inform preventative strategies / AW ;
assess effectiveness of different (contraceptive) methods (to prevent
disease) ;

4(b)(i) (named) oestrogen ; 2


(named) progesterone ;

4(b)(ii) (FSH would) stimulate an egg / follicle, to mature / develop / grow 3 I production (of eggs)
/ ripen ; ora
(FSH would) stimulate (release of) oestrogen / LH ; ora
(FSH would) lead to ovulation ; ora
(FSH would) increase the chance of fertilisation / pregnancy / AW ; ora A FSH is a fertility drug

4(b)(iii) implant / patch / injection / IUD / IUS (containing contraceptive hormones) ; 1 I birth control pills
spermicide ;

4(b)(iv) abstinence / body temperature / cervical mucus / natural contraception ;; 2 I birth control pills
diaphragm ; A cap
(named) surgical (sterilisation) method ;; A (named) tubes tied

4(b)(v) some females could use more than one method of contraception ; 1
some people may not have completed the survey, correctly A not used a method regularly (so not answered
/ honestly / AW ; all questions accurately)

© UCLES 2018 Page 8 of 11

Page 845 of 877


0610/42 Cambridge IGCSE – Mark Scheme May/June 2018
PUBLISHED
Question Answer Marks Guidance

5(a) three pairs of legs ; 3


three (named) body segments ;
wings ;
(pair of) antennae ;
compound eyes ;

5(b) C6H12O6 + 6O2 → 6CO2 + 6H2O (+ energy released) ;; 2 one mark for correct symbols
one mark for correct balancing

5(c)(i) volume ; 3
distance / length ;
control / maintain / regulate / stabilise / keep / constant / sustain ;

5(c)(ii) carbon dioxide will affect, results / volume of gas 1


(in respirometer) / carbon dioxide could kill the larvae ; A to measure (changes in) oxygen only

5(c)(iii) growth / development ; 2


active transport ;
protein synthesis ;
cell division / mitosis ;
passage of nerve impulses ;
muscle contraction ; A movement / breathe / egestion /digestion
/ excretion
AVP ; e.g. metabolism / (description of) metamorphosis

© UCLES 2018 Page 9 of 11

Page 846 of 877


0610/42 Cambridge IGCSE – Mark Scheme May/June 2018
PUBLISHED
Question Answer Marks Guidance

5(d) prediction 4
as temperature increases the respiration rate will increase ; ora
and then decrease ;

explanation:
there will be an optimum temperature (at a particular temperature) for max 3 for explanation
seed germination ;
ref to (respiratory / germination) enzymes ;
at high temperatures enzymes denature / described ;
at low temperatures not enough (kinetic) energy for, effective
collisions / biochemical reactions / respiration / digestion ; ora e.g. temperature will also affect the gas pressure
AVP ; in the respirometer

Question Answer Marks Guidance

6(a) (asexual) reproduction ; 1 R sexual reproduction

6(b)(i) image size ÷ magnification ; 1

6(b)(ii) 0.8 (µm) ; 1

6(c)(i) (Type 1) diabetes ; 1 A Type 2 diabetes

6(c)(ii) ref. to (human) gene / DNA that codes for (human) protein ; 5 procedures must be in the correct sequence
(human) DNA / gene, is, identified / isolated ;
DNA / gene / plasmid, cut (out) using restriction enzymes ;
forming, complementary / sticky, ends ;
DNA / gene / plasmid, cut with the same restriction enzymes ;
formation of recombinant, DNA / plasmid ; into plasmid
(DNA) ligase used to join plasmid and, gene / DNA ;
plasmids (with gene) inserted into bacteria ;
bacteria (with the plasmid), replicate / reproduce / multiply ;
AVP ; e.g. purification / identification of transformed bacteria /

© UCLES 2018 Page 10 of 11

Page 847 of 877


0610/42 Cambridge IGCSE – Mark Scheme May/June 2018
PUBLISHED
Question Answer Marks Guidance

6(d)(i) kills (named) microorganisms / pathogens ; 2


prevents contamination (by bacteria / microorganisms) ;
steam does not contaminant, product / medicines (with chemicals) ;
steam reaches all the crevices of fermenter ;

6(d)(ii) pH ; 3
temperature ;
oxygen ;
carbon dioxide ;
(named) nutrients ;
waste ;
turbidity ;
AVP ; (gas) pressure / rate of stirring / amount of (named) product

6(d)(iii) penicillin ; 1
AVP ;

6(e) disease resistance ; 4


large(r) / fast(er), yield ;
drought resistance ;
salt resistance ;
frost resistance ;
(named) nutritional enrichment ;
pest / insect, resistance ;
herbicide resistance ;
vaccine production ;
ref to benefits to, environment ;
ref to more desirable, product / increased income / AW ;
ref to a qualified benefit to humans ; e.g. food shortage / described
health benefit
AVP ; growth modification e.g. short stems / adaptations to extreme
environments / rapid improvement to crop / improvements using
characteristic that are not present in natural population

© UCLES 2018 Page 11 of 11

Page 848 of 877


0610/43 Cambridge IGCSE – Mark Scheme May/June 2018
PUBLISHED
Question Answer Marks Guidance

1(a) (food) is broken down into smaller pieces (without chemical change) ; 4

sites of mechanical digestion:


mouth / buccal cavity (in context mechanical) ;
stomach (in context of mechanical) ;

chewing / mastication ;
role of a named teeth ;;
ref to involvement of tongue ;
ref to movement of the jaw ;
churning / muscular, action of the stomach ;

1(b) part of the alimentary 3 one mark per row


enzyme substrate product(s)
canal
mouth amylase starch maltose
stomach pepsin protein peptides A protease (for enzyme)

small intestine / lipase fat fatty acids and R pancreas (for part of the alimentary canal)
duodenum / ileum glycerol
;;;

1(c)(i) glycogen ; 1

1(c)(ii) antibody ; 1

1(c)(iii) (thermal) insulation ; 1 A storage / protection

© UCLES 2018 Page 5 of 12

Page 849 of 877


0610/43 Cambridge IGCSE – Mark Scheme May/June 2018
PUBLISHED
Question Answer Marks Guidance

2(a)(i) (external) ears / pinna(e) ; 2


fur / hair ;
whiskers ;

2(a)(ii) inherited / genetic, feature ; 3


result of natural selection ;
increases fitness ;
increases chances of survival / AW ;
increases chances of, reproducing / AW ;

2(b) temperature ; 3
light (intensity) ;
water (supply) / idea that water is not available (as frozen) ; A humidity
(named) soil feature ;
(named) mineral ion ;
carbon dioxide ;
grazing / predation ;
(competition for) space ;
disease ;
(named) pollutant ;

© UCLES 2018 Page 6 of 12

Page 850 of 877


0610/43 Cambridge IGCSE – Mark Scheme May/June 2018
PUBLISHED
Question Answer Marks Guidance

2(c) little energy available from, herbivores / primary consumers / lower trophic 6
level(s) ;
few producers / low population of producers / AW ;
energy is lost, between / within, trophic levels / along food chain ; ora
ref to 10 % energy transfer / 90% energy loss (between trophic levels) ; ora
energy lost, in named process ;;;
low numbers of, prey / (primary) consumers / food ;

wolves not very successful at catching prey ;


reason why ; e.g. prey are widely dispersed / larger animals

lower reproductive rate / higher mortality of wolves ;

hunting / killed by people ;


reason why ; e.g. for fur / compete with humans for food

habitat destruction (by humans) ;


reason why ; e.g. road building / oil exploration / melting of snow

disruption of food web (described) ;


disease ;
inbreeding / reduced genetic diversity ;
climate change / global warming ;

© UCLES 2018 Page 7 of 12

Page 851 of 877


0610/43 Cambridge IGCSE – Mark Scheme May/June 2018
PUBLISHED
Question Answer Marks Guidance

3(a)(i) A (upper) epidermis ; 2


B palisade (mesophyll) ;

3(a)(ii) (cell surfaces are sites of) gas exchange ; 3


movement of gases by diffusion ;
ref. to efficient / faster / AW, gas exchange / diffusion / photosynthesis ;
carbon dioxide is, raw material / needed, for photosynthesis ;
absorption of carbon dioxide (when light available) ;
loss of oxygen (when light available) / absorption of oxygen ;
oxygen is required for (aerobic) respiration ;
more evaporation ;
idea of maximising light absorption ;

3(a)(iii) allows for, movement of (named) gases / diffusion / gas exchange, throughout 2
the whole of the leaf ;
ref. to faster / efficient / AW, diffusion / gas exchange ;
allows / AW, photosynthesis / respiration / transpiration / evaporation ;
ref. to storage of carbon dioxide ;
(air spaces) connect (to outside air) via stomata ;

3(b)(i) no / little, water ; 2 A drought / no, rainfall / precipitation /


high temperature ; irrigation
low humidity / dry air ;
high wind speed ;
long day length / high light intensity ;
high salinity / salt ;
freezing ;
disease ;
(soil) waterlogging / low oxygen concentration / pH ;
mineral / magnesium, deficiency ;

© UCLES 2018 Page 8 of 12

Page 852 of 877


0610/43 Cambridge IGCSE – Mark Scheme May/June 2018
PUBLISHED
Question Answer Marks Guidance

3(b)(ii) ref to osmosis ; 4


water, lost from / moves out of, cells / vacuoles ;
down water potential gradient ;
pressure of, water / cell contents, on (inelastic) cell wall decreases ;
correct ref. to turgor / turgidity / flaccid / plasmolysed ;
ref. to plants / cells, rely on water, for (structural) support / to
prevent wilting ; ora
water in cells not being replaced as quickly (as it is being lost) ;
AVP ;;

3(b)(iii) stomata close ; 2


to prevent more water loss ;
water conserved for, other processes / other parts of plant ; A reduces transpiration
decrease surface area, exposed to the Sun / for absorption of heat ;

Question Answer Marks Guidance

4(a)(i) chemical / substance, produced by a gland ; 2


transported in the blood (plasma) ;
alters the activity of one of more specific target, organs / tissues / cells ;

4(b)(i) retina ; 1

4(b)(ii) fovea ; 1

4(b)(iii) sensory (neurone / nerve cell) ; 1

4(b)(iv) optic ; 1

© UCLES 2018 Page 9 of 12

Page 853 of 877


0610/43 Cambridge IGCSE – Mark Scheme May/June 2018
PUBLISHED
Question Answer Marks Guidance

4(b)(v) spinal cord ; 1

4(b)(vi) adrenal (gland) ; 1

4(c) organ effect of the hormone 4

heart increased, pulse / heart, rate / beat ;


conversion of glycogen to glucose /
liver
increased blood glucose (concentration) ;
lungs increase, (rate) / depth, of breathing ;
dilated pupils /
eyes
radial muscles (in iris) contract ;

4(d) nervous system, responds quickly / immediately ; ora 4


idea that (nerve) impulses travel to, (specific) muscles / (adrenal) glands
/ effector(s) ;
effects of endocrine system are long lasting ; ora
hormones / adrenaline, travels throughout the body / allows multiple
(target) organs to respond (to same signal) ;
idea that less energy required than to have nerves going to every, cell / tissue ;
stimulate both voluntary and involuntary responses (simultaneously) ;
more effective / enhanced, response (than using one system alone) ;

4(e)(i) auxin ; 1

4(e)(ii) 2,4-D is a weedkiller / AW ; 2


sprayed on crops / applied to leaves ;
specific to broadleaved weeds ;
does not affect, cereals / narrow-leaved crop plants ;
causes uncontrolled growth ;

© UCLES 2018 Page 10 of 12

Page 854 of 877


0610/43 Cambridge IGCSE – Mark Scheme May/June 2018
PUBLISHED
Question Answer Marks Guidance

5(a) C6H12O6 + 6O2 → 6CO2 + 6H2O (+ energy released) ;; 2 one mark for correct symbols
one mark for correct balancing

5(b) 0.42 (ppm s–1) ; 1

5(c) to allow oxygen to enter the chamber ; 2


keep the crickets respiring aerobically ;
to remove carbon dioxide ;
to prevent death of crickets ;
ref. to ethical treatment of animals ;
maintaining similar conditions / resetting, for repeat readings / AW ;

5(d) heat (energy) is released by crickets ; 2


movement / ref. to kinetic energy ;
pressure increase ;
increased carbon dioxide leading to greenhouse effect ;
small closed space ;

5(e) rate of oxygen consumption increases with body mass of crickets 4 A respiration for oxygen consumption
(for each temperature) ;
any suitable data quote comparing rate at different masses (at same
temperature) ;
rate of oxygen consumption increases with temperature ;
any suitable data quote comparing rate at two temperatures (for the same
body mass) ;

© UCLES 2018 Page 11 of 12

Page 855 of 877


0610/43 Cambridge IGCSE – Mark Scheme May/June 2018
PUBLISHED
Question Answer Marks Guidance

6(a) name of the 4 one mark per row


process / event letter from Fig. 6.1
organ
meiosis to produce pollen grains C anther
pollination D stigma
development of seeds E ovary
protection of flower in the bud A sepal
;;;;

6(b)(i) image size ÷ magnification ; 1

6(b)(ii) 82 (µm) ; 1

6(b)(iii) (covered in) spikes / sticky ; 2


(pollen) sticks to, insect / animal (bodies / legs / AW) ;
large(r) size (in comparison with wind) ;
AVP ;

6(c)(i) ovule ; 1

6(c)(ii) (nucleus) containing one set of (unpaired) chromosomes ; 1

6(c)(iii) so that chromosome number does not double (at fertilisation) ; 1


so that chromosome number remains constant from generation to generation ;

© UCLES 2018 Page 12 of 12

Page 856 of 877


0610/41 Cambridge IGCSE – Mark Scheme October/November 2018
PUBLISHED
Question Answer Marks Guidance

1(a)(i) sun / light ; 1

1(a)(ii) C; 1

1(a)(iii) ingestion / feeding / AW ; 1

1(a)(iv) energy is lost (from the food chain as it is transferred from one trophic level to 3 A energy transfer is inefficient
the next) / energy decreases up the trophic levels ;
only 10% energy transferred ; ora
energy is lost as heat / in respiration / in (named) metabolic
processes / movement ;
not all organisms (in one trophic level) are eaten / not all parts of the
organisms are eaten ;
not all nutrients in the organisms are, eaten / digested / absorbed
some energy is lost in, excretion / urine / faeces ; A egestion
some energy is transferred to decomposers ;

1(b)(i) organisms that get energy from dead / waste, (organic) material ; 1

1(b)(ii) respiration ; 1

1(c) combustion / burning ; 5


(more / less) fossil fuel is used ;
concentration of (atmospheric) carbon dioxide is increasing ;
deforestation described ;
trees not replanted / fewer trees ; ora
described effect on photosynthesis ;
carbon dioxide released (into the atmosphere), as the trees are burnt
/ decay ;
causing, global warming / enhanced greenhouse effect ;
ref. to tundra thaw and methane ;
rate of fossilisation is slower than rate of combustion / fossil fuels are non-
renewable ;
positive human activities / carbon capture technology ;
(idea of) loss of equilibrium / balance ;

© UCLES 2018 Page 5 of 11

Page 857 of 877


061
10/41 Cambridg
ge IGCSE – Marrk Scheme Octob
ber/November 2018
PUBLISHED
Question A
Answer M
Marks Guidan
nce

2(a) fung
gus ; 1

2(b) smaall / no, clear area a / AW ; ora 2 A more bacteria growing


(antiibiotic in disc), not
n killing bacteriia / (continued) rreproduction ; orra

2(c)(i) 1 correct ref. to mutation


m (of bacte
eria) / have resisstance gene ; 6
2 mutation is a ch hange in the DNA A / base sequen nce ;
3 mutations can be b caused by, (io onising) radiation / (named) chemicals ;
4 variation (in ability of bacteria to o survive antibio otic treatment) ;
5 ref. to (natural) selection / evolu ution ;
6 bacteria with resistance (survive e and) reproducce / breed / multip ply
/ produce offsprring ;
7 bacteria with, no o / little, resistan
nce, die / do not rreproduce ;
8 (bacteria that su urvive) pass on tthe resistance, a allele / gene, to more
m
bacteria ;
9 bacteria reproduce quickly ;
10 exposure to anttibiotics acts as a selection presssure ;
11 only use antibio otics when essen ntial ;
12 complete the full course of presscribed antibioticcs ;
13 isolation of patieents with infectio ons ;
14 improved, healtthcare / sanitatio on / nutrition / goo
od diet / hygiene /
cleanliness / scrreening / AW ;
15 a
and 16
AVP ;;

2(c)(ii) virusses, are not alive


e / not made of ccells / AW ; ora 1
virusses do not have, a cell wall / nam
med cell compon nent ;

© UCLES 2018 Page 6 of 11

Page 858 of 877


0610/41 Cambridge IGCSE – Mark Scheme October/November 2018
PUBLISHED
Question Answer Marks Guidance

2(d) small / take up little space ; 3


reproduce rapidly / easy to grow ;
contain plasmids ;
transformation described / genetic modification / inserting genes ;
no ethical concerns ;
same genetic code as other organisms ;
same DNA ;
can make complex molecules / AW ;
AVP ;

Question Answer Marks Guidance Notes

3(a) no, cytoplasm / (named organelle) / hollow ; 3


ref. to lignin (in walls)
(cell walls) are waterproof / water impermeable / AW
(secondary) thickening of cell wall ;
long / elongated (cells / vessels / tubes) ;
(bordered) pits (for water movement between vessels) ;
no, (perforated) end / cross walls (between cells) / end plates to connect
vessels (end to end) ;

3(b) (water enters) root hair (cells) / M ; 5


by osmosis ;
the soil has a higher water potential than the root (cells) ; ora
water moves from an area of high(er) water potential to low(er) water
potential ;
active transport of ions to create a water potential gradient ;
(across / through partially permeable), membrane(s) ;
ref to root cortex / L – cortex / M to L to (K) to J ;
AVP ;

3(c)(i) 87 ;; 2

© UCLES 2018 Page 7 of 11

Page 859 of 877


0610/41 Cambridge IGCSE – Mark Scheme October/November 2018
PUBLISHED
Question Answer Marks Guidance

3(c)(ii) the nearer the tip / zone 1, the lower flow rate ; ora 3
flow rate increases (from tip to bulb) in both treated and healthy roots ;
flow rate is greater in zone 1 in the treated roots ;
flow rate is lower in zones 2 and 3 in the treated roots ; ora
comparative data quote with units ;

3(c)(iii) xylem vessels are dead, so toxins / treatment have no effect ; 2


osmosis / water flow into root, does not rely on living cells / energy
/ is passive / AW ;
AVP ;

Question Answer Marks Guidance

4(a) 5 one mark for each correct row


function name of part letter on
Fig. 4.1

carries impulses to the brain optic nerve Y;

focuses light onto the back of the lens S;


eye

controls the tension of the ciliary, muscles Q;


suspensory ligaments / body

tissue the detects light and colour retina W;

location of most cone cells fovea X;

4(b)(i) antagonistic ; 1

4(b)(ii) accommodation ; 1

© UCLES 2018 Page 8 of 11

Page 860 of 877


0610/41 Cambridge IGCSE – Mark Scheme October/November 2018
PUBLISHED
Question Answer Marks Guidance

4(c) cones are less sensitive in low light ; 2


cones detect colour ;
rods work in low light but can’t detect colour / AW ;

4(d)(i) XBXb ; 1

4(d)(ii) XbY ; 1

4(d)(iii) solid shaded square on Fig. 4.2 ; 1

4(d)(iv) one X chromosome from each parent / an X from father ; 2


mother does not have any colour-blind alleles / father passes on one colour-
blind allele ;
(all female offspring are) heterozygous / XBXb ;

Question Answer Marks Guidance Notes

5(a)(i) chemical substance produced by a (endocrine) gland ; 3


carried by the blood ;
alters the activity of specific target organs / AW ;

5(a)(ii) (insulin) stimulates enzymes (production) ; 2


conversion of glucose to glycogen ;
glycogen is stored / insoluble ;
increased, uptake / absorption / respiration, of glucose by liver (cells) ;

5(b)(i) deamination / removal of nitrogen containing part (of amino acids) ; 2


to form urea ;
(part of) amino acid converted to ammonia ;
ammonia converted to urea ;

5(b)(ii) (protein) synthesis ; 1

© UCLES 2018 Page 9 of 11

Page 861 of 877


0610/41 Cambridge IGCSE – Mark Scheme October/November 2018
PUBLISHED
Question Answer Marks Guidance

5(c)(i) aerobic / using oxygen ; 2


respiration / (to produce) carbon dioxide and water;
ref. to enzymes ;
AVP ; converted back to, glucose

5(c)(ii) as alcohol consumption increases risk of dying of liver disease increases ; 4


similar trend in males and females ;
comparative data quote with units for g per day ;
men exponential / women are not exponential / AW ;
at low consumption females have higher risk ; ora
same risk at 112 g per day ;

Question Answer Marks Guidance

6(a)(i) reflex (action) ; 1

6(a)(ii) contains antibodies / passive immunity / idea of fighting infections ; 4


bonding with mother /AW ;
is at a suitable body temperature ;
sterile / less risk of infection / contamination ;
convenience / always available / no preparation ;
cheap / free ;
easy to digest / less risk of colic / less risk of diabetes in child ;
no additives / less risk of allergies ;
idea of volume is controlled / no over-feeding ;
nutrient requirements met / change with age / change with development ;
AVP ;;

© UCLES 2018 Page 10 of 11

Page 862 of 877


0610/41 Cambridge IGCSE – Mark Scheme October/November 2018
PUBLISHED
Question Answer Marks Guidance

6(b)(i) 3
enzyme substrate product(s)

amylase starch glucose / maltose ;

maltase maltose glucose ;

protease protein amino acids ;

6(b)(ii) high temperatures denature enzymes / AW ; 2


low temperatures result in low energy / fewer collisions / slower reactions
/ AW ;
enzymes work best / most efficient at optimum temperature ;

6(b)(iii) pH ; 1
enzyme concentration ;
substrate concentration ;

© UCLES 2018 Page 11 of 11

Page 863 of 877


0610/42 Cambridge IGCSE – Mark Scheme October/November 2018
PUBLISHED
Question Answer Marks Guidance

1(a) 3
name of an organism from Fig.1.1

producer algae / (phyto)plankton / clover / grass ;

secondary marsh rice rat / (stone) crab / mycid


consumer shrimp / blenny / (bald) eagle ;

an animal that feeds (bald) eagle / blenny / (spotted) sandpiper ;


at two trophic levels

1(b) nitrification ; 2 A oxidation


(nitrifying) bacterium / bacteria ;

1(c) root hairs ; 3


(by) active, transport / uptake ;
across (cell) membranes ;
against a concentration gradient / low to high concentration / AW ;
by proteins (molecules / pumps / carriers) ;
use of, energy / ATP ;
(also by) diffusion ;

1(d) ribosome / (rough) endoplasmic reticulum ; 1

1(e) nitrogen fixation ; 1

© UCLES 2018 Page 5 of 12

Page 864 of 877


0610/42 Cambridge IGCSE – Mark Scheme October/November 2018
PUBLISHED
Question Answer Marks Guidance

1(f) little energy is transferred from one trophic level to the next /AW ; 3
not enough energy, at the top of the pyramid / at higher trophic levels, to
support a large number of organisms ;
named example of energy loss ;
heat / in respiration / in (named) metabolic processes / movement
/ excretion / urine / faeces
not all organisms / parts of organisms (in one trophic level), are,
eaten / digested ;
AVP ;

Question Answer Marks Guidance

2(a) a version of a gene ; 1

2(b) change in base (sequence of DNA) ; 3


DNA / gene / base sequence, codes for, protein / enzyme ;
ref. to mRNA ;
different (sequence of) amino acids in, protein / polypeptide / enzyme ;
(mutant / changed) enzyme / active site, has different, shape / structure ;
(active site / enzyme) not complementary to substrate / enzyme-substrate
complexes cannot form / substrate will not fit into or bind ;

2(c) the allele for dwarfism is, recessive / t ; 2


both parents are heterozygous (so do not express the allele) ;

2(d)(i) ref. to asexual reproduction ; 3


(plantlets / cells / offspring grow by) mitosis ;
all cells / new plants, are genetically identical ;
AVP ;

© UCLES 2018 Page 6 of 12

Page 865 of 877


0610/42 Cambridge IGCSE – Mark Scheme October/November 2018
PUBLISHED
Question Answer Marks Guidance

2(d)(ii) competition for resources as all individuals are close together ; 3


increased risk of inheriting harmful, alleles / features / trait ;
no / little, (genetic) variation ;
no new adaptive features ;
no evolution / no (natural) selection / no artificial selection / AW ;
no / little, ability to respond to (named) environmental change ;
all individuals are susceptible to the same, diseases / pests ;
higher risk of extinction ;

Question Answer Marks Guidance

3(a)(i) thick / strong, (cell) wall ; 2


withstanding, tension / collapse / hydrostatic pressure / AW ;

lignin (in walls) / walls are impermeable ;


prevents collapse / waterproofing ;

wide / AW ;
transport large volumes of water ;

no (cell) contents / empty / dead cells / like pipes / like tubes ;


no / little resistance to flow of water / allows water to flow easily / lots of water
/ continuous columns of water / no obstruction ;

no, cross walls / end walls ;


no / little, resistance to flow of water / allows water to flow easily / lots of
water / continuous columns of water / no obstruction ;

(bordered) pits ;
lateral transport / AW ;

© UCLES 2018 Page 7 of 12

Page 866 of 877


0610/42 Cambridge IGCSE – Mark Scheme October/November 2018
PUBLISHED
Question Answer Marks Guidance

3(a)(ii) evaporation from (cell walls) in mesophyll ; 4


diffusion of water vapour through stomata ;
reduction of, pressure / water potential, at top (of plant) resulting in water
moving upwards ;
continuous column of water (in the xylem) ;
cohesion of water (molecules) ; A if described incorrectly
cohesion described as, forces / attraction, between water molecules ;
transpiration pull ;
water enters or leaves xylem, by osmosis / down water potential gradient ;
AVP ;

3(a)(iii) support / described ; 1

3(b) increase / decrease (in rate of transpiration) ; 3


more / less, evaporation ;
increase / decrease, rate of diffusion (of water vapour) ;
ref. to (kinetic) energy of (molecules of) water ;
stomatal pores become, wider / narrower ; A stomata close
guard cells become, turgid / flaccid ;

Question Answer Marks Guidance

4(a) chemical substance produced by a (endocrine) gland ; 3


carried by the blood ;
alters the activity of specific target organs / AW ;

4(b)(i) (A) 210 (mg 100 per cm3) and (B) 88 (mg 100 per cm3) ; 1

4(b)(ii) 136 (%) ;; 2

4(b)(iii) increases / decreases, more steeply / faster ; 2


increases to / has, a higher concentration ;
reaches a peak / decreases, (much) later ;
does not return to, starting concentration / original value / normal / AW ;
any comparative use of figures with correct units ;

© UCLES 2018 Page 8 of 12

Page 867 of 877


0610/42 Cambridge IGCSE – Mark Scheme October/November 2018
PUBLISHED
Question Answer Marks Guidance

4(b)(iv) increase in glucose concentration detected (by pancreas) ; 4


insulin is, secreted / released (into the blood) ;
role of the pancreas in, detection / secretion of hormones ;
insulin, stimulates / AW, liver / muscle ;
increase, uptake / respiration, of glucose ;
glucose is converted to glycogen ;
activation / AW, of enzymes (in liver cells) ;
(blood) glucose concentration, decreases / maintained ;
ref. to homeostasis / negative feedback ;

4(b)(v) tiredness / lethargy / fatigue / described ; 3


breathlessness ;
dizziness / fainting / light-headedness / coma ;
frequent urination / AW ;
sticky / sweet, urine ;
urinary tract infection / UTI ;
recurrent thrush ;
thirsty (all the time) / drinking lots of water ;
dry mouth ;
weight loss ;
hunger / eating a lot of food ;
sweet-smelling breath ;
change(s) in behaviour ; e.g. irritability / confusion / mood swings
nausea / vomiting ;
blurred vision / blindness ;
cuts / grazes / wounds, that do not heal ;
AVP ;

© UCLES 2018 Page 9 of 12

Page 868 of 877


0610/42 Cambridge IGCSE – Mark Scheme October/November 2018
PUBLISHED
Question Answer Marks Guidance

5(a) red blood cell: 6


feature:
red blood cells smaller than (named) white blood cell(s) / ora ;
biconcave (disc / shape) / no nucleus ;
role:
contains haemoglobin / transports oxygen / transports carbon dioxide ;

lymphocyte:
feature:
little cytoplasm / large(r) nucleus / nucleus fills most of the cell ;
role:
ref. to active immunity / responds to, antigen(s) or vaccine(s) / produce,
antibodies or antitoxins / ref. to memory cells ;

phagocyte:
feature:
lobed / irregular-shaped / C-shaped / AW, nucleus ;
role:
engulf pathogens / phagocytosis / AW ;

5(b)(i) fibrinogen fibrin ; 1

5(b)(ii) prevent blood loss ; 2


prevent entry of (named), pathogens / microbes ;
ref. to wound healing / tissue repair ;

5(c)(i) (P) XHXh ; 3


(Q) XhY ;
(R) XHY ;

5(c)(ii) 0.25 / 25% / 1 in 4 / ¼ ; 1

5(c)(iii) gene is located on, a sex chromosome / X or Y / X / Y ; 2


characteristic is more common in, males / one sex (than the other) ;

© UCLES 2018 Page 10 of 12

Page 869 of 877


0610/42 Cambridge IGCSE – Mark Scheme October/November 2018
PUBLISHED
Question Answer Marks Guidance

6(a)(i) dry scaly skin ; 2


leathery / soft-shelled, eggs ;

6(a)(ii) cellulose / cell wall ; 2


chloroplast / chlorophyll ;
starch grains ;
(large / permanent / central) vacuole ;

6(b)(i) amylase ; 1

6(b)(ii) mouth ; 2
small intestine ;

6(c) monitoring / AW, population(s) / individual(s) ; 5


habitat, protection / restoration ;
reducing / prevention, of pollution ;
removal / AW, of alien species ;
preventing colonisation by alien species ;
hunting ban / prevent poaching ;
government / legislation, to protect species ;
create, exclusion zones / reserves (so not disturbed by people) ;
specific, times when / areas where, hunting / AW, not allowed ;
international agreements to limit trade ;
removal to, zoos / botanical gardens / wildlife parks ;
captive breeding / breeding programme (in situ or ex situ) ;
seed banks / frozen zoos / cryopreservation / AW ;
artificial insemination / IVF / use of surrogates / AW ;
reintroduction programmes ;
education / awareness ;

© UCLES 2018 Page 11 of 12

Page 870 of 877


0610/42 Cambridge IGCSE – Mark Scheme October/November 2018
PUBLISHED
Question Answer Marks Guidance

6(d) food ; 3
drugs / medicines ;
(named) fuel / biomass for energy ;
timber / building materials / AW ;
water ; ignore rain
oxygen ;
(named) mineral ;
gene(s) ;
clothing / fur ;
AVP ;;;

© UCLES 2018 Page 12 of 12

Page 871 of 877


0610/42 Cambridge IGCSE – Mark Scheme March 2018
PUBLISHED
Question Answer Marks Guidance

1(a)(i) each row in this order: 5 6 correct = 5 marks


4/5 correct = 4 marks
F A E 3 correct = 3 marks
2 correct = 2 marks
C B D 1 correct = 1 mark
;;;;;

1(a)(ii) prokaryote ; 1

1(b) presence of genetic material / DNA / RNA ; 1


presence of protein ;

1(c)(i) (actual length of bacterium) = size / length, of the image ÷ magnification ; 1

1(c)(ii) 2.6 (µm) ; 1

1(d)(i) 1 produces a toxin ; 4


2 bacteria / toxin, attach to the wall of the, small / large, intestine ;
3 correct ref to chloride ions ;
4 secretion / loss, chloride ions, into the, small intestine ;
5 causing a water potential gradient / water potential of the intestinal lumen is
lowered ;
6 causing osmotic movement of water into the gut / water flows from, the cells /
blood, into the, lumen / gut ;
7 loss of salts from the blood ;
8 causing, diarrhoea / dehydration ;

1(d)(ii) oral rehydration (therapy / salts / treatment / solution) ; 1


in-take of water, sugar and, salt / ions ;
antibiotics ;

© UCLES 2018 Page 5 of 10

Page 872 of 877


0610/42 Cambridge IGCSE – Mark Scheme March 2018
PUBLISHED
Question Answer Marks Guidance

2(a) describe and compare 6


1 COPD higher in villages than cities ; ora
2 COPD increasing in both areas ;
3 increasing more rapidly in villages ;
4 fluctuation / COPD decreases, in cities in 2001 ;
5 data quote comparing villages and cities including year and million ;

suggest
6 lack of healthcare in villages ;
7 more people smoke in villages / passive smoking ;
8 lack of awareness / education, in villages ;
9 pollution in villages ;
10 poor quality housing in villages ;
11 differences in diet ;
12 AVP ; e.g. lack of physical activity ;

2(b)(i) 1 nasal hairs, trap particles / AW ; 4


2 goblet cells secrete mucus ;
3 particles trapped in the mucus ;
4 cilia moving the mucus ;
5 mucus (containing particles) moved, away from the gas exchange surface
/ towards the throat / AW ;
6 mucus, swallowed / AW ;
7 AVP ; phagocytes / sneezing

2(b)(ii) more oxygen ; 2


less carbon dioxide ;
less water vapour ;

2(c)(i) intercostal ; 1

2(c)(ii) (pressure) decreases and (volume) increases ; 1

© UCLES 2018 Page 6 of 10

Page 873 of 877


0610/42 Cambridge IGCSE – Mark Scheme March 2018
PUBLISHED
Question Answer Marks Guidance

3(a)(i) 0.2 ; 1

3(a)(ii) pyramid shape with four trophic levels widest at the bottom ; 3 A ecf from part (i)
bars drawn at correct width (± half a small square) ;
each bar labelled with trophic level ;

3(b) decomposer ; 1

3(c)(i) photosynthesis ; 3
ref. to chlorophyll ;
light energy is transferred to chemical energy ;
(named) glucose formed (from carbon dioxide and water) ;
named example of carbohydrate molecule used to make biomass ; e.g. cellulose, sucrose, starch,
protein, DNA, tissues

3(c)(ii) energy is lost between the trophic levels / energy decreases up the trophic levels ; 3
not all of the organism is, eaten / digested / absorbed ;
energy is lost, as heat / in respiration / in metabolic processes / named metabolic
process / movement ;
energy lost in, excretion / faeces / urine ;
(so) less energy to support the next trophic level ;

3(d) 1 prevents extinction / protection of endangered species ; 3


2 maintains genetic diversity / biodiversity / AW ;
3 maintaining habitat / ecosystem / breeding grounds ;
4 maintain, nutrient recycling ;
5 maintain, resource provision / food / drugs ;
6 maintain, food chains / food webs / trophic levels / description of ;
7 prevent soil erosion / flooding ;
8 AVP ; as a leisure facility / tourism / education

© UCLES 2018 Page 7 of 10

Page 874 of 877


0610/42 Cambridge IGCSE – Mark Scheme March 2018
PUBLISHED
Question Answer Marks Guidance

4(a) function letter name 4 1 mark for each correct row

releases oestrogen F ovary


site of fertilisation A oviduct
site of implantation E uterus lining
dilates during the process of birth C/D vagina (C) / cervix (D)
;;;;

4(b) 23 ; 2
46 / 23 pairs ;

4(c) 1 cases increases then decrease ; 3


2 large increase between 10–14 and 15–19 ;
3 most cases in the 15–19 age group ;
4 from 15–19 number of cases decrease / from 20–24 number of cases steep
decrease ;
5 no cases above 55 years old / in 55–64 age group / 65+ age group ;
6 data quote with number of cases and age group ;

4(d)(i) antibiotics ; 1

4(d)(ii) HIV ; 1

4(d)(iii) (named) bodily fluids / sexual fluid ; 3


barrier ;
condom / femidom ;

© UCLES 2018 Page 8 of 10

Page 875 of 877


0610/42 Cambridge IGCSE – Mark Scheme March 2018
PUBLISHED
Question Answer Marks Guidance

5(a)(i) advantages 4
1 lower (dry) mass ;
2 fewer weeds / lower weed density ;
3 less competition ;
4 therefore higher yield of crop ;

disadvantages
5 more treatments ;
6 higher cost / time / effort ;
7 idea of increased environmental impact ;
8 increased health risks ;

5(a)(ii) increased strength of wind ; 2


increased precipitation ;
resistance ;
type of weed ;
AVP ;; e.g. ref. to amount / too much time between treatments

5(a)(iii) 1 absorbed by (broad leaved) weeds / selective for weeds ; 3


2 less absorption by (narrow leaved) crops ;
3 increase the growth (rate) of weeds ;
4 plant cannot produce enough, glucose / photosynthesise fast enough ;
5 weeds cannot maintain rate of growth ;
6 AVP ; e.g. falls over and can’t absorb sunlight

5(b) gravi / geo ; 2


tropism ;

© UCLES 2018 Page 9 of 10

Page 876 of 877


0610/42 Cambridge IGCSE – Mark Scheme March 2018
PUBLISHED
Question Answer Marks Guidance

6(a) breakdown of large to small molecules ; 2


from insoluble to soluble ;

6(b)(i) test-tube 1 5
1 (less cloudy), slower break down of egg white solution / protein ;
2 (no HCl so) pH of the solution is too high ; ora
3 high pH denatures pepsin / enzyme ;

test-tube 2
4 hydrochloric acid causes a low pH ;
5 pepsin works best in / optimal activity, low pH / acidic conditions ;

test-tube 3
6 pepsin / enzyme, unable to break down, protein / egg white solution ;
7 boiling denatures, pepsin / enzyme ;

8 ref to enzyme-substrate complex / fewer successful collisions ;


9 high pH / boiling, changes shape of active site ;

6(b)(ii) as a control ; 2
to show that pepsin is responsible for the protein digestion ;
to show that hydrochloric acid does not digest the protein ;

6(b)(iii) stomach ; 1

6(c) maltose broken down ; 3


to glucose ;
on the membranes of the epithelial lining ;
(acts) in the small intestine / duodenum ;

© UCLES 2018 Page 10 of 10

Page 877 of 877

You might also like